Sie sind auf Seite 1von 402

Part 1: Neurologic critical care

Instructions: For each question select the most correct answer.

1. A 50-year-old man is brought to the emergency department by


paramedics after collapsing at a tennis club. A bystander
provided CPR, and on arrival to the scene the paramedics
successfully cardioverted the patient from ventricular
fibrillation. Patient is now unresponsive to stimuli and receiving
mechanical ventilation. His temperature is 37.4°C (99.3°F), HR is
110/min, BP is 135/78 mm Hg, and RR is 20/min.

Which of the following interventions is most likely to improve


neurologic outcome in this patient?

A. Amiodarone
B. Normothermia
C. Primary seizure prophylaxis with phenytoin
D. Hypothermia (32°C - 34°C [89.6°F-93.2°F])
E. Beta blockade

2. A 75-year-old woman with a past medical history of breast


cancer presents to the emergency department after being found
comatose. She was last seen well yesterday by her daughter. On
admission to the ICU, her BP is 147/88 mm Hg, HR is 108/min,
and RR is 14-20/min. she is intubated and mechanically
ventilated but not sedated, Her examination shows clear lung
fields and normal findings of cardiac and abdominal
examinations. There is no evidence of pressure sores. Cranial
nerve examination shows symmetric face, normal pupillary
reaction, and normal oculocephalic reflexes. Her motor
responses are flexor bilaterally, and toes are upgoing. Urine
toxicology results are normal and her WBC count is 11,600/μL,
with 80% neutrophils and 5% bands. She has a creatinine level of
256 mg/dL, normal result on arterial blood gas testing and
chemistry panel, and a normal CT and CT angiography of the
head and neck.

Which of the following is the most appropriate next step?

A. Cerebral MRI
B. Comprehensive blood hepatic profile
C. Blood cultures
D. Electroencephalogram
E. Random cortisol

3. A 68-year-old man with significant history of hypertension,


diabetes mellitus, and benign prostatic hypertrophy is admitted
to the hospital for onset of confusion, memory loss, and agitation
for the last 4 weeks. In the hospital he becomes very agitated,
requiring a 1-on-1 supervision, chemical restrains , and oxygen
through a Venturi mask and has to be transferred to the ICU.
Neurological examination findings are significant for expressive
aphasia, normal cranial nerves, and a motor examination
showing extrapyramidal rigidity and stimulus-induced
myoclonus. Serology and blood chemistry, and lumber puncture
results are normal. MRI is shown in the Figure.
Which of the following is the most likely diagnosis for this
patient’s altered mental status?

A. Acute ischemic stroke


B. Spongiform encephalopathy
C. Status epilepticus
D. Hepatic encephalopathy
E. New-onset Alzheimer dementia
4. A 43-year-old woman is brought to the hospital for abnormal
mental status during the past 6 hours. Initially, she felt dizzy and
nauseated and fell asleep. The husband then called the
ambulance as she was not waking up. She received fluids,
naloxone, thiamine, and dextrose en route to the emergency
department without improvement. Her temperature is 37°C
(98.5°F), BP is 150/102 mm Hg, HR is 108/min, RR is 14-20/min,
and oxygen saturation is 92% on 2 L per nasal cannula. The
patient is lethargic with Glasgow coma scale score of 8 (eye
response of 2, motor response of 4, and verbal response of 2),
pupils are midpoint and fixed, and oculocephalic reflexes are only
present on vertical movement of the head. Her reflexes are 3+
throughout and she has bilateral upgoing toes. Abnormal
asynchronous fast tonic movements of the arms and legs are also
noted. Arterial blood gas studies show pH of 7.20, paco2 of 60 mm
Hg, pao2 of 62 mm Hg, and oxygen saturation of 90%. The patient
is intubated and mechanically ventilated. CT shows a hyperdense
signal at the level of the basilar artery.

Which of the following interventions is most appropriate at this


point?

A. Systemic thrombolytics
B. Immediate IV antibiotics
C. IV phenytoin
D. Local thrombolytics
E. Intracranial pressure monitor

5. A 65-year-old woman with an unknown medical history is


admitted to the hospital through the emergency department
With concerns of fevers, shortness of breath, productive cough,
and myalgias. Chest radiograph on admission shows a right
lower lobe infiltrate and IV antibiotics are started for the
treatment of a community-acquired pneumonia. Laboratory
studies show an elevated white blood cell count with a
neutrophil predominance, mild transminitis, hypoalbuminemia,
anemia, thrombocytopenia, and an elevated prothrombin
time/partial thromboplastin time. During the night, the patient is
confused, agitated, and paranoid. The nurses note that she is
making up vivid and elaborate stories to account for her
presence in the hospital. On examination, she is awake, alert with
fluent speech, and moving all extremities equally. She does not
cooperate with the cranial nerve examination but shows
limitation of eye movements.

Which of the following IV treatments is it most appropriate to


give this patient next?

A. Antibiotics
B. Thiamine and dextrose
C. Naloxone
D. Haloperidol
E. Dextrose solution

6. A 25-year-old graduate student is transferred to the ICU from the


student health service after presenting with changes in mental
status and a seizure. The patient had returned from a white-
water rafting vacation with friends when he had onset of acute
febrile illness characterized by headache, disorientation, and
behavioral abnormalities. He was aphasic and appeared to have
hallucinations before presentation. He had right arm motor
weakness and a focal seizure confined to that area. In the ICU, the
patient progresses to unresponsiveness. There is no significant
past medical history, and the patient has not traveled outside of
the united states in the past 10 years. Temperature is 39.3°C
(102.6°F) rectally, RR is 18/min, HR is 105/min, and BP is
110/70 mm Hg. on physical examination, the patient is
unresponsive with a slightly rigid neck. His pupils are
midposition, reactive to light, and optic disks appear to be flat.
Findings of hear, lung, and abdominal examinations are normal.
Neurologic examinations reveals hyperreflexia in the right upper
extremity with an upgoing toe on that side. Cerebrospinal fluid
from a lumbar puncture reveals an opening pressure of 35cm
H2O and WBC count of 200/μL, predominantly monocytic;
closing pressure is 5 cm H2O. protein level is mildly elevated,
glucose level is normal, Gram stain in negative, and RBC count is
1,000/μL.

Which of the following is the most appropriate intervention for


this patient?

A. IV dexamethasone
B. Immediate neurosurgical evaluation
C. IV acyclovir
D. IV amphotericin B
E. Preforming another lumbar puncture in 6 hours

7. A 27-year-old woman is admitted for lethargy and headache


during the last 7 days. Her past medical history is significant for
oral contraceptive use and recent appendectomy. On physical
examination, her temperature is 38.1°C (100.6°F), BP is 138/86
mm Hg, HR is 100/min, and RR is 24/min. neurological
examination reveals a lethargic patient who arouses to voice but
is oriented to self. Her cranial nerve examination findings are
normal except for papilledema. Motor examination findings are
normal, with no spasticity and movement of all 4 extremities to
simple commands. Reflexes are 3+ throughout. No ataxia is
revealed and she does not cooperate for the gait or sensory
examination. CT shows petechial hemorrhages over the left
parietal convexity.

At this point, which of the following interventions is most


appropriate?

A. Hyperventilation to achieve a Paco2 of 28-32 mm Hg


B. Starting IV vancomycin and cefepime
C. Starting IV dexamethasone
D. Infusing IV fluids and heparin
E. Starting IV nicardipine

8. A 27-year-old woman is found on the floor by her husband, with


tonic-clonic movements and foaming from the mouth. Emergency
medical services administers IV lorazepam, with control of the
seizure in the field. On arrival to the emergency department, she
is intubated for airway protection using etomidate and
succinylcholine. On arrival to the ICU, temperature is 41.1°C
(106°F), BP is 90/48 mm Hg, HR is 128/min, and RR is 18/min.
her arterial blood gas results show pH of 6.98, pao2 of 60 mm
Hg,paco2 of 120 mm Hg, and oxygen saturation of 99%.

The most appropriate next step in the management of this


patient is to:

A. Administer IV bicarbonate.
B. Decrease the minute ventilation by lowering tidal volume.
C. Start dantrolene.
D. Start IV hydrocortisone.
E. Increase minute ventilation

9. The locked-in syndrome is best characterized by which of the


following clinical descriptions:
10. A 45-year-old, right-handed man has sudden onset of confusion.
On examination he is alert, not cooperative, and unable to follow
commands. On direct examination, he cannot read or write. On
further questioning he is unable to repeat, but keeps on saying
“mani-panimani.” The motor examination shows normal strength
of the face, arm, and leg. He cannot cooperate with the sensory
examination, but does not seem to be ataxic. His gait and reflexes
are normal.

The patient is most likely experiencing which of the following


types of aphasia?

A. Broca aphasia
B. Wernicke aphasia
C. Transcortical motor aphasia
D. Conduction aphasia
E. Global aphasia

11. A 43-year-old man ingests approximately 20 grams of


acetaminophen in a suicide attempt and is treated with N-
acetylcyteine. In the hospital, he develops acute liver failure,
becoming stuporous within 2 days of admission. His ammonia
level is 123 mg/dL. An electroencephalogram is performed
showing diffuse slowing but no ictal activity.

Apart from a liver transplant, which of the following


interventions is most likely to prevent neurological
deterioration?

A. Prophylactic antiepileptic drugs


B. IV corticosteroids
C. Hyperosmolar therapy for intracranial pressure control
D. Lactulose for correction of hyperammonmia

12. A 78-year-old patient with past medical history of hypertension,


diabetes, and atrial fibrillation on anticoagulation is brought to
the hospital after a motor vehicle accident. CT of the head
shows an intracranial hemorrhage and he is taken emergently
for surgical evacuation. His postoperative course is uneventful
until urine output begins to increase (200-300 mL/h) on day 3.
He is still intubated and mechanically ventilated.

Central venous pressure is estimated at 3 mm Hg. BP is 85/55


mm Hg, HR is 108/min, RR is 24/min, and Spo2 is 98%. The rest
of the examination is unchanged. Serum sodium level is 136
mEq/L.

Half-normal saline (0.45% sodium) is initiated at 100 mL/h; 12


hours later the sodium level is 132 mEq/L. His urine output has
been 250 mL/h for the past 10 hours. Urine electrolytes show
sodium concentration of 90 mEq/L.

Which of the following is the most likely diagnosis?

A. Cerebral salt wasting


B. Syndrome of inappropriate antidiuretic hormone
C. Central diabetes insipidus
D. Osmotic diuresis

13. A 72-year-old man was admitted to the ICU 2 weeks ago with a
left basal ganglia hypertensive hemorrhage without
intraventricular extension. The patient has a medical history
significant for chronic obstructive pulmonary disease,
hypertension, and hyperlipidemia. He remains in the ICU
because of an ongoing need for mechanical ventilation. On
examination he remains hemiparetic over his right side. Lower
extremity ultrasonography reveals a right common femoral
deep venous thrombosis.

Which of the following interventions is the most appropriate in


this patient?

A. Inferior vena cava filter placement


B. Localized thrombolytics
C. IV heparin (goal activated partial thromboplastin time 1,5-2
times baseline)
D. Antiplatelet therapy
E. Warfarin (goal international normalized ratio 2.0-3.0)

14.A 78-year-old man presents to the emergency department with


right hemiparesis and expressive aphasia. Onset of symptoms
was witnessed by his wife and occurred 4 hours ago. CT of the
head shows no evidence of developing hypodensity or
hyperdensity. Temperature is 36.7°C (98°F), HR is 94/min, BP
is 170/88 mm Hg, and RR is 18/min. initial laboratory results
are within noral limits.

Which of the following is a contraindication to treatment with IV


thrombolytics?

A. Hip replacement 6 months ago


B. Duration of symptoms
C. Blood pressure
D. History of diabetes and previous stroke
E. His age

15. A 62-year-old man is admitted to the hospital after sudden


onset of right hemiparesis and aphasia 5 hours ago. Currently he
is mute and is not following commands. Temperature is 37.3°C
(99.1°F), BP is 182/100 mm Hg, HR is 102/min, RR is 22/min,
and oxygen saturation is 96% on oxygen, 2 L/min via nasal
cannula.
A. Cerebral blood flow
b. Cerebral blood volume
c. Mean transit time

Based on the result of the CT perfusion study shown in the


Figure, which of the following intervention is most
appropriate?

A. Systemic tPA
B. External ventricular drain
C. Decompressive hemicraniectomy
D. Brain tissue oxygen monitor
E. Local tPA

16.A 38-year-old woman with episodic migraine and tobacco use is


brought to the emergency department for evaluation of
persisting headache. She report that her symptoms started 3
days ago with sudden onset onset of nausea, vomiting, and what
she describes as the worst headache of her life, because she
knew it could be a migraine, she took her prescribed triptan
dose. In the emergency department she receives IV fluids, opioid
analgesics, and antiemetics. Her pain is down to 4/10, and her
neurological examination findings are unremarkable. CT finding
of the head are reported as normal.

The most appropriate next step in the management of this


patient is :

A. MRI of the brain


B. Repeating the dose of triptan and administering oxygen, 2
L/min
C. Admitting her to the ICU for close monitoring
D. Lumbar puncture
E. Discharge home with follow-up with primary neurologist

17. A 52-year-old woman with a subarachnoid hemorrhage of Hunt


and Hess grade 3 and Fisher class 3 was admitted to the ICU
after endovascular coiling of a lift posterior communication
artery aneurysm. On index day 7, you are called to see the
patient because she has stopped following commands and has
new left facial droop and leg weakness. Temperature is 37.3°C
(99.1°F), HR is 98/min, BP is 190/92 m Hg (mean arterial
pressure of the 123 mm Hg), RR is 20/min, and central venous
pressure is 10 cm H2O. Her angiogram is shown in the Figure.
which of the following is the most appropriate immediate
intervention for this patient?

A. Decrease blood pressure


B. Initiate heparin infusion
C. Administer bolus of normal saline
D. Administer local tPA
E. Perform angioplasty

18. A 64-year-old, hypertensive patient with spontaneous in


intracerebral (intraparenchymal) hemorrhage.
Which of the following locations of intracerebral hemorrhage
would be most likely to require surgical intervention (ie,
craniotomy)?

A. Basal ganglia
B. Cerebellum
C. Internal capsule
D. Parietal lobe (unilateral)
E. Pons

19. A 40-year-old man is admitted to the ICU with progressive


inability to walk during the preceding 24 hours. His BP is
140/80 mm Hg, pulse rate is 88/min, RR is 20/min, and
temperature is 37.3°C (99°F). he is alert and oriented but
reports cramping pain in his legs bilaterally with paresthesias.
Neurological examination shows 2/5+ strength in the proximal
and distal muscle groups of the lower extremities and absent
ankle and knee deep tendon reflexes. Rectal sphincter tone is
normal.

Which of the following interventions is the most appropriate


initial therapy to improve neurologic outcome with the least
adverse effects?

A. Plasmaphersis and concomitant IV immunoglobulin


B. Plasmaphersis followed by IV immunoglobulin
C. IV immunoglobulin only
D. High-dose corticosteroids

20. A 34-year-old man is admitted to the ICU for diabetics


ketoacidosis. The patient has a past medical history significant
for tobacco and cocaine abuse. He responds well to treatment
and is improving to the point where transfer to the regular
medical floor is considered. You are called to the bedside by the
nurse because the patient reports new-onset headache with new
ptosis. On physical examination, his temperature is 36°C (96.7°),
BP is 140/78 mm Hg, HR is 98/min, and RR is 18/min. he is
alert, oriented, and cooperative. There is no evidence of aphasia
or neglect. The right eye is ptotic with hypotropia/exotropia.
The right pupil is 5 mm and nonreactive and the left is 3 mm and
reactive. His face is not weak but looks asymmetric because of
the right eye ptosis. No papilledema is noted. The rest of the
neurologic examination findings are normal.

Which of the following is the most appropriate next step in this


patient’s care?

A. Electroenecephalography
B. Checking blood glucose level
C. Performing the pyridostigmine test
D. MR angiography of the brain
E. Noncontrast CT

21. An 18-year-old male is brought to the emergency department


after a motor vehicle crash. He was intubated in the field with
cervical pine in-line immobilization and given IV fluids. On
arrival to the emergency department, he was hypotensive and
received 4 L of crystalloid fluids with marked improvement in
the hemodynamics. On arrival to the ICU, his Glasgow coma
scale score remains 6 (verbal response, 1;motor response, 3; eye
response, 2).

For which of the following interventions is there the strongest


evidence for reducing this patient’s mortality risk?

A. Avoidance of hypotension (systolic BP <90 mm Hg)


B. Decompressive hemicraniectomy
C. Hyperosmolar therapy
D. Induced hypothermia
E. Intracarnial pressure monitoring

22. The patient whose brain image is shown in the Figure has
suffered severe traumatic brain injury. He remains comatose.
Which of the following treatments can be eliminated from
consideration?

A. Sedation and analgesia


B. Intracranial pressure monitoring
C. Head elevation to 30 degrees
D. Treatment od coagulopathy
E. IV corticosteroids

23. Which of the following intracranial pressure waveforms is most


indicative of low cerebral compliance?

24. The only intervention proven to decrease the incidence of


epilepsy after severe traumatic brain injury is:

A. Prophylaxis with phenytoin


B. Early decompressive hemicraniectomy and clot evacuation
C. Intracranial pressure monitoring with a goal <15 m Hg
D. IV corticosteroids
E. Primary prevention

25. A 48-year-old man was admitted to the trauma ICU after


multiorgan trauma. He had a fractured femur with “open book”
hip fracture and severe traumatic brain injury. The orthopedic
surgeon have inserted an intraparenchymal intracranial
pressure (ICP) monitor. Currently, he is receiving midazolam,
0.5 mg/h, and fentanyl, 25 μg/h. his temperature is 37.5°C
(99.4°F), BP is 150/104 mm Hg, HR is 112/min, and he is over-
breathing the ventilator. His pupils are 3 mm bilaterally and
reactive, and withdraws symmetrically with both arms. His head
remains elevated at 30 degrees, but the ICP monitor is reading
35 cm H2O. Blood gas result show pH of 7.38, Paco2 of 43 mm
Hg, Pao2 of 110 mm Hg, and oxygen saturation of 99% (0.4).

The next step to attempt to decrease his ICP is:

A. Immediately give a bolus of 20% mannitol solution, 0.25-


1g/kg over 5 minutes
B. Immediately give a bolus of 23.4% hypertonic saline solution,
20 mL over 10 minutes
C. Immediately give a bolus of fentanyl, 50 μg, and increase
sedation and analgesia
D. Target paco2 to 28-35 mm Hg
E. Start mild induced hypothermia to 33°C-35°C (91.4°F-94.9°F)

26. A 57-year-old man is admitted to the ICU after a fall from a 6-


foot ladder. An initial trauma survey reveals an isolated
traumatic brain injury with small, bifrontal contusions
associated with cerebral edema, cortical subarachnoid
hemorrhage, and intraventricular hemorrhage in the fourth
ventricle; he has moderate hydrocephalus. The patient was
intubated in the field and has a Glasgow coma scale score of 6.
Elevated intracranial pressure (ICP) is suspected.

Which of the following intervention can lower ICP immediately?

A. 23.4% hypertonic saline, 30-mL bolus


B. Head elevation
C. External ventricular drain and cerebrospinal fluid diversion
D. Mannitol
E. Hyperventilation to keeps Paco2 within 28-32 mm Hg for <30
minutes

27. The patient with the brain imaging shown in the figure had
severe traumatic brain injury due to fall from the second floor of
his home. On arrival he was awake, conversant, and wanted to
go he was found comatose with a 7-mm un reactive pupil on the
right and hemiplegia on the left.
The survival of this patient will depend on:

A. Burr holes
B. Emergent craniotomy
C. External ventricular drain
D. Corticosteroids
E. Starting 3% saline solution at 30 mL/h

28. A 28-year-old woman is admitted to the trauma ICU after falling


from her horse. She was intubated in the field and received 6 L
of crystalloid fluids in the emergency department. Temperature
is 38°C (100.3°F), BP is 68/42 mm Hg, HR is 41/min, RR is
16/min on the ventilator, and oxygen saturation is 100% (1.0).
her neurologic examination reveals a Glasgow Coma Scale score
of 7 (verbal response, 1;eye response, 3; motor response, 3),
pupils are reactive to light, and she has good gag reflex but no
cough reflex. Her motor examination reveals in arms and legs.
Primary and secondary surveys have excluded life-threatening
injuries and a focused assessment with sonography for trauma
(FAST) ruled out abdominal hemorrhage. Spinal CT is shown in
the figure. Urinary output is 0.3 mL/kg/h during the last 2
hours.
Her shock state will likely respond to the following:

A. Phenylephrine, 5-10 μg/kg/min


B. Blood transfusion
C. Neurosurgical decompression
D. Dopamine, 5-10 μg/kg/min
E. IV methylprednisolone
29. A 72-yea-old woman is involved in a car crash. She is confused
immediately after the collision and is taken to the emergency
department. On arrival, she has recovered her baseline mental
status. Head CT reveals a small, acute right frontal subdural
hematoma without mass effect. A neurosurgical consultant
recommends conservative management and observation.

Which of the following anticonvulsant regimens will most


effectively prevent posttraumatic seizure for this patient?

A. Phenytoin for 7 days


B. Phenytoin for 6 weeks, followed by outpatient EEG
C. Valproate for 1 month
D. Levetiracetam for 1 month

30. A 58-year-old, otherwise healthy man is brought to the


emergency department after a witnessed cardiac arrest. After 26
minutes CPR, he recovered his pulse. The patein is admitted to
the ICU for hypothermia protocol. After rewarming, he remains
on high-dose vasopressors (norepinephrine and vasopressin) to
keep mean arterial pressure greater than 65 mm Hg. Seventy-
two hours after his cardiac arrest, his neurological examination
findings are characterized by a Glasgow Coma Scale score of 3,
and absent cranial nerve function (papillary, oculocephalic, cold-
caloric, and gag/cough response are absent). During motor
examination, a right hand twitch is noted as well as left
flexion/rotation of the neck after painful stimulation. He
following brain perfusion study (cerebral scintigraphy with
technetium Tc99m-hexametazime[HMPAO]) is performed based
on the equivocal finding of his motor examination.
When the family request the intensivist’s expert opinion, it is
most appropriate to tell them that:

A. The patient may not be brain dead and the findings of his
motor examination suggest that medical therapy should
continue
B. The patient is brain dead and there is no reversibility
C. The patient should have a cerebral angiography to confirm
brain death
D. A trail of superamaximal elevation of mean arterial pressure
may be helpful
E. Seizure are likely, so trail of IV midazolam may be indicated
31. A 65-year-old man has suffered a devastating intracerebral
hemorrhage. An extraventricular drain was inserted for
hydrocephalus, and he remained in the ICU for 12 hours. He is
currently receiving norepinephrine at maximum doses to keep a
mean arterial pressure greater than 65 mm Hg, and his urine
output has been 300 mL/h. on physical examination, his BP is
98/58 mm Hg, HR is 118/min, RR is 14/min on the ventilator,
oxygen saturation is 98% (0.6), and temperature is 36°C
(96.7°F). His pupils are 5 mm and nonreactive, with no corneal
reflex, no oculocephalic reflexes and no cold-caloric responses.
Gag and cough reflexes are absent, as are spontaneous
respirations. During nail-bed pinching, he has a triple flexion
response of the both lower extremities. The team repeats the
examination within 6 hours and finds similar results, but at this
time fasciculations are apparent in the upper and lower
extremities

On the basis of his neurological examination findings, the team


should:

A. Perform an apnea test


B. Continue ongoing therapy ad have a family discussion
C. Declare him brain dead
D. Active organ donation network
E. Rewarm the patient to 37°C (98.5°F) and repeat examination

32. A 32-year-old woman admitted to the ICU for hypoxemic


respiratory failure, septic shock, and acute respiratory distress
syndrome secondary to H1N1 flu. Because of severe untreatable
hypoxemia, the team decides to bridge her to extracorporeal
membrane oxygenation. Thirteen days after admission, the
patient remains critically ill with mild improvement of
oxygenation. Her temperature is 36.7°C (98°F), BP is 120/78
mm Hg, HR is 118/min, RR is 18/min, and oxygen saturation is
92%. Without sedatives and paralytics, she appears comatose;
Glasgow Coma Scale score is 3; pupils are 5 mm and
nonreactive; and no corneal, oculocephalic, cold-caloric, or
gag/cough responses are seen. Spontaneous respirations are
absent. Motor examination is unrevealing. CT shows diffuse
cerebral edema.

The next step in the management of this patient is to:

A. Declare her brain dead


B. Perform a modified apnea test
C. Perform a classic apnea test
D. Perform an MRI of the brain
E. Continue medical management as she is likely to survive

33. Which of the following precludes the diagnosis of brain death?

A. Absence of respiratory drive


B. Absence of cranial nerve responses
C. Body temperature of 34°C (93.2°F)
D. Known cause of coma
E. Paco2 of 46 mm Hg

34. The persistent vegetative state is particularly characterized by:

A. Sleep and awake cycles


B. Organized EEG patterns
C. Absent N20 responses on somatosensory evoked potential
testing
D. State of awareness
E. Purposeful motor function
Answers:

1-D; 2-D; 3-B; 4-D; 5-B; 6-C; 7-D; 8-C; 9-A; 10-B; 11-C; 12-A; 13-C;
14-D; 15-C; 16-D; 17-E; 18-B; 19-C; 20-D; 21-A; 22-E; 23-E; 23-B;
24-E; 25-C; 26-E; 27-B; 28-D; 29-A;30-B; 31-A; 32-B; 33-C; 34-A

RATIONALE (1) Answer: D

The current advanced cardiac life support guidelines put an


important emphasis on appropriate postresuscitation support, in
an effort to improve outcomes of patient with cardiac arrest.
Increasing attention has been places on the institution of
measures that may improve long-term, neurologically intact
survival. Among the choices provided in the question, the correct
answers is D, hypothermia to 32°C-34°C (89.6°F-93.2°F) for 12-
24 hours. Therapeutic hypothermia for patients without a
hospital cardiac arrest who have a return of spontaneous
circulation and present to the hospital unresponsive has been
studied in 2 recently published, randomized clinical trials. Both
studies demonstrated that therapeutic hypothermia can improve
long-term neurological outcomes and survival in patient with
ventricular fibrillation in out-of-hospital cardiac arrest. Other
important measures to improve outcomes include the active
treatment of seizures if present, and control of hyperglycemia
associated with critical illness. Amiodraone and beta-blockers
have not been associated with improved neurological outcomes
in patients with cardiac arrest; thus, options A and E are
incorrect. Tight glycemic control has received a lot of attention in
the past several years based on the findings of a study that
demonstrated improved survival in a surgical critical care
population in which glucose levels were kept between 80 and 110
mg/dL. No studies have demonstrated that tight glycemic control
improved neurological outcomes following cardiac arrest;
therefore, option B is incorrect. However, it is recognized that
hypoglycemia can be deleterious to neurological recovery, and it
is recommended that it be treated. Although aggressive treatment
of seizures when present is recommended by the advanced
cardiac life support guidelines, there is no literature to support
primary seizure prophylaxis in these patients. Therefore, option C
is also incorrect.

RATIONALE (2) Answer: D

Patients who come to the emergency department in a coma can


be categorized into a number of groups: metabolic disarray,
overdoses and poisoning, vascular disease, infection, and seizure.
Of these groups, a commonly overlooked diagnosis is
nonconvulsive status epilepticus. According to Towne et al2, 8%
of patient evaluated for coma in their study had nonconvulsive
status epilepticus. The evaluation of the patient with coma should
include a search for metabolic abnormalities and drugs of abuse,
an investigation of medication in the home, a search for infection
including meningitis, and an evaluation by both physical
examination and imaging for vascular disease. If all of these tests
show no reason for the coma, nonconvulsive status epilepticus
should be considered. The gold standard test is
electroencephalography. Patient with previous stroke, brain
tumors, and previous history of seizures are at particular risk if
continuous seizures.

RATIONALE (3) Answer: B

Creutzfeldt-jakob disease (CJD), or spongiform encephalopathy, is


an incurable and invariably fatal degenerative neurological
condition. Symptoms of CJD are rapidly progressive demetia,
memory loss, personality changes, and hallucinations. Clinically,
patients may have aphasia, dysarthria, spasticity, and myoclonus.
The diagnosis requires a high level of suspicion. Lumber puncture
is always normal, but a high 14:3:3 protein level may be seen,
though its specificity is very poor. Electroencephalogram may
show diffuse slowing or triphasic waves. Fluid-attenuated
inversion recovery (FLAIR) or diffusion-weighted imaging (DWI)
MRI may show hyperintensity in basal ganglia and cortex (MRI in
this case shows DWI hyperintensity in cortical areas of the insula
and the parietal lobe of the left hemisphere). Apart from CJD,
other entities such as endovascular lymphomatosis, Hashimoto
thyroiditis, Whipple disease, and paraneoplastic syndrome may
present with rapidly progressive dementias.

RATIONALE (4) Answer: D

The clinical syndrome localized to the brainstem this patient is


comatose form a basilar artery thrombosis. CT shows a
hyperdense sign in the basilar artery suggesting a fresh colt. As
she is past the 4.5-hours window, she is no longer a caddidate for
systemic thrombolytics (ie, IV tPA). Intra-arterial thrombolysis is
an option for treatment of selected patients who have major
stroke of led than 6 hours’ duration hue to occlusions of the
middle cerebral artery and who are not otherwise candidates for
IV rtPA. Posterior circulation occlusions, on the other hand, may
be amenable to endovascular management within up to 24 hours
of symptom onset. However this treatment is supported by small
studies and expert opinions. Alternatives for the management of
ischemic stroke beyond the usual time windows using
endovascular approached include concentric retrievers or
fibrinolysis catheters, and/or local intraarterial thrombolysis
(depending on individual assessment). However, these
interventions have not been shown to improve outcomes after
stroke. Treatment requires the patient to be at an experienced
stroke center with immediate access to cerebral angiography and
qualified interventionists.

RATIONALE (5) Answer: B

The patient presents with a community-acquired pneumonia and


is found to be confused, confabulating, and without recent
memories of her illness. Her examination shows limited eye
movements, and laboratory tests show evidence of malnutrition.
With Korsakoff syndrome, patients typically present with
anterograde memory impairment. Wernicke encephalopathy is
characterized by confusion, gait ataxia, and oculomotor
abnormalities in an alcoholic patient, but is now also appreciated
to occur in any malnourished state including end-stage cancer,
intractable vomiting after gastric reduction procedures,
hyperemesis gravidarum. On MRI imaging, there is mammillary
body shrinkage, and involvement of the medial thalamic and
periaqueductal nuclei. Treatment consists of parenteral
administration of thiamine, 100mg/day for several days, as the
gastrointestinal mucosa absorbs poorly in the malnourished
state. Glucose should not be administered before thiamine, to
avoid precipitating Wernicke or causing an early form of the
disease to progress.

RATIONALE (6) Answer: C

This patient most likely has herpes simplex encephalitis based on


the physical examination findings, the clinical presentation, and
cerebrospinal fluid analysis. viral encephalitis begins with the
acute onset of a febrile illness. Additional findings include fever,
headache, disorientation, alterations in behavior and speech, and
neurologic findings that occasionally may be focal. These clinical
findings distinguish a patient with encephalitis from a patient
with viral meningitis. Characteristically, patients with viral
meningitis have nuchal rigidity, headache, and photophobia but
lack focal neurologic findings or significant changes in mental
status. The lumbar puncture is characteristic in this case,
revealing a cerebrospinal fluid pleocytosis with monocytic
predominance, a normal glucose level, and red blood cells
suggesting herpes simplex encephalitis. Herpes simplex virus has
a predilection for the temporal lobe and electroencephalography
may reveal focal findings. In addition, CT or MRI may reveal
changes in the temporal lobe area. For bacterial meningitis, Gram
stain might be expected to have positive results. Although fungal
meningitis would be a concern, this usually presents in a subacute
or chronic manner.

Viruses characteristically gain access to the central nervous


system through either hematogenous or neuronal spread. It is felt
that herpes simplex gains access to the central nervous system
via a neuronal pathway, with some suggestion that the olfactory
tract may be the access choice. Herpes simplex virus is the most
common cause of nonepidemic acute focal encephalitis in the
United States. Infections with this virus may occur any time of the
year and more common in patients younger than 20 and older
than 50 years of age. Blood in the cerebrospinal fluid is
characteristic of herpes simplex encephalitis. A definitive
diagnosis is established by brain biopsy, although this is usually
reserved for patient for whom therapy with acyclovir, based upon
a high index of suspicion, along with clinical findings and
cerebrospinal fluid values.

RATIONALE (7) Answer: D

Acute thrombosis of intracranial venous draining systems may


present as secondary hemorrhages from infarcted brain areas.
The clinical presentation depends generally on the venous
anatomy, but the onset of headache, abnormal mental status,
seizures, and papilledema suggests the diagnosis. Superior
sagittal sinus thrombosis is involved is about 70% of the cases;
the transverse, straight, and sigmoid sinuses may also be
involved. Acute sinus thrombosis may be seen in patients with
predisposing factors such as infection (otitis media),
prothrombotic states (hormone-related, cancer, and pregnancy),
and inherited hypercoagulable states (activated protein C
resistance by factor V Leiden mutation, protein C and S
deficiency). Full anticoagulation is indicated despite the presence
of hemorrhagic infarction, accompanied by judicious IV fluid
hydration, seizure management, and measures to control
intracranial pressure, such as head elevation and analgesia. For
rapidly progressive cases, transvenous endovascular
thrombolytic therapy is a therapeutic option.

RATIONALE (8) Answers: C

The patient has malignant hyperthermia, a genetic condition with


autosomal dominant inheritance and an incidence of 1 in every
15,000 episodes of general anesthesia or exposure to
succinylcholine. The clinical picture is characterized by a sudden
rise of Paco2 or end-tidal carbon dioxide, muscle rigidity,
hyperthermia, depressed consciousness, and autonomic
instability, and could lead to myonecrosis, rhabdomyolysis, and
acute renal failure. The syndrome is explained by excessive
calcium efflux from the sarcoplasmic reticulum in response to
halogenated inhalational agents as a result of uncoupling of
oxidative phosphorylation with dramatically increased metabolic
rate.

RATIONALE (9) Answer: A

Locked-in syndrome (LIS) consists of quadriplegia and anarthria


in the setting of preserved awareness and arousal. It not a
consciousness disorder per se but may be clinically confused with
one given the limited expressive capability of these patient. It is
associated with acute injury to the ventral pons, just below the
level of the third nerve nuclei, thus classically sparing vertical eye
movements and blinking. More rostral lesions may induce a
“total” LIS, in which even eye and lid movement is lost,
prohibiting all communication. Common etiologies of the LIS are
pontine infarction, hemorrhage, and trauma. The
electroencephalogram is usually normal.

RATIONALE (10) Answer: B

This patient has suffered a stroke of the inferior division of the


dominant (left) middle cerebral artery (MCA) and has Wernicke-
type aphasia (receptive or sensory). Apart from posterior
cerebral artery and top-of-the-basilar strokes, sudden onset of
Wernicke aphasia secondary to cerebral infraction is most often
caused by cardioembolic phenomena. The typical picture is that
of a patient with abnormal language examination findings and
inability to read, write, or follow commands. Patients with
inferior division strokes and Wernicke aphasia have normal
motor examination findings but a hemianopia may be missed if
they are not thoroughly examined during direct confrontation. A
“word salad” speech with neologisms or paraphasias is part the
typical picture. Patient with nondominant inferior division MCA
strokes usually present with confusional states, which tend to be
missed by inexperienced examiners. Truncation of the upper
division of the MCA will produce a language dysfunction
characterized by expressive or motor aphasia (Broca-type
aphasia), where comprehension may be preserved. In the
hyperacute phases of MCA occlusion, patients manifest global
language disruption, and in this state the abnormality is so
profound that patients are globally aphasic with nul
comprehension or expression. In transcortical motor aphasias,
patient have no motor expression but retain the ability to repeat
verbal commands. In transcortical sensory aphasias, patients
have no comprehension but retain the ability to follow verbal
commands. Subcortical strokes (thalamus, caudate, arcuate
fasciculus, and cingulate gyrus) are often associated with
transcortical type aphasias. Conduction aphasia, also called
associative aphasia, is a relatively rare from of aphasia. An
acquired language disorder, it is characterized by intact auditory
comprehension, fluent (yet paraphasic) speech production, but
poor speech repetition.
RATIONALE (11) Answer: C

Acute liver failure (ALF) is a catastrophic multisystem illness and


defined as the onset of encephalopathy and coagulopathy within
24 weeks of a hepatic insult in a previously healthy person.
Survival in ALF depends primarily on liver transplantation and
the treatment of multiorgan dysfunction in the ICU. Cerebral
edema occurs in up to 75% of patient with grade III and IV
hepatic encephalopathy and frequently progresses to intracranial
hypertension. Cerebral edema and herniation are the leading
cause of death to ALF. Though no randomized prospective
controlled trial has determined the optimal management of ALF
and intracranial hypertension, a protocol of cerebral perfusion
pressure optimization (CPP = MAP – ICP) greater than 60 mm Hg,
hyperosmolar therapy with mannitol and 3% saline solution,
initial hyperventilation to keep Paco2 at 30-35 m Hg, mild induced
hypothermia to 33°C (91.4°F), and/or pentobarbital was highly
successful in controlling intracranial pressure and mitigating the
risk of neurologic deterioration.

RATIONALE (12) Answer: A

This represents a typical case of cerebral salt wasting syndrome.


This controversial entity is typically encountered in the first 1-2
weeks after neurosurgical procedures (as in the case above) but
can also occur with infectious causes (meningitis) or
intracerebral bleeding. An increase in brain natriuretic peptide is
thought to be the inciting factor that results in reduced renal
reabsorption of solutes, diuresis, and thus volume depletion and
hypotension.

The finding of volume depletion is paramount for diagnosing


cerebral salt wasting syndrome in order to distinguish it from the
syndrome of inappropriate diuretic hormone (SIADH), in which
patients are euvolemic. The clinician should suspect cerebral salt
wasting in the appropriate clinical setting (after neurosurgery or
with other intracranial disease) when facing hypovolemic
hyponatremia with high urinary sodium levels indicating an
improper renal response to volume depletion.
In the current case, the patient has overt signs of volume
depletion with hypotension, tachycardia, and low central venous
pressure and thus does not have SIADH (option B is wrong).
Central diabetes insipidus and osmotic diuresis are associated
with hypernatremia, so options C and D are incorrect. Volume
expansion (with normal saline) is the initial treatment for
cerebral salt wasting. Salt tablets can be substituted later in the
course of the disease, which is usually transient and resolves
within a month of onset.

RATIONALE (13) Answer: C

According to the guidelines for prevention of stroke in patients


with ischemic stroke or transient ischemic attack, 1 for patient
who develop an intracranial hemorrhage, subarachnoid
hemorrhage, or subdural hematoma, all anticoagulants and
antiplatelets should be discontinued during the acute period for
at least 1 to 2 weeks after the hemorrhage, and the anticoagulant
effect should be reversed immediately with appropriate agents
(ie, vitamin K, fresh frozen plasma) (class III, level of evidence B).
Hence, resumption of anticoagulation may be considered at or
beyond 2 weeks from ictus. These guidelines also suggest that IV
heparin with partial thromboplastin time 1.5 to 2.0 times normal
may be a safer option when early reinstitution of anticoagulation
is indicated. Furthermore, eighth edition American college of
chest physicians guidelines2 state that for patients with
objectively confirmed deep venous thrombosis, short-term
treatment with IV unfractionated heparin, subcutaneous
lowmolecular-weight heparin, monitored subcutaneous
unfractionated heparin, fixed-dose subcutaneous unfractionated
heparin, or subcutaneous fondaparinux is recommended (grade
1A).

RATIONALE (14) Answer: D

Based upon emerging data, especially Hacke et al,2 the American


heart association/American stroke association has espoused an
expanded time window for the use of IV tPA. Patients that meet
appropriate criteria may now be treated between 3 and 4.5 hours
after symptom onset. However, several criteria in addition to
those associated with the 3- hour window must be met.
Additional exclusion criteria include age older than 80 years,
taking oral anticoagulants with an international normalized ratio
(INR) <1.7 (thus, anyone receiving oral anticoagulation
regardless of INR), a baseline national institutes of health stroke
scale score greater than 25, and history of stroke and diabetes.

RATIONALE (15) Answer: C

CT-assisted dynamic perfusion imaging (CTP) has evolved in


recent years as an adjuvant for the assessment of acute ischemic
stroke. The method permits quantitative determination of
cerebral blood flow (CBF), cerebral blood volume (CBV), and
mean transit time (MTT). CBF is the most important parameter
(CBF = CBV/MTT). It indicates how much blood is flowing
through the brain tissues in a specific period, and it is measured
in milliliters of blood per 100 grams of brain tissue per minute.
Normal values for CBF are between 50 and 80 mL/100 g/min.
Areas of the brain with high energy requirements, such as the
cortical surface or the basal ganglia, exhibit CBF values some 2–3
times higher than those for white matter. CBV is defined as the
percentage of blood vessels in a specific volume of tissue. Highly
vascularized areas of the brain such as the basal ganglia or the
cortical surface therefore have a higher CBV than the less
vascularized cerebral white matter. The CBV, however, is also a
functional parameter and changes if vessel size changes in the
context of vascular autoregulation. MTT is a direct measurement
of delayed perfusion. There is a direct correlation between it and
cerebral perfusion pressure. Even slight disturbances to the blood
supply can lead to the MTT’s being prolonged. In clinical studies
on strokes, the MTT has been a very sensitive indicator of
disruption in regional perfusion of the brain. In this case, CBF is
low in the left middle cerebral arterty (MCA) territory secondary
to an M1-segment occlusion. The CBV is low and MTT is very
prolonged, indicating a stroke. This pattern suggests that the
patient will likely develop malignant infarction, increased
cerebral edema, and intracranial hypertension, and may require a
decompressive hemicraniectomy to survive. About 80%–90% of
patients with complete MCA strokes develop malignant stroke.
The only intervention shown to improve mortality and functional
outcome after malignant MCA stroke is decompressive
hemicraniectomy. The patient is beyond the 4.5-hour window for
IV tPA, and intracranial pressure or brain tissue oxygen monitors
have not been shown to improve outcomes after malignant MCA
stroke. Intra-arterial tPA has been suggested for the management
of MCA occlusions within 6–8 hrs. of symptom onset. The
presence of low CBF in the presence of normal or mildly
decreased CBV and increased MTT suggests an area at risk or
penumbra. Some interventionists have used CTP to triage
interventions such as mechanical thrombectomy or intra-arterial
tPA.

RATIONALE (16) Answer: D

Worst headaches of life (WHOL) or thunderclap headaches


should alert practitioners to the possibility of subarachnoid
hemorrhage (SAH) from aneurysmal rupture. Several other
entities such as cervicoarterial dissections or ruptured
arteriovenous malformations may present as WHOL. Ruling out
these potentially life-threatening conditions is very important
when assessing patients with WHOL or thunderclap headaches. A
negative CT finding does not exclude the diagnosis of SAH, and
the sensitivity of CT decreases after the first 24 hours. An
inexpensive, high-yield, and relatively safe lumbar puncture is
indicated in these cases. SAH with negative CT findings and
positive lumbar puncture findings is otherwise known as a Fisher
class I SAH and carries a modest but significant chance of
rebleeding and vasospasm. Though MRI is sensible in subacute
SAH, sensitivity and specificity is suboptimal in acute SAH. Initial
misdiagnosis after SAH increases the probability of poor outcome.

RATIONALE (17) Answer: E

The patient has developed cerebral vasospasm of the right


anterior cerebral artery and the distal M1 segment of the middle
cerebral artery, as seen on cerebral angiogram. The outcome after
subarachnoid hemorrhage (SAH) depends on several factors,
including the severity of the initial event, medical management,
surgical variables, and the incidence of complications. Cerebral
vasospasm occurs in 70% of patients with SAH, correlating with a
baseline mortality 1.5–3 times greater. The diagnosis of
vasospasm may be suspected based on clinical examination
findings and daily variations of transcranial Doppler
ultrasonography results, with cerebral angiography serving as the
gold standard for its confirmation. Most often, vasospasm begins
on day 3 after SAH and reaches a maximum on days 6–8.
Symptoms are related to the region of cerebral ischemia. If the
vasospasm is severe enough and remains untreated, cerebral
infarction may occur. One of the most important aspects of
vasospasm is its failure to consistently respond to treatment.
Although it is well recognized that the volume of blood on CT is
the most reliable prognostic factor for vasospasm, the precise
pathophysiology of vasospasm is poorly understood. Angioplasty
and intraarterial vasodilators, either alone or in combination, are
the mainstay of therapy for vasospasm after SAH. Angioplasty
commonly results in excellent, and nearly always permanent,
angiographic reversal of vasospasm. By stretching the artery,
balloon dilatation leads to an immediate and profound
impairment in smooth muscle function, with in vitro and in vivo
studies demonstrating both a functional and morphologic change
in these fibers. Observations indicate that earlier treatment with
balloon angioplasty for patients with medically intractable
vasospasm may result in better clinical outcomes. Rosenwasser et
al3 sought to define a time frame in which balloon angioplasty
would be maximally effective and determined that early
treatment within 2 hours was associated with better
angiographic outcomes and clinical improvement. Maintenance of
normovolemia is key in the management of SAH patients; though
the prophylactic use of the so called “triple-H therapy”
(hypervolemia, hypertension, hemodilution) is not indicated as
initial management, it should be reserved for those patients with
confirmed vasospasm. The rationale of triple-H is that the
maintenance of high circulating blood volume will enhance

RATIONALE (18) Answer: B

Although spontaneous intracerebral (intraparenchymal)


hemorrhage (ICH) is more than twice as common as spontaneous
subarachnoid hemorrhage, there have been comparatively few
randomized trials of ICH treatment. Furthermore, as treatment
strategies worldwide have been widely divergent, there has been
no consensus on medical or surgical management of ICH. In 1999,
a writing group from the Stroke Council of the American Heart
Association (AHA) reviewed the available data and published
guidelines for ICH management.1 The most common cause of ICH
remains hypertension. Hypertensive hemorrhages secondary to
small-vessel disease most often occur in the putamen, global
pallidum, thalamus, internal capsule, deep periventricular white
matter, pons, and cerebellum. Other causes of ICH include illicit
drug abuse (usually cocaine), vascular malformations or
aneurysms, hemorrhage into cerebral infarcts or brain tumors, or
complications of thrombolytic or anticoagulant therapy.
Recommendations from the AHA scientific statement included the
following:
1. The initial study of choice for suspected ICH is noncontrast CT
of the head.
2. Angiography should be considered for surgical candidates
without a clear cause of hemorrhage (ie, young, normotensive
patients). However, angiography is not necessary for older
hypertensive patients who have a deep hemorrhage in the basal
ganglia, thalamus, cerebellum, or brainstem if CT does not suggest
a structural lesion. MRI and MR angiography may be useful in
diagnosing cavernous malformations and may obviate the need
for angiography in selected patients.
3. Compared with management of ischemic stroke, hypertension in
ICH should be treated more aggressively (without inducing
hypotension), in order to reduce ongoing bleeding from small
vessels.
4. Clinical deterioration is the most important indication for surgical
intervention. Patients with a moderate or large lobar hemorrhage
who are clinically deteriorating should be considered for
craniotomy. Surgical management should also be considered for
patients with relatively small cerebellar hemorrhages (~3 cm)
who are clinically deteriorating or have brainstem compression
and/or obstructive hydrocephalus. Nonsurgical management is
appropriate for patients with small hemorrhages, mild neurologic
deficits, very poor prognosis, or Glasgow Coma Scale score 4.
A cerebellar hemorrhage is more likely to require surgical
intervention (ie, craniotomy), because there is less room in the
posterior fossa for expansion of blood and brain edema.
Brainstem compression could occur with relatively small
hemorrhages.

RATIONALE (19) Answer: C

This patient most likely has Guillain-Barré syndrome (GBS) as


manifested by an ascending motor paralysis with loss of deep
tendon reflexes. GBS is a heterogenous condition with multiple
variants including immune-mediated, demyelinating, and axonal
polyneuropathies. About two-thirds of patients have a history of
respiratory tract or gastrointestinal infection, particularly
Campylobacter jejuni, but organisms including cytomegalovirus,
Epstein-Barr virus, Haemophilus, Mycoplasma, enterovirus,
hepatitis A or B, herpesvirus, or Chlamydia have also been
implicated. The most common variant is the acute inflammatory
demyelinating polyneuropathy in 80%–95% of the cases,
followed by acute motor axonal neuropathy, particularly in Asia;
acute motor-sensory axonal neuropathy; and the Miller Fisher
syndrome, characterized by areflexia, muscle weakness, and
ophthalmoparesis or ataxia, among others.

Diagnosis is based on the clinical presentation and supported by


cerebrospinal fluid examination, which shows a typical cyto-
protein dissociation. Additionally, electromyography and nerve
conduction studies provide some support but are usually normal
in early stages. However, absent F waves in the lower extremities
are a supportive finding. The treatment of choice is either
plasmapheresis or IV immunoglobulin. The combination of the 2
treatments has not been found to improve outcome more than a
single intervention and exposes the patient to increased risks.
Greater adverse effects are associated with plasmapheresis than
with IV immunoglobulin. Corticosteroids do not affect the
neurologic outcome in GBS.

RATIONALE (20) Answer: D

This patient has a neurological emergency. The “reddest” sign in


emergency neurology is a third nerve palsy with external
opthalmoplegia (non–pupillary sparing third nerve palsy), which
implies the mechanical disruption of parasympathetic fibers from
the Edinger-Westphal nucleus by an aneurysm of the posterior
communicating artery. The best option for this patient would be
MR angiography of the brain. This would most likely identify the
aneurysm. Electroencephalography is not indicated as this
presentation does not suggest an electrical problem. This patient
has diabetes and is recovering from diabetic ketoacidosis.
However, blood glucose abnormalities (high or low) will not
explain the clinical findings in this patient. Pyridostigmine testing
is unlikely to provide diagnostic yield as symptoms suggest a
vascular syndrome rather than a subacute or chronic syndrome
usually associated with myasthenia gravis. Finally, noncontrast
CT would only be helpful if there has been a subarachnoid
hemorrhage. The headache in this patient may indicate a sentinel
hemorrhage from the aneurysm. However, even if this were the
case, the CT may not be sensitive enough to identify the bleed,
and in the case of an unruptured aneurysm it would not identify
the vascular malformation causing these symptoms.

RATIONALE (21) Answer: A

Both prehospital and in-hospital hypotension with systolic BP


less than 90 mm Hg in adults have been associated with a
doubling of mortality in severe brain injury. The goal of fluid
resuscitation in the prehospital and hospital settings is to
optimize cardiac output, cerebral blood flow, brain tissue
perfusion (cerebral perfusion pressure is equal to mean arterial
pressure minus intracranial pressure), and prevent secondary
brain injury. No randomized, prospective, controlled trial could
be designed to test the hypothesis that optimizing blood pressure
after traumatic brain injury (TBI) improves outcomes on the
basis of ethical principles. The landmark study by Cooper DJ et
al2 demonstrated that a regimen of hypertonic saline was not
superior to conventional crystalloid therapy for resuscitation
after TBI in hypotensive patients with Glasgow Coma Scale scores
3-8. Furthermore, decompressive hemicraniectomy,
hyperosmolar therapy, induced hypothermia, and protocols of
intracranial pressure monitoring have not been associated with
improvements in mortality after TBI.

RATIONALE (22) Answer: E

After severe traumatic brain injury, corticosteroids are


contraindicated. The recommendation is based on the results of a
large multicenter study (CRASH) that showed this intervention to
be harmful with higher mortality, hyperglycemia, and infection
rate in the treatment group. The patient will benefit from
sedation, analgesia and head elevation at 30 degrees for ICP
control; in this patient, indications for ICP monitoring include the
presence of an abnormal CT scan, mass effect, and effacement of
the cisterns. Recognition of and correction of coagulopathy with
FFP, platelets, Vitamin K, and/or desmopressin acetate is critical
to decrease risk of contusion expansion.

RATIONALE (23) Answer: B

The intracranial pressure (ICP) waveform is a modified arterial


pressure tracing with 3 main components (P1, P2, and P3). In a
normal ICP waveform, P1 should have the highest upstroke, P2
should be in between, and P3 should have the lowest upstroke (as
in waveform A above). The first peak, or P1, is known as the
vascular, or “percussive,” wave and results from arterial pressure
transmitted from the choroid plexus. The second peak, or P2, is
known as the “tidal” wave and its amplitude varies with brain
compliance. A lower brain compliance (cerebral edema,
hydrocephalus, etc) increases P2. The third peak, or P3,
represents the dicrotic notch and is caused by the pressure
transmitted during diastole after aortic valve closure.

RATIONALE (24) Answer: E

No intervention has been proven to prevent long-term epilepsy


after severe traumatic brain injury (TBI). The best preventive
strategy has been the reduction of the incidence of TBI in all of its
forms (primary prevention). Early seizures may be prevented
with early use of antiepileptics with little risk of medication side
effects for up to 1 week after severe TBI. Prevention of early
seizure activity after TBI may be beneficial because it may
prevent spikes in intracranial pressure.

RATIONALE (25) Answer: C

This patient is most likely in pain. Adequate pain control and


sedation are fundamental steps in the management of
traumatized patients. Pain, fever, and increased central venous
pressure have been associated with subtle but significant changes
of the intracranial pressure. Hyperosmolar therapy with 20%
mannitol solution and 23.4 % hypertonic saline solution should
be used in cases of imminent transtentorial herniation, but not
routinely. Thus, options A and B are incorrect. Hyperventilation
to Paco2 between 28 and 35 mm Hg may be used in selected
cases when transtentorial herniation is evident and the patient is
receiving mechanical ventilation. Mild induced hypothermia
(option E) should be considered as a rescue therapy for
intracranial pressure when other alternatives have failed.

RATIONALE (26) Answer: E


Hyperventilation has an immediate and profound lowering effect
on intracranial pressure (ICP). This effect, however, is not
sustained, lasting between 30 minutes and 6 hours. Further, very
aggressive hyperventilation to Paco2 less than 25 mm Hg may
induce cerebral ischemia. In the management of increased ICP,
hyperventilation should be used in the setting of impending
herniation for short durations only; it should serve only as a
bridge to more definitive management.

RATIONALE (27) Answer: B

Neurosurgical consultation is an important step in the


management of severe traumatic brain injury (TBI). An epidural
hematoma requires immediate diagnosis; high index of suspicion;
and emergent referral for craniotomy, evacuation, and
hemostasis. The typical clinical scenario is that of a patient with a
history of TBI who has a transient loss of consciousness, recovers
to baseline, and then becomes comatose. Burr holes (option A)
are indicated for the management of acute subdural hemorrhage
and would not be indicated in this case. An external ventricular
drain (option C) may be required after craniotomy for ICP
monitoring but may not be indicated as a first-line therapy
because of the high risk of subsequent herniation. Corticosteroids
(option D) are contraindicated in severe TBI, and though
sometimes required for management of ICP and cerebral edema,
the benefit of hypertonic saline in transtentorial herniation has
been demonstrated with rapid changes in serum sodium
concentration of more than 5 mEq/L or a sodium goal of more
than 145 mEq/L without any risk of central pontine myelinolysis.
A drip of 30 mL/h of 3% saline solution will probably not
increase the serum sodium level acutely. Therefore, option E is
incorrect.

RATIONALE (28) Answer: D

The patient has suffered a type III fracture of C2 with possible


transection of the spinal cord at the C1–C2 level. The de-
efferentation of sympathetic pathways will cause a neurogenic
shock syndrome evidenced by hypotension, bradycardia, and
organ perfusion failure. Spinal shock refers to the syndrome
characterized by absence of spinal cord function in the setting of
spinal cord injury (hyporeflexia, sphincter incontinence, and
neurogenic shock). In this case, a vasopressor with strong beta1-
adrenergic activity is preferred, such as dopamine. A vasopressor
with only alpha-adrenergic activity, such as phenylephrine
(option A) may aggravate the patient’s bradycardia by unopposed
baroreflex response. Blood transfusion (option B) may not be
indicated in the absence of hypovolemia, and though she may
require corticosteroids and surgical decompression, they would
not be associated with rapid improvement of her hemodynamics.
Thus, options C and E are incorrect.

RATIONALE (29) Answer: A

Posttraumatic seizure is common following traumatic brain


injury (TBI). Risk of seizure within the first 7 days after TBI can
be effectively reduced with prophylactic anticonvulsant
medications. Incidence of late posttraumatic seizure following
TBI is not reduced with the use of prophylactic anticonvulsants.
Further, long-term anticonvulsant use has been associated with
worsened cognitive outcomes in brain-injured patients. Use of
prophylactic anticonvulsants for periods longer than 7 days
following TBI is not recommended, regardless of the specific
agent used.

RATIONALE (30) Answer: B

The cerebral scintigraphy with technetium Tc-99m-


hexametazime (HMPAO) shows absent blood flow into the cranial
vault. The test has confirmed the clinical suspicion of brain death.
No further actions are required and the patient can be
pronounced dead. Confirmatory testing is usually necessary when
equivocal findings in the neurological exam confound the
interpretation of absence of neurological function.

RATIONALE (31) Answer: A

For some patients with diagnoses suggesting brain death,


complex, non–brain-mediated, spontaneous movements can
falsely suggest retained brain function. Additionally, ventilator
autocycling may falsely suggest patient-initiated breathing. The
patient cannot be declared brain dead until apnea is confirmed. In
this case, the team should proceed with an apnea test that will
suffice as confirmation of absence of brain function.

RATIONALE (32) Answer: B

The American Academy of Neurology (AAN) published a 1995


practice parameter to delineate the medical standards for the
determination of brain death. The guideline was reviewed in
2010. The parameter emphasized the 3 clinical findings necessary
to confirm irreversible cessation of all functions of the entire
brain, including the brainstem: coma (with a known cause),
absence of brainstem reflexes, and apnea. A classic apnea test is
necessary for the confirmation of brain death and cannot be
easily performed in patients receiving extracorporeal membrane
oxygenation (ECMO). A modified apnea test can be performed in
ECMO patients by mixing the oxygen mixture with carbon dioxide
to achieve a Paco2 >20 mm Hg greater than the baseline. If no
respirations are seen, the patient can then be pronounced brain
dead. An MRI (option D) will add little to the diagnosis of brain
death, but may help to understand the etiology of brain injury.

RATIONALE (33) Answer: C

The presence of hypothermia precludes the diagnosis of brain


death; a core temperature greater than 36°C (96.7°F) should be
achieved before diagnosis of brain death. Hypercapnia does not
preclude the diagnosis of brain death if it is not accompanied by a
severe metabolic disarray. However, for the apnea test,
normocapnia (Paco2 of 35–45 mm Hg) should be established
first.

RATIONALE (34) Answer: A

Coma and other states of impaired consciousness are signs of


extensive dysfunction or injury involving the brainstem,
diencephalon, or cerebral cortex and are associated with a
substantial risk of death and disability. Neurologic prognosis is
determined by the underlying etiology and may be predicted by
the combination of clinical signs and electrophysiological tests.
The persistence of sleep-awake cycles sets patients in a persistent
vegetative state (PVS) apart from patients in a coma, but patients
in PVS do not have sensation or a state of awareness. The
minimally conscious state is characterized by a partial state of
awareness and intermittently purposeful motor activity. Absent
somatosensory evoked potential (SSEP) responses are not usually
seen in patients with PVS and absence of N20 response favors a
severe disruption of ascending spinocortical circuits such as in
cardiac arrest.
PART 2: CARDIOVASCULAR CRITICAL CARE
Instructions: For each question, select the most correct answer.

1. A 28-year-old, lethargic woman is admitted to the ICU with the


following data: temperature, 38.8°C (101.8°F); HR, 128/min;
RR, 26/min; BP, 68/36 mm Hg; mean arterial pressure, 46 mm
Hg; Spo2, 96% on oxygen, 2 L/min; cardiac output, 9 L/min;
central venous pressure, 3 mm Hg; systemic vascular
resistance, 382 dyne·s/cm5; pulmonary artery pressure, 15/8
mm Hg; and mean pulmonary artery pressure, 10 mm Hg. The
hospital is currently experiencing a shortage of several
vasoactive medications, limiting options for therapy.

The administration of which of the following agents is the most


appropriate initial step in the management of this patient?

A. Phenylephrine
B. Milrinone
C. Dobutamine
D. Vasopressin

2. A 19-year-old male was intubated in the field following a


motorcycle crash and transported to a nearby trauma center.
He has been lethargic since the accident but has maintained
normal vital signs with minimal intervention throughout the 6-
hour stay in the emergency department and ICU. Painful
stimulus elicits minimal movement of bilateral upper
extremities and no movement in the lower extremities. On
arrival in the ICU, vital signs are as follows: temperature,
35.5°C (95.8°F); HR, 67/min; BP, 108/62 mm Hg; Spo2, 97% on
volume-control ventilation; tidal volume, 480 mL; RR, 14/min;
positive end-expiratory pressure, 5 cm H2O; and FIO2, 0.5.
About 1 hour later, the registered nurse calls for help as the
patient now has a temperature of 34.9° (94.8°F); HR of 22/min;
BP of 58/20 mm Hg, and Spo2 of 97%.

Which of the following is the most likely cause of this patient’s


clinical deterioration?

A. A large tear in the inferior vena cava


B. Blunt injury to the heart
C. Complete transection of spinal cord at the C6 level
D. Sepsis from aspiration pneumonia

3. An otherwise healthy 17-year-old boy who tripped and fell into


a bonfire arrives at your trauma center after transfer from a small
community hospital. On arrival in the ICU, physical examination
reveals temperature of 35°C (94.9°F); HR of 118/min; BP of
94/50 mm Hg on phenylephrine, 250 µg/min; and Spo2 of 90%
on 60% oxygen via face mask. Neurologic examination reveals a
somnolent patient able to follow simple commands and to
withdraw from painful stimulus in all 4 extremities. He has
regular tachycardia, with no murmurs. Lung examination shows
coarse rhonchi in all fields. He has a partial- and full-thickness
burn involving the entire abdomen and chest (anterior and
posterior), both upper extremities, and the entire right lower
extremity. Spared skin includes the entire left lower extremity
and the head and neck. The physician who initiated the transfer
gave the following information: The patient’s burn occurred 6
hours ago. He has two 18-gauge IVs and has received 800 mL of
0.9% normal saline. More fluid was initially ordered, but the
oxygen saturation was progressively worsening, so the team
decided to use phenylephrine for blood pressure support in lieu
of crystalloid administration.

Which of the following is the best immediate intervention?


A. Placement of a central venous line for monitoring of central
venous pressure and administration of fluid and drugs.
B. Placement of an arterial line for beat-to-beat measurement of
blood pressure and serial blood gas sampling.
C. Endotracheal intubation, mechanical ventilation, and
placement of a pulmonary artery catheter to guide careful
administration of IV fluids.
D. Endotracheal intubation, mechanical ventilation and rapid IV
administration of multiple liters of warmed isotonic crystalloid
solution

4. An intubated and heavily sedated patient with heart failure


arrives from the catheterization lab to the ICU after left-sided
coronary angiography (without intervention) and placement of a
central venous catheter in the right internal jugular vein. Vital
signs are as follows: Temperature, 36.7°C (98°F); HR, 74/min in
sinus rhythm; BP, 110/62 mm Hg on dobutamine infusion, 7
µg/kg/min; Spo2, 97% on volume control ventilation; tidal
volume, 480 mL; frequency, 14/min; positive end-expiratory
pressure, 5 cm H2O; FIO2, 50%. The vital signs 8 minutes after
arrival in the ICU are as follows: Temperature, 36.7°C (98°F); HR,
124/min in sinus rhythm; BP, 72/30 mm Hg on dobutamine
infusion, 7 µg/kg/min; and Spo2, 82% on unchanged ventilator
settings. The ventilator alarm indicates high airway pressure.

Which of the following is the most important initial step in the


management of this patient?

A. Increase the dobutamine infusion.


B. Add epinephrine infusion.
C. Perform emergent needle decompression of the chest.
D. Obtain immediate chest radiography.
5. An otherwise healthy 27-year-old woman is walking out of the
hospital cafeteria when she falls to the ground. She is
somnolent and unresponsive to command, with noisy
breathing and a barely palpable radial pulse. She is rapidly
transported to the emergency department. Upon arrival, her
temperature is 37.2°C (98.9°F), HR is 132/min, BP is 58/34
mm Hg, RR is 28/min, and Spo2 is 92% on 80% FIO2 via
nonrebreather mask. Physical examination reveals an
unconscious woman with flushed skin in respiratory distress.
Lung examination reveals bilateral wheezing.
Which of the following is the most appropriate initial
intervention?

A. Administration of nebulized albuterol


B. IV administration of normal saline, 2 L
C. IV administration of hydrocortisone
D. IV (or intramuscular) administration of epinephrine

6. Which of the following statements best characterizes the


appropriate method of measuring central venous pressure
(CVP)?

A. CVP should be measured from the base of the v wave.


B. CVP should be measured at end-expiration if mechanically
ventilated and end-inspiration if spontaneously breathing.
C. CVP can be estimated by adding 5 cm H2O to the measured
distension of the jugular veins relative to the sternal angle.
D. If the midpoint of the right atrium is used as reference level
for CVP monitoring, measurements are only reliable when the
patient is supine.

7. Which of the following factors alters the interpretation of the


pulmonary artery occlusion pressure as an indicator of left
ventricular end-diastolic pressure (preload)?
A. Increased pulmonary vascular resistance
B. Change in pulmonary artery catheter position from West
zone 1 to West zone 2
C. Mitral insufficiency
D. Changes in right ventricular compliance

Which of the following statements best represents current


recommendations and the management of the rhythm shown in
the Figure?

A. 2010 American Heart Association Advanced Cardiac Life


Support Guidelines recommend adenosine for initial
management of this arrhythmia.

B. Three stacked shocks should be administered if a


monophasic defibrillator is used to treat this rhythm.

C. Fibrinolytic therapy during CPR for patients presenting with


this rhythm and suspected/ confirmed coronary occlusion has
not been shown to improve outcome.

D. Amiodarone has been shown to be superior to lidocaine for


increasing the rate of survival to hospital discharge in patients
presenting this rhythm.

9. A 60-year-old man is admitted to the ICU after cardiac


surgery. Findings of physical examination and chest
radiography are consistent with pulmonary edema. A
vasoactive drug is administered.
Which of the following drugs was administered?

A. Nitroglycerin
B. Milrinone
C. Dopamine
D. Norepinephrine

10. A 54-year-old man is admitted to the emergency department


with severe substernal chest pain and acute shortness of
breath. The patient has a history of type 2 diabetes mellitus
and hypertension controlled with medications. Examination
reveals diaphoresis and respiratory distress. HR is 120/min,
BP is 95/50 mm Hg, and RR is 24/ min. Lung examination
reveals bilateral crackles; heart examination is significant for
regular tachycardia, with normal S1 and S2. An S3 is present
without murmurs or rubs. Chest radiography shows changes
consistent with pulmonary edema. Twelve-lead ECG is
significant for T-wave inversions in V1–V5. Patient is taken
for emergent cardiac catheterization. The following
hemodynamic values are obtained:

Oxygen consumption is calculated at 180 mL/min/m2.


Significant triple vessel disease with a lesion in the proximal
left anterior descending coronary artery is found. An
intraaortic balloon pump is placed in preparation for an
emergent coronary artery bypass graft.

Which of the following hemodynamic parameters would be


most consistent with an effective intraaortic balloon
counterpulsation?

11. A 50-year-old man with a past medical history significant for


hypertension presents to the emergency department with
severe substernal chest pain and shortness of breath that
started 30 minutes ago while eating at a nearby restaurant.
On physical examination, the patient is diaphoretic. BP is
90/50 mm Hg, HR is 75/min, and RR is 28/min. Physical
examination reveals irregular tachycardia with no murmurs
and a positive S3. Lung auscultation reveals crackles in both
bases. The rest of his examination findings are
unremarkable. An ECG is shown in the Figure.
Which of the following management options is most appropriate
at this point?
A. Aspirin, IV nitroglycerin, low-molecular-weight heparin, and
beta-blockers
B. Aspirin, low-molecular-weight heparin, beta-blockers, and
glycoprotein IIb/IIIa inhibition
C. Aspirin and urgent catheterization
D. Aspirin, beta-blockers, and rtPA

12. A 55-year-old man presents to the emergency department


with hypoxemia, hypotension, and no past medical history of
cardiopulmonary disease. He has type 2 diabetes mellitus
and chronic renal insufficiency with a creatinine level of 2.2
mg/dL. Chest radiograph shows no infiltrates, and his lung
examination findings are normal. Blood pressure initially
normalizes with a fluid bolus, but then decreases, requiring
dopamine, 10 µg/kg/min, to maintain a systolic BP of 90–
100 mm Hg. Oxygenation is good with oxygen, 5 L via nasal
cannula. An echocardiogram reveals a dilated right atrium
and right ventricle and no other abnormalities. A catheter
inserted in the right side of the neck reveals a central venous
pressure of 12 mm Hg and an Scvo2 of 60%. CT and remote
interpretation are immediately available. Leg
ultrasonography, perfusion lung scanning, and angiography
are only available by call-in.

Which of the following is the most appropriate management


of this patient’s illness?

A. Dobutamine and thrombolytic therapy


B. Fluid bolus and thrombolytic therapy
C. Dobutamine and helical chest CT
D. Fluid bolus and helical chest CT

13. A 56-year-old woman being treated for unresectable lung


cancer is admitted to the ICU with hypotension that is poorly
responsive to fluid resuscitation, leading to initiation of
vasopressors (dopamine, 10 µg/kg/min) to maintain
adequate blood pressure. A pulmonary artery catheter
reveals equalization of pressures (right atrial, right
ventricular diastolic, pulmonary artery occlusive pressure).
Emergent echocardiogram reveals right atrial and right
ventricular collapse with systole. Cardiac index is 1.0
L/min/m2. A 2.5-cm circumferential pericardial effusion is
present. Emergent pericardial centesis removes
approximately 60 mL of fluid. Blood pressure rises
significantly, and the patient is weaned from dopamine.
Cardiac index increases. The patient is transferred to the
cardiac catheterization laboratory. Mean right atrial pressure
is 31 mm Hg and intrapericardial pressure is 31 mm Hg. Two
hundred milliliters of sanguinous fluid is removed from the
pericardial space, and a pigtail catheter is left for drainage.
Mean right atrial pressure is reduced to 23 mm Hg, while the
intrapericardial pressure decreases to 8 mm Hg. ECG
confirms that most of the fluid has been removed. Patient is
now slightly hypertensive and not receiving vasoactive
drugs.

In addition to hemodynamically significant pericardial


effusion, which of the following conditions is likely to be
present?

A. Primary pulmonary artery hypertension


B. Secondary pulmonary artery hypertension
C. Right ventricular infarction
D. Effusive-constrictive pericarditis

14. A patient with a known hypertrophic cardiomyopathy and


dynamic left ventricular outflow tract obstruction is
intubated for community-acquired pneumonia. Urine output
is minimal, with no response to a fluid bolus. Patient has BP
of 118/74 mm Hg and HR of 120/min.

Which of the following therapies is most appropriate as part


of the treatment?

A. Furosemide
B. Inotropic-dose dopamine
C. Beta blockade
D. Nitroglycerin

15. A 70-year-old man is admitted to the ICU after insertion


of an intraaortic balloon pump (IABP) for severe, intractable
angina while awaiting coronary artery bypass surgery. His
IABP is set at 1:1 and he is treated with heparin and
nitroglycerin infusions. During the course of the evening,
although his chest pain has lessened, he now has abdominal
pain of increasing severity.

In addition to checking the ECG for new changes, which of


the following is the most appropriate next step in his care?

A. Increase the nitroglycerin infusion dose.


B. Obtain a chest radiograph.
C. Reduce the IABP support from 1:1 to 1:2.
D. Check his stool for occult blood.

15. A 65-year-old woman with ischemic cardiomyopathy is


awaiting implantable cardioverterdefibrillator placement for
nonsustained ventricular tachycardia. An episode of stable
ventricular tachycardia quickly regresses to ventricular
fibrillation. The initial end-tidal carbon dioxide partial
pressure (PETCO2) is 10 mm Hg during the initial chest
compressions. After epinephrine, 1 mg, and defibrillation, the
Petco2 increases to 35 mm Hg.

Which of the following best represents the significance of the


PETCO2 readings for this patient?

A. The increase in PETCO2 indicates a return of spontaneous


circulation.

B. Increased PETCO2 indicates chest compressions can be


stopped.

C. Initial low PETCO2 values indicate inadequate chest


compressions.

D. Low PETCO2 values should be confirmed with blood gas


analysis.
17. Which of the following is a relative contraindication to
intraaortic balloon pump therapy?
A. Aortic dissection

B. Severe peripheral vascular disease

C. Aortic valve insufficiency

D. Patent ductus arteriosus

18. A 40-year-old woman supported with a left ventricular


assist device (LVAD) for nonischemic cardiomyopathy was
admitted to the ICU with a trace guaiac-positive lower
gastrointestinal bleed. She was hemodynamically stable until
she received sedation for a colonoscopy, when she became
hypotensive. Chest radiograph is shown in the Figure.
Which of the following is the most appropriate immediate
intervention?

A. Increase the blood pressure with administration of a pressor.

B. Increase the LVAD pump speed to increase the cardiac output.

C. Decrease the LVAD pump speed to increase the filling time of


the left ventricle.

C. Perform transfusion to maintain hemoglobin level of 8 g/dL


or until the systolic blood pressure is normal.

19. Following an uneventful mitral valve repair for severe mitral


regurgitation secondary to mitral valve prolapse, a 55-year-
old woman is admitted to the ICU. She is AAI paced via
transthoracic wires at 80/min. The PR interval is 0.29
seconds and the underlying rhythm is sinus bradycardia at
40/min.

The pacemaker should be changed to:

A. DDD pacing, with PR interval of 0.2 seconds

B. VOO pacing at 90/min

C. AOO pacing at 90/min

D. DDD pacing, with PR interval of 0.29 seconds

20. A 60-year-old woman is admitted to the ICU following a 3-


vessel aortocoronary bypass graft. She is paced at 96/min via
transthoracic epicardial wires. Without pacing or inotropic
support, she was in normal sinus rhythm at 80/min with a
cardiac index of 3.2.

Which of the following is the most appropriate rationale for


pacing?

A. Prophylaxis for heart block

B. Further increasing cardiac index for optimum myocardial


perfusion

C. Prevention of atrial fibrillation

E. Initial support of cardiac function while sedated and


mechanically ventilated

21. During a transfusion of fresh frozen plasma to normalize the


international normalized ratio of a 60-year-old man treated
with warfarin prior to repair of a hip fracture, he has “flash”
pulmonary edema and is admitted to the ICU.

Which of the following statements should guide management


of his cardiac dysfunction?

A. The diagnosis is left ventricular failure.

B. The diagnosis is biventricular failure.

C. The type of failure cannot be diagnosed based on his


symptoms.

D. The treatment goal of systolic failure is reduction of


symptoms.
22. Which of the following patients best meets criteria for
severe sepsis?

A. 59-year-old man hospitalized for 2 days following


laparoscopic cholecystectomy for acute cholecystitis who
develops fever to 38.5°C (101.2°F), WBC count of 14,000/µL,
and a urine culture positive for Escherichia coli

B. 41-year-old woman with diabetes and perineal pain, fever


to 38.9°C (102°F), and a blood glucose level above 400 mg/dL

C. 70-year-old man admitted with pneumonia with HR of 108


/min, BP of 105/70 mm Hg, RR of 22/min, positive sputum
culture for Klebsiella pneumoniae, and urine output of 10
mL/h after normal saline, 3 L

E. 45-year-old man with gram-positive bacteremia and


persistent hypotension despite 5 L of lactated Ringer solution
and high-dose norepinephrine infusion

23. 73-year-old woman presents to the emergency department


with increasing left lower quadrant abdominal pain. She has
fever to 38.8°C (101.8°F), irregular HR of about120/min, BP of
75/35 mm Hg, and RR of 24/min. On examination, she is
obtunded, and has significant pain with guarding on palpation
of her left lower quadrant. Laboratory values include WBC
count of 17,000/µL; hemoglobin level of 13.1 g/dL; creatinine
level of 1.7 mg/dL; glucose level of 255 mg/dL; lactate level of
5.6 mmol/L; and arterial blood gas results showing pH of 7.22,
Paco2 of 30 mm Hg, and Pao2 of 117 mm Hg. CT shows a large,
rim-enhancing fluid collection adjacent to her sigmoid colon,
and colonic thickening consistent with acute diverticulitis. IV
access is obtained, and lactated Ringer solution, 4 L, is
administered. BP rises to 82/48 mm Hg, and the patient is
started on norepinephrine infusion. Broad-spectrum
antibiotics are administered. Blood cultures are positive for
Enteroccocus. An interventional radiology– placed drain
reveals a significant amount of purulent material.

Which of the following data points is significant in making the


diagnosis of septic shock?

A. Fever to 38.8°C (101.8° F) and irregular HR of about


120/min

B. Positive blood cultures and purulent material from drain

C. Significant metabolic acidosis and signs of systemic


hypoperfusion

E. BP of 82/48 mm Hg despite adequate fluid resuscitation

24. A 23-year-old man is brought to the emergency department in a


cervical collar after a head-on motor vehicle collision. He was
unrestrained and used cocaine within the previous hour. He has
tachycardia and diaphoresis and reports abdominal and back
pain. His initial BP is 80/40 mm Hg, with HR of 100/min. A
central line is placed, and a 500-mL bolus of normal saline
solution brings his BP to 160/80 mm Hg and his HR to 80/min.
However, within 15 minutes, his BP falls to 60/30 mm Hg with a
central venous pressure of 9 mm Hg, his HR accelerates to
120/min, and he becomes unresponsive.

What is the most likely explanation for his fall in blood


pressure?

A. Splenic laceration
B. Myocardial infarction

C. Tension pneumothorax

E. Resolution of cocaine-induced hypertension

25. A 58-year-old migrant farm worker arrives at a rural hospital in


California, having suffered a traumatic amputation of his left arm
from a farm machinery accident. Although a tourniquet is in
place high in the axilla, it is uncertain how long ago the injury
occurred or how much blood has been lost. Despite some slow
oozing from his amputation site, ongoing blood loss now seems
to be controlled. HR is 90/min, BP is 110/90 mm Hg,
respirations are 22/min and shallow, and oral temperature is
34°C (93.2°). Although anxious, the patient seems coherent and
answers questions appropriately. Two peripheral IV catheters
(16- and 18-gauge) are present in theuninjured arm. Baseline
laboratory results include hemoglobin level of 8 mg/dL; platelet
count of 158,000/µL; arterial blood gas pH of 7.25, Paco2 of 45
mm Hg, and Pao2 of 75 mm Hg; and lactate level of 5 mg/dL. The
operating room will not be available for at least 45 minutes.

What are the most appropriate interventions that can be


accomplished in the time prior to the operating room becoming
available?

A. Type and cross for 4 units of packed red blood cells (PRBCs);
give 4 units of fresh frozen plasma (FFP), 6 units of platelets, and
sodium bicarbonate;warm patient to 35°C (94.9°F); place central
line.

B. Perform rapid-sequence intubation; administer sedation;


place nasogastric tube and suction gastric contents; place central
line; infuse 1,000 mL of hetastarch solution.
C. Type and cross for 6 units of PRBCs; give 6 units of FFP and 6
units of platelets; place urinary catheter; warm patient to 36°C
(96.7°F); infuse normal saline solution, 2 L.

D. Perform rapid sequence intubation; administer sedation,


sodium bicarbonate, and 2 units of type O non–cross-matched
PRBCs.

Questions 26 and 27 relate to the following vignette.

A 35-year-old, 70-kg (154-lb) police officer is brought to the


emergency department after falling from a motorcycle during a
high-speed chase. His uniform was removed in the field, and it is
evident that he has suffered significant abrasions. He is in a
cervical collar and his helmet has been removed. Large
contusions are present over the left hemithorax and right
abdomen, and his left femur has been placed in a traction splint.
Two 18-gauge peripheral IV lines have been inserted in bilateral
antecubital fossae, and he has received normal saline solution,
1.5 L, en route to the hospital. He is somnolent, and when roused,
he is confused. HR is 120/min, BP is 97/74 mm Hg, RR is 32/min,
and Spo2 is 93% on nonrebreather mask.Paramedics estimate
that he has lost at least 1 L of blood. His left hemithorax is dull to
percussion.

26. By Advanced Trauma Life Support (ATLS) standards, what


class of hemorrhage has this patient sustained?

A. Class I

B. Class II

C. Class III
D. Class IV

27. Which of the following interventions is the most appropriate


next step in this patient’s care?

A. Left chest tube placement

B. Right subclavian central line placement

C. Radiographs of the chest (anterior/posterior) and left femur

E. Exploratory laparotomy

Questions 28 and 29 relate to the following vignette.

A 63-year-old man with type 2 diabetes mellitus is brought to the


ICU after emergent decompression of a right temporal subdural
hematoma sustained after a fall of approximately 15 feet. He is
intubated and ventilated with FIO2 of 60%, synchronized
intermittent mandatory ventilation at 12/min, tidal volume of
500 mL, positive end-expiratory pressure (PEEP) of 5 cm H2O,
and Spo2 of 100%. He is estimated to have lost approximately 1 L
of blood from a scalp laceration preoperatively, and lost at least
another 1 L intraoperatively. He was given 8 units of packed red
blood cells (PRBCs), 6 units of fresh frozen plasma, and 6 units of
platelets intraoperatively. He remains hypotensive (80/40 mm
Hg) while being supported with dopamine, 10 µg/kg/min. HR is
100/min and central venous pressure is 18 mm Hg. The
anesthesiologist found sufficient left-sided filling volumes using
transesophageal echocardiography (TEE) toward the end of the
operation.

28. Which of the following TEE views would have best informed
the attending anesthesiologist of the patient’s volume status?
A. Deep transgastric long-axis view

B. Transgastric basal short-axis view

C. Mid-esophageal bicaval view

D. Transgastric midpapillary short-axis view

29. Which of the following would be the most appropriate initial


intervention to improve this patient’s hemodynamic status?

A. Discontinue PEEP.

B. Increase dopamine to 20 µg/kg/min.

C. Give PRBCs, 2 units, and normal saline, 1 L.

D. Give calcium chloride, 2 g.

30. The hemodynamic profile shown in the table is from a 62-


year-old man admitted to the ICU after coronary artery bypass
grafting:

Which of the following is the most likely cause of the changes


occurring after 30 minutes?
A. Anaphylactic reaction

B. Left ventricular ischemia

C. Pericardial tamponade

D. Pulmonary embolism

31. During evaluation of oliguria in a 62-year-old male following


operative repair of an aortic aneurysm, large V waves are noted
in a pulmonary artery occlusion pressure tracing.

Which of the following conditions is most likely associated with


the findings in this patient?

A. Pulmonary regurgitation

B. Mitral regurgitation

C. Aortic regurgitation

D. Coronary artery disease

32. In which of the following conditions would increasing


preload, systemic vascular resistance, and heart rate result in
improved hemodynamic stability?

A. Aortic stenosis

B. Asymmetric septal hypertrophy(idiopathic hypertrophic


subaortic stenosis)

C. Cardiac tamponade
D. Mitral regurgitation

F. Mitral stenosis

33.A 45-year-old woman with mitral valve stenosis is scheduled


for elective mitral valve replacement. Two minutes after
tracheal intubation, she develops atrial fibrillation with a
ventricular response rate of 150/min and a decrease in BP to
75/45 mmHg. Which of the following is the most appropriate
first step in management?

A. Increasing the concentration of anesthetic gas

B. Increasing the infusion rate of lactated Ringer solution

C. IV administration of digoxin

D. IV administration of propranolol

E. Electrical cardioversion

34.A 52-year-old woman with severe mitral stenosis is scheduled


for mitral valve replacement.

Which of the following preoperative findings is most likely to


be present?

A. Atrial fibrillation

B. Left ventricular hypertrophy

C. Elevated left ventricular end-diastolic pressure

D. Symptomatic improvement with systemic vasodilation


35.A 68-year-old man with stable angina treated with
propranolol and nifedipine is scheduled for coronary artery
bypass grafting. After induction with fentanyl and tracheal
intubation, HR decreases to 40/min and BP decreases to
70/40 mmHg. The ECG is shown in the figure.

Which of the following is the most appropriate first step in


management?

A. Infusion of epinephrine or dopamine

B. Administration of phenylephrine

C. Closed-chest CPR D. Defibrillation

E. Synchronized cardioversion

36. A 73-year-old man is admitted with chest pain radiating to the


interscapular region, HR of 90/min and BP of 190/80 mm Hg.
Initial CT shows a dissecting aneurysm of the descending
aorta and medical management is advised.

Which of the following is the first line of treatment?

A. Nitroprusside infusion

B. Nitroglycerin infusion
C. Labetalol infusion

D. Beta-blockers

37.A 76-year-old man with hypertension, type 2 diabetes


mellitus, hepatic insufficiency secondary to ethanol abuse, and
chronic kidney disease is admitted to the ICU with severe
chest pain radiating to the back. His initial examination is
remarkable for BP of 210/115 mm Hg, creatinine level of 2.7
mg/dL, international normalized ratio of 2.8, and total
bilirubin level of 4.8 mg/dL. Radiologic studies reveal an acute
dissection of the descending thoracic aorta starting 4 cm
below the left subclavian artery and extending into the
infrarenal abdominal aorta with the left artery supplied from
the false lumen.

Which of the following medication approaches is optimal for


the control of his blood pressure?

A. Esmolol followed by nicardipine

B. Esmolol followed by nitroprusside

C. Nitroprusside followed by metoprolol

C. Nitroprusside followed by nicardipine

38.A 70-year-old woman with hypertension and


hypercholesterolemia is admitted to the ICU after a 5-hour
onset of left hemiparesis and BP of 225/105 mm Hg on
presentation. CT shows a loss of gray-white distinction in right
frontal and parietal lobes consistent with ischemic stroke.
Which of the following treatments would provide optimal
management?

A. Systemic thrombolysis

B. Nitroprusside to reduce BP to 190/90 mm Hg

C. Labetalol to reduce BP to 130/75 mm Hg

D. Labetalol to reduce BP to 190/90 mm Hg

39.A 29-year-old presents with severe preeclapmsia (BP of


190/120 mm Hg) at 30 weeks’ gestation. Pertinent laboratory
values are: hemoglobin, 10.1 g/dL; platelets, 87,000/µL;
aspartate aminotransferase, 115 U/L; alanine
aminotransferase, 93 U/L; and alkaline phosphatase, 55 U/L.
A magnesium infusion is started withIV hydration and she is
admitted to the ICU.

Which of the following is the best approach for further


management of this patient?

A. IV hydralazine as needed while systolic BP is greater than


160 mm Hg

B. IV enalapril as needed while systolic BP is greater than 160


mm Hg

C. IV nicardipine infusion to keep systolic BP less than 160 mm


Hg

D. Emergent cesarean section


40. A 72-year-old man has hypertension, ischemic
cardiomyopathy with a left ventricular ejection fraction of
21%, and an implantable cardioverter-defibrillator (ICD) for
treatment of ventricular dysrhythmias. The ICD was
inactivated prior to a mitral valve annuloplasty for severe
mitral regurgitation. Postoperatively, he was supported with
norepinephrine and milrinone. Two hours after admission, he
developed atrial fibrillation with rapid ventricular response
and an HR of 120–130/min, resulting in decreased blood
pressure to 70/41 mm Hg and decreased cardiac index from
2.6 to 1.4 L/min/m2.

Which of the following is the best next step in management of


this patient?

A. Reactivating the ICD

B. Amiodarone infusion after initial bolus

C. External defibrillation

D. External synchronized cardioversion

PART 2: Cardiovascular Critical Care

ANSWERS:

1–A; 2–C; 3–D; 4–C; 5–D; 6–C; 7–A; 8–C; 9–B; 10–C; 11–C; 12–
D; 13–D; 14–C; 15–B; 16–A; 17–B; 18–A; 19–A; 20–C; 21–C;
22–C; 23–D; 24–A; 25–C; 26–C; 27–A; 28–D; 29–D; 30–B; 31–
B; 32–C; 33–E; 34–A; 35–A; 36–D; 37–A; 38–D; 39–C; 40–D

RATIONALE (1) Answer: A


The patient has distributive shock, as evidenced by
hypotension, preserved cardiac output, and profoundly low
systemic vascular resistance. The primary derangement in
this patient appears to be low vascular tone. As a direct-acting
alpha-adrenergic agonist, phenylephrine works to restore
systemic vascular resistance and increase blood pressure.
Milrinone is a phosphodiesterase inhibitor that increases
cardiac contractility and causes vasodilation. The patient’s
data indicate that cardiac contractility is robust and
peripheral vasodilation is the primary problem.
Administration of milrinone in this case would probably be
detrimental. Dobutamine is a synthetic sympathomimetic
with predominantly beta1-adrenergic effects. It typically
causes an increase in cardiac contractility and a small
decrease in systemic vascular resistance. The patient’s data
indicate that these effects would be unlikely to be of benefit.
Vasopressin binds to V1 receptors, causing contraction of
vascular smooth muscle cells, leading to increased systemic
vascular resistance. While the patient does require an agent to
increase vascular tone, vasopressin would be difficult to
rapidly titrate as the patient’s status changes over time. While
administration of vasopressin might be reasonable as an
adjunct to a titratable infusion, vasopressin is inferior to
phenylephrine as the initial step in the management of this
patient.

RATIONALE (2) Answer: C

This patient is suffering from the acute onset of profound


bradycardia and hypotension. While several anatomic and
physiologic insults can produce this combination, in the blunt
trauma patient, neurogenic shock is a common cause of this
picture. Keys to the correct answer in this question include
the history of high-velocity blunt trauma, the neurologic
examination findings, and the timing of cardiovascular
collapse. Neurogenic shock can manifest itself hours after
spinal cord injury. A large tear in the inferior vena cava would
result in immediate and profound hypotension at the scene of
the accident, detectable long before hour 6 of an emergency
department and ICU stay. Blunt trauma to the heart would
probably produce clinical signs within a short time after
injury. Sepsis from aspiration pneumonia, while common in
blunt trauma patients, would not typically produce shock
within the first 24 hours after the accident. Neurogenic shock
is a form of distributive shock and is characterized by the
triad of hypotension, bradycardia, and hypothermia resulting
from functional sympathectomy and/or from loss of cardiac
inotropy/chronotropy secondary to high spinal cord injury.
Functional sympathectomy results in loss of vascular tone and
profound hypotension. It is more common in injuries at the
midthorax and higher. In high cord injuries, loss of cardiac
accelerator function and unopposed parasympathetic tone
contribute to bradycardia, which exacerbates impaired
cardiac output and thus exacerbates shock. Anticholinergics
(eg, atropine) or beta-adrenergic agonists are useful in
increasing the heart rate. Alpha-adrenergic agonists are useful
in restoring peripheral vascular tone, improving venous
return, and thus increasing cardiac output.

RATIONALE (3) Answer: D

Large burn injury causes a combination of hypovolemic and


distributive shock, at times combined with direct myocardial
depression (in the immediate postburn phase). The
predominant component of postburn shock is hypovolemia,
largely the result of microvascular injury leading to pathologic
leaking of fluid from the intravascular compartment. In large
burns (>20% total body surface area [TBSA]), this fluid leak
results in the need for large-volume fluid administration to
avoid organ failure and death. The patient in this vignette is
critically ill and has multiple systems requiring intervention.
The question asks what the “best immediate intervention” is.
Considering the degree and trajectory of hypoxia in this
previously healthy 17-year-old, the mechanism and extent of
injury, and the resuscitation that is about to occur, the
intensivist is obligated to secure the airway and initiate
mechanical ventilator support immediately. Deferring
necessary airway management for the placement of lines
which are not lifesaving (arterial, central venous, pulmonary
arterial) is a common and sometimes fatal error in the care of
severely burned and injured patients. By the “rule of nines,”
this patient’s burn can be estimated at approximately 72% of
TBSA. Regardless of whether the intensivist uses a
conservative formula such as the modified Brooke formula (2
mL fluid per kg per percent TBSA in 24 hours) or the
traditional Parkland formula (4 mL fluid per kg per percent
TBSA in 24 hours), this patient is clearly in need of a large
volume of IV fluids. While the placement of a central venous,
arterial, or pulmonary arterial catheter might help ongoing
decision making with regard to fluid resuscitation, this
patient’s history and physical examination findings are
sufficient to know that he is in need of immediate fluid bolus.
Delaying fluid administration for the placement of invasive
monitors would be inappropriate in this case and might result
in ongoing damage to underperfused tissues. Other important
considerations in this patient include his hypothermia, his
existing (and soon to worsen) lung injury, possible inhalation
injury, and the likely restrictive effects of a whole-torso burn
on respiratory mechanics.

RATIONALE (4) Answer: C


This patient, who has recently undergone placement of a
central venous catheter, is demonstrating several of the
classic signs of tension pneumothorax: rapidly progressive
and profound hypotension, tachycardia, hypoxia, and
inadequate ventilation. While this constellation of findings is
common among ICU patients, the proximity in time to the
central line placement and the low tidal volumes with high
airway pressures necessitate strong consideration of tension
pneumothorax on the differential. The rate of clinical
deterioration requires immediate action to correct the
problem. When tension pneumothorax is being strongly
considered in an ICU patient who is near cardiac arrest,
emergent needle decompression of the affected hemi-thorax
is essential. As a heart failure patient, this patient would
tolerate the stress of tension pneumothorax even more poorly
than most. Increasing the dobutamine infusion would have
little to no effect in this case. The shock state induced by
tension pneumothorax is caused by elevated intrathoracic
pressure leading to poor venous return to the heart, thus
reducing cardiac output. Inotropics such as dobutamine are
not the treatment of choice for any shock state related to
extremely compromised venous return. Epinephrine might
improve the blood pressure temporarily, but until the
underlying problem—inadequate venous return—is solved,
the patient is at risk of progressing toward cardiac arrest.
While many clinicians associate chest radiography with
pneumothorax, the decision to wait for chest radiography
while a tension pneumothorax expands is a potentially fatal
error. This patient’s systolic BP has dropped from 110 to 54
mm Hg and the Spo2 has dropped from 97% to 61% in a
matter of minutes. The time that it takes for a radiograph to
be obtained and processed is excessive. Alternatively,
intensivists’ increased use of point-of-care ultrasonography
for detection of pneumothorax has been advocated as a fast
and accurate alternative to radiography.

RATIONALE (5) Answer: D

This woman’s history and physical examination are most


consistent with anaphylaxis, which is a form of distributive
shock. Anaphylaxis involves the exposure to a trigger in a
susceptible person, which leads to mast cell activation. The
subsequent release of chemical mediators such as histamine
and numerous cytokines results in a complex clinical picture
that can include systemic vasodilation, hypotension,
tachycardia, urticaria, brochospasm, arrhythmias, coronary
vasoconstriction, and myocardial depression.
Prompt administration of epinephrine is an essential therapy
for patients with anaphylactic shock. The mechanisms of
epinephrine beneficial to anaphylaxis patients include alpha1-
agonist effects (restoring vascular tone and blood pressure
and reducing mucosal edema), beta1-agonist effects
(increasing the rate and force of cardiac contractions), beta2-
agonist effects (bronchodilation), and stabilization of mast
cells. While all 4 answers describe potential desirable
measures for a patient suffering from anaphylactic shock, the
question asks for the most appropriate initial intervention. In
addition to these measures, the administration of
antihistamine agents (both H1 and H2 blockers) is advised.
The proximity of the patient to a recent meal implicates
unrecognized ingestion of a food allergen as a likely cause of
anaphylaxis in this case. Accidental ingestion of peanuts or
tree nuts is a common cause of anaphylaxis in the setting of
food ingestion.

RATIONALE (6) Answer: C


Proper positioning of the pressure transducer is critical for
measurement of central venous pressure (CVP) because
seemingly small errors in transducer height amplify errors in
measuring cardiac filling pressure. The commonly accepted
reference point for CVP measurement is the midpoint of the
right atrium, which can be identified on physical examination
at a vertical distance 5 cm below the sternal angle. This is true
whether the patient is supine or sitting at a 60-degree angle.
Therefore, patients do not have to be supine when this
reference level is used, and option D is incorrect. More
commonly in the ICU, the midthoracic position at the level of
the fifth rib is used for reference. This reference is easier to
teach but should only be used for measurements in the supine
position because this reference point changes in relation to
the right atrium with changes in posture. The recommended
position for measurement of the CVP is the base of the c wave,
because this corresponds to the pressure in the atrium before
the onset of systole and therefore is the best estimate of
preload. Thus, option A is incorrect. CVP can be estimated on
physical examination by measuring the distension of the
jugular veins relative to the sternal angle and then adding 5
cm H2O to this measurement. Therefore, option C is correct.
CVP measurements are referenced to atmospheric pressure,
but physiologically it is transmural pressure (the difference
between intracardiac and intrathoracic-extracardiac
pressure) that determines ventricular preload. Increased
intrathoracic or elevated intrapericardiac pressure (eg,
cardiac tamponade, high positive end-expiratory pressure,
large pleural effusion) may reduce venous return while
paradoxically increasing the CVP. Unfortunately, pleural
pressure is not easily measured in critically ill patients. At
end-expiration the lungs recoil inward and the chest wall
outward, which gives a pressure around the heart of
approximately –2 to –3 cm H2O, not extremely different from
atmospheric pressure. Accordingly, vascular pressures
(including CVP) should be measured at end-expiration
whether the patient is ventilated with positive pressure
ventilation or breathing spontaneously. (Option B is
incorrect.)

RATIONALE (7) Answer: A

Pulmonary artery occlusion pressure (PAOP) is obtained by


inflation of the balloon at the tip of the pulmonary artery
catheter. After inflation of the balloon, there is a continuous
column of blood from the pulmonary artery to the left
ventricle during diastole. Consequently, PAOP is considered
an approximation of left ventricular end-diastolic pressure
(LVEDP). However, the assumption that PAOP always reflects
LVEDP is not always valid. When the tip of the catheter is in
West zone 1 or 2, alveolar pressure interrupts the blood
column. Consequently, the measured PAOP is higher than
end-diastolic pulmonary pressure and pulmonary venous
pressure. Thus, for accurate measurement, the catheter tip
must be in West zone 3 of the lung. Other factors that can
adversely alter interpretation of the relationship between
PAOP and LVEDP include increases in pulmonary vascular
resistance (option A) (eg, chronic obstructive pulmonary
disease, acidosis, pulmonary emboli), changes in intrathoracic
pressure (eg, positive end-expiratory pressure), changes in
the intracardiac environment (eg, mitral stenosis where PAOP
reflects increased left atrial pressure and not LVEDP), and
changes in ventricular compliance (eg, myocardial ischemia,
myocardial hypertrophy or dilatation, aortic or pericardial
disease).

RATIONALE (8) Answer: C


The 2010 American Heart Association (AHA) Advanced
Cardiac Life Support (ACLS) guidelines recommend adenosine
as a safe and potentially effective therapy in the initial
management of stable, undifferentiated, regular,
monomorphic, wide-complex tachycardia, not for ventricular
fibrillation, which the rhythm strip shows. Thus, option A is
incorrect. The AHA ACLS guidelines recommend 1 shock of
150 or 200 J (based on manufacturer guidelines) if using a
biphasic defibrillator or 360 J if using a monophasic
defibrillator. This takes the place of the 3 stacked shocks of
200, 300, and 360 J that were recommended in previous ACLS
guidelines (option B). The 3-shock recommendation was
based on the low first-shock efficacy of monophasic
dampened waveforms and efforts to decrease transthoracic
impedance with delivery of shocks in rapid succession.
Modern biphasic defibrillators have a high first-shock efficacy
in termination of ventricular fibrillation. In addition, 3 stacked
shocks delay resumption of chest compressions.
Approximately 70% of adults who have an out-of-hospital
cardiac arrest have underlying acute myocardial infarction or
pulmonary embolism. Therefore, it has been hypothesized
that thrombolysis during CPR might improve survival. A
recent double-blind, multicenter trial compared thrombolysis
(with tenecteplase) to placebo in adult patients with
witnessed out-of-hospital cardiac arrest. The trial was
terminated prematurely for futility after enrolling a total of
1,050 patients. No significant differences were found between
the thrombolysis and placebo groups in the primary end point
of 30-day survival or in the secondary end points of hospital
admission, return of spontaneous circulation, 24-hour
survival, survival to hospital discharge, or neurologic
outcome. There were more intracranial hemorrhages in the
thrombolysis group. There is no evidence that routine
administration of either amiodarone or lidocaine during
cardiac arrest increases the rate of survival to hospital
discharge. The Amiodarone in Out-of-Hospital Resuscitation
of Refractory Sustained Ventricular Tachycardia (ARREST)
trial compared an amiodarone 300-mg bolus with placebo in
patients with out-of-hospital cardiac arrest who had received
3 or more shocks, had no pulse, and had ventricular
fibrillation or ventricular tachycardia. More patients in the
amiodarone group had admission to the hospital with
successful resuscitation than the placebo group although
there were no significant differences in the rates of survival to
hospital discharge between the amiodarone-treated group
and placebo. The Amiodarone versus Lidocaine in Prehospital
Ventricular Fibrillation Evaluation (ALIVE) trial compared an
amiodarone 5-mg/kg bolus with a lidocaine 1.5-mg/kg bolus
in patients with out-of-hospital ventricular fibrillation. More
patients in the amiodarone group had successful survival to
hospital admission than the lidocaine group, but the study
found no significant difference in the rates of hospital
discharge between the amiodarone and lidocaine groups.
Therefore, option D is incorrect.

RATIONALE (9) Answer: B

Milrinone, a second-generation phosphodiesterase inhibitor,


is a nonadrenergic inotrope. Inhibiting phosphodiesterase
increases intracellular cyclic adenosine monophosphate,
resulting in positive inotropy and peripheral vasodilation. The
drug may be utilized for decompensated heart failure, as well
as for weaning from cardiopulmonary bypass after cardiac
surgery. Milrinone decreases both left ventricular afterload
and pulmonary vascular resistance. Cardiac output is
increased. In this patient, the vasodilatory properties of
milrinone decreased systemic blood pressure. The cardiac
index increased. Pulmonary artery occlusion pressure
decreased. Nitroglycerin (option A) is a venous vasodilator
that would decrease systemic blood pressure and right atrial
pressure, but the cardiac index would not be increased.
Dopamine (option C) has both inotropic and chronotropic
properties. Dopamine would increase the heart rate and
cardiac index, but would not lower the blood pressure.
Norepinephrine (option D) would have inotropic effects, but
the alpha-adrenergic effects would raise the systemic blood
pressure and increase systemic vascular resistance. Despite
the favorable hemodynamic profile for cases of
decompensated congestive heart failure (CHF), a recent
randomized controlled trial of milrinone (versus placebo) in
CHF failed to show a decrease in the number of days
hospitalized for cardiovascular causes. There was no
significant difference in mortality. The authors concluded that
IV milrinone should not be used routinely for cases of
decompensated CHF.

RATIONALE (10) Answer: C

The intraaortic balloon pump (IABP) is an important and


established therapy for various serious cardiac conditions.
The underlying principle of the IABP is that inflating the
balloon during early diastole will displace blood in the aorta,
increasing the diastolic BP, and by this mechanism improve
coronary perfusion pressure; deflation of the balloon just
before systole will decrease left ventricle aft erload by
producing a rapid decrease in aortic pressure. This, in turn,
will decrease myocardial oxygen demand and consumption.
Hemodynamic effects that would be expected in this clinical
scenario include the following:

 Increase in the diastolic arterial pressure, systolic arterial


pressure, and mean arterial pressure;
 Decrease of the pulmonary artery occlusion pressure;

 Improved left ventricular performance and ejection


fraction, with increased cardiac index.

 Decreased myocardial oxygen demand that would result


in decreased oxygen consumption.

 Only the values in option C match all these changes,


making it the best answer.

RATIONALE (11) Answer: C

This patient is having an acute ST-elevation anterior myocardial


infarction. In addition, he presents signs of developing
cardiogenic shock with tachycardia, low blood pressure, and
signs of a cardiogenic pulmonary edema. The SHOCK (Should
We Emergently Revascularize Occluded Coronaries for
Cardiogenic Shock) randomized trial demonstrated that, in
patients with acute myocardial infarction complicated by
cardiogenic shock, early mechanical revascularization reduced
mortality as compared with initial medical stabilization followed
by late or no revascularization. Based on these findings, in their
most recent guidelines the American College of Cardiology and
the American Heart Association elevated early mechanical
revascularization for cardiogenic shock to a class I
recommendation for patients younger than 75 years with ST
elevation or a left bundle-branch block acute myocardial
infarction. Although all the other options include treatments
that are important for patients with acute myocardial infarction,
some of them may be deleterious in patients developing
cardiogenic shock, and may have negative impacts on patients
with hypertension. In hospitals without the capability for onsite
cardiac interventions, stabilization and transfer to an institution
capable of performing revascularization is probably the best
strategy with acute myocardial infarction and cardiogenic shock.
The Danish Trial in Acute Myocardial Infarction (DANAMI) trial
compared primary mechanical revascularization (angioplasty)
with thrombolytic therapy in patients with ST-elevation acute
myocardial infarction. Patients randomized at sites without
cardiac catheterization facilities were transferred for
revascularization. In this trial, the combined outcome of death,
recurrent myocardial infarction, and stroke was improved in
patients randomized to the mechanical revascularization arm
(whether it was done onsite or the patient was transferred to
another facility).

RATIONALE (12) Answer: D

This patient’s pulmonary artery hypertension is probably due to


a pulmonary embolism. He requires vasopressor levels of
dopamine and has a central venous pressure (CVP) of 12 mm
Hg. It is generally accepted that fluid resuscitation effects are
variable in patients with suspected pulmonary embolism,
depending upon the degree of pulmonary artery hypertension
and current intravascular volume status. A CVP of 12 mm Hg,
however, would be considered inadequate, 20–25 mm Hg being
the optimal range. Thrombolytic therapy (options A and B) may
on occasion be utilized in patients who are at high risk for
pulmonary embolisms but have only echocardiographic support
for pulmonary embolism. In this patient, a definitive test would
be optimal management, as he is adequately oxygenated with
supplemental oxygen, requires levels of dopamine that are not
considered high, and has a relative contraindication to
thrombolytic therapy. Since CT of the chest is the only test
currently available, the choice is between dye exposure, which
in a patient with chronic renal insufficiency carries a significant
risk, versus thrombolytic exposure, with its risk of intrarenal
hemorrhage. The risks associated with the use of the dye in a
patient with renal insufficiency, even if it results in the need for
renal replacement therapy, when weighed against the risk of an
intracranial hemorrhage, give the edge to immediate definitive
testing for the presence or absence of a pulmonary embolism.
Delaying testing in this patient may allow his condition to
deteriorate. Given these considerations, urgent thrombolytic
therapy would not be the most appropriate choice.

RATIONALE (13) Answer: D

Effusive-constrictive pericarditis is a pericardial syndrome


characterized by tense pericardial effusion with a tamponade, in
addition to a visceral pericardial constriction. The diagnostic
criterion is the failure of the right atrial pressure to fall by 50%
or more, or to a level below 10 mm Hg after intrapericardial
pressure is normalized. Although the most frequent cause is
idiopathic, followed by neoplastic, a higher percentage of
patients with radiation-induced pericarditis will also have an
effusive-constrictive pericarditis. Following the drainage of the
pericardial effusion, there is typically some improvement in the
cardiac index, although it may not be as dramatic as in this case.
In some cases of constrictive pericarditis associated with
idiopathic acute exudative pericarditis, spontaneous resolution
is possible. This is also called transient cardiac constriction. In
other cases, surgery may be required to relieve visceral
constriction. Prior to pericardiocentesis, the right atrial
transmural pressures are very low (less than 2 mm Hg), with a
pulsus paradoxus of 10 mm Hg or more. These are typical
criteria for a cardiac tamponade and typically resolve with the
drainage of the pericardial effusion. However, constriction and
elevated right atrial pressure remain. Radiotherapy is a likely
risk factor for this patient given her medical history of lung
cancer not in remission and her previous therapy.

RATIONALE (14) Answer: C

Therapy for hypertrophic cardiomyopathy is directed at the


dynamic left ventricular (LV) outflow tract obstruction. The
obstruction causes an increase in LV systolic pressure which
leads to a complex interplay of abnormalities that decrease
cardiac output. In the intensive care setting, this condition often
deteriorates with volume depletion, and with the institution of
inotropic agents. In that circumstance, the infusion of fluids and
the discontinuation of inotropic agents is the initial therapy. A
beta-blocker should also be added; however, if hypotension is
present, a vasoconstrictor such as phenylephrine should be
administered first. Acute onset of atrial fibrillation may result in
severe hemodynamic compromise due to the loss of atrial
contractions. Prompt cardioversion should occur in this
circumstance. The first-line approach to the relief of symptoms
is to block the effects of catecholamines that exacerbate the
outflow tract obstruction, and to slow the heart rate to enhance
diastolic filling. Beta-blockers are generally the initial choice to
accomplish these goals. Verapamil, the calcium channel blocker,
can also be used. Sudden death has been reported in patients
with severe pulmonary hypertension and severe outflow
obstruction who are given verapamil. This drug should be given
with caution in patients with this combination of findings.
Nitroglycerin (option D) would decrease cardiac filling and is
problematic.

RATIONALE (15) Answer: B

The major known complication of intraaortic balloon pump


(IABP) therapy is noncardiac ischemia. Since the IABP is usually
inserted via a femoral artery, limb ischemia is the most common.
Malposition of the IABP can result in occlusion of the celiac,
superior mesenteric, or renal arteries. After comparison of
position of the IABP in chest radiographs and CTs, visualization
of the tip at the level of the carina can be used as confirmation of
the correct position. The tip of the IABP catheter is usually seen.
However, the balloon itself will not be visible unless the
radiography is performed during balloon insufflation. Because
this patient’s angina has resolved, there is no indication for
increase of the nitroglycerin (option A). Reduction of the IABP
support might increase the angina and is not an appropriate
intervention at this time, so option C is incorrect. It is likely that
occult blood in the stool would be secondary to anticoagulation
with heparin and nondiagnostic, so option D is also incorrect.

RATIONALE (16) Answer: A

The most recent guidelines for Advanced Cardiac Life Support


(ACLS) from the American Heart Association include the use of
continuous waveform capnography. Studies have demonstrated
high sensitivity and specificity of waveform capnography.
During cardiac arrest, PETCO2 less than or equal to 10 mm Hg
indicates that the cardiac output is inadequate to achieve a
return of spontaneous circulation. Although there is no
established value that optimizes the chance of return of
spontaneous circulation, a marked increase in Petco2 would
indicate sufficient perfusion. Chest compression should continue
for an additional 5 cycles after a return to spontaneous
circulation indicated by Petco2. Monitoring Petco2 has the
potential to guide the adequacy of CPR.

RATIONALE (17) Answer: B


The intraaortic balloon pump (IABP) is the most used
mechanical device for cardiac support. The traditional thought is
that coronary artery blood flow is augmented during inflation of
the IABP during isovolemic diastole while afterload and wall
tension are decreased when the balloon is deflated during
systole. The latter are likely the primary mechanisms of action.
The IABP is absolutely contraindicated in patients with aortic
dissection (because of the possibility of insertion in a false
lumen), aortic valve insufficiency (because it is ineffective and
worsens regurgitation), or a patent ductus arteriosus (because
of increased left-to-right shunt). Severe vascular disease,
although a comorbidity, does not exclude percutaneous IABP
placement in the brachial or external iliac arteries or surgically
in the descending thoracic aorta.

RATIONALE (18) Answer: A

Ventricular assist devices are inserted in patients with advanced


heart failure. Most often these are left ventricular assist devices
(LVAD) to improve patient outcome. These have been used as a
bridge to cardiac transplantation or as “destination therapy” to
maintain some quality of life if a transplant is not possible.
Adjustment of an LVAD can be technically challenging and
should be performed by qualified individuals. After the initial
surgical placement, it is often performed with monitoring via
transesophageal echocardiogram. The LVAD functions by
moving blood from the apex of the left ventricle to the aorta.
Increasing or decreasing the pump speed may actually decrease
the cardiac output by altering ventricular filling and decreasing
the output to the aorta. Thus, options B and C are incorrect.
Since LVAD patients receive anticoagulation, trace-positive
gastrointestinal bleeds are common, but still need to be worked
up to determine severity. In this patient, there is no evidence for
significant anemia. Therefore, option D is incorrect. Since the
hypotension occurred with the administration of sedation, it is
likely that it was secondary to a reduction in vascular tone.
Increasing the blood pressure with pressor administration
(option A) would be appropriate until a definitive diagnosis for
the hypotension is identified.

RATIONALE (19) Answer: A

Atrioventricular (AV) conduction defects are present in


approximately 25% of patients after cardiac surgery and are
most common after aortic and mitral valve surgeries. The
nomenclature for pacemakers is chamber paced (atrial [A],
ventricular [V], atrial and ventricular [D]), chamber sensed, and
function (inhibited [I], not effected [O], and triggers or inhibits
[D]). VOO is not indicated since the patient has atrial activity, so
option B is incorrect When the PR interval exhibits a prolonged
first-degree AV block, faster pacing with AOO does not achieve
adequate conduction since the AV node remains refractory
when the next beat occurs and creates an iatrogenic second-
degree block. Therefore, option C is incorrect. DDD pacing with a
shorter PR interval will markedly improve hemodynamics by
eliminating the first-degree AV block. Thus, option A is correct.

RATIONALE (20) Answer: C

Atrial overdrive pacing has been shown to be beneficial for the


prevention of atrial fibrillation. Postoperative atrial fibrillation
occurs in 30%–40% of patients in the second postoperative day
after cardiac surgery. The pathogenesis is multifactorial and
related to pericarditis, supraventricular premature contractions,
electrolyte disturbances, and sympathetic tone. Atrial overdrive
pacing for the first 24 hours after cardiac surgery has been
shown to markedly decrease the incidence of postoperative
atrial fibrillation. Therefore, option C is correct. Heart block
secondary to edema or surgical disruption of the conduction
system is seen immediately in the operating room and in the
ICU, so option A is incorrect. The cardiac index of 3.2 without
inotropics would not require augmentation whether or not the
patient was supported with mechanical ventilation. Therefore
options B and D are incorrect.

RATIONALE (21) Answer: C

The diagnosis of systolic and diastolic heart failure cannot be


made based on history and physical examination.
Echocardiography is needed. Both classifications of heart failure
could give this patient pulmonary edema with a worse
prognosis for diastolic heart failure. The primary goal of
treatment for diastolic dysfunction is to alleviate symptoms with
diuretics, beta-blockers, and inhibitors of the renin-angiotensin
system.

RATIONALE (22) Answer: C

A 2001 Consensus Statement by the Society of Critical Care


Medicine, European Society of Intensive Care Medicine,
American College of Chest Physicians, American Thoracic
Society, and Surgical Infection Society revised the definition of
sepsis to include infection with clinical or laboratory evidence of
inflammation. The definition of severe sepsis remains
unchanged as sepsis with an organ dysfunction or evidence of
organ hypoperfusion. Tissue hypoperfusion can occur in the
absence of hypotension and can be present for several hours
before clinical evidence is manifest. Clinical recognition of organ
dysfunction includes areas of mottled skin, capillary refill
greater than 3 seconds, urine output less than 0.5 mL/kg for at
least 1 hour or need for renal replacement therapy, lactate level
less than 2 mmol/L, abrupt change in mental status or abnormal
EEG findings, platelet count less than 100,000/µL, acute lung
injury/acute respiratory distress syndrome, and cardiac
dysfunction. Septic shock refers to refractory hypotension
despite adequate volume resuscitation, often necessitating the
use of vasopressors.

RATIONALE (23) Answer: D

Septic shock is defined as the presence of severe sepsis and


systemic mean arterial pressure less than 60 mm Hg (or <80
mm Hg with previous hypertension) after 40–60 mL/kg of
saline, 20–30 mL/kg of starch, and/or a pulmonary capillary
wedge pressure between 12 and 20 mm Hg. The need for
vasopressors in the setting of adequate fluid resuscitation also
defines septic shock. Refractory septic shock refers to the need
for high-dose vasopressors, such as dopamine >15 µg/kg/min,
or norepinephrine or epinephrine >0.25 µg/kg/min to maintain
mean arterial pressure greater than 60 mm Hg (or >80 mm Hg
with previous hypertension). The presence of hypotension in
the setting of aggressive fluid resuscitation in a patient with
infection is the hallmark of septic shock. Signs and symptoms of
systemic hypoperfusion may occur several hours before onset of
shock, or may occur even in the presence of normal blood
pressure parameters. Multiple-Organ Dysfunction Syndrome
(MODS) refers to the decline in organ function to the point that
homeostasis cannot be maintained without intervention, as
outlined in the 1991 American College of Chest
Physicians/Society of Critical Care Medicine consensus
statement.1 Primary MODS occurs as a result of a well-defined
insult in which organ dysfunction occurs early and can be
directly attributed to the insult itself (eg, renal failure due to
rhabdomyolysis). Secondary MODS is organ failure not in direct
response to the insult itself, but as a consequence of a host
response (eg, acute respiratory distress syndrome in patients
with pancreatitis). In the context of the definitions of sepsis and
systemic inflammatory response syndrome, MODS represents
the more severe end of the spectrum of severity of illness
characterized by systemic inflammatory response syndrome or
sepsis. There are no universally accepted criteria for MODS, and
there is considerable overlap with severe sepsis. Marshall6
critically evaluated the definitions of MODS adopted in the
clinical literature and provided a rationale for the physiologic
descriptors commonly used to define this syndrome, focusing on
markers of dysfunction for 6 organ systems: Pao2/FIO2, serum
creatinine level, Glasgow Coma Scale score, platelet count,
serum bilirubin level, and pressure-adjusted heart rate (heart
rate multiplied by the ratio of central venous and mean arterial
pressures).

RATIONALE (24) Answer: A

This patient displays the classic “transient responder” profile of


ongoing blood loss. Given his mechanism of injury, an occult
intra-abdominal source is likely. While myocardial ischemia is
possible even in young people without coronary artery disease
while under the influence of cocaine, his hypertension appears
not to rise to a level consistent with this presentation. Further,
with the placement of a central line and the ability to monitor
central filling pressures, a normal value tends to rule out both
cardiogenic and obstructive shock. A decrease in blood pressure
would be expected with the resolution of cocaine-induced
hypertension (option D), but this would probably be
accompanied by a resolution of tachycardia as well. The
reemergence of hypotension and tachycardia simultaneously is
most suspicious for occult hemorrhage.

RATIONALE (25) Answer: C


Delays in definitive treatment are common, particularly in mass
casualty situations or austere locations, and constitute
opportunities for preoperative optimization of the patient’s
status. This patient is showing classic symptoms of a class II
hemorrhage, appearing anxious, with shallow tachypnea and a
narrowed pulse pressure. Further, the patient is hypothermic,
acidotic, and possibly trending toward coagulopathy (the “lethal
triad”). While several of the individual therapies listed may be
beneficial, the only combination that coherently serves the
patient’s best interests is option C. The patient is oriented and
able to protect his airway; therefore, rapid-sequence intubation,
maintenance of sedation/anesthesia, and ventilator
management (options B and D) are most appropriately deferred
to the operating room. Since this patient’s narrowed pulse
pressure serves as a compensatory mechanism against shock,
induction of anesthesia remote from definitive treatment may
cause a sudden loss of peripheral vascular resistance and
assumes the unnecessary risk of cardiovascular collapse,
another argument for deferring until the operating room.
Placement of a nasogastric tube and removal of stomach
contents (option B) would not be a high priority in this patient,
especially once a definitive airway has been placed.
Hyperventilation and sodium bicarbonate (options A and D) are
of only transient value in the treatment of acidosis, and given
the patient’s increased lactate, he is more in need of fluid
resuscitation. His 2 large-bore IV lines are probably adequate
for resuscitation, making central line placement (options A and
B) a low priority and possibly an unnecessary risk.
Hydroxyethyl starches (option B) have been associated with a
dose-dependent decrease in factor VIII and von Willebrand
factor activity and may effect platelet function and prothrombin
time, both of which could be deleterious in a patient nearing
coagulopathy. While the patient will probably need blood and
factor replacement soon, he seems sufficiently stable to await
type and cross of an appropriate number of blood products;
therefore, option D is incorrect. Warming this patient is of
critical importance. Hypothermia contributes significantly to
morbid myocardial events, impairs platelet function and blood
loss, increases the risk of wound infections, and lengthens
hospital stay. A forced-air warmer should be placed as soon as
possible and exposed areas should be covered. If blood products
are begun in the emergency department, they should be
delivered through a fluid warmer. An increase in the patient’s
urinary output will indicate the effectiveness of ongoing fluid
resuscitation. Ideally, a urinary catheter will be placed
preoperatively, particularly if any significant delay is
anticipated; if not, one should be inserted soon after induction
and intubation.

RATIONALE (26) Answer: C

This patient exhibits the symptoms of a class III hemorrhage,


suggesting a loss of 30%–40% of his blood volume: confusion,
pulse rate greater than 120/min, decreased blood pressure,
decreased pulse pressure, and RR of 30–40/min. Estimates of
blood loss are often underestimated, as 30%–40% of the blood
volume of a patient of this size would be expected to be roughly
1.5–2 L.

RATIONALE (27) Answer: A

He is unstable and in class III hemorrhagic shock. A cause for


this shock is evident, in the form of a left hemothorax. After
securing his airway, this condition should be treated with a left
chest tube, possibly with progression to a left thoracotomy to
control bleeding. If available, sterile collection and
autotransfusion of the patient’s shed blood should be
considered. With two large-bore IV lines, the patient likely has
sufficient vascular access for resuscitation. Were a central line to
be placed after chest tube placement, a left-sided approach
would be prudent, as the risk of line-associated pneumothorax
would be obviated and the unaffected right lung would not be
placed at risk. Thus, option B is incorrect. Obtaining a chest
radiograph (option C) would be unnecessary in a clinically
diagnosed hemothorax; this study should be deferred until the
patient is stable in order to confirm appropriate placement of
both the endotracheal tube and the chest tube. While an
exploratory laparotomy (option D) may be necessary should the
patient continue to manifest signs of ongoing occult blood loss, it
should only be undertaken after the readily obvious sources of
loss, namely the left hemothorax and left femur fracture, have
been addressed.

RATIONALE (28) Answer: D

The transgastric midpapillary short-axis view will show the


cross-sectional chamber size (and therefore the volume status)
of the left ventricle, and is readily applied for globally assessing
both volume and left ventricular (LV) function. While the deep
transgastric long-axis view (option A) focuses on the LV, it is
difficult to use for volume assessment, as the probe may not be
centered in the LV chamber. This view is most often used for
interrogation of the aortic valve and the left ventricular outflow
tract. While close to and in the same orientation as the
midpapillary view, the transgastric basal short-axis view focuses
on the cross-sectional view of the mitral valve at the base (most
superior aspect) of the LV. Very little information about the LV
volume status can be gained from this view in isolation from
other views. Thus, option B is incorrect. The mid-esophageal
bicaval view (option C) focuses on the right atrium, the superior
vena cava, the inferior vena cava, and the interatrial septum.
While inspecting the relative fullness of the venae cavae can give
some indication as to volume status, the LV will not be present
in this view.

RATIONALE (29) Answer: D

The citrate in banked blood products chelates divalent cations


such as calcium and magnesium. After a massive transfusion,
hypocalcemia may decrease cardiac inotropy and blood
pressure. As the percentage of total calcium made up by ionized
calcium is small, as there is a large safety margin prior to the
dangers of hypercalcemia setting in, and as awaiting return of an
ionized calcium level from the lab may be time consuming,
empiric replacement of calcium should be considered in this
patient. While decreasing positive end-expiratory pressure
(PEEP) (option A) may increase preload in some cases, this
patient’s preload, as assessed both by central venous pressure
and transesophageal echocardiography, is more than adequate
to overcome the modest level of PEEP currently employed.
Dopamine dosages of 5–10 µg/kg/min should have the greatest
effect on beta1-adrenergic receptors, increasing cardiac
inotropy. Increasing this infusion to 20 µg/kg/min (option B)
would shift dopamine’s profile to favor alpha1-adrenergic
effects, producing vasoconstriction and increased peripheral
vascular resistance. While measured blood pressure may
increase, this increase to forward flow may actually decrease
cardiac output even further. In the absence of ongoing blood
loss, anemia, or hypovolemia, additional blood or fluid
resuscitation would not be indicated in this patient. Further
banked blood may even exacerbate this patient’s hypocalcemia.
Therefore, option C is incorrect

RATIONALE (30) Answer: B


This patient has developed acute left ventricular ischemia
immediately after coronary artery bypass grafting with a
significant elevation of left -sided filling pressures. Marked
elevation in the pulmonary artery diastolic pressure (PADP) but
not the pulmonary artery occlusion pressure (PAOP) suggests
the presence of pulmonary hypertension. Marked elevations in
the central venous pressure, PADP, and PAOP, with equalization
of diastolic filling pressures, is consistent with cardiac
tamponade. Either an anaphylactic reaction or a septic shock
will be characterized by lower filling pressures in the face of
hypotension and tachycardia.

RATIONALE (31) Answer: B

In the central venous pressure tracing, a normal V wave


indicates venous return. However, giant V waves can be seen
with tricuspid regurgitation or mitral regurgitation due to
papillary muscle ischemia on the pulmonary artery occlusion
pressure trace. Eventually, coronary artery disease could
provoke ischemia and then acute mitral valve regurgitation.

RATIONALE (32) Answer: C

Increasing preload, systemic vascular resistance (SVR), and


heart rate will best maintain hemodynamic stability in patients
with pericardial tamponade; the ultimate treatment will be the
evacuation of the effusion. In patients with aortic stenosis,
increasing preload and systemic vascular resistance while
maintaining sinus rhythm will be recommended. Mitral stenosis
should be approached by increasing left ventricular preload,
keeping sinus rhythm, and maintaining SVR at normal values. In
patients with mitral regurgitation, avoiding bradycardia,
carefully increasing left ventricular preload, and keeping SVR
low to allow forward blood flow are key. Patients with
idiopathic hypertrophic subaortic stenosis benefit from preload
and afterload augmentation and from slow heart rates.

RATIONALE (33) Answer: E

The lack of tolerance to rapid atrial fibrillation could be


explained as a twofold problem: First, the patient is being
induced into general anesthesia which per se could have caused
some transient hypotension. Second, the sudden loss of atrial
power could cause hemodynamic instability. Electrical
cardioversion should be the first line of treatment to help
reestablish normal hemodynamics. Deepening the anesthetic
could only cause more hypotension. A rapid bolus of fluid could
be helpful, but would be transient. Beta-blockers and or digoxin
have shown some benefit by controlling the rate, but they lack
cardioverter properties.

RATIONALE (34) Answer: A

Atrial fibrillation is a common feature of mitral stenosis because


of significant dilation of the left atrium. The left ventricle is
chronically underloaded and contractility is usually preserved.
Afterload should be maintained, and large increases should be
avoided because of possible left ventricular dysfunction.

RATIONALE (35) Answer: A

This patient has a Mobitz type II atrioventricular block. The


advanced cardiac life support (ACLS) bradycardia algorithm
should be followed; if transcutaneous pacing is not available or
fails to improve this condition, atropine should be considered. If
there is still no response, an infusion of epinephrine or
dopamine should be started. Phenylephrine could certainly be
considered; however, it could lead to more bradycardia by
causing a vagal reflex. CPR should become part of the treatment
if pulse and blood pressure cannot be detected.

RATIONALE (36) Answer: D

If the patient is stable, the BP should be lowered to 90–120 mm


Hg, first by using beta-blockers, then considering vasodilators.
Starting vasodilators first could increase shear forces and dP/dt,
potentially expanding the dissection.

RATIONALE (37) Answer: A

This patient has type B aortic dissection. The cornerstone of


management is long-term blood pressure and heart rate control.
In the acute setting, aortic dissection presents as a hypertensive
emergency that requires aggressive blood pressure
management. The propagation of the dissection primarily
depends on the mean arterial pressure and shearing force
created by the high velocity of left ventricular contraction
(aortic pulse dP/dt). Reducing the rate of rise of the aortic pulse
by decreasing the force of the left ventricular contractions
retards the progression of the dissection and decreases the risk
of aortic rupture. Beta-adrenergic blockade with peripheral
vasodilatation is the basis of modern management of aortic
dissection. Beta-blocker therapy should be started before a
vasodilator to limit reflex catecholamine release secondary to
the direct vasodilatation that will worsen the dissection.
Esmolol, with its quick onset, short duration of action and easy
titratability is the ideal beta-blocker for this circumstance. If
heart rate and blood pressure are not controlled with esmolol
alone, the addition of a vasodilator will be beneficial.
Nicardipine is a calcium-channel blocker with a high vascular
selectivity, fast onset, and relatively short half-life that produces
primarily arterial vasodilatation. Nitroprusside has been
traditionally used as an arterial vasodilator, but cyanide toxicity
may be problematic for patients with liver and kidney
dysfunction.

RATIONALE (38) Answer: D

Increased blood pressure in patients with acute ischemic stroke


is the mechanism to maintain perfusion of affected cerebral
tissue. A reduction of blood pressure to normotensive values has
the potential for further cereberal ischemia. Systemic
thrombolysis is a treatment option if given within 3 hours of the
onset of stroke. The American Stroke Association recommends
withholding antihypertensive treatment unless other
noncerebral organ damage is present, the patient is a candidate
for thrombolytic therapy, or systolic BP is greater than 220 mm
Hg and/or diastolic BP is greater than 120 mm Hg. The
treatment goal is a 15%–20% reduction in blood pressure
within 24 hours. Nitroprusside causes cerebral vasodilation and
increases intracerebral pressure. It can produce a “cerebral
steal” syndrome as a result of increasing vasodilation more in
systemic arterioles than in the cerebral vascular bed. Labetalol
or nicardipine are better choices for management of severe
hypertension in the setting of acute ischemic stroke.

RATIONALE (39) Answer: C

Major steps in the management of patients with severe


preeclampsia are hydration and magnesium sulfate for seizure
prophylaxis and blood pressure control. Treatment of severe
hypertension in these settings is primarily directed toward
reducing the risks of intracerebral hemorrhage (systolic BP
>160 mm Hg is an important factor in stroke with preeclampsia)
and cardiac failure while maintaining uteroplacental blood flow.
The American College of Obstetricians and Gynecologists
recommends treating severe hypertension during pregnancy
with a goal to reduce systolic BP to 140–160 mm Hg and/or
reduce diastolic BP to 90–105 mm Hg. Hydralazine was
traditionally used in pregnancy-induced hypertension and
preeclampsia, but it has a delayed onset and has been linked to a
higher incidence of maternal hypotension, placental abruption,
and emergent cesarean sections. Angiotensin-converting
enzyme inhibitors are contraindicated in pregnancy. Delivery is
the ultimate treatment for preeclampsia when the fetus is at a
more advanced gestational age.

RATIONALE (40) Answer: D

Emergent synchronized cardioversion is the treatment for new-


onset atrial fibrillation with hemodynamic instability. Unlike
defibrillation that is performed irrespectively of the cardiac
cycle, synchronized cardioversion is produced by synchronizing
discharge with the R wave. Timing the discharge prevents the
delivery of a high-energy shock within a vulnerable period of the
cardiac cycle (T wave), which might result in ventricular
fibrillation. Higher success rates are achieved when direct-
current cardioversion is performed earlier, from onset of atrial
fibrillation, with delivery of a higher-energy, biphasic shock. The
main purpose of the implantable cardioverter-defibrillator (ICD)
in this patient is the potential treatment of a lethal ventricular
dysrhythmia due to his cardiomyopathy. The ICD is
programmed to detect ventricular tachycardia and perform
pacing or cardioversion according to the algorithm. Modern
ICDs have special algorithms to distinguish between ventricular
and supraventricular dysrhythmias or sinus tachycardia in
order to prevent misfire of the device. External cardioversion
will not damage modern implantable pacemakers and ICDs. If
several attempts of external cardioversion are unsuccessful,
then the patient should be given amiodarone to facilitate
external cardioversion. Beta-blockade may result in further
hypotension, may not be efficacious, and may not have an
immediate effect.
PART 3: Respiratory Critical Care

Instructions: For each question, select the most correct answer.

1. Following a motor vehicle collision, a 20-year-old man is


admitted to the ICU with injuries including a fracture of the
left humerus and a possible closed head injury. Glasgow Coma
Scale score is 11 and he is in a cervical collar. Within the first
hour, his neurologic status deteriorates to a Glasgow Coma
Scale score of 7, and he requires endotracheal intubation for
airway protection.

To accomplish this intubation, which of the following


sequences provides the optimum conditions?

A. No sedation, cervical collar in place, fiberoptic intubation

B. No sedation, cervical collar in place, direct laryngoscopy

C. Rapid-sequence induction with propofol and


succinylcholine, in-line manual stabilization, cervical collar
off, direct laryngoscopy

D. Rapid-sequence induction with etomidate and vecuronium,


in-line manual stabilization, cervical collar in place, direct
laryngoscopy

E. Propofol, in-line manual stabilization, fiberoptic intubation

2. Following a modified radical neck dissection, a 72-year-old


man is admitted to the ICU for routine progressive care. Past
medical history includes hypertension treated with lisinopril.
Shortly after admission, the nurse notices that his tongue
appears to be larger than upon admission. When you arrive,
the nurse informs you that the tongue has now doubled in size
since you were called to evaluate him. The patient has become
increasingly anxious.

Which of the following is the most appropriate treatment at


this time?

A. Nebulized epinephrine

B. IV diphenydramine, 50 mg, and IV dexamethasone, 12 mg

C. Observation

D. Immediate intubation by an anesthesiologist with a


surgeon present

E. Elevation of the head of the bed to 30°

3. A thoracoabdominal esophagectomy is planned for a 46-year-


old man with an otherwise unremarkable past medical
history. Unfortunately, the laparotomy reveals diffuse
metastatic disease and the procedure is aborted. He is
brought to the ICU because he is still chemically paralyzed.
Prior to extubation, which of the following would confirm
return of muscle strength?

A. A return of the train-of-four to 4 equal, strong twitches

B. Inspiratory force of –20 cm H2O

C. 600-mL tidal volume on pressure support ventilation at 16


cm H2O

D. Ability to follow commands


E. RR greater than 18/min

4. Which of the following patients is the best candidate for


noninvasive ventilation?

A. 55-year-old man with ST-elevation myocardial infarction,


pulmonary edema; BP of 90/64 mm Hg, HR of 110/min, RR of
25/min, pulse oximetry of 90% on nonrebreather mask, arterial
blood gas (ABG) pH of 7.28, Paco2 of 45 mm Hg, and Pao2 of 58
mm Hg

B. 68-year-old man receiving home oxygen for chronic


obstructive pulmonary disease that is in exacerbation with mild
lethargy, BP of 140/92 mm Hg, HR of 110/min, RR of 30/min,
pulse oximetry of 96% on nonrebreather mask, ABG pH of 7.20,
Paco2 of 70 mm Hg, and Pao2 of 120 mm Hg

C. 28-year-old woman with an exacerbation of asthma, BP of


130/88 mm Hg, HR of 118/min, RR of 20/min, pulse oximetry
of 97% on 2 L/min of oxygen via nasal cannula, ABG pH of 7.45,
Paco2 of 35 mm Hg, and Pao2 of 120 mm Hg

D. 59-year-old man with bronchiectasis, worsening shortness of


breath, significant ongoing hemoptysis, BP of 118/68 mm Hg,
HR of 105/min, RR of 32/min, pulse oximetry of 92% on
nonrebreather mask, ABG pH of 7.42, Paco2 of 30 mm Hg, and
Pao2 of 65 mm Hg

E. 62-year-old woman with coronary artery bypass surgery


followed by extubation immediately postoperatively and
respiratory distress 8 hours later, BP of 108/70 mm Hg, HR of
72/min, RR of 25/min, pulse oximetry of 94% on
nonrebreather mask, ABG pH of 7.37, Paco2 of 34 mm Hg, and
Pao2 of 76 mm Hg
5. A 62-year-old woman is admitted to the ICU with an
exacerbation of acute chronic obstructive pulmonary disease. An
arterial blood gas study showing pH of 7.15, Paco2 of 78 mm Hg,
Pao2 of 68 mm Hg, and oxygen saturation of 96% is obtained
during administration of supplemental oxygen. Noninvasive
ventilation is initiated with expiratory pressure of 4 cm H2O,
inspiratory pressure of 10 cm H2O, and FIO2 of 50%. An hour
later, the patient’s BP has dropped from 140/85 to 110/60 mm
Hg. On examination, the patient seems more lethargic. Sao2 is
now 89%. An ECG shows T-wave inversions in the anterior
leads. Which of the following the best next step in this patient’s
management?

A. Measure arterial blood gases

B. Increase oxygen to maintain saturation at 92% or higher

C. Orally intubate the patient and provide mechanical ventilator


support

D. Increase the expiratory pressure setting by 5 cm H2O

E. Increase the inspiratory pressure setting by 3 cm H2O

6. Which of the following patient data are utilized in the


calculation of the appropriate tidal volume for a patient with
acute respiratory distress syndrome (ARDS) in accordance with
ARDSNet recommendations?

A. Body mass index

B. Weight

C. Height
D. Age

7. A 64-year-old woman weighing approximately 60 kg (132


lbs), is admitted to the ICU from the emergency department
following intubation and initiation of mechanical ventilation. No
past medical history is available. Chest radiograph reveals a
right lower lobe infiltrate consistent with pneumonia.
Ventilator settings are FIO2 of 0.5, assist-control volume
ventilation, rate of 16/min (total assisted rate of 20/min), tidal
volume of 600 mL, square inspiratory waveform, peak
inspiratory flow rate of 80 L/min, and positive end-expiratory
pressure of 5 cm H2O. The patient exhibits dyssychronous
breathing. Pulse oximetry oxyhemoglobin saturation is 98%.
Flow-over-time waveform is shown (see Figure below). Which
of the following is most likely to be effective in improving the
dyssynchronous breathing?

A. Decreasing peak inspiratory flow rate

B. Switching to decelerating inspiratory flow waveform

C. Increasing set ventilator rate to >20/min

D. Positioning patient with left side down


E. Increasing set positive end-expiratory pressure

8. A 28-year-old man weighing 80 kg (176 lbs) is admitted to


the ICU following intubation and initiation of mechanical
ventilation for community-acquired pneumonia associated
with severe hypoxemia. His breathing pattern is
dyssynchronous. Ventilation settings on assist-control, volume-
cycled ventilation are FIO2 of 0.6, rate of 14/min (total rate
assisted is 25/min), tidal volume of 700 mL, positive end-
expiratory pressure (PEEP) of 10 cm H2O, and peak inspiratory
flow rate of 60 L/min, with a square inspiratory flow
waveform. Pulse oximetry reveals an oxyhemoglobin
saturation of 96%. Pressure-over-time waveform is shown in
the Figure. Which of the following is most likely to relieve the
patient’s dyssynchronous breathing?

A. Increasing peak inspiratory flow rate

B. Switching to a decelerating inspiratory flow waveform

C. Increasing ventilator rate

D. Increasing PEEP E. Decreasing PEEP

9. A 25-year-old, asthmatic woman undergoes emergent


laparotomy for a perforated appendix. Postoperatively, she
develops profound dyspnea and acute respiratory failure. She
is intubated with a rapid-sequence induction. Mechanical
ventilation is initiated in an assist-control mode at 16/min,
tidal volume of 550 mL, positive end-expiratory pressure of 0
cm H2O, and FIO2 of 1.0. In the postanesthesia recovery room,
the patient is deeply sedated, with RR of 16/min on a set rate of
16/min with the ventilator. The flow graphic waveform shown
in the Figure is displayed on the ventilator. Which of the
following would be most effective in correcting this problem?

A. Decrease the inspiratory flow rate.

B. Increase tidal volume.

C. Decrease respiratory rate.

D. Increase positive end-expiratory pressure.

10. A 58-year-old patient is intubated with an exacerbation of his


chronic obstructive pulmonary disorder. There is no
pneumonia present, but secretions are thick and yellow.
Bronchodilator and steroid therapies are initiated. On day 4 of
mechanical ventilation, the patient suffers respiratory distress
followed by cardiac arrest. Respiratory support data over the 5
days leading up to the arrest are provided below:

Which of the following is the most likely reason for the


respiratory arrest?

A. Flash pulmonary edema

B. Endotracheal tube obstruction

C. Tension pneumothorax

D. Pulmonary embolus
11. A 50-year-old woman intubated 7 days for pneumonia and
severe sepsis is being evaluated for weaning from mechanical
ventilation. Current ventilator settings include volume-assist
mode with a tidal volume of 450 mL, rate of 12/min, FIO2 of
0.35, and positive end-expiratory pressure of 5 cm H20. She
has stable vital signs off sedative or vasoactive drips and is
awake and able to follow simple commands. Her hand grip is
weak bilaterally, lungs are clear, and heart rate is regular with
no murmurs or gallop. She is placed on continuous positive
airway pressure for 3 minutes, during which time her RR is
25/min with an average tidal volume of 350 mL, and a negative
inspiratory force measured at –30 cm H20. An arterial blood
gas study is pending.

Which of the following is the most appropriate course of


actionregarding this patient’s weaning from mechanical
ventilation?

A. Patient should not be extubated as she is likely to fail


extubation.

B. Patient should be placed on pressure support ventilation of 5


cm H2O for 120 minutes.

C. Patient should be extubated at this point without further


action.

D. Patient should be extubated and placed on noninvasive


ventilation.

E. Patient should be extubated based on arterial blood gas


results.
12. A 75 year-old man with hypertension, chronic obstructive
pulmonary disease (COPD), and type 2 diabetes was admitted
to the ICU with pneumonia and has been on mechanical
ventilation for 12 days. He has completed a course of
antibiotics and has also received aggressive treatment for
COPD with albuterol, ipratropium, and steroids. Current
ventilator settings are assist-control mode with tidal volume of
400 mL, rate of 14/min, FIO2 of 0.30, and positive end-
expiratory pressure of 5 cm H2O. His oxygen saturation is
90%–92% and his peak airway pressure is 12 cm H2O. On
these settings, HR is 88/min, BP is 128/ 72 mm Hg, and RR is
20/min. On examination, he is awake and able to follow
commands. He has generalized decreased muscle strength
(upper extremities 4+/5 and lower extremities 4/5). Lungs are
clear to auscultation (with prolonged expiratory phase), and
cardiac examination findings are unremarkable.
Spontaneousbreathing trials have failed on previous days but
the patient is now more awake and cooperative. He is placed on
pressure support of +5 cm H2O and after 40 minutes seems to
develop increased distress and anxiety. HR is 108/min, BP is
140/86 mm Hg, and RR is 30/min. Arterial blood gases show a
pH of 7.35, Paco2 of 40 mm Hg, Pao2 of 58 mm Hg, and an Sao2
of 88%. Blood work sent at this time shows normal electrolyte
level and a serum protein level of 7 mg/dL (serum protein level
drawn earlier in the day was 5.5 mg/dL). An ECG done during
the weaning trial shows sinus tachycardia with no ST-segment
changes.

Which of the following is the most likely cause of this patient’s


failure to wean from the ventilator?

A. Acute delirium

B. Increased airway resistance


C. Cardiac dysfunction

D. Inspiratory muscle fatigue

E. Endocrine dysfunction

13. A 46-year-old woman is admitted to the ICU from the floor


after developing respiratory distress with increased work of
breathing and increased oxygen requirement. The patient has
history of systemic lupus erythematosus (SLE) and diabetes.
She was admitted 2 days ago with fever and dyspnea. On
admission to the hospital, significant laboratory data included:
WBC count of 12,000/µL, hemoglobin level of 11 g/dL, platelet
count of 110,000/µL, and creatinine level of 1.3 mg/dL. Initial
chest radiograph showed a possible consolidation versus
atelectasis at the right lung base. The patient was admitted for
further workup and started on antibiotics for possible
infection. Temperature is 37.3°C (99.1°F), HR is 120/min, RR is
32/min, and BP is 155/89 mm Hg. Arterial blood gas findings
obtained prior to intubation show a pH of 7.32, Pao2 of 58
mmHg, Paco2 of 48 mm Hg, and hemoglobin level of 7.4 g/dL.
Chest radiograph is shown in the Figure.
Which of the following interventions would be most
appropriate as initial therapy for this patient?

A. Trimethoprim/sulfamethoxazole

B. Furosemide

C. Methylprednisolone

D. Factor VIIa

E. Rituximab
14. A 28-year old man presents to the emergency department
with worsening shortness of breath and a cough productive of
blood-tinged sputum and lasting 2 days. He denies fever, chills,
or chest pain but admits to shortness of breath while jogging on
the day of presentation. He denies injection drug use or
tuberculosis exposure. Patient smokes 1 pack of cigarettes a
day. Patient has no past medical problems and recently had a
normal physical examination with normal blood findings at
work. Physical examination reveals an anxious young male in
respiratory distress; temperature is 37.3°C (99°F), RR is
28/min, HR is 110/min, BP is 138/70 mm Hg, and Spo2 is 87%.
There are no petechiae or ecchymoses; cardiac examination
shows tachycardia with normal S1 and S2, and no murmurs or
gallops. Lung auscultation reveals crackles over both bases;
abdomen is soft, nontender, and nondistended, with no
organomegaly. Chest radiograph reveals bilateral patchy
alveolar infiltrates. Additional testing reveals hemoglobin level
of 10 g/dL, WBC count of 10,800/µL with normal differential,
platelet count of 450,000/µL, and serum creatinine level of 3.5
mg/dL. Urinalysis shows 10 RBCs, RBC casts, proteinuria, and
no bacteria.

The patient develops worsening respiratory distress with


hypoxemia and is intubated prior to transfer to the ICU.

Which of the following diagnostic tests is most likely to


influence the therapeutic approach to this patient’s underlying
condition?

A. Bronchoscopy with bronchoalveolar lavage

B. CT angiogram of the chest

C. Antineutrophil cytoplasmic autoantibody titer


D. Antibasement membrane antibody titer

E. Bone marrow biopsy

15. A 21-year-old man is transferred to the ICU with an


exacerbation of his asthma. He was recently admitted to the
hospital after a motor vehicle collision that resulted in cervical
spine instability that required immobility using a halo-vest. From
reviewing his chart, it is clear that he frequently visits the
emergency department for acute respiratory distress. During the
past 6 months, he has been treated for 5 acute asthma
exacerbations, and once required mechanical ventilation. His
asthma appears to be refractory since he has poor response to
bronchodilators and steroids; he has persistent wheezing and
increased work of breathing.

On examination, the patient is anxious, with tachypnea and


audible wheezing. In the ICU initially, temperature is 37.5°C
(99.4°F); HR is 110/min; RR is 25/min; BP is 134/78 mm Hg;
and Spo2 is 100% on nonrebreather mask. Arterial blood gas
results show pH of 7.30, Paco2 of 50 mm Hg, and Pao2 of 90
mm Hg.

Because of his cervical spine immobility, the anesthesiologist


decides to intubate the patient awake with fiberoptics.

Which of the following findings is most likely observed during


the intubation?

A. Bilateral true vocal cord fixation

B. Unilateral vocal cord paralysis


C. True vocal cord abduction on inspiration, and partial
adduction during expiration

D. True vocal cord adduction during inspiration

16. A 76-year-old man presents to the emergency department


with worsening shortness of breath. Medical history is significant
for chronic obstructive pulmonary disease on home oxygen and
peptic ulcer disease with a recent episode of upper
gastrointestinal bleeding. In the emergency department, his
respiratory status quickly deteriorates and he requires
intubation. The patient is admitted to the ICU with a diagnosis of
respiratory failure secondary to community-acquired pneumonia.
After 2 weeks of mechanical ventilation, a tracheostomy is
performed because the patient is not making progress towards
weaning from mechanical ventilation. He is transferred to a
chronic ventilatory unit in the hospital for weaning. Twelve days
after his tracheostomy, his secretions have become bloody.
Evaluation reveals stable hemodynamics with no change in
hemoglobin or hematocrit. Respiratory and nursing staff are
asked to limit suctioning through the tracheostomy to avoid
traumatic bleeding. Two days later, the patient has massive
bleeding through his tracheostomy. Despite rapid and aggressive
measures to control the hemorrhage, the patient quickly
exsanguinates and dies.

Which of the following is the most likely diagnosis?

A. Pulmonary embolism with lung infarction

B. Trauma from overaggressive suctioning

C. Trachea-innominate artery fistula


D. Pulmonary alveolar hemorrhage
E. Upper gastrointestinal hemorrhage

17. A 60 year-old man presents to the emergency department


with acute-onset shortness of breath and chest pain. The
patient has a recent diagnosis of colon cancer and had a
colectomy 8 days ago. He has a medical history of hypertension
and type 2 diabetes mellitus. On physical examination, he is
alert with mild respiratory discomfort, temperature of 37.9°C
(100.1°F), HR of 117/min, RR of 22/min, and BP of 135/84 mm
Hg. Patient weighs 120 kg (264 lbs) and his body mass index is
32. Oxygen saturation on room air is 90% and is 96% with 3
L/min of supplemental oxygen. Lung auscultation findings are
normal, and cardiac examination reveals regular tachycardia
with normal S1 and S2 and no murmurs or gallops. Abdomen is
soft and nontender with a clean surgical scar. ECG shows sinus
tachycardia with no signs of cardiac ischemia. Chest radiograph
is shown in the Figure. Serum creatinine concentration is 2.5
mg/dL (was 1.2 mg/dL before surgery).
Which of the following is the most appropriate initial
management?

A. CT angiography prior to initiating treatment

B. IV unfractioned heparin, 80-U/kg-bolus followed by 18-


U/kg/h IV (based on ideal body weight)

C. Subcutaneous enoxaparin, 120 mg/kg every 12 hours


D. IV argatroban, 2 mg/kg/min

E. IV unfractioned heparin, 9,600-U bolus followed by 2,100


U/h

18. A 48-year-old woman presents to the emergency department


with acute-onset shortness of breath and chest pain. She has a
history of breast cancer and is currently undergoing
chemotherapy. On examination, temperature is 37.2°C (98.9°F),
HR is 120/min, RR is 22/min, and BP is 110/68 mm Hg. CT
angiography shows bilateral filling defects consistent with
pulmonary embolism. A bedside echocardiogram reveals a
dilated right ventricle with decreased contractility (free-wall
hypokinesis) and paradoxical movement of the interventricular
septum. The intensivist is consulted to determine if ICU care is
warranted. Which of the following management decisions
should be instituted based on the findings on the
echocardiogram?

A. Lower extremity ultrasonography before further therapeutic


decisions
B. Systemic thrombolytic therapy
C. Admission to the ICU and treatment with unfractionated
heparin
D. No ICU admission, but inferior vena cava filter placement

19. Which of the following statements is most correct with regards


to the utilization of low molecular weight heparins (LMWH) in
the prophylaxis and treatment of pulmonary embolism (PE) in
critically ill patients?

Which of the following statements should guide the utilization


of low-molecular-weight heparins (LMWHs) in the prophylaxis
and treatment of pulmonary embolism (PE) in critically ill
patients?

A. LMWH is contraindicated in critically ill patients with renal


failure.

B. LMWH is superior to unfractioned heparin in treating PE.

C. LMWH can be utilized in critically ill patients suspected of


having developed heparin-induced thrombocytopenia.

D. Subcutaneous LMWH administration in surgical critically ill


patients can result in lower plasma drug levels.

E. LMWHs are not indicated for critically ill patients with PE.

20. A 52-year-old man is admitted to the ICU from the emergency


department with a diagnosis of acute pulmonary embolism.
The patient is intubated and receiving unfractioned heparin,
1,000 U/h as a continuous infusion on arrival to the ICU. Initial
examination reveals a mildly sedated patient with a
temperature of 37.9°C (100.1°F), HR of 110/min, RR of 20/min,
and BP of 135/82 mm Hg. Shortly after arrival, you are called
to the bedside due to hypotension. The patient’s HR is 125/min
and BP is 88/54 mm Hg. Findings of focused ultrasonography
done at the bedside are shown in the Figure.
Which of the following therapeutic interventions is most
appropriate?

A. Hold heparin and perform pericardiocentesis.

B. Administer bolus of 0.9% normal saline.

C. Initiate norepinephrine infusion.

D. Initiate systemic thrombolyitics.

21. A 35-year-old woman is transferred intubated to your ICU from


an outside hospital with respiratory failure. The patient
presented to the outside hospital yesterday with a 3-day
history of high fevers, myalgia, and increased shortness of
breath. She has no significant past medical history. She was
initially treated for a possible bacterial pneumonia and for
influenza but developed progressive hypoxemia with increased
oxygen requirements.

On arrival to the ICU, she is sedated with propofol. On physical


examination, her temperature is 38.1°C (100.5°F), HR is
108/min, BP is 128/78 mm Hg, and RR is 26/min. The patient
weighs 100 kg (220 lbs), height is 157 cm (5 ft 2 in), and body
mass index is 41. She responds to stimulation. Lung exam has
coarse breath sounds bilaterally, cardiac examination reveals
tachycardia with regular rate and normal heart sounds, and the
abdomen is soft and nontender. Chest radiograph from the
outside hospital is shown in the Figure.

Which of the following are the most appropriate initial


ventilator settings for this patient?
A. Pressure-controlled ventilation with tidal volume of 600 mL;
RR, 18/min; FIO2, 100%; positive end-expiratory pressure
(PEEP), 12 cm H2O

B. Assist-control with tidal volume of 400 mL; RR, 20/min,


FIO2, 100%; PEEP, 8 cm H2O

C. High-frequency oscillatory ventilation with frequency of 6


Hz; amplitude, 70 cm H2O; peak airway pressure, 30 cm H2O;
FIO2,100%; inspiratory time, 33%; bias flow, 40 L/min

D. Airway pressure release ventilation, with Thigh of 5


seconds; Tlow 0.6 seconds; Phigh 30 cm, H2O; Plow 0 cm H2O;
FIO2 100%

22. A 44-year-old woman is admitted to the ICU from the


emergency department, where she presented with severe
acute pancreatitis. She required intubation and seems to have
developed acute respiratory distress syndrome (ARDS). On
physical examination, she is intubated and sedated.
Examination of the lungs reveals diffuse crackles and rhonchi,
cardiac examination reveals regular tachycardia with normal
heart sounds, and her abdomen is very tender and has
decreased bowel sounds. Chest radiograph shows diffuse
bilateral infiltrates. Initially, the patient had a good oxygen
saturation on a FIO2 of 50% and positive end-expiratory
pressure of 8 cm H2O. Now on arrival to the ICU her oxygen
saturation has dropped and her FIO2 has been increased to 10
0%. A pressure-volume curve is shown in the Figure.
Please select the point on this pressure-volume curve that
would be most appropriate as the initial target to set PEEP in
this patient.

A. Point A

B. Point B

C. Point C

D. Point D

23. A patient with acute respiratory failure is in the ICU on


mechanical ventilation. The graph of airway pressure over
time for this patient’s current mechanical ventilation settings
is shown below (figure 23).
Figure 23. PAW = Airway pressure in cm H2O Which of the
following changes would be most appropriate to correct a
high carbon dioxide level?

A. Increase Thigh

B. Increase frequency by 1–2 Hz

C. Increase Tlow

D. Increase amplitude (Power) by 5–10 cm H2O

E. Decrease Phigh

24. 61-year-old man with a history of myasthenia gravis is


admitted to the ICU for progressive dyspnea and weakness. The
patient’s wife reports that he stopped taking his medications
several days ago. On admission to the ICU, the patient has shallow
breaths and difficulty speaking. Lung examination findings are
normal, with clear breath sounds on both sides. A chest
radiograph shows no infiltrates. Bedside respiratory testing
shows a vital capacity of 1.6 L (40% of the predicted value) and a
maximum negative inspiratory force of –15 cm H2O. Treatment
with pyridostigmine and steroids is initiated. During the next day,
his overall weakness remains unchanged. During morning
rounds, you notice that he is having difficulty with secretions and
the nurse reports that he has been coughing when given water
with his pills.

Which of the following courses of action is most appropriate for


the management of this patient at this point?

A. Order the patient to be kept fasting and administer all


medications intravenously.

B. Continue to monitor vital capacity and intubate if it drops


below 1.0 L.

C. Initiate continuous positive airway pressure via face mask.

D. Initiate bilevel positive airway pressure ventilation via nasal


mask.

E. Orally intubate the patient and initiate mechanical ventilation.

25. A 52-year-old man is evaluated in the emergency department


for a 3-day history of increased shortness of breath with cough
productive of yellow sputum. The patient has a history of chronic
obstructive pulmonary disease (on home oxygen) and continues
to smoke 2 packs of cigarettes a day.

On examination, his temperature is 38.1°C (100.5°F), HR is


110/min, BP is 132/84 mm Hg, and RR is 28/min. The patient
has a prolonged expiratory phase on auscultation, with crackles
in the left lower lung field and intercostal retractions during
respiration. His cardiac examination findings are significant for
tachycardia with normal heart sounds and no murmurs or
gallop. Patient is awake and neurologic examination is nonfocal.
WBC count is 18,000/µL with 10% bands. On 2 L/min of oxygen
via nasal cannula, arterial blood gas (ABG) results show pH of
7.26, Pao2 of 58 mm Hg, and Paco2 of 65 mm Hg. Therapy with
IV methyprednisolone, nebulized albuterol/ipratropium, and IV
levofloxacin is initiated. The patient is started on noninvasive
positive-pressure ventilation with an FIO2 of 70%, inspiratory
airway pressure (IPAP) of 10 cm H2O, and an expiratory airway
pressure (EPAP) of 5 cm H2O.

The patient is admitted to the ICU. Approximately 2 hours after


arrival, the patient is more lethargic, although he seems to
respond to stimulation. Temperature is 37.3°C (99°F), HR is
128/min, BP is 88/54 mm Hg, and RR is 32/min. ECG shows
what seems to be multifocal tachycardia. Repeat ABG results
show pH of 7.24, Pao2 of 60 mm Hg, and Paco2 of 68 mm Hg. The
physician orders a bolus of IV fluids.

Which of the following is the most appropriate next step in this


patient’s management?

A. Increase IPAP to 14 cm H2O.

B. Increase EPAP to 8 cm H2O.

C. Intubate and begin mechanical ventilation.

D. Increase IPAP and EPAP by 2 cm H2O.

E. Continue current settings and recheck ABG in 30–60 minutes.


26. A 29-year-old woman with a history of asthma presents to
the emergency department in respiratory distress with a 2-day
history of increasing shortness of breath after a febrile illness.
The patient states that, despite using her routine albuterol and
inhaled steroids, her symptoms have worsened. Her temperature
is 37.3°C (99°F), HR is 110/min, RR is 34/min, and BP is 130/78
mm Hg. The patient weighs 60 kg (132 lbs). Auscultation of the
chest reveals diffuse wheezing with decreased breath sounds.
Chest radiograph shows hyperinflated lungs with no evidence of
infiltrates. An arterial blood gas (ABG) study on 4 L/min of
oxygen shows pH of 7.38, Pao2 of 79 mm Hg, and Paco2 of 45
mm Hg. She is started on continuous albuterol nebulizer and
given IV methylprednisolone. Approximately 1 hour later, she is
more lethargic, and after a repeat ABG shows pH of 7.31, Pao2 of
70 mm Hg, and Paco2 of 52 mm Hg, she is intubated.

On arrival to the ICU, the patient is heavily sedated and


paralyzed. Ventilator settings are assist-control mode with tidal
volume of 600 mL, RR of 16/min, FIO2 of 60%, and positive end-
expiratory pressure (PEEP) of 5 cm H2O. With these settings, the
peak airway pressure is 55 cm H2O and an ABG shows pH of
7.37, Pao2 of 90 mm Hg, and Paco2 of 46 mm Hg. Her vital signs
are stable. Treatment with inhaled albuterol/ipratropium and IV
steroids is continued. One hour later, when an ABG shows pH of
7.33, Pao2 of 70 mm Hg, and Paco2 of 50 mm Hg, the physician
on call increases the RR to 24/min. An ABG done after the
ventilator changes shows pH of 7.32, Pao2 of 70 mm Hg, and
Paco2 of 46 mm Hg, but the patient becomes hypotensive and
tachycardic (BP is 72/48 mm Hg and HR is 138/min). Physical
examination reveals a midline trachea and bilateral hyper-
resonance of the chest.

Which of the following interventions is most appropriate as the


next step in this patient’s management?
A. Administer a bolus of IV fluids.

B. Perform bilateral needle decompression of the chest.

C. Decrease respiratory rate on ventilator.

D. Disconnect patient from ventilator.

27. A 28-year-old woman with no previous medical history


presents to the emergency department with a 2-day history of
lower right abdominal pain that has progressively increased and
today has been associated with nausea and vomiting. A right
ovarian torsion is diagnosed. The patient undergoes a right
salpingo-oophorectomy under general anesthesia that is
uneventful. Approximately 1 hour after admission to the
postoperative care unit, the patient reports increased shortness
of breath and coughs frothy, blood-tinged sputum. She is started
on supplemental oxygen, 4 L/min, to keep her Sao2 above 90%.
On examination, the patient has bilateral crackles on
auscultation and mild tachycardia. A portable chest radiography
reveals a normal cardiac silhouette with diffuse bilateral
infiltrates.

Which of the following interventions is most appropriate for this


patient’s condition?

A. Supplemental oxygen and supportive care

B. Levofloxacin and metronidazole

C. Methylprednisolone

D. Furosemide
28. A 25-year-old man with asthma that is managed with
albuterol and inhaled steroids is admitted to the hospital after a
3-day history of increasing shortness of breath, coughing, and
wheezing. He has had multiple asthma exacerbations during the
past year and has taken a short course of systemic
corticosteroids. He admits to poor adherence with his asthma
regimen when he is feeling well. Initial assessment in the
emergency department reveals severe bronchospasm and
tachypnea. After aggressive treatment with
albuterol/ipratropium, magnesium, and corticosteroids, the
patient shows improvement and is admitted to the medical
floors. He is continued on IV corticosteroids,
albuterol/ipratropium inhalers, and levofloxacin. Several hours
after admission, he starts having increased respiratory distress
unresponsive to bronchodilators and is transferred to the ICU,
where he is intubated and started on mechanical ventilation.
Because of refractory bronchospasm, he requires sedation and
neuromuscular paralysis. On day 3 in the ICU, he develops new
fevers. Chest radiography shows diffuse alveolar infiltrates.
Nondirected bronchial aspirates are obtained and sent for Gram
stain and culture. Antibiotic coverage is expanded to vancomycin
and cefepime. On day 4, the lab reports that the Gram stain from
the sputum has gram-positive, dichotomously branching hyphae.
Clinically, the patient has persistent fevers and increased oxygen
requirements.

Which of the following is most appropriate for the management


of this patient based on the available information?

A. Change vancomycin to linezolid.

B. Initiate fluconazole.
C. Initiate voriconazole.

D. Initiate trimethoprim/sulfamethoxazole.

E. Keep current antibiotic regimen pending cultures.

29. You are part of the critical care rapid response team called to
the general medical floor to evaluate a 41-year-old woman with
acute shortness of breath. The patient has a history of sickle cell
disease and was admitted 4 days ago with upper extremity and
flank pain reminiscent of previous vaso-occlusive sickle cell
crisis. Temperature is 38.4°C (101°F), HR is 110/min, RR is
38/min, BP is 165/ 92 mm Hg, and oxygen saturation is 74%. On
examination, the patient is in distress from labored breathing.
Lung auscultation reveals diffuse crackles with expiratory
wheezes, and an emergent portable chest radiograph shows new
diffuse, bilateral pulmonary infiltrates. Bedside focused
ultrasonography of the heart reveals a dilated right ventricle
with decreased right ventricular contractility, normal left
ventricular contractility, and no pericardial effusions or regional
wall motion abnormalities. The patient is intubated after
becoming lethargic and transferred to the ICU.

Which of the following interventions is most likely to produce


rapid improvement in the patient’s oxygenation?

A. IV heparin infusion

B. Broad-spectrum antibiotics

C. Blood transfusion

D. Corticosteroids
E. Furosemide

30. You have completed placement of a left subclavian central


venous catheter under ultrasound guidance in a mechanically
ventilated patient. You plan to perform ultrasonography of the
lung to determine if there is a pneumothorax after your
procedure. Which of the following ultrasonography findings
would be most useful in evaluation of the lung for a potential
pneumothorax?

A. Identification of the lung point

B. Absence of pleural sliding

C. Presence of A-lines

D. Absence of B-lines

31. A 32-year-old woman is admitted to the ICU with severe


respiratory failure due to H1N1 influenza with superimposed
bacterial pneumonia. She is placed on high-frequency oscillatory
ventilation because of refractory hypoxemia on conventional
ventilation. Her current settings are as follows: FIO2, 60%;
frequency, 6 Hz; amplitude, 68 cm H2O; airway pressure, 29 cm
H2O; inspiratory time, 33%; and bias flow, 35 L/min. Arterial
blood gas results show a pH of 7.13, Pao2 of 70mm Hg, and
Paco2 of 77 mmHg.

Which of the following interventions would be most appropriate


to address the increased Paco2?

A. Decrease frequency to 5 Hz

B. Increase frequency to 7 Hz
C. Decrease power by 5 cm H2O

D. Increase mean airway pressure to 34 cm H2O

32. A 24-year-old man develops severe respiratory failure with


acute respiratory distress syndrome after a near-drowning
episode. Because of refractory hypoxemia requiring high positive
end-expiratory pressure (PEEP) and FIO2 of 100% on pressure-
control ventilation, he is placed on high-frequency oscillatory
ventilation (HFOV) as salvage therapy. The patient tolerates
HFOV well for the first 24 hours. Oxygenation improves and the
current FIO2 is down to 60%. However, during the past hour
there has been an increase in the amplitude from 70 to 85 cm
H2O in order to maintain a good chest wiggle going to the mid-
thighs.

Which of the following interventions is most likely to improve


the increased amplitude requirements?

A. Bilateral chest tubes

B. Suctioning of respiratory secretions

C. Increased level of sedation

D. Deflation of endotracheal cuff

E. Disconnecting the patient from the ventilator

33. A 44-year-old man admitted to the ICU with severe acute


respiratory distress syndrome develops refractory hypoxemia
that is difficult to correct with conventional mechanical
ventilation. Salvage therapy with high-frequency oscillatory
ventilation (HFOV) is initiated. Which of the following
parameters should be increased in order to improve oxygenation
and wean down FIO2 in this patient on HFOV?

A. Amplitude

B. Frequency

C. Power

D. Bias flow

34. You are consulted regarding a 37-year-old man who presents


to the emergency department with increased dyspnea. As part of
your initial evaluation, you perform a bedside ultrasonography
of his right chest (see Figure).

Which of the following can be diagnosed from this


ultrasonogram?

A. Pneumothorax
B. Chest mass with diaphragm invagination

C. Pleural effusion

D. Pericardial effusion

E. Diaphragmatic paralysis

35. A 60-year-old, active man with known idiopathic pulmonary


fibrosis (IPF) is admitted to the hospital with rapidly increasing
shortness of breath. Although his IPF has been stable over the
last 2-and-a-half years, he now has had increasing dyspnea
during the past 2 weeks, with chest CT showing the rapid
progression of his disease. During his hospital stay, the patient
has a bronchoscopy with no growth of organisms on his
broncheoalveolar lavage. CT angiogram shows no evidence of
pulmonary embolism and echocardiogram shows normal
ejection fraction with no valvular abnormalities. The patient has
a mildly elevated serum brain natriuretic peptide (BNP) level.

Two days after admission, the patient is somnolent and hypoxic.


His family produces his living will, which says the patient would
not like artificial life support if no easily reversible cause of
respiratory failure is behind his clinical deterioration or if he
could not get a lung transplant.

Which of the following is most appropriate at this point?

A. Intubate the patient and treat him for pulmonary edema


because of his elevated BNP level.

B. Intubate the patient and transfer him to a tertiary care facility


that offers lung transplant.
C. Offer the patient and his family hospice because of his acute
exacerbation of idiopathic pulmonary fibrosis with known high
mortality.

D. Intubate the patient and treat him with high-dose steroids.

E. Start the patient on noninvasive ventilation and antibiotics for


possible subclinical infection.
PART 3: Respiratory Critical Care

ANSWERS: 1–C; 2–D; 3–A; 4–B; 5–C; 6–C; 7–E; 8–A; 9–C; 10–B;
11–B; 12–C; 13–C; 14–D; 15–D; 16–C; 17–E; 18–C; 19–D; 20–B;
21–B; 22–A; 23–C; 24–E; 25–C; 26–D; 27–A; 28–C; 29–C; 30–A;
31–A; 32–B; 33–D; 34–C; 35–C

RATIONALE (1) Answer: C

The goals during the intubation of this patient are to prevent


pulmonary aspiration, maintain cervical spine stability, and
minimize the risk of hypercapnia and hypoxemia. Because the
patient was admitted after a trauma, a full stomach must be
assumed. Fiberoptic-guided intubation requires time to
anesthetize the posterior pharynx with local anesthesia and will
also blunt any airway reflexes in response to ventilation. Mask
ventilation is not indicated in a patient with a full stomach unless
it is used as a rescue for a failed intubation. Although the
patient’s neurologic status has declined, it is unlikely that direct
intubation without Cervical collars often prevent the use of a
jaw-thrust maneuver that can aid in the visualization of the vocal
cords. With in-line manual stabilization, the stability of the
cervical spine can be maintained so that the cervical collar can be
removed, facilitating the endotracheal intubation.

RATIONALE (2) Answer: D

Angioedema that can cause a rapid and progressive airway


compromise can occur with the use of angiotensin-converting
enzyme (ACE) inhibitors. ACE inhibitors cause 25%–39% of
nonhereditary angioedemas. It has been suggested that the
incidence of angioedema with lisinopril is greater than with the
other ACE inhibitors. The exact pathophysiologic mechanism is
unknown, but it may involve inhibition of kinase activity with the
accumulation of tissue mediators. Up to 20% of patients may
present with acute dyspnea, dysphagia, dysphonia, and stridor.
As many as 22% of patients with angioedema induced by ACE
inhibitors require airway intervention, which may include
cricothyrotomy if attempts at direct visualization of the larynx
fail. Once the airway is secured, the angioedema is self-limiting
and resolves. There is no evidence that pharmacologic
intervention effectively treats this type of angioedema.

RATIONALE (3) Answer: A

Residual neuromuscular blockade is a common problem. All of


the above choices should be considered when determining if a
patient is suitable for extubation. The only direct measurement
of muscle strength is the assessment of twitch strength after
motor nerve stimulation. The most common method to
accomplish this is train-of-four stimulation of the ulnar nerve at
the wrist. With train-of-four stimulation, 4 stimuli at 2 Hz are
repeated every 10–12 seconds. A comparison of the strength of
the first twitch (T1) of the adductor pollicis brevis muscle is
made 2 seconds after the fourth twitch (T4). When the strength
of the first twitch is reduced to 90%, 80%, and 75% of the
maximum twitch height in the absence of neuromuscular
blockade, 1, 2, or 3 twitches will be elicited. At a T4 to T1 ratio of
0.75 (3 twitches), patients will manifest a 5-second head lift,
achieve a vital capacity of 15–20 mL/kg, have an effective cough,
and be able to generate an inspiratory force of –25 cm H2O.
However, upper esophageal sphincter tone is reduced at T4 to T1
ratios less than 90%, with the risk of aspiration.

An inspiratory force of –20 cm H2O (option B) may be


inadequate and is not a firm indicator for sufficient reversal of a
neuromuscular blockade to maintain a patent airway. An
adequate tidal volume on pressure support ventilation of 16 cm
H2O may be markedly reduced at lower levels of ventilatory
support. Thus, option C is incorrect. Patients may follow
commands and have respiratory rates within normal ranges
without complete return of muscle function after neuromuscular
blockade, so options D and E are also incorrect.

RATIONALE (4) Answer: B

It is important to know the indications as well as the


contraindications for noninvasive positive-pressure ventilation
(NPPV), and the respiratory conditions in which NPPV offers
benefit. Patients with chonic obstructive pulmonary disease
(COPD) exacerbations respond well to NPPV, with decreased
rates of intubation and improved mortality. Although depressed
mental status and inability to cooperate are typical
contraindications to NPPV, the patient described in option B has
only mild lethargy, which may be related to the increased carbon
dioxide, partly from over-oxygenation–induced increase in dead
space.

Recent studies also suggest that NPPV can be utilized effectively


in selected patients with a depressed level of consciousness.
Other contraindications to NPPV are hemodynamic instability, as
illustrated by the 55-year-old man with ST-elevation myocardial
infarction and pulmonary edema (option A). This patient is in
cardiogenic shock with a metabolic acidosis and should be
immediately intubated. The inability to control secretions,
vomiting, and hemoptysis (as in option D) preclude use of NPPV
because of aspiration risk. NPPV is unlikely to benefit the patient
with mild disease, as in the asthmatic patient in option C. In
asthma, NPPV should be reserved for patients with moderate to
severe asthma who fail to respond to initial therapy and who do
not qualify for immediate intubation. Several studies have shown
that NPPV does not prevent reintubation in patients with
postextubation respiratory failure (option E).

RATIONALE (5) Answer: C

The patient in this question was initially placed on noninvasive


positive pressure ventilation (NPPV) for treatment of an acute
chronic obstructive pulmonary disease (COPD) exacerbation.
After initiation with appropriate settings, the patient develops
hemodynamic changes that include a drop in blood pressure,
ECG changes suggesting ischemia, and a worsening clinical status
as evidenced by increased lethargy and increased hypoxemia.
Although the institution of NPPV is often beneficial for patients
with COPD exacerbations, it is important to recognize when to
proceed with endotracheal intubation and provision of
conventional mechanical ventilator support. Studies have shown
that delaying needed intubation can add to patient morbidity
and mortality.

Contraindications and factors that may indicate a high risk of


failure with NPPV include the following: respiratory arrest,
severe acid-base abnormalities, medical instability of the patient
(hemodynamic instability, uncontrolled gastrointestinal bleed,
acute myocardial ischemia, uncontrolled arrhythmias), inability
to protect airway, excessive secretions, uncooperativeness or
agitation, and recent upper airway or upper gastrointestinal
surgery.

This patient shows clinical deterioration, with evidence of


hemodynamic instability and myocardial ischemia. Therefore, he
is not a candidate to continue on NPPV and should be intubated
immediately. In addition, predictors of failures of NPPV in
patients with acute exacerbation of COPD include RR greater
than 35/min, Acute Physiology and Chronic Health Evaluation II
(APACHE II) score greater than 29, and Glasgow Coma Scale
score less than 11.

RATIONALE (6) Answer: C

The Acute Respiratory Distress Syndrome (ARDS) Network


clinical trial of low tidal volumes demonstrated significant
improvements in mortality when low-volume ventilation was
utilized in patients with ARDS.1 In that study, patients were
given a tidal volume of 6 mL/kg of predicted body weight (PBW).
The formulas for PBW are given below:

Male = 50 + 2.3 [height (inches) – 60] or 50 + 0.91 [height (cm) –


152.4]

Female = 45.5 + 2.3 [height (inches) – 60] or 45.5 + 0.91 [height


(cm) – 152.4]

It is important to utilize the PBW when calculating tidal volume


for patients with ARDS, because patients who are volume
overloaded or who have a high body mass index may have a
much higher actual body weight than a PBW. In those cases, the
use of an actual weight may provide deleterious tidal volumes to
the patient. In order to calculate the PBW, one must know the
patient’s height.

RATIONALE (7) Answer: E

This flow-over-time waveform demonstrates the presence of


auto positive end-expiratory pressure (auto-PEEP), signified by
significant expiratory flow still occurring at end expiration (see
Figure, arrow 2). In addition, it demonstrates a dramatic drop in
flow rate that occurs near the end of expiration which is not
associated with inspiratory triggering (see Figure, arrow 1). This
occurs because the patient has made a significant inspiratory
effort, creating a negative intrapleural pressure and therefore
decelerating the expiratory flow rate. The patient’s effort,
however, does not trigger the next inspiration and subsequently
subsides, followed by an unassisted ventilator triggering based
on time interval from last inspiration.

Since the patient must first overcome the auto-PEEP before


encountering the normal triggering mechanisms, the patient’s
work of breathing may be offset to some degree by increasing
the set ventilator PEEP, because the degree of inspiration effort
to trigger is decreased as the difference between set PEEP and
total PEEP decreases. This occurs because the ventilator now
triggers from the set PEEP level (plus some change in pressure
or flow) instead of from atmospheric pressure. Decreasing the
peak inspiratory flow rate (option A) in a patient on volume
ventilation would have no effect on this cause of dyssynchrony,
but in fact would worsen the auto-PEEP by prolonging
inspiration. The inspiratory flow waveform is already set as
decelerating flow, so option B is incorrect. Switching to a square
inspiratory flow waveform would, however, improve auto-PEEP,
as it would shorten the inspiratory time. Increasing the set
ventilator rate to 20/min (option C) would also have no effect in
this patient, as she is already breathing at a rate of 20/min. If the
rate was increased higher than 20/min, it would exacerbate the
auto-PEEP by further decreasing expiratory time. Positioning the
patient left side down would have no predictable effect on this
clinical problem, so option D is incorrect.
RATIONALE (8) Answer: A

The pressure-over-time waveform shown for this patient is


typical for volume ventilation with rising pressure throughout
inspiration. The waveform, however, exhibits a decay in pressure
after the onset of inspiration (see Figure, arrow A). This drop in
pressure is related to significant inspiratory effort occurring
after inspiration is triggered. This typically occurs when a
patient is not satisfied with the inspiratory flow rate setting.

This is particularly problematic in young, healthy individuals


with good respiratory muscle strength who are in respiratory
distress. Increasing the peak inspiratory flow rate (PIFR) should
accommodate the patient’s desires and decrease dyssynchrony
(see Figure, arrow B). Note that in addition to smoothing out the
inspiratory flow waveform, the peak inspiration pressure also
rises with the higher rate. However, tidal volume and inspiratory
plateau pressure do not change.

Switching to a decelerating inspiratory flow waveform (option B)


will likely worsen dyssynchrony, as the PIFR will steadily
decrease after onset of inspiration. Increasing the set ventilator
rate (option C), unless it is increased higher then 30/min
(current assisted rate), would have no effect on any parameters,
and if increased above 30/min, it would have no effect on
dyssnychrony. Although this patient’s positive end-expiratory
pressure might need to be increased, it is not a reason for the
dyssynchrony, and with a FIO2 of 0.6 and oxyhemoglobin
saturation of 96%, this is not a critical issue. Therefore, option D
is incorrect. Decreasing the positive end-expiratory pressure to
less than 10 cm H2O with an FIO2 of 0.6 is probably not an
appropriate intervention. Thus, option E is incorrect.

RATIONALE (9) Answer: C

The arrow on the flow-time diagram shown in the Figure below


indicates intrinsic positive end-expiratory pressure (or auto-
PEEP).
Intrinsic PEEP is present when the end-expiratory lung volume
represents a volume at which air would normally continue to
escape if there was no impediment to expiration. When the
expiratory time is insufficient to allow full exhalation of a
ventilator breath, expiratory flow is still occurring when the next
ventilator breath is delivered (arrow). Since the expiratory flow
does not reach zero, there is also a pressure gradient at end-
expiration. Therefore, PEEP exists even when it is not set on the
ventilator (or in excess of what is set on the ventilator). The
presence or absence of intrinsic PEEP may be confirmed by
checking graphic flow waveforms on the ventilator. Since severe
bronchospasm increases the necessary expiratory time, patients
with either status asthmaticus (as in this case scenario), or
severe chronic obstructive pulmonary disease exacerbation are
at risk for intrinsic PEEP. The patient in this case scenario should
be ventilated with a strategy to limit intrinsic PEEP by
decreasing inspiratory time relative to expiratory time. This is
best accomplished by decreasing the respiratory rate, giving the
patient more time to exhale between breaths. Decreasing the
tidal volume will also result in a small improvement in intrinsic
PEEP, but to a much lesser degree than decreasing the rate. With
volume-cycled ventilation, the inspiratory time is also dependent
on the peak inspiratory flow rate, which determines how fast a
breath will be delivered. The choice of inspiratory flow
waveform also influences the inspiratory to expiratory ratio.
When providing volume-cycled ventilation in the presence of a
bronchospasm-induced intrinsic PEEP, the peak inspiratory flow
rate should be set at 80–100 L/min, with a square waveform in
order to limit the inspiratory time, leaving more time in the
respiratory cycle for expiration. However, it must be emphasized
that the inspiratory to expiratory ratio should not be the primary
goal in managing intrinsic PEEP. Rather, the chief determinant of
intrinsic PEEP is the absolute time for expiration of each breath.
That is why decreasing the respiratory rate is the critical
intervention.

RATIONALE (10) Answer: B

This patient has an endotracheal tube obstruction. The key to the


identification of this problem relates to an understanding of the
components of peak inspiratory pressure and inspiratory
plateau pressure. Peak inspiratory pressure is created by the
pressure required to overcome resistance in the endotracheal
tube and airways, as well as the elastance of the lung
parenchyma, pleural space, and chest wall. Inspiratory plateau
pressure is pressure generated to overcome the elastance of the
lung parenchyma, pleural space, and chest wall and is not related
to endotracheal tube and airways resistance. In this patient, it is
noted that peak inspiratory pressure is steadily rising between
day 1 and day 4, whereas inspiratory plateau pressure remains
constant. This is compatible with increasing endotracheal tube
obstruction, in this case due to inspissated mucus. Flash
pulmonary edema and tension pneumothorax (options A and C,
respectively) would be associated with increases in both peak
inspiratory pressure and inspiratory plateau pressure.
Pulmonary embolus (option D) might not produce any changes
in inspiratory pressure, but if it produced significant
parenchymal lung problems, it would be associated with the
same pattern as flash pulmonary edema or tension
pneumothorax.

RATIONALE (11) Answer: B

Mechanical ventilation is an essential therapeutic intervention in


patients with acute respiratory failure. However, there are
several complications associated with prolonged mechanical
ventilation. Therefore, liberating patients from mechanical
ventilation as their acute illness resolves is an important
challenge for intensivists. Our understanding of weaning from
mechanical ventilation has evolved thanks to several clinical
trials. Today it is apparent that when assessing for readiness of
weaning, clinicians should use liberal oxygenation criteria.
Furthermore, the routine use of weaning criteria (predictors)
and arterial blood gas results is usually not required. In patients
who appear to be ready for weaning, the rapid shallow breathing
index (RSBI = respiratory frequency ÷ tidal volume) is one
weaning predictor that can quickly assess a patient’s readiness.
When done on continuous positive airway pressure, it can
identify patients who are unlikely to do well (RSBI >105). If it is
less than 105, it does not predict success but suggests that a
patient should proceed to a spontaneous breathing trial (SBT).
Based on results from clinical studies, SBTs are best done on
pressure support ventilation of 5 cm H2O or via T-piece and
should last from 30 to 120 minutes. One study found that the
median time to weaning trial failure in patients with chronic
obstructive pulmonary disease on mechanical ventilation for 15
or more days was 120 minutes. Based on this finding, it may be
appropriate in some patients to extend the SBT to 120 minutes.
The patient in this case appears to be ready for an SBT. Based on
the provided information, the best option is to proceed with SBT
on pressure support ventilation of 5 cm H2O for 120 minutes. If
the patient tolerates this trial, she should be extubated.

RATIONALE (12) Answer: C

About 20%–30% of patients are difficult to wean from invasive


mechanical ventilation. The pathophysiology leading to weaning
failure is complex and several causes contribute to difficult
weaning. In this particular case, cardiac dysfunction is the most
likely cause of the patient’s failure to wean. The transition from
mechanical ventilation to spontaneous breathing can impose a
significant strain on the cardiovascular system. Changes in
intrathoracic pressures during this transition produce changes in
cardiac preload and afterload. In addition, the increased oxygen
consumption of the respiratory muscles can contribute to the
increased strain on cardiovascular performance. In patients with
chronic obstructive pulmonary disease, weaning from
mechanical ventilation can be associated with decreased left
ventricular ejection fraction (probably due to increased
afterload), and reduced myocardial compliance. Elevated end-
diastolic left ventricular pressures can produce bronchial wall
edema and increase the work of breathing. Clinically, the
combined increase in arterial pressure and heart rate during
unsuccessful weaning is quite suggestive of weaning failure from
cardiac dysfunction. Patients with cardiac dysfunction during
weaning often have decreased mixed venous oxygen saturation
(Svo2). A normal Svo2 with no changes during a failed
spontaneous weaning trial almost rules out cardiac dysfunction
as the cause for the failure. Patients with cardiac dysfunction
during weaning can also show increased brain natriuretic
peptide levels in blood. Signs of weaning-induced
hemoconcentration, such as increased protein blood levels, are
highly suggestive of hydrostatic pulmonary edema. The ECG in
this case did not show significant changes. However, ST changes
and T-wave inversions during a failed weaning trial suggest
cardiac dysfunction/ischemia. This patient has various clinical
signs suggesting weaning failure from cardiac origin. Further
evaluation can be done with echocardiography and/or
pulmonary artery catheter during weaning trials. In some
instances, evaluation for possible underlying cardiac ischemia
may be warranted. Treatment of cardiac dysfunction may
include afterload reduction, inotropes, and in some cases
revascularization.

The patient shows no clinical evidence of acute delirium (option


A is incorrect). Inspiratory muscle fatigue is extremely unlikely
in a patient who fails a weaning trial yet has a normal Paco2 as in
this case (option D is incorrect). Finally there are no clear clinical
signs suggesting increased airway resistance or endocrine
dysfunction. However, in the absence of a strong suspicion for an
alternative cause, the clinician should have a low threshold to
work up potential endocrine causes of weaning failure
(hypothyroidism and adrenal insufficiency).

RATIONALE (13) Answer: C

This patient with known systemic lupus erythematosus (SLE)


has developed a clinical picture most consistent with acute
diffuse alveolar hemorrhage (DAH). DAH is characterized by the
clinical constellation of hemoptysis (not always present),
anemia, diffuse radiographic pulmonary infiltrates, and
hypoxemic respiratory failure. DAH associated with SLE most
often presents with a histological pattern of pulmonary
capillaritis. In up to 30% of patients, hemoptysis is absent.
Diagnosis can be established with bronchoscopy, after sequential
bronchoalveolar lavage reveals increasing red blood cell counts
or the presence of hemosiderin-laden macrophages.
Methylprednisolone is recommended for the initial treatment of
DAH related to SLE. Pulse-dose steroids (methylprednisolone, 1
g in divided doses for 3 days) can be used to treat DAH or acute
lupus pneumonitis (also a consideration in this case). In patients
not responding to steroids, the addition of cyclophosphamide or
other treatments such as azathioprine or IV immunoglobulin
should be considered. In cases refractory to steroids,
plasmapheresis has been used with limited success. Although
patients with immunosupression could be at risk for
Pneumocystis carinii pneumonia, this does not seem a likely
diagnosis in this patient; hence trimethoprim/sulfamethoxazole
(option A) is incorrect. The picture is not consistent with
pulmonary edema, making furosemide incorrect (option B).
Factor VIIa (option D) has been proposed as a possible rescue
therapy in patients with severe DAH due to microscopic
polyangiitis with unremitting respiratory failure. However, it is
not considered a first-line therapy in patients with DAH and SLE.
Finally, rituximab (a monoclonal antibody) has been utilized in
patients with antineutrophil cytoplasmic autoantibody–
associated vasculitis that is refractory to standard therapy. This
patient was treated with methylprednisolone, 1 g/day for 3 days,
with good clinical response (see figure 13b, radiograph taken 2
days after initiating steroids).
Figure 13b

RATIONALE (14) Answer: D

This patient’s clinical picture is consistent with diffuse alveolar


hemorrhage (DAH) and acute renal failure. In a previously
healthy young male, this presentation of an acute pulmonary-
renal syndrome is highly suggestive of Goodpasture syndrome
(antiglomerular basement membrane disease). Goodpasture
syndrome is a classic pulmonary-renal syndrome with high
mortality if untreated. The disease only affects the lungs and the
kidneys. DAH is a common manifestation, especially in cigarette
smokers. Patients with Goodpasture tend to be males in their 20s
who also smoke. More than 90% of patients with Goodpasture
syndrome have circulating antibasement membrane. A positive
titer for the antibasement membrane antibody would confirm
the diagnosis in this patient and would be most helpful in
dictating therapy (option D is correct). Although a bronchoscopy
with bronchoalveolar lavage would help establish the diagnosis
of alveolar hemorrhage, it would not provide a specific
underlying diagnosis linked to therapy. Antineutrophil
cytoplasmic autoantibody titer (option C) is helpful in
differentiating DAH due to vasculitis such as Wegener
granulomatosis from microscopic polyangiitis. However, the
clinical presentation of this patient is not consistent with those
diseases and clearly fits the profile of Goodpasture syndrome.
There is a small percentage of patients with Goodpasture
syndrome in whom the antibasement membrane antibody titer
may be negative. In these patients, a renal or lung biopsy with
immunofluorescence revealing linear antibody deposition within
the alveolar or glomerular basement membrane confirms the
diagnosis. A bone marrow biopsy (option E) would not be helpful
in the diagnosis of Goodpasture syndrome. Patients with
Goodpasture syndrome should be treated with
immunosuppression (cyclophosphamide and steroids) and
plasmapheresis. Early renal replacement therapy should also be
considered. With early dialysis, plasma exchange and
immunosuppression, mortality decreases significantly.

RATIONALE (15) Answer: D

This patient most likely has vocal cord motion disorder or vocal
cord dysfunction. This is a syndrome that commonly presents as
asthma. During the respiratory cycle, the vocal cords normally
partially abduct with inhalation and partially adduct with
exhalation. This physiological movement allows the unimpeded
movement of air inward to the lungs and outward to the
atmosphere, while maintaining alveolar patency. In paradoxical
vocal cord motion disorder (PVCM) or vocal cord dysfunction,
the vocal folds will adduct during inhalation and/or exhalation.
This typically leads to episodic or recurrent episodes of dyspnea,
wheezing, and/or stridor. Because of these manifestations, PVCM
is often confused with asthma and patients receive steroid
treatment without symptom relief. Rarely, this can lead to
intubation and mechanical ventilation for “refractory asthma.”
The gold standard for diagnosing PVCM is laryngoscopy.

RATIONALE (16) Answer: C

Bleeding is one of the most important complications associated


with percutaneous and surgical tracheostomies. Early bleeding,
within days of the procedure, is more likely caused by trauma
during the procedure itself and usually resolves with minimal
intervention aimed at correcting any existing coagulopathy and
proper local hemostasis. Bleeding can also result from more
serious complications such as an arteriotracheal fistula. There
are 2 different mechanisms by which a tracheostomy can lead to
an arteriotracheal fistula. The first is erosion of the curved part
of the tracheostomy through the artery beneath it; the second is
erosion by the tip or the cuff of the tracheostomy through the
anterior wall of the trachea. Arteriotracheal fistulas (tracheo-
innominate artery fistula, tracheo-brachiocephalic artery fistula)
are a rare but fatal complication, occurring in 0.7% of
tracheostomies. Mortality is attributed to acute hemorrhage,
which classically occurs 2–4 weeks after the procedure. A so-
called “sentinel bleed,” a milder bleeding episode that occurs
prior to the acute hemorrhage, has also been described. It is
important for clinicians to recognize the potential for
devastating bleeding complications in patients with
tracheostomy. If bleeding is profuse and suspected to be from an
arteriotracheal fistula, the only chance for survival is immediate
occlusion of the bleeding site (usually done by placing a finger
directly on the bleeding site inside the trachea) en route to the
operating room for emergent surgical repair.
RATIONALE (17) Answer: E

The patient’s clinical presentation is most consistent with acute


pulmonary embolism (PE). The patient has significant risk
factors for PE (recent surgical intervention/hospitalization and
cancer). Ideally, thromboembolic disease should be confirmed
prior to initiating treatment. However, in situations with a high
clinical suspicion, it is appropriate to initiate anticoagulation if
there are no clear contraindications pending a definitive
diagnosis. CT angiography is commonly the initial diagnostic test
when pulmonary embolism is suspected. In this case, the
increased creatinine would preclude performing a CT
angiography (choice A is incorrect). Low-molecular-weight
heparin (LMWH) and unfractioned heparin are the first-line
agents for anticoagulation. Long-term anticoagulation with
LMWH is preferred in cancer patients with thromboembolic
disease. With renal failure, the dose of enoxaparin should be
reduced to 1 mg/kg every 24 hrs, so option C is incorrect.
Argatroban is a direct thrombin inhibitor that is utilized in
patients with heparin-induced thrombocytopenia (HIT). There
are no clinical data documenting efficacy of thrombin inhibitors
as first-line therapy for PE and they should not be utilized
outside of the context of HIT (option D is incorrect). For this
particular patient, the best option would be to initiate
anticoagulation with unfractioned heparin. Treatment with
unfractioned heparin should be based on the patient’s actual
weight, not his ideal weight, so option B is incorrect. The correct
answer is option E, starting unfractionated heparin with a bolus
of 80 U/kg (patient’s weight of 120 kg = 9600-U bolus) followed
by 18 U/kg/h (patient’s weight of 120 kg = approximately 2100
U/h). The dosing should be titrated to activated partial
thromboplastin time or heparin levels.

RATIONALE (18) Answer: C


The results on the echocardiogram identify this patient as having
a higher risk for complications and mortality. Echocardiography
has an important role in the management of patients with
pulmonary embolism (PE). Patients with documented PE and
echocardiographic findings consistent with right ventricular
strain/failure (right ventricular [RV] dilation, RV decreased
contractility, abnormal movement of the interventricular septum
into the left ventricle) have a higher risk of death. Early
identification of these patients can aid clinicians in triage
decisions and prognostication. Studies have shown that when
patients with PE have echocardiographic signs of RV dysfunction,
short-term, all-cause mortality and PE-related mortality increase
compared to patients without evidence of RV dysfunction (11%
versus 0.5%). Triage to the ICU of high-risk patients with PE and
RV dysfunction makes sense if one considers the increased risk
of death and potential decompensation. Although some have
proposed that RV dysfunction could be a potential indication for
the use of systemic thrombolytics, there is no clinical trial data to
support this practice (option B is incorrect). Similarly, findings of
RV dysfunction at present are not utilized in the decision process
of placing inferior vena cava filters in patients with PE (option D
is incorrect). Finally, option A is incorrect because the decision to
initiate anticoagulation is based on clinical suspicion and the
findings of CT angiography. In the presence of PE,
echocardiographic findings of RV dysfunction are prognostic.
However, RV dilation is not diagnostic of PE and can also occur in
patients with chronic pulmonary hypertension (cor pulmonale)
from other causes.

RATIONALE (19) Answer: D

Proper anticoagulation is the basis for treatment in patients


diagnosed with pulmonary embolism. Traditionally unfractioned
heparin (UF) as a continuous IV infusion has been utilized in
critically ill patients with PE. Low-molecular-weight heparins
(LMWHs) are an alternative to UF. In clinical trials LMWHs have
been shown to be non inferior to UH for treatment of deep vein
thrombosis and PE. However, these studies did not include large
numbers of critically ill patients. Moreover, uncertainty exists
regarding the subcutaneous dosing route for critically ill patients
with PE. LMWHs are administered via the subcutaneous route.
Subcutaneous LMWH leads to lower plasma drug levels,
measured as anti-factor Xa activity, in surgical critically ill
patients with or without shock when compared to non-ICU
patients despite receiving the same dosing regimen (option D is
correct). LMWH have a lower incidence of causing heparin
induced thrombocytopenia (HIT) than unfractioned heparin
(UH). However, patients treated with UF with probable or
confirmed HIT should not be changed to LMWH (option C is
incorrect). Both UF and LMWH should be avoided in these
circumstances. If there is a need for anticoagulation, patients
with HIT should be changed to a thrombin inhibitor (argatroban
or lepirudin). LMWH can be utilized in patients with renal
failure. In patients with a creatinine clearance < 30 ml the dose
and frequency should be adjusted (option A is incorrect).

RATIONALE (20) Answer: B

In a patient with diagnosed pulmonary embolism, development


of hypotension heralds a higher risk of death. The bedside
ultrasonography reveals a dilated right ventricle; as can be seen
on the apical 4-chamber view provided, the right ventricle is
almost the same size as the left ventricle. Although it is not
possible to determine by echocardiography if existing right
ventricular dysfunction is acute or chronic, in the setting of this
case it is most likely acute and due to the pulmonary embolism.
Treatment of hypotension in the context of pulmonary embolism
should take into consideration the underlying pathophysiology.
A large clot burden will produce an acute rise in pulmonary
vasculature pressures, which result in acute right ventricular
strain (dilation of the right ventricle on echocardiography with
decreased contractility). As the right ventricular cardiac output
drops, so will blood flow to the left ventricle. The under-filled left
ventricle will also drop its cardiac output, resulting in arterial
hypotension with a reflex increase in systemic vascular
resistance. The first attempt to break this cycle shoud be the
judicious administration of IV fluids with the aim of increasing
venous return to the right ventricle and increasing its cardiac
output. In cases where fluids do not improve blood pressure
quickly, vasopressor support may be needed. Although there are
no clinical studies evaluating vasopressors in pulmonary
embolism, some experts recommend norepinephrine as the
agent of choice (based on animal data). Because the principal
problem is related to the clot burden in the pulmonary
vasculature, current guidelines recommend the use of systemic
thrombolytics in patients with pulmonary embolism and shock.
Thrombolytic therapy can lead to rapid improvement of
hemodynamic alterations seen with massive pulmonary
embolism. In this case, the first step would be the administration
of volume. If the patient does not respond, support with
vasopressors and administration of systemic thrombolytics
would be warranted. Option A is incorrect because there is no
evidence of pericardial effusion in this case.

RATIONALE (21) Answer: B

This patient’s presentation is most consistent with acute


respiratory distress syndrome (ARDS) due to pneumonia
(bacterial and/or viral). The cornerstone of mechanical
ventilation for patients with ARDS or acute lung injury is the
prevention of ventilator-induced lung injury through a protective
lung strategy. The landmark study by the ARDSNet group1
demonstrated that low tidal volume (6 mL/ kg of predicted body
weight) with limitation of plateau airway pressures to less than
30 cm H2O was associated with significant improvement in
patient mortality. Based on this important clinical trial and many
other studies, current recommendations for patients with ARDS
use a low tidal volume calculated from the patient’s predicted
body weight. It is essential for clinicians to understand that lung
size does not increase proportionally with patient’s weight.
Average lung size is determined by the patient’s height and sex.
Therefore, when starting mechanical ventilation for ARDS, one
must use a predicted body weight and not the actual body
weight. Especially in patients with high body mass indexes, the
use of actual weight can lead to large and harmful tidal volumes.
Therefore, option A is incorrect because the tidal volume is
based on actual weight. Finally, high-frequency oscillatory
ventilation and airway pressure release ventilation are
nonconventional modes of ventilation. To date there are no
clinical studies showing that either one is superior to
conventional ventilation with low tidal volume. Therefore, they
are reserved for salvage therapy in cases where conventional
ventilation with adequate lung protective settings has failed.
They would not be utilized as the initial settings in a case such as
the one presented in this question; therefore, options C and D are
incorrect.

RATIONALE (22) Answer: A

Point A corresponds to what is referred to as the lower inflection


point on the inspiratory limb of the pressure-volume curve.
Positive end-expiratory pressure (PEEP) and FIO2 are the
principal tools on the ventilator that can be adjusted to improve
oxygenation. When PEEP produces alveolar recruitment, shunt is
decreased and Pao2 improves. Furthermore, when set above the
lower inflection point, PEEP will prevent the collapse of alveoli at
the end of expiration. By decreasing the opening and closing of
alveoli, PEEP can provide protection to the lung against
increased injury from shearing forces (atelectrauma). This
concept is best understood by evaluating the pressure-volume
curve. However, in practice, setting the best PEEP may be more
difficult. Currently, clinicians set PEEP based on tables utilized in
clinical trials by ARDSNet (www.ardsnet.org). Optimal PEEP can
also be set at the bedside by increasing PEEP in 2- to 3-cm H2O
increments and finding the level that provides the best arterial
oxygen saturation (>88% and <95%) with the lowest possible
FIO2. Many experts believe that higher levels of PEEP are best
for patients with ARDS. The rationale behind this process is
based on understanding the advantages of setting PEEP above
the lower inflection point and the ability to decrease FIO2 with
higher PEEP. Three clinical trials tested modest versus high
levels of PEEP in ARDS and failed to show conclusive benefits in
patient survival. These studies did report improved Pao2 to FIO2
ratio in patients treated with higher PEEP. Two of these studies
also reported decreased rates of refractory hypoxemia, lower
rate of death with refractory hypoxemia, and decreased use of
rescue therapies in the patients treated with higher PEEP. A
PEEP setting of 8–15 cm H2O is probably a safe starting point for
most patients with ARDS. There is still no evidence as to the best
way to select PEEP in patients with ARDS. For the time being
clinicians must evaluate the benefits and risks of high PEEP in
each individual patient. Point B on the pressure-volume curve
corresponds to the upper deflection point. Ventilation beyond
this point would likely cause overdistension of alveoli and lead to
volutrauma. Points C and D do not have any particular
significance.
RATIONALE (23) Answer: C

This patient is being mechanically ventilated with airway


pressure release ventilation (APRV). APRV is a newer
nonconventional mode of ventilation that has been proposed as
an alternative for an open-lung mechanical ventilation strategy.
APRV is thought to provide the benefits of a near-permanent
recruitment maneuver with the ability to augment ventilation in
patients with poor lung compliance. APRV can be thought of as
continuous positive airway pressure with regular brief releases
in pressure. APRV is a time-cycled, pressure-limited mode of
mechanical ventilation that cycles between a high pressure
(determined by Phigh and Thigh) and a shorter lower pressure
or release pressure (determined by Plow and Tlow)Oxygenation
depends on the FIO2, the set high airway pressure (set by Phigh),
and the time Plow. this high pressure-phase will last
(determined by Thigh). The release phase determines ventilation
(carbon dioxide dump) and is determined by the difference
between Phigh and Plow (Plow is usually set at 0), the time spent
in the low-pressure phase (determined by Tlow), and
spontaneous breathing by the patient throughout the whole
respiratory cycle.

RATIONALE (24) Answer: E

The main cause of mortality in neuromuscular diseases such as


myasthenia gravis is respiratory failure and pneumonia. Close
monitoring of respiratory muscle strength with serial
measurements of vital capacity and maximum negative
inspiratory force can help identify impending respiratory failure.
Usually, rapidly declining values or a vital capacity less than 1 L
indicate the need for mechanical respiratory support.
Noninvasive modalities are definitely helpful in treating patients
with neuromuscular disease–induced respiratory failure.
Normally bilevel positive airway pressure is required. However,
when there is evidence of inability to protect the airway as in our
patient, respiratory support with endotracheal intubation or
tracheostomy is required. Intubation for mechanical ventilation
may also be required if the patient does not respond to
noninvasive positive-pressure ventilation.

RATIONALE (25) Answer: C

This patient presented to the emergency department with an


exacerbation of chronic obstructive pulmonary disease (COPD)
triggered by a community-acquired pneumonia. Noninvasive
positive-pressure ventilation (NPPV) is a valid therapeutic
option in acute COPD exacerbations. In patients with COPD who
are able to cooperate and protect their airway and are
hemodynamically stable, a trial of NPPV is certainly warranted.
In patients with acute COPD exacerbations, NPPV can often
prevent intubation and mechanical ventilation. One could argue
that in this patient with pneumonia and productive cough, NPPV
may not be the best option. Hence, one of the most important
aspects of utilizing NPPV is understanding when to transition to
more aggressive respiratory support with intubation and
conventional mechanical ventilation. To this effect, what many
experts have called the “1-hour check” is extremely important. If
patients are not steadily improving after 1–2 hours of effective
NPPV, the clinician should strongly consider moving to
intubation and mechanical ventilation. In this patient, not only
do the arterial blood gas results fail to show any substantial
improvement but more importantly his clinical condition has
deteriorated. The patient is more lethargic and has developed
hypotension and tachycardia with multifocal tachycardia on ECG.
At this point, adjusting the NPPV settings is unlikely to lead to
significant clinical improvement as it seems that the patient is
developing severe sepsis and perhaps shock from his
pneumonia. At this point, the best option is to intubate the
patient and control the airway before the patient becomes more
unstable. Contraindications to the use of NPPV include
hemodynamic instability (shock, uncontrolled cardiac ischemia,
or arrhythmia), upper gastrointestinal bleeding,
agitated/uncooperative patient, inability to protect airway,
swallowing impairment, excessive secretions, and recent upper
airway or gastrointestinal surgery.

RATIONALE (26) Answer: D

This patient with a severe asthma exacerbation has likely


developed dynamic hyperinflation of the lungs (auto-positive
end-expiratory pressure) leading to increased intrathoracic
pressure and hypotension (from decreased cardiac output).
Although tension pneumothorax is certainly a consideration in a
mechanically ventilated patient with severe asthma, dynamic
hyperinflation is more likely on the basis of the course of events
in this case. In patients with severe airway obstruction,
exhalation time is prolonged. It may be so prolonged that all the
tidal volume may not be exhaled in the allotted time and, as a
new breath is delivered, the patient stacks breaths with
progressive increase in lung volume at the end of expiration
(functional residual capacity [FRC]). Since asthmatics usually
start with a higher baseline FRC, if dynamic hyperinflation
occurs, FRC may be pushed close to total lung capacity and can
produce severe hemodynamic instability. The hemodynamic
profile of dynamic hyperinflation is similar to that seen with
tension pneumothorax or acute cardiac failure (increased central
venous and pulmonary wedge pressures, decreased cardiac
output, and hypotension). The mistake in the management of this
case was to continue to decrease exhalation time (increasing RR)
in order to correct the high carbon dioxide level. . Mild to
moderate elevations in carbon dioxide are usually well tolerated
by patients on mechanical ventilation (permissive hypercapnia).
In a patient with asthma on mechanical ventilation, special
attention should be directed to airway pressures and preventing
dynamic hyperinflation. If she is disconnected from the
ventilator, the patient will have enough time to exhale the
accumulated tidal volume; this should result in improved
hemodynamics. In this case, the pressure immediately recovered
and the heart rate decreased. When the patient was placed back
on the ventilator, both the tidal volume (set to high initially) and
the rate were decreased. Decreasing the rate, although
eventually needed, would not take precedence over
disconnecting the patient from the ventilator in this case because
of the severe hemodynamic compromise and the need to act
emergently

RATIONALE (27) Answer: A

This patient has most likely developed negative-pressure


pulmonary edema (NPPE). NPPE may occur in the acute setting
in association with laryngospasm and other upper-airway
obstructive processes such as epiglottitis, near drowning, and
strangulation. It is not a frequent complication of general
anesthesia but can occur, especially in young, healthy adults after
surgery. NPPE has also been described in chronic conditions
such as obstructive sleep apnea, nasopharyngeal mass, and
tonsillar enlargement. Clinical findings of NPPE include cough,
dyspnea, tachypnea, tachycardia, hypoxemia, frothy secretions,
and in rare instances, frank pulmonary hemorrhage. NPPE is
usually triggered by laryngospasm causing inspiration against a
closed glottis, inducing highly negative intrathoracic pressures
with a concomitant increase in venous return. The consequence
of this phenomenon is increased pulmonary blood flow with
increased hydrostatic pressure leading to noncardiogenic
pulmonary edema. This is usually a selflimited phenomenon and
resolves in 24 hours with appropriate supportive care. There is
no evidence suggesting that corticosteroids or diuretics will
accelerate recovery. The clinical symptoms and findings of NPPE
are nonspecific and should lead the clinician to a broad
differential diagnosis. However, when these symptoms occur in
healthy young adults in the perioperative period as in this case,
NPPE should be highly suspected.

RATIONALE (28) Answer: C

This patient was admitted with severe status asthmaticus and


now is clinically deteriorating. The changes in his chest
radiograph and fevers suggest a new acquired pulmonary
process. The findings on the sputum are highly suggestive of
Aspergillus fumigatus. The potential for invasive pulmonary
aspergillosis (IPA) complicating acute severe asthma should be
considered. The risk of developing IPA is highest among patients
with neutropenia, hematological malignancy, or following bone
marrow or solid organ transplant. However, the combination of
respiratory disease (asthma, chronic obstructive pulmonary
disease) and corticosteroids also increases the risk of developing
IPA. Current guidelines recommend early initiation of antifungal
therapy in critically ill patients with suspected IPA while
diagnostic evaluation is conducted. Early empiric treatment is
warranted considering the high mortality of IPA in critically ill
patients. For primary treatment of IPA, voriconazole is
recommended as firstline agent. In seriously ill patients, the
parenteral formulation is recommended. There is no indication
to change vancomycin to linezolid. Fluconozale is not
recommended for the treatment of Aspergillus/IPA. There is no
evidence suggesting Pneumocystis carinii pneumonia (PCP), so
adding trimethoprim/sulfamethoxazole is not warranted.
Finally, waiting for confirmation of cultures would delay
treatment and probably have a negative impact on the patient’s
outcome considering the severity of the current clinical situation.
Further diagnostic tests with cultures obtained from
bronchoalveolar lavage, serological testing for Aspergillus
(antibody titers), and CT imaging can help establish the
diagnosis. Patients may acquire Aspergillus species either in the
ICU or prior to entering it. Patients with asthma are commonly
colonized by Aspergillus fumigatus. Fifty-two percent of critically
ill patients with Aspergillus in respiratory tract samples show no
evidence of invasive disease and are thought to be colonized,
whereas the remaining 48% have IPA.

RATIONALE (29) Answer: C

The patient’s clinical presentation is most consistent with acute


chest syndrome (ACS). ACS is a serious complication of sickle cell
disease that is defined by the presence of a new pulmonary
infiltrate on chest radiograph with chest pain, tachypnea,
tachycardia, dyspnea, fever, hypoxia, crackles, and/or wheezes.
The most common causes of ACS are fat emboli from the bone
marrow and infections. It is proposed that in ACS the injured
lung is unable to reoxygenate sickled red blood cells. This leads
to activation of the endothelium with aggregation of red blood
cells in the pulmonary microvasculature, resulting in increased
lung inflammation and potential lung infarcts. The mainstay of
treatment in ACS is hydration, oxygen therapy, and analgesia.
Because infections are common triggers of ACS, antibiotic
coverage is recommended. Multiple studies have shown that
blood transfusions can rapidly improve oxygenation and have a
positive impact on the course of ACS. The role of exchange
transfusions in ACS is not as clear. However, they should be
considered in critically ill patients with deteriorating
cardiopulmonary status because they may improve outcomes.
The enlarged right ventricle in this patient is due to chronic
pulmonary hypertension associated with her sickle cell disease.
There is no indication that the patient has pulmonary embolism,
hence heparin is unlikely to help. Although antibiotics are
indicated in ACS, they are unlikely to produce rapid
improvement in oxygenation. Corticosteroids and furosemide
have not been shown to be effective in the treatment of ACS.

RATIONALE (30) Answer: A

The use of bedside ultrasonography as a point-of-care diagnostic


tool is rapidly gaining acceptance in the ICU. Within the growing
number of applications for critical care ultrasonography,
evaluation of the lung is a relatively novel one. Normally,
ultrasound is not transmitted through anatomical structures
filled with gas, and the lung parenchyma is not visible beyond
the pleura. The injured lung produces characteristic artifacts
resulting from the abnormal gas-tissue interface. Recent studies
have described these artifacts and their utility in evaluating
several pathological conditions in the lung.

One of the first structures one can identify with ultrasonography


is the pleura. In normal conditions, a hyperechoic sliding line
(moving forward and back with ventilation) is seen below the rib
line and is called the pleural line. The presence of pleural sliding
signifies a normal pleural space (no pneumothorax) for the
region of lung being directly visualized but does not rule out a
partial pneumothorax elsewhere. If there is air between the
parietal and visceral pleurae, as occurs in a pneumothorax, there
will be no sliding of the pleural line. However, this finding is not
specific for pneumothorax as it can occur in other situations such
as presence of chest tubes, pleural adherences, bullous
emphysema, and advanced chronic obstructive pulmonary
disease.
A-lines are artificial horizontal lines seen below the pleural line.
A-lines will be present if there is a pneumothorax, but they will
be motionless since there is no pleural sliding. B-lines are
artificial vertical lines seen in up to a third of patients with
normal lungs. B-lines also move with the pleural line and efface
A-lines. Because B-lines are not always seen, their absence is not
particularly useful in diagnosing a possible pneumothorax.
However, the presence of B-lines can rule out a pneumothorax in
the portion of lung being evaluated.

Finally, probably the most valuable sign in the diagnosis of a


pneumothorax is identification of the lung point. The lung point
is the point where the normal lung pattern (pleural sliding
and/or the presence of B-lines) replaces the pneumothorax
pattern (absent lung sliding and horizontal A-lines). Utilization of
M-mode can facilitate detection of the lung point. Studies have
shown that, in trained hands, ultrasonography has better
sensitivity and specificity for diagnosing pneumothorax than
conventional chest radiography (86% and 97% versus 28% and
100%, respectively). In addition, lung ultrasonography was
shown to be significantly faster than chest radiography.

RATIONALE (31) Answer: A

High-frequency oscillatory ventilation (HFOV) has been


proposed as an alternative mode for delivering low-tidal-volume,
controlled-pressure ventilation in patients with severe acute
respiratory distress syndrome. HFOV, commonly utilized in
neonatology and only recently recognized as potentially useful in
adults, is based on the principles of the open-lung ventilation and
aims to improve oxygenation by keeping the lung uniformly
inflated for an extended period of time. In HFOV, the lung is kept
open by a set mean airway pressure (oxygenation). High
frequency oscillations (>180/min) produce inspirationand
expiration with small volumes around this mean airway pressure
(ventilation). Theoretically, this will improve recruitment
(oxygenation) and minimize ventilator-induced lung injury by
limiting volutrauma and atelectrauma. In HFOV, alveolar
ventilation (carbon dioxide removal) is controlled by the power
of oscillation, which determines amplitude, and by the frequency
of oscillations measured in hertz. The speed of oscillation is
determined by the frequency. One hertz (Hz) is equal to 1 breath
per second or 60 breaths per minute. A frequency of 6 Hz
therefore is equal to 360 breaths per minute. In HFOV, because
of the fixed inspiratory to expiratory ratio, as frequency
increases, the time allotted for the piston to move air is
decreased. In other words, changes in frequency are inversely
proportional to tidal volume. With HFOV, in order to increase
alveolar ventilation (increase tidal volume) and remove more
carbon dioxide, the frequency must be decreased. This is usually
done by decreasing frequency in 1-Hz decrements to a minimum
of 3 Hz. Alveolar ventilation (carbon dioxide removal) may also
be increased by increasing the power to increase amplitude. Of
the possible answers, only option A (decreasing frequency to 5
Hz) would result in improved carbon dioxide removal. If
adjustments in frequency and amplitude fail to correct increased
carbon dioxide, one last option is to deflate the endotracheal cuff
and produce an air leak. This needs to be accompanied by an
increase in mean airway pressure (by 5 cm H2O) and is usually
the least preferred method of correcting high carbon dioxide
levels. Mean airway pressure determines oxygenation, and
increasing it will not affect carbon dioxide level.

RATIONALE (32) Answer: B

High-frequency oscillatory ventilation (HFOV) is a unique mode


of ventilation that requires the critical care team to monitor
patients with unique potential complications in mind. The
amplitude is set by increasing power until the patient has a clear
chest wiggle that travels down to the mid-thighs. Because the
HFOV ventilator is poorly alarmed, it is essential to monitor the
patient closely for potential problems. Clinical assessment and
evaluation of the patient’s chest wiggle are needed to recognize
potential problems such as endotracheal tube (ET) obstruction,
tension pneumothorax, or pulmonary overdistension.
Observation of the patient for equal and continuous chest wiggle
should be performed at the initiation of HFOV and regularly
thereafter. If the patient’s chest wiggle abruptly changes or
decreases, potential causes to be considered include ET
obstruction (ie, mucus plug) and tension pneumothorax
(especially if the chest wiggle becomes asymmetric). Increases in
amplitude (by 10 cm H2O or more) reflect changes in pulmonary
dynamics. The most common cause of changes in pulmonary
dynamics leading to increased amplitude is airway secretions.
Situations like the one described in this case usually indicate the
need for aggressive suctioning of respiratory secretions. It is
important to emphasize that suctioning during HFOV must be
done with a closed circuit because disconnecting the patient will
lead to derecruitment and likely hypoxemia. When patients are
on HFOV, every effort must be made to avoid disconnection from
the oscillator in order to prevent derecruitment. There is no
indication in this case of a potential pneumothorax; therefore,
the chest tubes are not indicated. Increasing sedation would not
be correct based on the available information, although it is
important to remember that patients on HFOV require heavy
sedation and often neuromuscular blockade. Deflation of the ET
cuff is sometimes utilized to increase carbon dioxide removal.

RATIONALE (33) Answer: D

The principal determinants of oxygenation during high-


frequency oscillatory ventilation (HFOV) are the mean airway
pressure and FIO2. Oxygenation can be improved by increasing
mean airway pressure, FIO2, or by increasing the percent of
inspiratory time (usually set a 33%). Bias flow is the rate at
which gas flows through the ventilator circuit. An increase in
bias flow will increase mean airway pressure, thereby increasing
oxygenation. Generally, it is accepted that, as a starting point on
HFOV, the bias flow should be set around 40 L/min. The
maximum bias flow possible on current HFOV machines is 60
L/min. It is important to recognize that even a maximal flow rate
on HFOV is insufficient to support spontaneous breathing efforts.
For this reason, patients should be deeply sedated (and often
paralyzed) during HFOV.

RATIONALE (34) Answer: C

Ultrasonography is increasingly being used as a diagnostic


modality in the critically ill. The patient has a pleural effusion as
pleural fluid appears hypoechoic (black) on ultrasonogram. The
pleural fluid is demonstrated in the Figure by the star. The
patient’s liver is the salt-and-pepper structure demonstrated by
the square. The patient’s diaphragm is demonstrated by the
straight arrow and is above the patient’s liver. The curved arrow
represents atelectatic lung due to the patient’s pleural effusion.
This still image from the ultrasonography does not explain the
patient’s respiratory distress. The pleural effusion in this picture
does not appear large. However, on only 1 view, it is difficult to
fully appreciate the extent of this effusion; more views are
needed to complete the examination. There is no evidence of a
pneumothorax, which would be demonstrated by absence of
lung sliding in the proper clinical context, or of a mass in the
chest or a pericardial effusion. Diaphragmatic paralysis can be
identified in a live ultrasonography clip that shows respiratory
motion, but it would not be a diagnosis made with a still
ultrasonography image such as the one presented in this case. A
normal ultrasonogram of the patient’s right thorax would
demonstrate the patient’s liver, diaphragm, and horizontal
reflections from the patient’s chest wall know as A-lines
secondary to the piezoelectric effect induced by
ultrasonography.

RATIONALE (35) Answer: C

Patients with idiopathic pulmonary fibrosis from the time of


diagnosis have a median survival of approximately 3.5 years.
Acute exacerbations are associated with a mortality of 60%–70%
over 3–6 months. As was done with this patient, it is important
to rule out reversible causes of respiratory distress in idiopathic
pulmonary fibrosis patients with acute deterioration. This was
done with the bronchoscopy, which ruled out pulmonary
infection; the normal ejection fraction on the echocardiogram,
which ruled out congestive heart failure; and the CT, which ruled
out pulmonary embolism and other lung disease such as
pneumothorax. The patient’s mildly elevated brain natriuretic
peptide level would not explain his rapid deterioration. Neither
steroids nor any other pharmaceutical therapy have proven
benefit in idiopathic pulmonary fibrosis. Noninvasive ventilation
is not indicated for somnolent patients with idiopathic
pulmonary fibrosis, and the patient has no indication for
antibiotics. Idiopathic pulmonary fibrosis is now the leading
indication for pulmonary transplantation in the United States.
Early after diagnosis, patients should be referred to transplant
centers to be evaluated for possible transplant. Exacerbations of
the disease that occurred in this patient can lead to rapid
deterioration. Without prior evaluation, it is unlikely the patient
would receive a lung transplant because acutely ventilated
idiopathic pulmonary fibrosis patients have high mortality even
after transplant. However, in lung transplant recipients, those
with the worst long-term prognosis now have 1-year survival of
83% and 5-year survival of 54% according to a scientific registry
of transplant recipients.
PART 4: Critical Care Infectious Diseases

Instructions: For each question, select the most correct answer.

1. A 44-year-old woman with an allergy to sulfa drugs is


admitted to the ICU following a motor vehicle accident. She is
intubated and mechanically ventilated. On hospital day 5, she
develops fever, purulent sputum, and a left lower lobe infiltrate.
Gram stain from bronchoalveolar lavage (BAL) shows many
leukocytes and gram-negative bacilli. Culture from BAL and
blood grows Enterobacter cloacae sensitive to third-generation
cephalosporins, piperacillin, and
trimethoprim/sulfamethoxazole. She is started on IV ceftriaxone.

Seven days later, she continues to have intermittent fevers and


produce purulent sputum; the left lower lobe infiltrate is
unchanged. Repeat BAL and blood cultures grow E cloacae that is
now sensitive to trimethoprim/sulfamethoxazole and resistant
to all beta-lactams except meropenem.

Which of the following is the most appropriate next step?

A. Continue ceftriaxone to complete a 14-day course and repeat


cultures on day 14 to ensure she has cleared her bacteremia.

B. Discontinue ceftriaxone and start piperacillin/tazobactam plus


amikacin for synergy.

C. Discontinue ceftriaxone, desensitize to


trimethoprim/sulfamethoxazole, and start
trimethoprim/sulfamethoxazole to complete a 2-week course.

D. Discontinue ceftriaxone and start meropenem to complete a 2-


week course.
2. A 37-year-old man seen 1 week ago in urgent care clinic for an
influenza-like illness is now admitted to the ICU with severe
community-acquired pneumonia. Chest CT demonstrates
bilateral cavitary infiltrates and a loculated right lower lobe
effusion. Empirical therapy for methicillin-resistant
Staphylococcus aureus (MRSA) is started pending sputum and
blood culture results.

Which of the following statements should guide antibiotic


management of this patient’s suspected MRSA pneumonia?

A. Vancomycin trough concentrations of 15–20 µg/mL are most


appropriate.

B. Linezolid is associated with lower mortality rates than


vancomycin.

C. If the vancomycin minimum inhibitory concentration is


greater than 2 µg/mL, daptomycin should be used.

D. Empirical coverage for MRSA should be continued even if


cultures do not grow the organism.

3. A 69-year-old woman is transferred to the ICU from an outside


hospital. Two weeks ago, she underwent uncomplicated partial
colectomy for diverticulitis. One week ago, she was admitted to
the outside hospital for fevers and abdominal pain. Blood
cultures were positive for vancomycin-resistant Enterococcus
faecium (VRE) and a highly-resistant strain of Klebsiella
pneumoniae. She was started on tigecycline but developed
respiratory failure and was intubated and mechanically
ventilated 5 days ago. Chest radiography shows bilateral
multifocal opacities consistent with bilateral pneumonia and
acute respiratory distress syndrome.

Which of the following statements about tigecycline should guide


this patient’s treatment?

A. Tigecycline has minimal activity against anaerobic bacteria.

B. Most strains of Pseudomonas are sensitive to tigecycline.

C. Compared to other antibiotics, tigecycline is associated with


an increased risk of death, especially in patients with ventilator-
associated pneumonia.

D. Tigecycline has poor activity against carbapenemase-


producing, gram-negative bacteria.

4. A 74-year-old woman is admitted to the ICU for fever to 38.5°C


(101.2°F), altered mental status, abdominal pain, and
hypotension. She resides in a nursing home and presented to the
emergency department after several days of frequent, loose,
watery bowel movements and abdominal cramping. Her
worsening shock necessitates endotracheal intubation,
aggressive IV fluid resuscitation, and vasoactive agent support.

At admission, WBC count is 26,000/µL (12% bands), serum


creatinine level is 1.9 mg/dL (baseline 0.7 mg/dL), and serum
lactate level is 27.9 mg/dL.

Clostridium difficile toxin assay detects toxin A and B.

Which of the following is the most appropriate next step in the


menagement of this patient?
A. Oral vancomycin

B. Emergent colectomy

C. Diagnostic colonoscopy

D. CT of the abdomen

E. IV metronidazole

5. A 27-year-old man with a history of IV drug use is admitted to


the ICU for fevers, altered mental status, and hypotension. Blood
cultures at the time of admission are growing methicillin-
resistant Staphylococcus aureus (MRSA).

Which of the following statements is most appropriate to guide


the management of his MRSA bacteremia?

A. Gentamicin should routinely be added to vancomycin.

B. Rifampin should routinely be added to vancomycin.

C. Echocardiographic evaluation for endocarditis is not


recommended unless repeat blood culture results are positive
for MRSA.

D. Appropriate initial antibiotics include vancomycin or


daptomycin.

6. A 65-year-old woman presents to the emergency department


with fever and altered mental status. She has a history of atrial
fibrillation treated with amiodarone and is severely allergic to
penicillin. Chest radiography shows a questionable retrocardiac
opacity. Blood and urine cultures are obtained. She is started on
moxifloxacin and admitted to the floor for community-acquired
pneumonia. Overnight, she becomes increasingly delirious and is
transferred to the ICU for hypotension.

Which of the following statements is correct regarding


moxifloxacin therapy?

A. Moxifloxacin concentrates optimally in the urine and can be


used for suspected urinary tract infections.

B. Moxifloxacin should generally be avoided in patients with


prolonged QT intervals.

C. Compared to ciprofloxacin, moxifloxacin has greater in vitro


activity against Pseudomonas aeruginosa.

D. In contrast to ciprofloxacin, moxifloxacin demonstrates


activity against vancomycin-resistant Enterococcus.

7. A 75-year-old man with diabetes and end-stage renal disease


on hemodialysis is admitted to the medical ICU for fever. Four
weeks before admission, he was started on linezolid for a
diabetic foot infection.

Besides thrombocytopenia, which of the following side effects is


associated with long durations of therapy with linezolid?

A. Guillain-Barré syndrome

B. Hyperglycemia due to pancreatic dysfunction

C. Lactic acidosis

D. Hypertriglyceridemia
E. Fanconi syndrome

8. A 52-year old man is admitted to the ICU for sepsis and


multisystem organ failure and requires a urinary catheter for
accurate monitoring of urine output.

Which of the following strategies helps to prevent catheter-


associated urinary tract infections?

A. Sterile equipment and aseptic technique for catheter insertion

B. Routine cleaning of the urethral meatus with antiseptic


solution after catheter insertion

C. Keeping the collecting bag above the level of the bladder to


promote reflux of urine into the bladder and not emptying it
routinely

D. Bladder irrigation and instilling antiseptic or antimicrobial


agents into the collecting bag

9. A 76-year-old woman is admitted to the ICU from a nursing


home for health care–associated pneumonia. She has a history of
infection with multidrug-resistant organisms.

Which of the following statements should guide the use of


colistin in this patient’s treatment?

A. Colistin is nephrotoxic but not neurotoxic.

B. Colistin is neurotoxic but not nephrotoxic.


C. Colistin retains activity against most strains of multidrug-
resistant, gram-negative infections.

D. Unlike tobramycin, colistin cannot be nebulized for inhaled


use.

10. A 46-year-old woman with a recent bilateral lung transplant


for cystic fibrosis is transferred into the ICU with dyspnea,
hypoxia, and fever. She is receiving immunosuppression with
mycophenolate, tacrolimus, and prednisone.

Yesterday, she underwent bronchoscopy with transbronchial


biopsy, which is reported today as showing acute-angle
branching hyphae. After a brief conversation with the infectious
diseases consultant, the patient is started on IV voriconazole.
Doses of which of the following of her medications should be
reduced?

A. Mycophenolate

B. Voriconazole

C. Tacrolimus

D. Prednisone

E. Propofol

11. A 63-year-old man was admitted to the ICU 4 days ago after
undergoing a partial colectomy for a perforated diverticulum. He
has been febrile since postoperative day 1 despite treatment
with vancomycin and piperacillin/tazobactam. This afternoon he
developed hypotension requiring blood pressure support. Which
antimicrobial medication should be added empirically to his
regimen?

A. Amphotericin B

B. Liposomal amphotericin B

C. Caspofungin

D. Fluconazole

12. A 34-year-old woman with recently diagnosed AIDS is


admitted to the ICU with fever, hypoxia, respiratory failure, and
pancytopenia. Six weeks ago she was started on highly active
antiretroviral therapy with tenofovir, emtricitabine, and
ritonavir-boosted lopinavir. Her chest radiograph shows
bilateral infiltrates with an interstitial pattern. The evaluation of
her pancytopenia included a bone marrow biopsy, which showed
trilineage hypoplasia and macrophages with multiple
intracellular yeast forms consistent with histoplasmosis. Which
of the following is the preferred initial treatment?

A. Lipid preparation of amphotericin B

B. IV fluconazole

C. IV voriconazole

D. Caspofungin

E. Posaconazole

13. A 58-year-old man with a kidney transplant is admitted with


confusion. In the emergency department, he was intubated for
airway protection and underwent a lumbar puncture with an
opening pressure of 32 cm H2O. Analysis of cerebrospinal fluid
showed 12 RBCs per µL, 83 WBCs per µL, glucose level of 38
mg/dL, and protein level of 181 mg/dL. Gram stain showed
polymorphonuclear cells but no organisms, and results of India
ink capsule stain were negative. Which of the following is the
most appropriate initial regimen of antimicrobials?

A. Ceftriaxone alone

B. Ampicillin and ceftriaxone

C. Vancomycin and ceftriaxone only

D. Vancomycin, ceftriaxone, and amphotericin B

14. A 23-year-old man with acute myelogenous leukemia


recently underwent induction chemotherapy. He has had
neutropenia for 18 days with fever the last 5 days. Yesterday, he
reported headache, and a sinus CT showed opacification of his
left maxillary and ethmoid sinuses that was of concern for
possible fungal sinusitis. He is admitted to the ICU after
endoscopic evaluation of his sinuses, during which several areas
of necrotic mucosa were debrided; frozen pathologic sections
showed a few hyphae invading blood vessels. Which of the
following is the most appropriate empiric therapy for this
infection?

A. Fluconazole

B. Micafungin

C. Liposomal amphotericin B
D. Voriconazole alone

E. Voriconazole and caspofungin

15. A 63-year-old woman with a past medical history of type 2


diabetes mellitus and hypertension is admitted to the ICU with a
diagnosis of community-acquired pneumonia. Intially she is
hypotensive and hyperglycemic. However, her blood pressure
normalizes with a bolus of normal saline. Unfortunately, her
respiratory status declines, requiring intubation and mechanical
ventilation. Shortly after intubation, bronchoalveolar lavage is
performed, with Gram stain showing many white blood cells,
many gram-positive cocci in pairs, and few yeasts. The most
appropriate initial therapy for this patient is:

A. Vancomycin and piperacillin/tazobactam

B. Ceftriaxone and azithromycin

C. Ceftriaxone and fluconazole

D. Levofloxacin and fluconazole

16. A 57-year-old man was admitted to the ICU 5 days ago with
an ST-elevation myocardial infarction. Yesterday he developed a
fever and cultures of blood, sputum, and urine were sent. Chest
radiography showed a new right lower lobe infiltrate. Results of
urinalysis were entirely within normal limits. The patient was
started on piperacillin/tazobactam for the pneumonia and is
afebrile today. The microbiology laboratory reports that more
than 100,000 colony-forming units per millilter of yeast are
growing from the urine culture. The most appropriate next
course of action is:
A. No intervention

B. Oral fluconazole

C. IV caspofungin

D. Amphotericin B bladder washes

E. IV fluconazole

17. A previously healthy, 47-year-old man is admitted to the ICU


after decompensation following a diagnostic bronchoscopy
earlier today. He has a 6-week history of a right middle lobe
infiltrate that has not responded to a course of
amoxicillin/clavulanate or levofloxacin. The patient is an avid
hunter and spent 2 weeks deer hunting in middle Tennessee
before the development of the pneumonia. The most likely
diagnosis is:

A. Coccidioidomycosis

B. Aspergillosis

C. Mucormycosis

D. Blastomycosis

18. A 54-year-old man underwent a single lung transplant 14


days ago for idiopathic pulmonary fibrosis. He had a difficult
operation and primary graft dysfunction and has had a
prolonged ventilatory course. He has been on vancomycin and
meropenem for an aspiration pneumonia in the donor, and he
has been maintained on voriconazole and
trimethoprim/sulfamethoxazole as routine prophylaxis of fungi
and Pneumocystis. His lung has improved to the point of
initiating ventilator weaning. However, he is not waking up
despite being off midazolam for 6 hours. An MRI of the brain
shows no evidence of ischemia or hemorrhage. Which of the
following is the most likely explanation of his neurologic status?

A. He suffered a major cerebral ischemic event during lung


transplantation.

B. Trimethoprim/sulfamethoxazole inhibits the clearance of


midazolam.

C. Voriconazole inhibits the clearance of midazolam.

D. Voriconazole has major psychiatric side effects and the patient


is in a fugue state.

19. A 70-year-old man with benign prostatic hypertrophy was an


unrestrained passenger in a rollover motor vehicle crash. He is
brought to a trauma center in shock and respiratory failure. He
undergoes an emergent exploratory laparotomy and
splenectomy and requires multiple thoracostomy tubes to
manage bilateral pneumothoraxes. His ICU course is complicated
by an abdominal evisceration, Enterobacter line sepsis, and
ventilator-associated pneumonias with Pseudomonas and
Acinetobacter. After stabilizing from these conditions, he
develops bright red blood per rectum, diarrhea, leukopenia, and
fever. A flexible sigmoidoscopy is performed and reveals
multiple oozing ulcerations in the colon with the pathology
shown in the Figure.
Which of the following is the most appropriate antimicrobial
medication to add?

A. IV ceftriaxone for disseminated pneumococcal disease

B. IV acyclovir for herpes simplex virus colitis

C. Vancomycin per nasogastric tube plus IV metronidazole for


Clostridium difficile colitis

D. IV ganciclovir for cytomegalovirus colitis

E. IV metronidazole for Entamoeba histolytica colitis


20. A 51-year-old woman recently underwent an allogeneic bone
marrow transplant for refractory multiple myeloma, with
successful engraftment. She is readmitted with suspected graft-
versus-host disease (GVHD) because of recently developed
shortness of breath, increased liver enzymes, and conjunctivitis.
She is also given IV levofloxacin for a suspected urinary tract
infection because of blood and white blood cells on urine
microscopy. She is given high-dose steroids and thymoglobulin
for GVHD. Twenty-four hours later, she develops respiratory
distress, fulminant hepatitis, and bloody urine and is transferred
to the ICU. Chest radiographs show progressive bilateral
infiltrates in a pattern consistent with acute respiratory distress
syndrome.

The most likely etiology of this course is:

A. Cytomegalovirus

B. Adenovirus

C. Legionella species

D. Parainfluenza 3

E. Toxicity from chemotherapy

21. A 35-year-old man with HIV diagnosed 4 years ago is


admitted with fever, weight loss, and night sweats. A diagnosis of
lymphoma is made, and he is treated with rituximab plus
cyclophosphamide, doxorubicin, vincristine, and prednisone (R-
CHOP). A few days later, he is transferred to the ICU for
neutropenia and hypotension. He is treated with vancomycin
and meropenem but remains hypotensive. After 48 hours,
caspofungin is added, but his blood pressure remains low.
Adrenal insufficiency is diagnosed and he is started on stress-
dose steroids. Seventy-two hours later, he develops a burning,
painful rash with vesicles on his back, left chest, and left upper
abdomen.

Which of the following steps is most appropriate?

A. Substitute IV linezolid for vancomycin.

B. Begin IV acyclovir.

C. Begin valacyclovir.

D. Substitute amphotericin B for caspofungin.

22. A 26-year-old graduate student in chiropterology returned 2


days ago from a 2-week field study of eating habits of fruit bats
in the Democratic Republic of the Congo and Uganda. During her
flight back, she started to feel fever, malaise, and nausea. Shortly
after arriving in the US, she developed diarrhea and vomiting. A
local emergency department, suspecting food poisoning from the
fish she ate on her flight, prescribed her ciprofloxacin and
metronidazole. The next day she returned with continued
symptoms, rigors, and abdominal pain, at which time a
maculopapular rash was observed. She progressed to delirium,
hypotension, and abrupt prostration. She was intubated for
airway protection. Soon afterward, she developed bloody
pulmonary secretions, conjunctival hemorrhages, and bloody
diarrhea.

Which of the following interventions is most likely to benefit the


patient?

A. Ribavirin
B. Acyclovir

C. Oseltamivir

D. Aggressive fluid repletion

E. Replacement of coagulation factors

23. A 65-year-old, male resident of an assisted living center has


asthma, hypertension, and coronary artery disease. He is
transferred to an acute care facility with acute respiratory
distress and fever in mid January after spending the holidays
with children and grandchildren. Unable to maintain his
oxygenation, he is intubated and placed on mechanical
ventilation. His chest radiograph shows bilateral infiltrates.
Results of a respiratory viral panel are positive for influenza A.
There have been sporadic reports of oseltamivir-resistant strains
in the current season.

Which of the following is the best therapeutic option for this


patient?

A. Oseltamivir, enteral

B. Zanamivir, inhaled

C. IV amantadine

D. IV immunologlobulins

E. IV foscarnet
24. A 44-year-old woman with weakness and difficulty in
breathing presents to the emergency department reporting
fever, abdominal pain, and a pruritic maculopapular rash on her
back and chest for the past 24 hours. She was diagnosed 10 years
ago with HIV, which had been controlled with
lamivudine/zidovudine and lopinavir/ritonavir for the past 3
years. Approximately 2 weeks ago, her physician added abacavir
to her regimen to achieve an undetectable viral load. She also has
diastolic dysfunction, hypertension, and hypothyroidism. She
only takes lisinopril in addition to antiretrovirals. She is found to
be hypotensive, tachycardic, and tachypneic. Radiography shows
an interstitial pneumonitis, but oxygen saturation is 95% on
room air.

Which of the following is the most likely cause of this condition?

A. Congestive heart failure

B. Acute renal failure due to antiretrovirals

C. Hypersensitivity reaction due to an antiretroviral

D. Pneumocystis pneumonia

E. Acute HIV syndrome

25. A 22-year-old man with HIV and a history of epilepsy is


admitted to an ICU with status epilepticus. He is intubated for
airway protection and his seizures are quickly controlled. His
CD4 cell count is 485/µL and his HIV viral load is undetectable.
He receives tenofovir, emtricitabine, and efavirenz for HIV.
Findings of CT and MRI of the brain are negative. Which of the
following is the most appropriate management of this patient’s
illness?
A. Stop his antiretroviral medications until he is back to baseline.

B. Reduce the dosages of his antiretroviral medications in case


the seizures were induced by lactic acidosis.

C. Change his antiretroviral medications to include an unboosted


protease inhibitor.

D. Continue his current antiretroviral regimen.

E. Perform a lumbar puncture to evaluate for encephalitis.


PART 4: Critical Care Infectious Diseases

ANSWERS:

1–D; 2–A; 3–C; 4–B; 5–D; 6–B; 7–C; 8–A; 9–C; 10–C; 11–C; 12–A;
13–D; 14–C; 15–B; 16–A; 17–D; 18–C; 19–D; 20–B; 21–B; 22–E;
23–A; 24–C; 25–D

RATIONALE (1) Answer: D

Enterobacter cloacae possesses an inducible ampC chromosomal


beta-lactamase. In Enterobacter infections, mutants can produce
high levels of beta-lactamase. These mutant strains can emerge
during therapy with a third-generation cephalosporin such as
ceftriaxone and result in treatment failure, even if the isolates
appear susceptible on initial testing. For serious infections
caused by Enterobacter, carbapenems are first-line therapy.

RATIONALE (2) Answer: A

Vancomycin generally has poor penetration into pulmonary


tissue and lung epithelial lining fluid. The pharmacodynamic
parameter that best predicts efficacy of vancomycin is the ratio
of the area under the curve (AUC) to the minimum inhibitory
concentration (MIC) [AUC/MIC]. In Staphylococcus aureus lower
respiratory tract infections, an AUC/MIC greater than 400 is
associated with improved clinical response and microbiologic
eradication. On the basis of currently available data, vancomycin
trough concentrations of 15–20 µg/mL are needed to achieve
this target. In prospective studies, linezolid and vancomycin
were associated with comparable cure rates. Thus, option B is
incorrect. Daptomycin is inactivated by pulmonary surfactant
and should not be used in patients with pneumonia, so option C
is also incorrect. Although empirical therapy for methicillin-
resistant S aureus (MRSA) should be considered in patients with
severe community-acquired pneumonia and necrotizing or
cavitary infiltrates, empirical coverage for MRSA should be
discontinued if sputum or blood cultures do not grow the
organism. Therefore, option D is incorrect.

RATIONALE (3) Answer: C

Tigecycline has broad-spectrum activity against gram-positive,


gram-negative, and anaerobic organisms. It also has in vitro
activity against methicillin-resistant Staphylococcus aureus,
vancomycin-resistant enterococci, and many highly resistant
gram-negative bacteria including strains that produce
carbapenemases; however, clinical data is limited. Tigecycline
generally has poor activity against most strains of Pseudomonas.
On July 6, 2010, the U.S. Food and Drug Administration (FDA)
updated the Warnings and Precautions and Adverse Reactions
sections of the tigecycline drug label to include information
regarding increased mortality risk (seen most clearly in patients
treated for hospital-acquired pneumonia, especially ventilator-
associated pneumonia

RATIONALE (4) Answer: B

Colectomy may be a lifesaving intervention for persons who are


severely ill from Clostridium difficile infection (CDI), as
illustrated in this case; therefore, option B is correct. Studies
indicate that in the setting of CDI, clinical deterioration
characterized by cardiopulmonary compromise , peritoneal
signs, or toxic megacolon should lead to urgent consideration of
surgical intervention. Furthermore, conservative medical
management with antibiotic therapy (options A and E) or
radiographic evaluation (option D) in the setting of severe CDI
has been associated with higher mortality compared with
surgical intervention. Endoscopic evaluation (option C) is not
indicated given the documentation of a positive stool assay
result for C difficile, and in fact, increases the risk of bowel
perforation

RATIONALE (5) Answer: D

Vancomycin or daptomycin are reasonable choices for MRSA


bacteremia. Echocardiography generally should be performed in
all adult patients with MRSA bacteremia to evaluate for
endocarditis, so option C is incorrect. Routine addition of
gentamicin and/or rifampin is not recommended for MRSA
bacteremia or native valve endocarditis, so options A and B are
incorrect.

RATIONALE (6) Answer: B

Unlike ciprofloxacin and levofloxacin, moxifloxacin does not


concentrate in the urine and should not be used for suspected
urinary tract infections (therefore, option A is incorrect). All of
the fluoroquinolones can potentially cause prolongation of the
QT interval (option B). Compared to moxifloxacin, ciprofloxacin
generally has better in vitro activity against most strains of
Pseudomonas (therefore, option C is incorrect). None of the
fluoroquinolones demonstrates activity against vancomycin-
resistant Enterococcus, so option D is incorrect.

RATIONALE (7) Answer: C

Although rare, long durations of linezolid treatment are


associated with an increased risk of lactic acidosis. Additional
potential risks after prolonged linezolid therapy include optic
and peripheral neuropathy, serotonin syndrome (especially in
the setting of concominant selective serotonin reuptake inhibitor
therapy), and myelosuppression, including thrombocytopenia.
Linezolid has not been associated with Guillain-Barré syndrome
(option A), hyperglycemia (option B), hypertriglyceridemia
(option D) or Fanconi syndrome (option E).

RATIONALE (8) Answer: A

To prevent catheter-associated urinary tract infections, sterile


equipment and aseptic technique should be used for insertion
(option A). Routine cleaning of the urethral meatus with
antibiotic solution after catheter insertion is not recommended
and may actually increase the risk of bacteruria, so option B is
incorrect. The collecting bag should remain below the level of the
bladder to prevent reflux of urine into the bladder and should be
emptied routinely; thus, option C is incorrect. Procedures such as
bladder irrigation and instilling antiseptic or antimicrobial
agents into the collecting bag have not been shown to provide
benefit and should not be performed. Therefore, option D is
incorrect

RATIONALE (9) Answer: C

Colisitin is generally the treatment option of last resort for


extremely multidrug-resistant, gram-negative infections. Thus,
option C is correct. Colistin can potentially cause both
nephrotoxicity and neurotoxicity, so options A and B are
incorrect. Colistin can be nebulized for inhaled use, so option D is
incorrect. However, premixing colistimethate into an aqueous
solution and storing it for longer than 24 hours results in
increased concentrations of colistin in solution, increasing the
potential for lung toxicity.

RATIONALE (10) Answer: C


Many antimicrobials have significant drug interactions,
especially those which affect hepatic metabolism. Many
medications are cleared through the cytochrome P-450 enzymes.
Some antimicrobials inhibit cytochrome P-450, especially
cytochrome 3A4. The antifungal class of triazoles, or “azoles,” are
well known inhibitors of cytochrome 3A4. Consequently, any
medication which is cleared through 3A4 will then accumulate
resulting in higher serum levels. Among the medications listed,
tacrolimus is cleared through 3A4. Typically, when tacrolimus is
co-administered with voriconazole, the same serum levels can be
achieved by reducing the dose of tacrolimus to one-half or one-
third the previous dose. The other medications listed are cleared
renally or through other degradation pathways. Antibiotics in
the rifamycin class (rifampin, rifabutin, rifapentene, etc) are
inducers of the cytochrome P-450 enzymes and rifampin is the
most potent. Therefore, when rifamycins are coadministered
with medications metabolized through cytochrome P-450 , the
result is lower drug levels of the coadministered medication. If
rifabutin had been started instead of voriconazole, then the dose
of tacrolimus would have needed to be increased to maintain
therapeutic levels. Rifampin is such a potent inducer of 3A4 that
it is difficult to maintain therapeutic levels of tacrolimus when
the 2 are coadministered.

RATIONALE (11) Answer: C

Abdominal surgery is a risk factor for the development of


candidiasis/candidemia. Since the patient has not responded to
appropriately broad antibacterial therapy, it is reasonable to
suspect that he may have a Candida infection; it may be localized
to the peritoneum or systemic in the bloodstream. Candida are
commensals of the human gastrointestinal tract. Thus, they are
frequently encountered in gastrointestinal-associated infections.
In 2009, the Infectious Diseases Society of America released
updated treatment guidelines for candidiasis. These guidelines
recommend that an antifungal from the echinocandin class be
used to treat candidiasis in moderately to severely ill patients.
The guidelines do not distinguish between the 3 FDA-approved
echinocandins. Since caspofungin is the only listed echinocandin,
it is the most correct choice (option C). The echinocandins are
fungicidal for Candida species. While fluconazole (option D) has
been used empirically for years to treat candidiasis, it is
fungistatic. Therefore, the recommendation is to use an
echinocandin in critically ill patients. Amphotericin B products
are also fungicidal for Candida species. However, all
amphotericin B– containing medications have increased toxicity
over the echinocandins. Because of this excess toxicity,
amphotericin B should be used as a second-line agent in patients
who have had an adverse or allergic response to echinocandins.
Therefore, options A and B are incorrect

RATIONALE (12) Answer: A

Amphotericin B, whether a lipid preparation or the deoxycholate


preparation, is the treatment of choice for disseminated
histoplasmosis (option A). Histoplasma capsulatum, the
causative agent of histoplasmosis, is endemic in the Mississippi
and Ohio River valleys. Patients with immunocompromised
immune systems, ie, AIDS, solid organ transplant recipients, or
recipients of tumor necrosis factor antagonists, are at increased
risk for developing disseminated disease. When Histoplasma
disseminates, it often goes to the bone marrow causing
pancytopenia. The liver and spleen are other organs commonly
affected. Pathologic specimens may show “foamy” macrophages,
or frequently granulomas, in less severely immunocompromised
patients. Fluconazole (option B) is less effective against
Histoplasma; therefore, it should not be used in the treatment of
disseminated histoplasmosis. While voriconazole and
posaconazole (options C and E) have in vitro activity against
Histoplasma, they have not been studied in controlled clinical
trials. Therefore, they cannot be recommended for routine use.
Itraconazole does have activity against Histoplasma and it is the
preferred oral option for less severe disease. However,
itraconazole has significant drug interactions with ritonavir and
coadministration is contraindicated. The echinocandins,
including caspofungin (option D), have no activity against
Histoplasma

RATIONALE (13) Answer: D

This patient has meningitis. In an immunocompromised patient,


such as a kidney transplant recipient, common and uncommon
causes must be suspected. Thus, the etiology could be usual
organisms like Streptococcus pneumoniae, or more unusual
organisms like Listeria monocytogenes or Cryptococcus
neoformans. Typically, pneumococcus meningitis produces more
inflammation than the 83 WBCs seen in this case. However, solid
organ transplant recipients often have a blunted inflammatory
response. This is especially true of those patients receiving
higher doses of steroids. Even so, pneumococcus is usually seen
readily on Gram stain of the cerebrospinal fluid. Ceftriaxone is
excellent treatment for pneumonoccus, but there are some
isolates which exhibit high-level resistance, in which case
vancomycin should be used. Listeria typically causes a
mononuclear pleocytosis in the cerebrospinal fluid. Thus, it is a
less likely cause of meningitis in this patient. However,
medications active against Listeria should be used until the
cultures have adequately incubated. Ceftriaxone does not treat
Listeria. Ampicillin and vancomycin are both active against
Listeria. Cryptococcus is frequently seen in transplant-recipient
meningitis. The elevated opening pressure, high protein level,
and moderate pleocytosis are typical features of a cryptococcal
meningitis. The India ink test is less sensitive than latex
agglutination for the cryptoccocal antigen. Therefore, false
negatives are possible, especially if there is a low organism
burden. Of the choices listed, only vancomycin plus ceftriaxone
plus amphotericin B (option D) is broad enough antimicrobial
coverage for all these possibilities.

RATIONALE (14) Answer: C

This patient clearly has invasive fungal sinusitis. The most


common causes of invasive fungal sinusitis are aspergillosis and
mucormycosis. Both diseases are characterized by necrosis,
hyphae, and blood vessel invasion. Only culture or special
pathologic techniques can distinguish between the 2. Of the
agents listed, only the liposomal amphotericin B (option C) has
activity in both diseases. Fluconazole (option A) does not treat
either aspergillosis or mucormycosis. Micafungin, voriconazole,
and caspofungin (options B, D, and E) have activity against
aspergillosis but not against mucormycosis

RATIONALE (15) Answer: B

Community-acquired pneumonia (CAP) guidelines recommend


coverage for both typical (Streptococcus pneumoniae,
Haemophilus influenzae) and atypical (Mycoplasma, Chlamydia)
organisms. However, organisms such as methicillin-resistant
Staphylococcus aureus and Pseudomonas are not usually
encountered in CAP. Therefore, ceftriaxone plus azithromycin is
a reasonable initial regimen until the causative agent has grown
in culture. Levofloxacin and other fluoroquinolones provide good
coverage for both typical and atypical organisms seen in CAP.
Vancomycin plus piperacillin/tazobactam (option A) would be a
good regimen for health care–associated pneumonia. However, it
wouldn’t cover atypical organisms sometimes seen in CAP. Based
on the Gram stain, this patient likely has an S pneumoniae
pneumonia. The yeasts seen on the Gram stain are likely
colonizers, ie, Candida. Candida species are almost never a cause
of pneumonia unless the patient has a prolonged neutropenia or
embolic disease from right-sided Candida endocarditis.
Therefore, the 2 choices including fluconazole (options C and D)
would not be correct

RATIONALE (16) Answer: A

The fever in this patient is caused by the pneumonia in the right


lower lobe. The yeast in the urine represents asymptomatic
candiduria as the urinalysis did not show any pyuria. Research
has shown that asymptomatic candiduria has the same response
rate whether the patient receives no treatment, systemic
treatment with fluconazole (options B and E), or amphotericin B
bladder washes (option D). There is no indication for systemic
echinocandin therapy (option C). Therefore, the most recent
guidelines on candidiasis suggest no treatment for asymptomatic
candiduria (option A

RATIONALE (17) Answer: D

This question requires understanding of basic fungal


epidemiology. The correct answer is blastomycosis (option D).
Blastomyces dermatitidis is a fungus found on decaying
vegetation and occurs in the southeastern US and the Mississippi
and Ohio river valleys. Most individuals who become infected
have hunting or hiking as an avocational activity. Outbreaks have
been observed surrounding earth-moving activities such as
construction. Most patients develop a pneumonia, and the
diagnosis of blastomycosis is suspected or confirmed after
failure to respond to repeated courses of antibacterial
medications. The diagnosis of blastomycosis can be made by
isolating the organism in culture from appropriate specimens
(sputum or bronchoalveolar lavage in the scenario), or by
detection of a fungal antigen in the urine similar to that for
histoplasmosis. Coccidioidomycosis (option A) occurs only in the
desert southwest, and aspergillosis and mucormycosis (options
B and C) occur only in immunocompromised patients.

RATIONALE (18) Answer: C

The correct answer is option C—voriconazole inhibits the


clearance of midazolam. Midazolam is cleared through the
cytochrome P-450 system, and voriconazole greatly inhibits the
activity of these enzymes. Thus, when coadministered,
midazolam accumulates and has delayed clearance. Therefore,
the sedative effect of midazolam is prolonged. While a stroke
(option A) could potentially be the cause of this patient’s
depressed neurologic status, the normal brain MRI finding
makes this diagnosis less likely. Trimethoprim/sulfamethoxazole
(option B) has no effect on the clearance of midazolam and is
rarely if ever implicated as a central nervous system depressant.
Voriconazole does have major psychiatric side effects, but a
fugue state (option D) has not been reported. The most common
psychiatric side effect is visual hallucinations. Typically, the
patient is aware of the hallucinations and is quite disturbed by
the adverse event. Very rarely, psychotic breaks are associated
with the visual hallucinations of voriconazole.

RATIONALE (19) Answer: D

Ganciclovir (option D) is the correct therapeutic choice for this


patient. The diagnosis is cytomegalovirus (CMV) colitis, based on
pathology showing viral inclusions and on the clinical
presentation. CMV infections occur, though rarely, in
nonimmunocompromised patients. Latent CMV can reactivate
and cause end-organ disease in situations with high levels of
tumor necrosis factor alpha, lipopolysaccharide, or exogenous
catecholamines. Therefore, it is not uncommon for CMV
replication to be detected in patients in the ICU. While the
patient is at risk for infection from encapsulated organisms
because of the splenectomy, the scenario is not consistent with
pneumococcal disease, so option A is incorrect. Herpes simplex
virus colitis is extremely rare and unlikely in this situation. Thus,
option B is incorrect. This patient is at high risk for developing
Clostridium difficile–associated colitis, but the pathology is
consistent with viral colitis, not pseudomembranous colitis.
Therefore, option C is incorrect. While bloody diarrhea can be a
presentation of amoebic colitis, this patient has no known risk
factors, so option E is also incorrect.

RATIONALE (20) Answer: B

Adenoviral disease is the most likely process in this patient.


Although conjunctivitis, respiratory distress, hepatitis, and
hemorrhagic cystitis all may be caused by graft -versus-host-
disease (GVHD), they are unlikely to occur simultaneously with
this process. Worsening of symptoms with GVHD-focused
immunosuppression points toward an infectious etiology. While
cytomegalovirus (option A) and Legionella (option C) may cause
many of these symptoms, hemorrhagic cystitis is unlikely. In this
patient population, parainfluenza infections can be severe but
are usually limited to pulmonary disease, so option D is unlikely.
Direct toxicity from chemotherapy (option E) should always be
considered, but given the spectrum of symptoms, it is also
unlikely in this patient. There is no established therapy for
adenoviral disease, but both cidofovir and ribavirin have been
used, as well as IV immunoglobulin.

RATIONALE (21) Answer: B


This patient has multidermatomal varicella zoster (shingles) that
requires acyclovir treatment. Valacyclovir is an enteral antiviral
and would not be indicated for the initial treatment of this
infection in an immunocompromised patient. There is no reason
to substitute linezolid for vancomycin or amphotericin B for the
echinocandin caspofungin.

RATIONALE (22) Answer: E

The diagnosis is Ebola; therefore, the correct therapy is


replacement of coagulation factors (option E). The bloody
secretions are a clue to viral hemorrhagic fever; the exposure to
fruit bats in central Africa makes Ebola virus the most likely
etiology. Treatment for viral hemorrhagic fever infection is
primarily supportive. There are no effective agents against Ebola,
so treatment is primarily supportive care. However, with Ebola
infections, aggressive volume resuscitation (option D) has been
linked to poor pulmonary outcomes. Careful monitoring of
coagulation factors and appropriate replacements have been
shown to support the patient while the immune system builds a
response to the virus

RATIONALE (23) Answer: A

Despite the reported sporadic resistance, oseltamivir is the best


option for this patient. Inhaled zanamivir (option B) should have
good activity against influenza but is relatively contraindicated
in patients with asthma and cannot be effectively aerosolized for
mechanical ventilation. Most circulating strains of influenza are
now amantadine resistant, ruling out option C. No data exists to
support the efficacy of IV immunoglobulin (option D) or
foscarnet (option E) in this setting. Currently, oseltamivir is only
available in oral formulation. IV oseltamivir and IV peramivir
(another influenza neuraminidase inhibitor) are in trials, but
may be available for compassionate use

RATIONALE (24) Answer: C

This patient has abacavir hypersensitivity syndrome or reaction.


While many antiretrovirals cause hypersensitivity, the reaction
to abacavir is well described and can be severe. It is associated
with the HLA-B57 haplotype. It can occur from 1 week to 2
months after starting abacavir and usually involves fever, rash,
nausea, vomiting, and abdominal pain. Rarely, the clinical course
can worsen, with associated hypotension, acute interstitial
pneumonitis, and cardiopulmonary failure.

RATIONALE (25) Answer: D

The correct management is to continue the current antiretroviral


medications. None of these medications potentiates seizure.
Unless there is a high suspicion of direct toxicity or change in
renal function, the antiretroviral medications do not need to be
adjusted.
PART 5: Hepatic, Gastrointestinal, Hematologic and Oncologic
Disease in the ICU

Instructions: For each question, select the most correct answer.

1. What are the risk factors that have the strongest correlation
with stress-related bleeding in critically ill patients?

A. Mechanical ventilation and glucocorticoids

B. Coagulopathy and sepsis

C. Mechanical ventilation and coagulopathy

D. Sepsis and renal failure

E. Mechanical ventilation and hypotension

2. A 62-year-old man with history of chronic obstructive


pulmonary disorder was admitted to the ICU with respiratory
failure. The patient has been on mechanical ventilation for 7
days. He is currently receiving antibiotics for pneumonia. He has
received intermittent doses of benzodiazepines and narcotics for
sedation and analgesia. Over the past 2 days, he has developed a
progressive abdominal distension with poor tolerance of enteral
feedings secondary to high gastric residuals. On examination,
you notice a distended abdomen with no evidence of surgical
scars. There is a significant tympany on percussion and diffuse
tenderness on palpation, with no rebound tenderness, guarding,
rigidity, or evidence of ascites. On auscultation, bowel sounds are
decreased on all quadrants. An abdominal radiograph (flat plate)
is obtained and shows dilatation of the cecum (12 cm) with no
other significant findings.
Which of the following management steps is most appropriate?

A. Surgical intervention

B. Central venous alimentation for 2 weeks

C. Neostigmine

D. Atropine

E. Colonoscopy

3. In which of the following clinical scenarios is the placement of


a large-bore nasogastric tube most indicated?

A. Suspected partial versus complete bowel obstruction

B. Acute pancreatitis

C. To assist with diagnosis of gastrointestinal bleeding

D. Refractory nausea and vomiting

E. Suspected esophageal perforation

4. A generally healthy, 64-year-old man presents to the


emergency department with acute hematemesis and
hypotension. Due to an ankle sprain, he has been taking
ibuprofen, 800 mg every 8 hours for the past 2 weeks. After
initial resuscitation and initiation of an IV proton pump inhibitor,
the patient is stabilized and admitted to the ICU. An
esophagogastroduodenoscopy (EGD) performed within 24 hours
reveals a 1.5-cm gastric ulcer with a visible vessel and adherent
clot on the lesser curve of the stomach. This is treated with
epinephrine and heater probe cautery. Two days later, the
patient develops acute hypotension, tachycardia, a drop in
hematocrit, and melena.

After resuscitation and stabilization, which of the following is the


best next step in management?

A. Urgent surgical intervention

B. Immediate CT of abdomen and pelvis

C. IV octreotide

D. Repeat EGD

E. Angiography

5. A 45-year-old, otherwise healthy man is brought by


ambulance to the emergency department because of a massive
upper gastrointestinal tract bleed. Initial laboratory results show
hemoglobin level of 3 g/dL, hematocrit of 9%, platelet count of
200,000/µL, total bilirubin level of 1.4 mg/dL, and international
normalized ratio (INR) of 1.2. He is resuscitated and admitted to
the ICU. An esophagogastroduodenoscopy (EGD) reveals a large
duodenal ulcer with active bleeding. He undergoes endoscopic
therapy and is placed on a proton pump inhibitor drip. Over the
course of the first 48 hours, the patient requires 12 units of
packed red blood cells. On hospital day 3, the patient’s
laboratory results reveal a platelet count of 90,000/µL, total
bilirubin level of 2.4 mg/dL (indirect bilirubin level of 1.8
mg/dL) and an INR of 2.2. His complete blood count is stable,
and testing for immunoglobulin G antibodies to Helicobacter
pylori returns positive results.
Which of the following is the most likely cause of the patient’s
thrombocytopenia?

A. Idiopathic thrombocytopenic purpura

B. Disseminated intravascular coagulation

C. Heparin-induced thrombocytopenia

D. Dilutional effect from massive transfusion of red blood cells

E. Hepatic failure from proton pump inhibitor therapy

6. A 69-year-old woman with a history of congestive heart failure


and chronic atrial fibrillation comes to the emergency
department because of the abrupt onset of right lower quadrant
abdominal pain and hematochezia. Current medications are
digoxin and furosemide. On physical examination, temperature is
37.9°C (100.2°F), pulse rate is 100/min with irregular rhythm,
and BP is 140/80 mm Hg. Moderate right lower quadrant
abdominal tenderness is noted on palpation. No masses are
evident. Rectal examination discloses bloody stool. Hemoglobin
level is 13.5 g/dL. Leukocyte count is 13,500/µL. After a colon
preparation, a colonoscopy is performed, which reveals mucosal
erythema, edema, and friability involving the cecum, ascending
colon, and proximal transverse colon only. Examination of a stool
specimen shows many leukocytes, but no ova or parasites.
Results of stool cultures are pending. The patient clinically
improves, and her abdominal pain is gradually improving.

Which of the following is most appropriate to recommend at this


time?

A. Supportive care and observation


B. Corticosteroid therapy

C. IV antibiotics

D. Mesenteric angiography

E. Surgical consultation

7. A 50-year-old woman with alcoholic cirrhosis and a model for


end-stage liver disease (MELD) score of 20, is admitted to the
ICU for management of unexplained hypoxemia. Her arterial
blood gas results on room air show pH of 7.43, Paco2 of 36 mm
Hg, and Pao2 of 55 mm Hg. Subjectively, she describes dyspnea
when she is moved from a supine to an upright position. Her
diagnostic evaluation so far has included a normal chest
radiograph and a low-probability ventilation-perfusion scan.

The next diagnostic test for the evaluation of her hypoxemia


should be:

A. Transthoracic echocardiography with a bubble study

B. IV contrast-enhanced chest CT

C. Dobutamine stress echocardiography

D. Cardiac MRI E. Bronchoscopy

8. A 45-year-old man with hepatitis C cirrhosis, a model for end-


stage liver disease (MELD) score of 29, and prior complications
of ascites and spontaneous bacterial peritonitis presents to the
ICU with fever and encephalopathy. Due to worsening
encephalopathy, endotracheal intubation is required for airway
protection.

The most appropriate management of hepatic encephalopathy in


this patient includes:

A. Placement of an intracranial pressure monitor

B. IV mannitol therapy

C. Enteral rifaximin therapy

D. IV hypertonic saline

E. Initiation of hypothermia

9. A 45-year-old man with alcoholic cirrhosis and a previously


documented hepatic hydrothorax presents with worsening
lethargy and hypotension. Initially, his temperature is 39.8°C
(103.6°F), pulse rate is 110/ min, BP is 80/50 mm Hg, and RR is
24/min. His examination reveals decreased breath sounds in the
right lung base, marked scleral icterus, asterixis, and no ascites.
Blood and urine cultures are obtained. A chest radiograph
demonstrates a moderate-sized right pleural effusion.

Additional infectious evaluation at this stage should include:

A. MRI with gadolinium of the brain

B. Thoracentesis with pleural fluid analysis

C. Lumbar puncture

D. Chest CT with contrast


E. Bronchoscopy with bronchoalveolar lavage

10. A 22-year-old woman presents with acute liver failure due to


acetaminophen toxicity. Her vital signs are stable. On
examination, grade 2 encephalopathy is found, and the INR is 7.0.
No signs of active bleeding are noted, and serum hemoglobin
level is 13 g/dL.

The most appropriate next step in the management of her severe


coagulopathy is:

A. Administration of fresh frozen plasma

B. Administration of recombinant factor VIIa

C. Plasmapheresis

D. Monitoring without intervention

E. Administration of factor IX complex

11. A 36-year-old woman undergoes a percutaneous liver biopsy


to assess for autoimmune hepatitis. Four days later, she presents
to the emergency department with hypotension, tachycardia,
melena, and a significant drop in hematocrit, with no associated
abdominal pain. After resuscitation, noncontrast abdominal CT
findings are negative for an intraperitoneal bleed.
Esophagogastroduodenoscopy reveals copious blood coming
from the ampulla of Vater.

Which of the following is the best next step in management?

A. Observation with blood transfusions as needed


B. Octreotide infusion

C. Angiography with possible embolization

D. Endoscopic retrograde cholangiopancreatography with stent


placement

E. Surgical consultation

12. A 62-year-old man with hepatitis C cirrhosis is transferred from


an outside hospital with suspected septic shock. Following
endotracheal intubation for worsening shock, aggressive
resuscitation with crystalloids and vasoactive agent support is
initiated, resulting in stabilization of the mean arterial pressure
above 65 mm Hg. Blood and urine cultures are sent, and broad-
spectrum antibacterial therapy is initiated. Twelve hours after
aggressive resuscitation, the patient demonstrates a worsening
clinical status characterized by worsening hypotension,
abdominal distension, high ventilator peak pressures, and
oliguria. Chest and abdominal radiographs are unremarkable. A
transthoracic echocardiogram demonstrates underfilling of the
right heart and collapse of the inferior vena cava. Abdominal
ultrasonography reveals hepatomegaly and a large amount of
ascites.

What is the best next step in management?

A. Abdominal MRI with and without contrast

B. Addition of antifungal therapy

C. Repeat echocardiography
D. Measurement of bladder pressure and paracentesis

E. Nephrology consultation for hemodialysis

13. A 60-year-old man with Child-Pugh stage C cirrhosis


secondary to hepatitis C and alcohol abuse presents with
confusion and is diagnosed with hepatic encephalopathy.
Treatment with lactulose is initiated.

On admission, the serum creatinine level is found to be 1.5


mg/dL, and over the next few days increases to 3.5 mg/dL
despite daily administration of albumin, placement of Foley
catheter, and avoidance of all potentially nephrotoxic drugs.

On physical examination, the patient has signs of chronic liver


failure, and his BP is 98/55 mm Hg, with HR of 70/min. He is
confused.

Laboratory results are as follows: sodium, 137 mEq/L;


potassium, 4.9 mEq/L; chloride, 92 mEq/L; bicarbonate, 17
mEq/L; blood urea nitrogen, 45 mg/dL; creatinine, 3.5 mg/ dL;
aspartate aminotransferase, 55 U/L; alanine aminotransferase,
68 U/L; international normalized ratio, 4.0; WBCs, 1,900/µL;
hemoglobin, 8.9 g/dL; and platelets, 56,000/µL. Urine sodium
level is less than 10 mg/dL.

Which of the following is the best next step in the management of


this patient?

A. Octreotide and midodrine

B. Transjugular intrahepatic portosystemic shunt

C. Ribavirin and interferon treatment


D. Dopamine

14. A 20-year-old with hemophilia A sustained significant chest


wall trauma in a motor vehicle accident. After thoracostomy for a
right pneumothorax, he has 100 mL/h of bloody output from the
chest tube. He has received transfusions multiple times in the
past and is known to have inhibitors to factor VIII.

Which of the following should be administered?

A. Recombinant factor VIII

B. Cryoprecipitate

C. Factor IX concentrate

D. Recombinant factor VIIa

E. Fresh frozen plasma

15. A 50-year-old patient with chronic kidney disease is admitted


to the ICU at 11 pm with altered mental status for 2 days, fever,
tachycardia, and hypoxemia. A chest radiograph reveals a right
lower lobe infiltrate. Prolonged bleeding from venipuncture sites
and bleeding from the nasal mucosa after passage of a
nasogastric tube is also noted. Platelet count is 100,000/µL,
prothrombin time is 11.5 seconds, partial thromboplastin time is
35 seconds, blood urea nitrogen level is 150 mg/dL, and
creatinine level is 10 mg/dL.

Which of the following treatments is most likely to result in rapid


improvement in the bleeding diathesis?
A. Platelet transfusion

B. Hemodialysis for uremia

C. Conjugated estrogens

D. Cryoprecipitate

E. Desmopressin acetate A

16. 55-year-old man with coronary artery disease and


hypertension is transferred from the telemetry unit to the ICU
after developing an altered mental status and fever. The patient
underwent percutaneous coronary angioplasty 1 month ago with
a stent placement and was recently admitted to the hospital for
evaluation of chest pains. On physical examination, temperature
is 39.9°C (103.8°F), HR is 115/min, RR is 25/min and BP is
120/90 mm Hg. There is no jugular vein distension, no S3, lungs
are clear on auscultation, abdominal examination findings are
unremarkable, and extremities are cool and mottled. Neurologic
examination reveals a confused patient who is able to move all
extremities spontaneously and opens his eyes to voice.

Laboratory data include the following: hemoglobin, 8.1 g/dL;


WBCs, 10,000/µL; platelets, 4,000/µL; partial thromboplastin
time, 25 seconds; international normalized ratio, 1.0;
prothrombin time, 12 seconds; sodium, 139 mEq/L; potassium,
4.5 mEq/L; chloride, 102 mEq/L; bicarbonate, 22 mEq/L; and
serum creatinine, 2.2 mg/dL. A peripheral blood smear is shown
in the Figure.
Which of the following is most appropriate as initial treatment
for this patient?

A. Argatroban

B. Dexamethasone

C. Plasmapheresis

D. Platelet transfusion

E. IV immunoglobulin

17. A 40-year-old woman is admitted to the hospital with


probable immune thrombocytopenic purpura. She has some
petechiae on her lower extremities and ecchymosis in areas of
mild trauma. Her platelet count is 5,000/µL and prothrombin
time and partial thromboplastin time are normal. She is started
on prednisone, 80 mg/day. On day 2 in the hospital, she slips and
falls, sustaining a femoral neck fracture. Surgical repair is
required in the next several days.
Which of the following is the most appropriate intervention to
increase the platelet count prior to surgery?

A. Platelet transfusion

B. Splenectomy

C. IV immunoglobulin

D. Rituximab

E. Danazol

18. A 65-year-old woman is admitted to the ICU after coronary


bypass surgery for acute myocardial infarction with cardiogenic
shock. The postoperative course is complicated by acute renal
insufficiency, and on postoperative day 5 the patient develops
acute dyspnea with hypoxemia. A diagnosis of left lower
extremity deep venous thrombosis is made by lower extremity
ultrasonography, with probable pulmonary embolism. The
patient is started on IV unfractionated heparin (weight-based
nomogram) to achieve a partial thromboplastin time (PTT) 1.5 to
2 times the normal value. Laboratory studies at this time reveal
normal electrolyte levels, a blood urea nitrogen level of 45
mg/dL, creatinine level of 2.8 mg/dL, WBC count of 9,000/µL,
hemoglobin level of 10.1 g/dL, platelet count of 250,000/µL, and
normal prothrombin time, international normalized ratio, and
partial thromboplastin time. After 2 days of therapeutic
anticoagulation with unfractionated heparin, the patient reports
right lower extremity pain. Examination of the right lower
extremity reveals a cold painful extremity with loss of distal
pulses. Laboratory studies reveal a blood urea nitrogen level of
50 mg/dL, creatinine level of 2.8 mg/dL, platelet count of
90,000/µL, international normalized ratio of 1.2, and PTT of 50
seconds.

Which of the following therapeutic options is most appropriate


at this time?

A. Increase unfractionated heparin to achieve a higher PTT.

B. Discontinue unfractionated heparin and start enoxaparin.

C. Discontinue unfractionated heparin and start warfarin.

D. Discontinue unfractionated heparin and start argatroban.

E. Discontinue anticoagulation.

19. A 22-year-old, otherwise healthy man suffered blunt


abdominal trauma in a motor vehicle collision. After exploratory
laparotomy with a splenectomy, he required transfusion of 4
units of packed red blood cells. Postoperatively, he had diffuse
oozing at the site of the incision. His laboratory studies
preoperatively revealed a platelet count of 210,000/µL,
prothrombin time of 12.8 seconds, and partial thromboplastin
time of 48 seconds.

Which of the following is the most likely etiology of this patient’s


bleeding?

A. Disseminated intravascular coagulation

B. Hypothermia

C. Von Willebrand disease


D. Salicylate use

E. Citrate toxicity

20. A patient with septic shock secondary to pneumonia is


admitted and treated in the ICU. On hospital day 3, his blood
pressure is stable, and he is being weaned from norepinephrine.
Hemoglobin is 8.5 g/dL, platelets are 35,000/µL, prothrombin
time is 18 seconds, and partial thromboplastin time is 40
seconds.

Which of the following best characterizes the blood products


that should be transfused at this time?

A. Red blood cells, platelets, and fresh-frozen plasma

B. Red blood cells and platelets only

C. Red blood cells only

D. Platelets only

E. No blood products

21. A 68-year-old man with a 100-pack-year smoking history is


admitted to the ICU with hypotension. He has a 2-month history
of weight loss and fatigue and a 3-day history of anorexia and
polyuria. On admission, his temperature is 37°C (98.6°F), BP is
90/50 mm Hg, HR is 114/min, and RR is 18/min. The patient is
lethargic but has no focal neurologic deficits. Initial laboratory
results are as follows: sodium, 148 mEq/L; potassium, 4.2
mEq/L; chloride, 110 mEq/L; carbon dioxide, 28 mEq/L; blood
urea nitrogen, 35 mg/dL; creatinine, 2.1 mg/dL; and glucose, 120
mg/dL. ECG reveals sinus tachycardia and shortened QT interval.
The admission chest radiograph is shown in the Figure.

In addition to IV normal saline solution, which of the following


treatments will most likely benefit this patient?

A. Methylprednisolone

B. Magnesium sulfate

C. Pamidronate

D. Sodium phosphate
E. Mannitol

22. A 28-year-old man with hemoglobin SC disease is admitted to


the hospital for leg and chest pain secondary to a vaso-occlusive
crisis. On admission, chest radiography shows mild cardiomegaly
but no infiltrates, and pulse oximetry on room air is 97%. He is
treated with IV hydration and analgesics.

On hospital day 3, the patient develops a fever of 38°C (100.3°F)


and shortness of breath. He continues to report sternal pain.
Oxygen saturation on room air is 85% and rales are noted in the
right lower lobe. Chest radiography confirms infiltrates in the
right and left lower lobe. His hemoglobin at this time is 9.2 g/dL,
with WBC count of 20,000/µL (unchanged from admission). The
patient is transferred to the ICU for further care.

Which of the following interventions is most appropriate for this


patient?

A. Exchange transfusion

B. Simple transfusion

C. Administration of IV dexamethasone

D. Bronchoalveolar lavage to exclude infection

E. Administration of inhaled nitric oxide

23. A 55-year-old woman with a history of breast cancer is


brought to the emergency department with back pain of several
weeks’ duration, urinary retention, constipation, and difficulty
walking since waking up several hours before presentation. On
examination, she has decreased sensation below the navel and
decreased strength in both lower extremities. Spinal MRI shows
a solitary mass compressing the cord at T9.

Which of the following treatments is most likely to result in


recovery of ambulation in this patient?

A. Chemotherapy alone

B. Radiation therapy

C. Surgical decompression plus radiation

D. Dexamethasone alone

E. Chemotherapy and corticosteroids

A 40-year-old, previously healthy man is brought to the


emergency department with sudden onset of severe headache.
He appears somnolent. Head CT without contrast reveals a
subarachnoid hemorrhage. Laboratory work-up reveals a WBC
count of 10,000/µL with frequent immature cells, platelet count
of 45,000/µL, prothrombin time (PT) of 15 seconds, and partial
thromboplastin time (PTT) of 50 seconds. He is admitted to the
neurological ICU for management.

Which of the following tests would be most helpful in


determining the cause of bleeding and guiding immediate
therapy?

A. Fibrinogen and D-dimer tests

B. PT and PTT mixing studies


C. Platelet function assay

D. Bone marrow biopsy

E. Review of the peripheral blood smear

25. An 18-year-old man is transferred from the hematology floor


to the medical ICU because of a potassium level of 6 mEq/L and
ECG changes on the third day after initiation of chemotherapy for
Burkitt lymphoma. Other laboratory results include a serum
creatinine level of 3 mg/dL, phosphorus level of 7 mg/dL,
calcium level of 6.5 mg/dL, and uric acid level of 16 mg/dL.

In addition to IV fluids and management of hyperkalemia and


hyperphosphatemia, which of the following interventions is most
appropriate?

A. Allopurinol

B. Immediate hemodialysis

C. Urine alkalization

D. Furosemide

E. Rasburicase

26. A 35-year-old woman presents to the emergency department


with fever and generalized weakness 10 days after starting her
third cycle of consolidation chemotherapy for acute myeloid
leukemia. She denies cough, mouth sores, and exposure to
illness. She has hypotension, tachycardia, and clear lungs. Her
central venous line is clean and nontender to palpation. WBC
count is 300/µL, hemoglobin level is 9 g/dL, and platelet count is
20,000/µL. After blood cultures are sent, she is admitted to the
medical ICU.

Which of the following management steps is most appropriate


for this patient?

A. Vancomycin alone

B. Amphotericin

C. Fluconazole

D. Waiting on culture results to guide antibiotic choice

E. Ceftazidime and vancomycin

27. A 45-year-old man is brought by his wife because of a 2-week


history of dyspnea, fatigue, and fever. On physical examination,
he is pale and has gum hypertrophy. BP is 100/70 mm Hg, pulse
rate is 95/min, temperature is 37.3°C (99°F), and pulse oximetry
is 87% on room air. Laboratory work-up reveals a WBC count of
300,000/µL with 95% blasts, platelet count of 25,000/µL, and
hemoglobin level of 8 g/dL. Chest radiography shows diffuse
interstitial infiltrates.

Which of the following is the hematologist most likely to


recommend for symptom control?

A. Leukapheresis

B. Exchange transfusion

C. Combination chemotherapy
D. Bone marrow biopsy

E. Emergent whole-body radiation

28. A 65-year-old woman presents with dyspnea and facial


fullness. The dyspnea has worsened significantly on the day of
admission. On examination, she has plethora, mild right arm
swelling, and prominent veins on the right side of her chest. BP is
130/85 mm Hg, pulse rate is 120/min, RR is 24 /min,
temperature is 37.3°C (99°F), and pulse oximetry is 87% on
room air. Chest radiography shows a right upper lobe density.

Which of the following is likely to provide the quickest relief and


is the best next option?

A. Warfarin

B. Diuretics

C. Steroids

D. Intravascular stenting

E. Radiation therapy

29. A 70-year-old woman is brought to the emergency


department because of confusion and somnolence at home. On
examination, she appears thin. Her vital signs are normal and she
is not orthostatic. Her mucous membranes are moist and her
skin turgor is normal. During the examination, she has a
witnessed seizure. Laboratory evaluation reveals sodium level of
120 mEq/L; blood urea nitrogen and creatinine levels are
normal. Complete blood count shows a WBC count of 8,500/µL,
hemoglobin level of 9 g/dL, and platelet count of 200,000/µL.
Chest radiography shows a large hilar shadow and chest CT
confirms a pulmonary mass.

Which of the following best explains the pathophysiology of this


condition?

A. Polydipsia that induces a hypervolemic state

B. Volume depletion due to poor oral intake

C. Inability to excrete dilute urine and retention of water due to


excessive release of antidiuretic hormone

D. Reduced cardiac output leading to the release of antidiuretic


hormone via the J receptors

E. Impaired renal absorption of sodium in the proximal renal


tubule
PART 5: Hepatic, Gastrointestinal, Hematologic and Oncologic
Disease in the ICU

ANSWERS: 1–C; 2–C; 3–A; 4–D; 5–D; 6–E; 7–A; 8–C; 9–B; 10–D;
11–C; 12–D; 13–A; 14–D; 15–E; 16–C; 17–C; 18–D; 19–C; 20–E;
21–C; 22–A; 23–C; 24–A; 25–E; 26–E; 27–A; 28–D; 29–C

RATIONALE (1) Answer: C

The risk of developing stress-related gastrointestinal bleeding


varies in critically ill patients. It is important to recognize
patients with increased risk factors in order to develop cost-
effective prophylactic regimens. In a landmark study on risk
factors, Cook et al1 found that the strongest independent risk
factors for the development of stress-related bleeding in ICU
patients were mechanical ventilation for more than 48 hours (RR
16/min, P <0.01) and coagulopathy (RR 4.3/min, P <0.01).
Coagulopathy was defined as a platelet count less than
50,000/mL, prothrombin time greater than 1.5 times control
value, or a partial thromboplastin time greater than twice the
control value. Hypotension was found to have an increased odds
ratio (3.7), but did not achieve statistical significance. Renal
failure, sepsis, liver failure, enteral feeding, and glucocorticoids
did not achieve statistical significance. In this study, the
incidence of clinically significant stress-related bleeding was
3.7% in patients with risk factors compared to 0.1% in patients
without risk factors.

RATIONALE (2) Answer: C

The patient’s presentation is consistent with acute colonic


pseudo-obstruction, also known as Ogilvie syndrome. Ogilvie
syndrome is characterized by gross dilation of the colon, usually
the cecum, in the absence of an anatomic obstructing lesion. The
small intestine might also be dilated in some cases. This
syndrome can be seen in critically ill patients admitted to the ICU
or long-term acute care centers. Risk factors include electrolyte
abnormalities and use of antimotility agents and narcotics. The
exact mechanism that produces colonic atony remains unclear.
However, autonomic nervous dysfunction is thought to play an
important role. Clinical presentation usually includes abdominal
distension, tympany, diffuse tenderness, and reduced bowel
sounds. Plain radiographs will reveal a dilated colon, most often
cecum. Complications such as ischemia and perforation are
uncommon with cecal diameters of less than 12 cm. The rate of
development and the duration of dilation probably relate better
to the rate of complications. Management involves supportive
measures, with treatment of the underlying disease and the
correction of any electrolyte abnormalities. Gastrointestinal
decompression with nasogastric and rectal tubes can be used.
Studies have demonstrated the effectiveness of IV neostigmine, 2
mg, in producing rapid decompression and preventing
recurrence. Neostigmine is well tolerated in general but may
produce bradycardia, which is responsive to atropine. Cases
refractory to conservative and medical treatment may require
colonoscopy for decompression. Colonoscopy in the ICU patient
with an unprepared bowel has a high complication rate and
should be avoided, if possible. Surgery is indicated in patients
where other therapeutic modalities fail or in those who develop
complications such as ischemia and perforation. Initiation of
central venous alimentation is not indicated in patients with
prompt response to therapy.

RATIONALE (3) Answer: A

Placement of a large-bore nasogastric (NG) tube has been


described as one of the most painful experiences a conscious
patient may endure. There are very few evidence-based
indications for use of a large-bore NG tube. There is even data to
suggest that patients do not need an NG tube after undergoing
bowel surgery. That being said, decompression of a suspected
bowel obstruction can be an important therapeutic maneuver
and may alleviate abdominal distension, nausea/vomiting, and
discomfort.

Although nasojejunal enteric feeds have proven benefit in


patients with acute pancreatitis (option B), a large-bore tube is
not necessary.

A large-bore NG tube is not necessary in diagnosing an upper


gastrointestinal tract bleed (option C). In fact, it can be
misleading. Aggressive resuscitation, medical therapy, and
urgent endoscopy is the standard of care for suspected upper
gastrointestinal tract bleeding. Although a large-bore NG tube
may theoretically assist with clearing out large blood clots from
the stomach and enabling better endoscopic viewing, adequate
clearance is rarely achieved and IV erythromycin is a less
invasive choice.

A large-bore NG tube is not indicated with refractory nausea and


vomiting (option D).

A Dobhoff (small-bore) feeding tube may be necessary, and


antiemetic therapy with IV fluids should be the initial
management during investigation of the cause of such
symptoms.

A large-bore NG tube is contraindicated in cases of suspected


esophageal rupture. Therefore, option E is also incorrect.

RATIONALE (4) Answer: D


The patient presents with a likely nonsteroidal antiinflammatory
drug–induced bleeding gastric ulcer with some high-risk features
(visible vessel, adherent clot). Medical and endoscopic therapy is
applied and the patient stabilizes. Forty-eight hours later, the
patient has a significant rebleed. Since he is stabilized, a repeat
esophagogastroduodenoscopy (EGD) to confirm the site of
rebleeding is the best answer. If the same ulcer is rebleeding,
repeat endoscopic therapy is certainly indicated and will reduce
the risk of rebleeding. The original ulcer characteristics (only 1.5
cm, unlikely to be malignant) do not mandate surgery. There is
no information given that suggests perforation; therefore, CT
(option B) is not needed.

If the patient were too unstable for upper endoscopy, an attempt


at angiography (option E) would be reasonable.

RATIONALE (5) Answer: D

Based on the clinical scenario and other laboratory


abnormalities, the most likely cause of thrombocytopenia in this
patient is a dilutional effect. Most of the literature regarding
dilutional coagulopathy in the setting of massive blood
transfusions relates to trauma. Traditionally, transfusing more
than 10 units of packed red blood cells has been considered
“massive,” and it is recommended to give fresh frozen plasma
after approximately 6 units of packed red blood cells have been
transfused. Dilution can cause a rise in prothrombin time, partial
thromboplastin time, and international normalized ratio (INR),
as well as a drop in platelet count. Although Helicobacter pylori
is associated with chronic idiopathic thrombocytopenic purpura,
the acuity of the thrombocytopenia and other lab abnormalities
makes this answer (option A) incorrect. Although disseminated
intravascular coagulation is a possible cause of
thrombocytopenia and elevated INR, there is no information to
suggest that the patient has acute infection or sepsis. Therefore,
option B is incorrect. There is no mention that the patient
received heparin therapy (and he probably did not since he
presented with a gastrointestinal bleed), and the timing of the
decrease in platelets would be a bit unusual for heparin-induced
thrombocytopenia. Therefore, option C is also incorrect. Proton
pump inhibitors are not likely to cause acute liver failure (option
E), and there is no clinical evidence of such a process in this case.
The elevated INR is due to a dilutional coagulopathy, and the
indirect bilirubin elevation is probably due to the blood
transfusions as well.

RATIONALE (6) ANSWER: E

This patient has developed isolated right-sided ischemic colitis,


probably secondary to an embolic event due to atrial fibrillation.
Right-sided ischemic colitis portends the worst prognosis of all
types of colonic ischemia with regard to subsequent mesenteric
ischemia. This pattern is seen with superior mesenteric artery
ischemia, and most patients will go on to require surgery. Since
this process rarely leads to recovery following conservative
management, supportive care and observation (option A) is not
the appropriate therapy. Imaging and a surgical consultation
(option E) should be performed. Corticosteroids (option B) and
antibiotics (option C) are not indicated since the etiology of the
colonic inflammation is not inflammatory or infectious.
Mesenteric angiography (option D) may further evaluate her
anatomy and assess for arterial stenosis, but it is invasive. Since
the patient is clinically improving, evaluation with MR
angiography or CT angiography may be more appropriate.

RATIONALE (7) Answer: A


The clinical presentation suggests hepatopulmonary syndrome
(HPS), which is characterized by intrapulmonary shunting–
induced hypoxemia due to pathologic pulmonary vascular
dilation, and consequent diffusion-impeded transfer of oxygen
from the alveolus to the pulmonary arteriole. The clinical
presentation is characterized by platypnea (subjective shortness
of breath when moved from a supine to a sitting position) and
orthodeoxia (desaturation when moved from a supine to a sitting
position). The proposed explanation for the exacerbation of the
hypoxemia in the sitting position is that more blood flow is
directed to the basilar West zone 3 lung segments, where the
pathologic pulmonary vasodilation is more pronounced. The test
of choice to support the diagnosis of HPS is a transthoracic
echocardiogram with a bubble study, which will demonstrate
delayed appearance of bubbles in the left heart in 4 to 6 cardiac
cycles. The other choices do not provide diagnostic information
regarding HPS.

RATIONALE (8) Answer: C

In the setting of hepatic encephalopathy (HE) associated with


acute-on-chronic liver failure, enteral administration of rifaximin
has been shown to reverse HE. The mechanism of action of
rifaximin is the reduction of gastrointestinal ammonia
production by the elimination of gut bacterial flora. In contrast to
HE associated with acute liver failure, the HE of acute-on-chronic
liver failure is typically not associated with intracranial
hypertension and cerebral edema. Therefore, there is no role for
empiric intracranial pressure monitoring, osmotherapy, or
hypothermia in HE associated with acute-on-chronic liver
failure. An additional therapeutic option in treating the HE of
acute-on-chronic liver failure is enteral or rectal lactulose.

RATIONALE (9) Answer: B


The presence of a hepatic hydrothorax increases the risk of
spontaneous bacterial empyema (SBEM); therefore, pleural fluid
analysis is indicated to evaluate for this infectious process. The
diagnostic criteria for SBEM are similar to those for spontaneous
bacterial peritonitis (SBP), and the pleural fluid analysis will
demonstrate a transudate with an absolute neutrophil count
greater than 250/mL, with or without a Gram stain and culture
positive for a specific microbial process. The etiology of SBEM
involves the development of an infection in a preexisting hepatic
hydrothorax, which occurs due to the passage of ascites from the
peritoneal to the thoracic cavity across diaphragmatic defects.
The treatment is similar to SBP, and involves a 5-day course of
broad-spectrum antibacterial therapy. There is no role for large-
bore chest tube drainage in the treatment of SBEM, and in fact,
this intervention could lead to hypovolemic shock and
reexpansion pulmonary edema.

RATIONALE (10) Answer: D

In the absence of bleeding, there is no indication to reverse the


coagulopathy associated with acute liver failure. Instead, since
the international normalized ratio is the best measure of hepatic
synthetic function, it should be used as a biochemical marker to
predict recovery of intrinsic hepatic function or the need for liver
transplantation. The risk of bleeding in the setting of
coagulopathy associated with acute liver failure has been
reported to be extremely low in multiple observational studies. A
potential explanation for these observations is the loss of both
procoagulant and anticoagulant factors in the setting of acute
liver failure, resulting in a normal hemostatic equilibrium.

RATIONALE (11) Answer: C


Hemobilia is a rare cause of upper gastrointestingal tract
bleeding that has increased in incidence due to more frequent
hepatobiliary procedures such as percutaneous liver biopsy,
percutaneous transhepatic cholangiography, endoscopic
retrograde cholangiopancreatography (ERCP) with
biopsy/brushing/ stenting, and interventional angiography.
Hemobilia is diagnosed by upper endoscopy, when blood is seen
flowing from the ampulla of Vater. Hemobilia is often treated
with angiography with embolization. Option A is incorrect since
the patient is actively bleeding and is presenting with
hemodynamic instability. Option B is incorrect as octreotide has
no role in treating hemobilia. Octreotide is indicated in cases of
portal hypertensive bleeding, including varices. Option C is the
correct answer. Option D is incorrect, and there is no role for
ERCP unless a biliary obstruction develops. Option E is not
correct— interventional radiology should be called (not
surgery). If an intraperitoneal bleed is seen, surgery should be
consulted.

RATIONALE (12) Answer: D

This is a case of abdominal compartment syndrome (ACS) due to


excessive ascites accumulation in a cirrhotic patient following
aggressive resuscitation for shock. Option A is incorrect; an MRI
may provide additional information regarding abdominal
pathology, but is not the next required step in this critically ill
patient.

Option B is incorrect; although antifungal therapy could be


considered, it does not address the immediate concern for ACS.
Option C is incorrect; a repeat echocardiography is not indicated.
Option D is correct; this will diagnose ACS, and a paracentesis
will decompress the peritoneum and relieve inferior vena cava
compression, thereby improving venous return and mean
arterial pressure. Option E is incorrect; though the oliguria may
progress to acute kidney injury, the best next step is acute
decompression of the abdomen.

RATIONALE (13) Answer: A

The patient has hepatorenal syndrome, a disease that can be


rapidly fatal. Criteria for diagnosis have been recently revised
and highlight the importance of infusion of albumin, 1 g/kg daily
for 2 days prior to making the diagnosis. Several therapies have
been studied and failed to show any survival advantage. The
combination of octreotide and midrodrine has been shown to
improve survival in such patients (option A is correct).

Dopamine administration has no known impact on hepatorenal


syndrome, so option D is incorrect. Treating hepatitis C with
antivirals in this patient is contraindicated. Thus, option C is
incorrect. Transjugular intrahepatic portosystemic shunt
treatment can help to improve short-term outcomes in patients
awaiting liver transplant, but since it can be associated with
complications, it should be considered as a last-resort therapy
after all others have failed. Therefore, option B is incorrect.

RATIONALE (14) Answer: D

One of the approved indications for use of recombinant factor


VIIa is serious bleeding in hemophiliacs with inhibitors. Factor
VIIa enhances thrombin generation, leading to hemostasis.
Factor VIII in any form (cryoprecipitate, plasma-derived, or
recombinant) would be ineffective due to the presence of
inhibitors. Factor XI requires factor VIII as a cofactor to activate
factor X. Fresh frozen plasma does not contain significant
amounts of factor VII to allow sufficient thrombin generation for
effective clotting. Activated prothrombin complex concentrates
are another hemostatic option for hemophiliacs with inhibitor
levels.

RATIONALE (15) Answer: E

Uremia is one of the most common causes of platelet dysfunction


in the critically ill, and bleeding from invasive procedures and
surgery may be difficult to control. Several options exist to
correct platelet dysfunction. Hemodialysis to lower the blood
urea nitrogen level would be optimal, but requires considerable
time to institute at night in many situations. Hemodialysis may
also contribute to bleeding by continuous platelet activation,
which is induced by the interaction between blood and artificial
surfaces and results in platelet exhaustion. Thus, option B is
incorrect. Continuous renal replacement techniques would not
be expected to improve the uremia rapidly. IV desmopressin
acetate (DDAVP), 0.3 µg/kg, has a rapid effect on improving
platelet function. The onset of action is within 1 hour and the
effects last 4–8 hours. Subcutaneous injection and nasal spray
are alternative routes of administration. DDAVP results in a 2- to
6-fold increase in both factor VIII and von Willebrand factor,
which increases platelet adhesiveness. Unfortunately, this effect
is short-lived and patients rapidly develop tachyphylaxis. DDAVP
is most helpful in controlling emergent bleeding while other
measures are being instituted. Platelet transfusions (option A)
would be of no benefit since transfused platelets are exposed to
the same uremic environment. Conjugated estrogens (option C)
are of value in improving platelet dysfunction, but the effect
takes several days. Therefore, they are not an option for
emergent interventions. Erythropoietin is another long-term
treatment option that improves platelet function.
Cryoprecipitate (option D) has been used in the past, but is no
longer recommended because it is not effective in many patients.
It also exposes the patient to transfusion-related complications.
Keep in mind that the use of antibiotics such as penicillins and
certain cephalosporins may also potentiate platelet dysfunction.

RATIONALE (16) Answer: C

This patient has thrombotic thrombocytopenic purpura (TTP),


most likely as a result of treatment with ticlopidine after his
cardiac catheterization. TTP is characterized by the pentad of
thrombocytopenia, microangiopathic hemolytic anemia, fever,
neurologic findings, and renal dysfunction. These clinical
features rarely present simultaneously. The peripheral smear
that is shown has fragmented red cells and schistocytes, which
are typical findings for microangiopathic hemolytic anemia. In
the context of the clinical presentation, TTP is the most likely
diagnosis. Patients do not require all elements of the pentad to
be fulfilled for diagnosis of TTP, and a high level of vigilance
must be maintained to identify these patients in a timely fashion
and institute appropriate treatment. TTP is associated with
invasive enteric infections, chemotherapeutic agents such as
mitomycin, cancer, HIV infection, and antiplatelet agents
(ticlopidine and clopidogrel). There is also an idiopathic form of
TTP. TTP seems to be initiated by endothelial damage that leads
to the accumulation of large von Willebrand factor multimers
that lead to platelet aggregation and thrombin formation in
affected organ systems. The treatment of TTP, regardless of the
cause, is the discontinuation of the offending agent followed by
daily plasma exchange (plasmapheresis plus infusion of fresh
frozen plasma). Prompt therapy with plasma exchange decreases
the mortality of TTP to 10%, and permanent organ dysfunction is
uncommon. In severe cases that are refractory to conventional
therapy, glucocorticoid therapy, and also splenectomy, have been
reported to be useful adjunctive measures. Patients with TTP in
general should not be given platelet transfusions (unless there is
life-threatening or intracranial bleeding). Argatroban is reserved
for patients who develop heparin-induced thrombocytopenia
and require systemic anticoagulation. There has been no proven
benefit or role for systemic corticosteroids in the initial
treatment of TTP. Low-molecular-weight heparin and
immunoglobulin are not indicated in TTP.

RATIONALE (17) Answer: C

Patients with immune thrombocytopenic purpura (ITP) usually


do not have significant bleeding because the few platelets
present are young and active in coagulation. This patient has a
comorbidity that makes urgent surgical intervention necessary.
Although steroids are the first-line treatment, platelet counts
may not increase for several days to weeks. IV immunoglobulin
would be the best intervention to accomplish a transient
increase in platelets during the operative intervention. It is
administered at 0.5–1 g/kg/day, along with methylprednisolone,
1 g/day. Depending on the response, platelet transfusions may
be needed at the time of surgery. Another option is the use of
anti-D immunoglobulin, but this is an option only in patients who
are Rh D–positive and have a spleen. The hemolysis that can
occur with anti-D immunoglobulin would also preclude its use in
a patient undergoing surgery. Platelet transfusions alone are
unlikely to benefit this patient, since they would be subject to the
same antibody destruction as endogenous platelets. Splenectomy
would not be warranted; rituximab and danazol are options for
chronic ITP and would be unlikely to have rapid effects on the
platelet count.

RATIONALE (18) Answer: D

This patient has developed heparin-induced thrombocytopenia


(HIT) type II, with arterial thrombosis of the right lower
extremity. There are 2 types of heparin-associated
thrombocytopenia. Type I occurs in 10%–20% of patients
receiving unfractionated heparin. Nonimmune factors result in
mild decreases in platelet counts after 1 to 4 days of therapy
with heparin. Most patients will normalize the platelet count in a
few days despite heparin continuation. Type II is more severe
and is associated with thrombotic complications in 30%–80% of
cases. Development of HIT type II is related to the formation of
heparin-platelet factor 4 complexes and the induction of
immunoglobulin G antiheparin-PF4 antibodies. HIT type II
usually develops after 5 days of heparin exposure. However, in
patients with prior exposure to heparin it may appear earlier.
HIT may also occur after heparin is discontinued. Cardiac and
orthopedic surgery patients and ICU patients are at higher risk of
developing HIT. Once HIT type II is suspected, all sources of
heparin, including low-molecular-weight heparins, should be
discontinued. Patients with thrombotic complications (venous or
arterial) from HIT, or other indications for anticoagulation
should be treated with antithrombin agents such as lepirudin or
argatroban, or danaparoid (not available in the United States).
However, lepirudin is renally eliminated and needs dose
adjustment for renal dysfunction. Therefore, in patients with
renal insufficiency, argatroban would be the preferred choice
since it is hepatically metabolized. Warfarin anticoagulation can
exacerbate the prothrombotic state of HIT type II prior to
achieving full anticoagulation with an international normalized
ratio greater than 2.0. As a consequence, warfarin should be
withheld until there is evidence of improvement and the patient
is fully anticoagulated with a parenteral antithrombin agent.

RATIONALE (19) Answer: C

Von Willebrand disease is the most common inherited


coagulation disorder. The most common type, type I, is due to
decreased release or production of von Willebrand factor (VWF).
Other types are due to abnormal VWF polymers or decreased
levels of VWF and factor VIII activity. Bleeding is usually only
noted with trauma, surgery, and other invasive procedures.
Impaired platelet adhesion at the vascular site of injury results in
bleeding. Coagulation studies reveal only a prolonged partial
thromboplastin time and increased bleeding time. Treatment of
von Willebrand disease includes desmopressin acetate (DDAVP),
factor VIII concentrate, and cryoprecipitate. DDAVP is effective in
type I disease only. Factor VIII concentrate is preferred over
cryoprecipitate because of lower infection risk.

Acquired von Willebrand disease has also been described.


Associated conditions include lymphoproliferative and
myeloproliferative disorders and cardiovascular defects such as
endocarditis and septal defects.

Salicylates (option D) would not prolong the partial


thromboplastin time, and citrate toxicity (option E) is unlikely
since only 4 units of red blood cells were transfused.
Hypothermia (option B) postoperatively can result in platelet
dysfunction, but is unlikely to result in an isolated prolonged
partial thromboplastin time. Likewise, the normal platelet count
and normal prothrombin time make disseminated intravascular
coagulation (option A) an unlikely cause of the oozing.

RATIONALE (20) Answer: E

In this stable patient recovering from septic shock without


evidence of bleeding, no blood products are needed at this time.
According to the Canadian Critical Care Trials Group prospective
study of transfusion thresholds2, transfusion of red blood cells is
recommended only when hemoglobin level decreases below 7
g/dL, and should target a hemoglobin level of 7 to 9 g/dL. The
effect of red blood cell transfusion in patients with sepsis has
been evaluated in several studies and shows an increase in
oxygen delivery but no increase in oxygen consumption. The
transfusion threshold recommended in a stable patient is
distinct from the target hematocrit of 30% in patients with low
central venous oxygen saturation during the first 6 hours of
resuscitation of septic shock. No clinical trials have addressed
the transfusion of platelets and fresh-frozen plasma (FFP) in
patients with sepsis, and recommendations are based on
consensus opinion and experience in other patient groups. In
patients with severe sepsis, platelets should be administered
when counts are less than or equal to 5,000/µL, regardless of
apparent bleeding. Platelet transfusion can be considered when
counts are 5,000 to 30,000/µL and there is a significant risk for
bleeding. Higher platelet counts may be required if surgery or
invasive procedures are planned. In the presence of active
bleeding or prior to surgical or invasive procedures, FFP is
indicated for coagulopathy due to a documented sufficiency of
coagulation factors. In the absence of bleeding, routine use of
FFP to correct laboratory abnormalities is not recommended.

RATIONALE (21) Answer: C

The clinical findings in association with the large lung mass are
suggestive of hypercalcemia. The main clinical manifestations of
hypercalcemia are gastrointestinal (anorexia, nausea, vomiting,
abdominal pain, constipation), cardiovascular (hypertension,
prolonged PR and QRS intervals, shortened QT interval,
bradyarrhythmias), renal (polyuria, nephrocalcinosis),
neurologic (apathy, lethargy, coma), and skeletal (bone pain).
Hypercalcemia due to lung cancer is usually associated with
non–small cell tumors, and the most common mechanism is
parathyroid hormone–related peptide. Other mechanisms of
hypercalcemia associated with malignancy include ectopic
production of vitamin D and bone metastasis. The initial step in
management of severe hypercalcemia involves replacing
intravascular volume with isotonic saline solution to increase
renal blood flow and enhance calciuresis. After volume
expansion is achieved, administration of a loop diuretic to
increase renal excretion of calcium may help. Potassium and
magnesium will usually require replacement during diuresis.
Additional specific therapy is usually required with extreme
elevations of calcium, and includes bisphosphonates and
calcitonin. Glucocorticoids may be helpful in hypercalcemia
associated with excess vitamin D, such as granulomatous
diseases or in patients with hematologic malignancy, such as
lymphoma or multiple myeloma.

Although phosphate is effective in lowering the serum calcium


concentration, this therapy may lead to the precipitation of
calcium-phosphate complexes, and is not recommended as initial
therapy in hypercalcemia.

RATIONALE (22) Answer: A

This patient’s clinical history is highly suggestive of acute chest


syndrome associated with sickle cell disease. Adult patients
typically develop the syndrome 2 to 3 days after hospitalization
for a vasoocclusive crisis. The most common clinical
manifestations are fever, cough, tachypnea, chest pain, and a
decrease in oxygen saturation. The etiology of the syndrome may
be infection (most commonly Chlamydia and Mycoplasma), fat
embolism, or atelectasis due to respiratory splinting. Patients
should receive oxygen therapy as needed, and antibiotics to
cover for typical pathogens (a cephalosporin and macrolide or
quinolone). Exchange transfusion would be preferred in this
patient over simple transfusion because of his higher
hemoglobin level. Simple and exchange transfusions are capable
of improving oxygenation. Exchange transfusion aims to
decrease hemoglobin-S to less than 30%–50%. The hemoglobin
should be maintained at =11 g/dL to avoid hyperviscosity, which
could result in additional complications such as stroke. Exchange
transfusion is often preferred in patients who have more severe
diseases, multilobe lung involvement, persistent or worsening
hypoxemia, neurologic abnormalities, or multiorgan failure.
Leukocyte-poor blood should be used in these patients. Although
dexamethasone, 0.3 mg/kg, was shown to reduce the length of
hospitalization in children with acute chest syndrome, a rebound
effect resulted in increased readmission in the treatment group.
No studies are available in adults, and use of dexamethasone
should be considered experimental. Thus, option C is incorrect.
Bronchoalveolar lavage (option D) should be reserved for those
patients who fail to respond to initial therapy. Although there are
anecdotal reports of use of inhaled nitric oxide (option E) in
mechanically ventilated patients, this application is also
experimental and should be reserved for patients with the most
severe diseases that are unresponsive to other mechanical
ventilation techniques

RATIONALE (23) Answer: C

Spinal cord compression is a growing problem among cancer


patients, partly because of improvements in local control and
increasing life expectancy. It affects 5%–10% of all cancer
patients and approximately 20,000 cases per year are diagnosed
in the United States. Incidence is highest among patients with
breast, lung, and prostate cancer. It generally carries a poor
prognosis, with the exception of hematological malignancies,
which are exquisitely chemosensitive and radiosensitive. Pain is
the most common symptom at presentation and can rapidly
progress to weakness, sensory changes, and bowel or bladder
dysfunction. Therefore, early diagnosis and intervention are key
to regaining or preserving function. MRI is the diagnostic test of
choice. It is important to image the entire spine because up to
38% of patients have multiple lesions. The mainstays of
treatment are steroids, radiation, and surgical decompression.
Dexamethasone is the preferred corticosteroid because of its
central nervous system penetration and ability to provide
immediate palliation. However, steroids alone are not sufficient
for long-term therapy. One randomized trial showed that
immediate surgical decompression followed by radiation
therapy was superior to radiation therapy alone. Compared to
the radiation-only group, the surgery group retained the ability
to walk in more cases (84% versus 57% of the time, P=0.001)
and for a longer duration (median 122 days versus 13 days,
P=0.003). Median survival was also significantly higher in the
surgery group compared to the radiation-alone group (126 days
versus 100 days, P= 0.0330). Strikingly, in the surgery group, 10
of the 16 paraplegic patients regained the ability to walk,
compared to 3 of the 16 paraplegic patients in the radiation-only
arm (P=0.012).There was no difference in days of
hospitalization, and 30-day morbidity was higher among
patients treated with radiation alone. The National
Comprehensive Cancer Network recommends decompressive
surgery and adjuvant radiation therapy in the setting of spinal
instability and no contraindication to surgery.

Chemotherapy may be an option for adjuvant therapy but would


not produce the rapid effect expected with surgical
decompression, with the exception of leukemias or certain
lymphomas

RATIONALE (24) Answer: A

Disseminated intravascular coagulation (DIC) is common in


patients with hematological malignancies. It contributes to
significant morbidity and mortality predominantly due to
bleeding, but also due to thrombotic complications, which are
more common in patients with solid tumors. Approximately 15%
of patients with acute leukemia present with overt DIC, with the
most striking presentation and harmful impact in patients with
acute promyelocytic leukemia (APL), the most curable of the
acute myeloid leukemias. Approximately 5%–10% of patients
with newly diagnosed APL die from bleeding complications
before initiation of chemotherapy. DIC is thought to result from a
combination of endothelial damage, expression of procoagulant
factors by malignant cells, and cytokine release. There is
generation of thrombin mediated by tissue factor, and a
hyperfibrinolytic state which is thought to predominate in APL.
The patient in question presented with a probable acute
leukemia and the data suggests the diagnosis of DIC. However, a
decreased fibrinogen level would further support the diagnosis
and trigger therapy with cryoprecipitate (in addition to fresh
frozen plasma and platelets), which is the optimal repletion
therapy for hypofibrinogenemia. D-dimer tests assist in the
diagnosis, but do not guide repletion therapy. Platelet function
assay is unreliable in the setting of thrombocytopenia, so option
C is incorrect. Prothrombin time and partial thromboplastin time
mixing studies are helpful when there are no other clues to the
diagnosis. In this case, the circulating immature cells and the
thrombocytopenia make acute leukemia with DIC the most likely
diagnosis. Therefore, option B is incorrect. Bone marrow biopsy
and review of the peripheral blood smear will confirm the
diagnosis of acute leukemia and will guide chemotherapy choice
but will not contribute to the immediate stabilization of the
patient with intracranial hemorrhage and DIC. Thus, options D
and E are incorrect.

RATIONALE (25) Answer: E


Tumor lysis syndrome results from the abrupt release of cellular
components into the blood after rapid lysis of malignant cells. It
occurs more frequently in patients with a large tumor burden,
tumors that are very sensitive to chemotherapy, and rapidly
proliferating tumors, such as Burkitt lymphoma and acute
leukemias. The hallmark of tumor lysis syndrome is the
combination of elevated levels of uric acid, potassium, and
phosphorus and decreased calcium levels. The clinical
consequences include cardiac arrhythmias; paresthesias; muscle
cramps, tetany, and seizures from electrolyte abnormalities; and
renal failure from uric acid nephropathy and precipitation of
calcium phosphate complexes due to the rapid increase in
phosphorus. Purine nucleic acids are catabolized to
hypoxanthine and xanthine by the enzyme xanthine oxidase,
which can be inhibited by allopurinol. These intermediaries are
further converted to uric acid, which is relatively insoluble in
water and at the lower pH in the kidneys. Therefore, in
hyperuricemic conditions, as the uric acid concentration
increases, the likelihood of crystal formation and deposition in
the tubules also increases and this can lead to renal failure. In
most nonhuman mammals, the enzyme urate oxidase further
converts uric acid into allantoin, which is 5–10 times more
soluble in urine than uric acid. Rasburicase is a recombinant
form of urate oxidase and was found to be effective at reducing
uric acid levels and improving renal parameters in patients with
hematological malignancies considered to be at high risk for
tumor lysis syndrome. Rasburicase is contraindicated in patients
with glucose-6-phosphate dehydrogenase deficiency. Allopurinol
(option A) is a reasonable alternative, but in the case of renal
failure and hyperuricemia, the breakdown of uric acid is more
likely to improve renal function and lower uric acid levels.
Immediate hemodialysis (option B) is not indicated unless
medical measures have failed to correct the metabolic
derangements. Urine alkalization can actually exacerbate the
condition by promoting calcium-phosphate crystal formation.
Therefore, option C is incorrect. Diuretics such as option D may
exacerbate the renal failure by promoting intravascular volume
depletion.

RATIONALE (26) Answer: E

Febrile neutropenia is defined as a single oral temperature of at


least 38.3°C (100.9°F), or a sustained temperature of 38°C
(100.3°F) over 1 hour with less than 500 neutrophils per
microliter, or less than 1,000 neutrophils per microliter and a
predicted decline to less than 500 neutrophils per microliter
over the next 48 hours. Febrile neutropenia carries a high
mortality when treatment is delayed; therefore, all patients with
febrile neutropenia should receive empiric antimicrobial
therapy. Initial antibiotic selection should take into
consideration potential site(s) of infection, known colonization
with resistant organisms and local resistance patterns, clinical
instability, recent antibiotic use (including prophylaxis), and
drug allergies. Monotherapy is recommended for the stable,
nontoxic patient. The routine use of empiric vancomycin is not
recommended, unless there is suspected catheter infection based
on examination findings, mucosal damage, known methicillin-
resistant Staphylococcus aureus (MRSA) colonization, or shock.
In that event, the continued use of vancomycin should be
reassessed within 2–3 days to prevent the increase in
vancomycin-resistant strains. Institution of early antifungal
therapy is not routinely recommended, although many patients
may be already taking fungal prophylaxis based on their disease
and risk factors for fungal infection. Antifungal therapy is
recommended in the event of persistent febrile neutropenia (>4
days) despite appropriate antibiotic coverage. The addition of an
aminoglycoside is also recommended as initial therapy for the
unstable febrile neutropenia patient.
RATIONALE (27) Answer: A

Leukostasis tends to occur at WBC counts above 100,000/µL


(hyperleukocytosis), but there is no clear cutoff for the diagnosis
or initiation of therapy. The clinical scenario should dictate the
most appropriate management strategy. Leukostasis is
uncommon in indolent or chronic leukemias and is most
common in those presenting with acute myeloid leukemia.
Abnormal expression of cell adhesion molecules by leukemic
blasts in addition to the elevation in the white blood cell count
with increased viscosity and sludging in the microvasculature
contribute to the complications of leukostasis. Hyperleukocytosis
was associated with increased 30-day mortality in prospective
and retrospective studies. Signs and symptoms of leukostasis
include respiratory distress due to leukemic infiltrates in the
pulmonary and cardiac microvasculature, and central nervous
system alternations, including hemorrhage that can present with
headache, visual disturbance, tinnitus, or overt neurologic
deterioration. It is therefore important to perform a detailed
physical examination, including a funduscopic examination for
signs of ongoing or impending clinical leukostasis. The most
rapid way to reduce the blast count is with mechanical
leukapheresis, and hydroxyurea is used concomitantly for
further cytoreduction. Hydroxyurea alone is appropriate in
hyperleukocytosis without signs or symptoms of leukostasis.
Bone marrow biopsy will confirm the diagnosis, but does not
help with the immediate need for cytoreduction. Combination
chemotherapy will be definitive therapy, but is associated with
increased morbidity and mortality in the setting of leukostasis.
There is no role for radiation therapy in this setting. In addition
to leukapheresis, it is important to institute hydration and tumor
lysis prophylaxis in patients presenting with malignant
hyperleukocytosis.
RATIONALE (28) Answer: D

Superior vena cava (SVC) syndrome results from occlusion of the


SVC due to tumor, fibrosis, or thrombosis. Physical examination
reveals distended neck veins and plethora and may reveal
adenopathy. Chest radiography may reveal a widened
mediastinum or a hilar mass, and Doppler ultrasonography or
contrast CT may guide diagnosis. SVC syndrome is generally not
considered an emergency but can progress to the point of airway
compromise. Immediate symptomatic management is therefore
indicated. Anticoagulation (option A) would be appropriate if
thrombosis were documented. Radiation therapy (option E)
requires a definitive diagnosis. Initial management includes
supportive care, such as providing supplemental oxygen,
elevating the head of the bed, and intravascular stenting to
relieve the obstruction (option D). Intravascular stenting allows
for prompt symptomatic relief and allows time to secure a
diagnosis. Steroids and diuretics (options C and B, respectively)
are often used, but efficacy data are lacking.

RATIONALE (29) Answer: C

The syndrome of inappropriate antidiuretic hormone (SIADH) is


a frequent cause of hyponatremia. It is historically associated
with bronchogenic carcinoma, but can occur as a result of certain
medications, extreme pain, pulmonary and central nervous
system disorders, and malignancy. The diagnosis requires a low
sodium level in the setting of euvolemia, concentrated urine,
hypo-osmolar plasma, and normal adrenal and thyroid function.
SIADH is thought to result from failure of the negative feedback
mechanisms that regulate release and inhibition of antidiuretic
hormone (ADH) or from ectopic production of ADH by certain
tumors. ADH excess induces water retention by the kidney
despite low serum osmolality and expanded intracellular
volume. The consequences of hyponatremia depend on the
severity and rapidity of the disorder, and include somnolence,
confusion, seizures, and coma. Treatment depends on the
chronicity of the syndrome and the clinical condition of the
patient and ranges from water restriction to hypertonic saline to
slowly increase the sodium level. The clinical scenario points to a
malignant cause for hyponatremia and makes option A unlikely.
The physical examination argues against volume depletion
(option B). Reduced cardiac output (option D) can lead to release
of ADH, but via the carotid sinus baroreceptors. The
hyponatremia in SIADH is not related to renal sodium losses, so
option E is incorrect.
PART 6: Renal and Metabolic Disorders in the ICU

Instructions: For each question, select the most correct


answer.

A 67-year-old alcoholic is admitted to the ICU with severe


intoxication (ethanol level 350 mg/dL) and requires
intubation for airway protection. BP is 102/58 mm Hg, HR is
101/min, RR is 14/min, and temperature is 37.1°C (98.7°F).
Laboratory results include the following: sodium, 118 mEq/L;
potassium, 3.1 mEq/L; chloride, 88 mEq/L; bicarbonate, 25
mEq/L; glucose, 120 mg/dL; blood urea nitrogen, 24 mg/dL;
creatinine, 1.3 mg/dL; urine osmolarity, 380 mOsm/kg; and
urine sodium, 5 mEq/L.

Which of the following interventions is most appropriate to


treat the hyponatremia?

A. Normal saline solution

B. 3% saline solution

C. Water restriction

D. Conivaptan

E. Furosemide

2. Which of the following is an advantage of using continuous


venovenous hemofiltration rather than intermittent
hemodialysis in a critically ill patient with acute renal failure?

A. Lower cost
B. Improved mortality

C. Reliable antibiotic dosing

D. Hemodynamic tolerance

E. Effective solute removal

3. A 54-year-old man found under a bridge is admitted to the


ICU with altered mental status. He is intubated for airway
protection. His BP is 100/56 mm Hg (after 1 L normal saline),
HR is 108/min, RR is 18/min, and temperature is 37.9°C
(100.2°F). Laboratory data are as follows: sodium, 142
mEq/L; potassium, 5.3 mEq/L; chloride, 102 mEq/L;
bicarbonate, 22 mEq/L; blood urea nitrogen, 50 mg/dL; and
creatinine, 4.8 mg/dL. Urine studies show specific gravity of
1.014, pH of 5.6, no white blood cells, rare red blood cells,
negative blood on dipstick, 1+ protein on dipstick, moderate
hyaline casts, urine sodium level of 54 mEq/L, and urine
creatinine level of 60 mg/dL. Which of the following is the
most likely cause of the acute kidney injury?

A. Hypovolemia

B. Nephrotoxin

C. Urinary retention

D. Glomerulonephritis

4. A 70-year-old woman with chronic kidney disease,


hypertension, and diabetes is admitted with nausea, fever,
and hypotension. Her BP is 88/50 mm Hg, HR is 110/min, RR
is 20/min, and temperature is 38.4°C (101°F). Laboratory
results are as follows: pH, 7.35; Paco2, 34 mm Hg; Pao2, 68
mm Hg (room air); sodium, 132 mEq/L; potassium, 4.0
mEq/L; chloride, 103 mEq/L; bicarbonate,18 mEq/L;
albumin, 2.1 g/dL; blood urea nitrogen, 28 mg/dL; and
creatinine, 1.8 mg/dL. Which of the following acid-base
disorders is present?

A. Anion gap metabolic acidosis alone

B. Non–anion gap metabolic acidosis alone

C. Non–anion gap metabolic acidosis and respiratory alkalosis

D. Anion gap and non–anion gap metabolic acidosis

5. A 55-year-old man is admitted to the ICU with fever and


hypotension. His medical history includes well-controlled
hypertension and a 2-month history of fatigue and weight
loss. On the day of admission, he had some dysuria and had
syncope when getting up to urinate. On admission, his BP is
70/40 mm Hg, pulse rate is 110/min, temperature is 39°C
(102.1°F), and RR is 18/min. The physical examination
reveals a thin man in mild distress. Laboratory results include
the following: sodium, 128 mEq/L; potassium, 5.6 mEq/L;
chloride, 102 mEq/L;bicarbonate,16 mEq/L; blood urea
nitrogen, 28 mg/dL; creatinine, 1.5 mg/dL; and glucose, 60
mg/dL. WBC count is 12,000/µL. Cardiac enzyme results are
negative. Urinalysis shows 50 WBCs and moderate bacteria.
After normal saline, 3 L over 2 hours, the patient is alert but
his BP is 80/50 mm Hg. Norepinephrine is initated and
titrated to 0.1 µg/kg/min to maintain BP of 90/56 mm Hg.
Broad-spectrum IV antibiotics are started.
Which of the following options is most appropriate at this
time?

A. Increase norepinephrine infusion.

B. Administer IV dexamethasone, 4 mg.

C. Infuse hetastarch, 500 mL.

D. Administer fludrocortisone, 50 µg.

6. A 65-year-old man with a heart transplant and chronic


kidney disease is admitted to the ICU with nausea, diarrhea,
and respiratory distress. BP is 100/62 mm Hg, HR is 115/min,
RR is 26/min, and temperature is 37.3°C (99°F). Arterial
blood gas results include pH of 7.23, Paco2 of 17 mm Hg, and
Pao2 of 235 mm Hg (50% ventimask). Laboratory results are
as follows: sodium, 123 mEq/L; potassium, 3.9 mEq/L;
chloride, 97 mEq/L; bicarbonate, 7 mEq/L; blood urea
nitrogen, 119 mg/dL; and creatinine, 5.1 mg/dL (baseline 2.1
mg/dL). Which of the following acid-base disorders is
present?

A. Anion gap metabolic acidosis alone

B. Anion gap metabolic acidosis and respiratory alkalosis

C. Non–anion gap metabolic acidosis alone

D. Anion gap metabolic acidosis and non–anion gap metabolic


acidosis

7. The rhythms shown in the Figure were noted on the


monitor of a 28-year-old man admitted with injuries after
being hit on his bicycle by an automobile. He suffered bilateral
femur fractures and a crush injury of the left ankle. Which of
the following is the most appropriate initial intervention?

First

Second
Third

Fourth

A. Epinephrine
B. Calcium chloride
C. Sodium bicarbonate
D. Sodium polystyrene sulfonate

8. A 25-year-old man with insulin-dependent diabetes mellitus is


admitted to the ICU with uncontrolled diabetes. He has a 1-day
history of nausea, vomiting, and diarrhea, and he withheld insulin
because of poor oral intake. His BP is 100/60 mm Hg, HR is
114/min, RR is 28/min, and temperature is 37.3°C (99°F).
Physical examination reveals dry mucous membranes and mild
diffuse abdominal tenderness. Laboratory examinations reveal
the following: sodium, 135 mEq/L; potassium, 3.5 mEq/L;
chloride, 110 mEq/L; carbon dioxide, 8 mEq/L; blood urea
nitrogen, 22 mg/dL; creatinine, 1.2 mg/dL; albumin, 4.1 g/dL;
and glucose, 350 mg/dL. Arterial blood gas measurements on
FIO2 of 0.21 are pH, 7.25; Paco2, 20 mm Hg; and Pao2, 95 mm Hg;
serum ketones are present at a 1:8 dilution.

Which of the following is the best explanation for the acid-base


abnormality in this patient?
A. Diabetic ketoacidosis alone

B. Diabetic ketoacidosis with hyperventilation

C. Diabetic ketoacidosis and diarrhea

D. Diabetic ketoacidosis and vomiting

9. A 65-year-old man weighing 70 kg (154 lbs) presents to the


emergency department with headache and lethargy. CT of the
head shows a brain lesion with mass effect and midline shift.
High-dose dexamethasone is started and the patient is
transferred to the ICU for close neuromonitoring. On the second
hospital day, he has worsening headache and increased lethargy.
He is intubated, and a repeat CT reveals worsening cerebral
edema in the area surrounding the mass, with increased midline
shift. At this time, blood chemistry is significant for sodium level
of 115 mEq/L.

Which of the following is the best next step in the management of


this patient?

A. Give 3% sodium chloride solution at 100 mL/h to increase


sodium level to 125 mEq/L over 12 hours.

B. Give 3% sodium chloride solution at 50 mL/h to increase


sodium level to 127 mEq/L over 24 hours.

C. Give 3% sodium chloride solution in repeat 100-mL boluses to


increase sodium level to 120 mEq/L within 1 hour.

D. Give 3% sodium chloride solution at 125 mL/h to increase


sodium level to 127 mEq/L over 8 hours.
10. A 75-year-old woman presents to the hospital with dizziness
and lethargy. Her past medical history is significant for
hypertension, for which she takes lisinopril, and for chronic back
pain treated with acetaminophen and ibuprofen. She has had no
previous abdominal surgeries. On physical examination, she
appears chronically ill with BP of 152/78 mm Hg and HR of
74/min. She is lethargic but arousable, with no focal neurologic
signs.

Laboratory data include the following: complete blood count,


normal; sodium, 138 mEq/L; potassium, 3.6 mEq/L; chloride, 98
mEq/L; bicarbonate, 9 mEq/L; glucose, 130 mg/dL; calcium, 8.9
mg/dL; lactic acid, 1.2 mmol/L; ammonia, normal; aspartate
aminotransferase, 65 U/L; alanine aminotransferase, 70 U/L;
international normalized ratio, 1.4; blood urea nitrogen, 30
mg/dL; and creatinine, 1.4 mg/dL (baseline 1.2 mg/dL). Testing
is negative for salicylates, urine drug screen results are negative,
acetaminophen level is less than 10 µg/mL, and ethanol level is
negative. Findings of chest radiography and urinalysis are normal
and blood cultures are obtained.

IV fluids are administered; 48 hours later, the anion gap is


normal, creatinine level is 1.2 mg/dL, and mental status is
improved. She had a similar episode 6 months ago, which also
resolved within 2–3 days after hospitalization.

Which of the following is the most likely diagnosis?

A. Laboratory measurement error

B. Acute renal failure

C. Adverse drug effect


D. Early sepsis

11. Induction chemotherapy was initiated in a 45-year-old man


with acute lymphoblastic leukemia 2 days ago. He now has
diffuse weakness and hyperkalemia. Upon evaluation the patient
is also noted to be short of breath. He has been receiving IV
normal saline at 100 mL/h since initiation of chemotherapy, in
addition to oral allopurinol.

On physical examination, his BP is 177/85 mm Hg, HR is 62/min,


RR is 28/min, Spo2 is 94% on 2 L/min of oxygen via nasal
cannula. He has decreased breath sounds bilaterally and crackles
in up to one-third of lung fields. Additionally, there is 2+ bilateral
lower extremity edema. His urine output has declined over the
past 8 hours to less than 50 mL/h.

Laboratory work shows the following: sodium, 147 mEq/L;


potassium, 7.2 mEq/L; chloride, 108 mEq/L; bicarbonate, 10
mEq/L; phosphorus, 13 mg/dL; calcium, 5.7 mg/dL; blood urea
nitrogen, 68 mg/dL; and creatinine, 3.0 mg/dL (baseline 1.6
mg/dL).

Which of the following is the most appropriate next step in


management of this patient?

A. Change IV fluids to 5% dextrose with sodium bicarbonate, 150


mEq/L.

B. Initiate rasburicase.

C. Increase the allopurinol dose.

D. Initiate hemodialysis.
12. An 80-year-old woman presents to the hospital with
shortness of breath and hemoptysis. She has been feeling sick
over the last few days, and this morning she started coughing
bright red blood. Her past medical history is significant for
hypertension and diabetes. On physical examination she has BP
of 120/65 mm Hg, HR of 95/min, RR of 32/min, and Spo2 of 90%
on nonrebreather mask. The rest of the physical examination is
unremarkable.

Her laboratory results are as follows: blood urea nitrogen, 98


mg/dL; creatinine, 7 mg/dL; sodium, 138 mEq/L; potassium, 4.5
mEq/L; chloride, 96 mEq/L; bicarbonate, 19 mEq/L; WBCs,
12,000/µL; hemoglobin, 11.5 g/dL; platelets, 140,000/µL; and
lactic acid, normal. Urinalysis shows specific gravity of 1.010, no
WBCs, leukocyte esterase negative, and 30–50 RBCs/hpf with
dysmorphic features. Antinuclear antibody testing is negative,
perinuclear antineutrophil cytoplasmic antibody testing is
positive, anti-glomerular basement membrane testing is negative,
and complement levels are normal.

Chest radiograph reveals bilateral alveolar opacities. After


intubation due to progressive hypoxemia and increased work of
breathing, a noncontrast CT of the chest suggesting alveolar
hemorrhage is obtained.

Which of the following is the most appropriate next step in this


patient’s management?

A. Oral prednisone and cyclophosphamide with initiation of


plasmapheresis

B. High-dose IV methylprednisolone daily for 3 days with oral


cyclophosphamide
C. Oral cyclophosphamide and mycophenolate for 7 days

D. Hemodialysis, oral prednisone, and azathioprine

13. Which of the following is required before initiation of enteral


nutrition in the critically ill patient?

A. Presence of bowel sounds

B. Bowel movements

C. Hemodynamic stability

D. Placement of a jejunal feeding tube

14. Which of the following patients would most likely have an


improved outcome from enteral nutrition with an immune-
modulating formula?

A. Mechanically ventilated patient with myasthenia gravis

B. Mechanically ventilated patient after emergent hemicolectomy


for bowel perforation

C. Mechanically ventilated patient with pulmonary edema


secondary to systolic heart failure

D. Acute stroke patient with dysphagia and hemiparesis who is


not mechanically ventilated

15. A 32-year-old patient with type 1 diabetes mellitus and


multiple admissions for diabetic ketoacidosis is admitted to the
ICU with a 2-day history of weakness, nausea, and vomiting. BP is
104/70 mm Hg, HR is 118/min, RR is 20/min, and temperature is
37.4°C (99.3°F). Laboratory data include the following: arterial
blood gas pH, 6.84; Paco2, 10.5 mm Hg; Pao2, 114 mm Hg;
sodium, 139 mEq/L; chloride, 105 mEq/L; potassium, 2.5 mEq/L;
bicarbonate, 5 mEq/L; and glucose, 568 mg/dL. After a 1-L bolus
of normal saline is administered, which of the following is the
most appropriate intervention?

A. IV regular insulin as a bolus

B. IV sodium bicarbonate as a bolus

C. IV and oral potassium

D. IV regular insulin as a bolus and IV potassium

16. A 62-year-old woman with hypertension and hyperlipidemia


was diagnosed with Graves disease 1 year ago. She presents with
palpitations, dizziness, shortness of breath, and intermittent
chest pain for the last 2 days. She admits to having run out of her
propylthiouracil 2 weeks ago. On examination, she is confused
and diaphoretic, with irregular HR of 132/min, BP of 138/82 mm
Hg, RR of 22/min, temperature of 38°C (100.3°F) and Spo2 of
92% on room air. Physical examination is remarkable for
restlessness, tremor of both hands, an irregular heart rhythm
with a grade II/VI systolic murmur, bibasilar rales, and warm
skin. She is admitted to the ICU. Which of the following is the
most appropriate first intervention?

A. Oral propylthiouracil
B. Oral saturated solution of iodine
C. Oral aspirin
D. IV diltiazem
PART 6: Renal and Metabolic Disorders in the ICU

ANSWERS:

1–A; 2–D; 3–B; 4–A; 5–B; 6–D; 7–B; 8–C; 9–C; 10–C; 11–D; 12–A;
13–C; 14–B; 15–C; 16–A

RATIONALE (1) Answer: A

This patient’s laboratory results are most consistent with


hypovolemic hyponatremia. Assessment of volume status is
usually the first step in evaluating hyponatremia, but
determination of status may be difficult. In this patient, there are
subtle suggestions of hypovolemia with a slightly elevated heart
rate and a blood urea nitrogen and creatinine level that are
probably high for a patient with chronic alcohol dependence. The
urine studies are helpful in assessing the cause of the
hyponatremia. The calculated serum osmolarity is 250 mOsm/kg
(2 × sodium + glucose ÷ 18 + blood urea nitrogen ÷ 2.8). The
serum osmolarity is higher, which could be consistent with
euvolemic hyponatremia (syndrome of inappropriate antidiuretic
hormone secretion [SIADH]) or hypovolemic hyponatremia. The
information that suggests hypovolemic hyponatremia is the low
urine sodium level. Urine sodium level is usually greater than 40
mEq/L in SIADH and other etiologies of euvolemic hyponatremia
and less than 20 mEq/L in hypovolemic states. A high urine
specific gravity would also suggest hypovolemic hyponatremia.
Secretion of antidiuretic hormone is appropriate in patients with
hypovolemia. The intervention of choice is the correction of the
hypovolemia rather than restriction of fluids or administration of
hypertonic saline. Vasopressin antagonists such as conivaptan
should not be used in hypovolemic hyponatremia because they
can cause hypotension and worsening hypovolemia.
RATIONALE (2) Answer: D

Acute renal failure worsens mortality in critically ill patients.


Renal replacement therapy (RRT) is frequently implemented to
control complications. There is ongoing debate over the most
effective type of RRT, but there are distinct advantages and
disadvantages of intermittent hemodialysis (IHD) and continuous
venovenous hemofiltration (CVVH). The major advantage of
CVVH is hemodynamic tolerance due to less fluctuation of volume
status. CVVH is commonly preferred over IHD in hypotensive
patients. The need for substitution fluids, anticoagulants, and
multiple filters make CVVH more costly than IHD despite the fact
that specialized personnel are not required for implementation.
Studies thus far have failed to show a mortality advantage for
either IHD or CVVH. Pharmacodynamic data exist for the dosing
of most antibiotics with IHD but not for CVVH. Underdosing of
antibiotics is a major concern, especially with high ultrafiltration
rates. Solute removal is more effective with IHD, which is the
reason CVVH must be used continuously

RATIONALE (3) Answer: B

The etiologies of acute renal failure are usually categorized as


prerenal, renal (intrinsic), and postrenal. The most common
cause of acute renal failure in ICU patients is acute tubular
necrosis (ATN) secondary to ischemia/hypoperfusion injury or
nephrotoxin or a combination of these factors. The laboratory
and urine findings in this patient suggest ATN due to the
following findings: blood urea nitrogen to creatinine ratio of
approximately 10:1, fractional excretion of sodium of 3%, bland
sediment, and urinary sodium concentration greater than 40
mEq/L. The formula for calculating the fractional excretion of
sodium is
FENa = ([urine sodium ÷ serum sodium] ÷ [urine creatinine ÷
serum creatinine]) × 100.

A nephrotoxin would be consistent with these findings.


Hypotension could also produce ATN but is not offered as an
option. Hypovolemia is a prerenal cause of acute renal failure
with findings of blood urea nitrogen to creatinine ratio greater
than 20, FENa less than 1%, and urinary sodium level less than 20
mEq/L. The urine specific gravity would also be higher. Urinary
retention is an example of postrenal etiology of acute renal failure
and would also result in a blood urea nitrogen to creatinine ratio
greater than 20 and variable FENa and urine sodium
concentration. The hallmark of glomerulonephritis is an active
urinary sediment with red blood cells and red blood cell casts.

RATIONALE (4) Answer: A

In this patient, the pH of 7.35 suggests an acidosis, and the low


bicarbonate level along with decreased Paco2 suggests a
metabolic process. The respiratory compensation is appropriate
using the formula

Paco2 = 1.5 × [bicarbonate] + 8 ± 2.

The anion gap is usually calculated as

sodium – (chloride + bicarbonate),

which in this patient yields an anion gap of 11 mEq/L. The


normal anion gap is usually 10–12 mEq/L, depending on the
laboratory normal range. The anion gap of 11 mEq/L might be
considered normal for this patient except that the albumin
concentration must be taken into account. The normal range for
anion gap is decreased in hypoalbuminemic patients. For every
decrease of 1 g/dL in albumin, a decrease of 2.5 to 3 mEq/L in
anion gap occurs. The following formula can be used to estimate
the true anion gap (AG):

AGcorrected = AGobserved + 2.5 × [normal albumin – measured


albumin (g/dL)]

In this patient, the anion gap is closer to 16 mEq/L, indicating an


anion gap acidosis. Correction of the anion gap for phosphate
concentration has been suggested, but it will not usually impact
interpretation of the acid-base disorder. The clinical scenario in
this case suggests infection and/or hypotension as the cause of
the acidosis. Options B and C are incorrect since an anion gap
metabolic acidosis exists and the Paco2 is appropriate
compensation for the acidosis rather than a primary disorder.
Option D is incorrect since the decrease in serum bicarbonate is
similar to the increase in anion gap (referred to as the delta gap).

RATIONALE (5) Answer: B

The clinical findings in this patient are consistent with acute


decompensation secondary to adrenal insufficiency. Clinical
findings may include hypotension, orthostasis, fatigue, fever, and
weight loss. In addition, patients may have nausea, vomiting,
weight loss, and abdominal pain. Diagnosis of adrenal
insufficiency is often delayed because of the nonspecific
symptoms. Laboratory findings that support a diagnosis of
adrenal insufficiency include hyponatremia, hyperkalemia,
prerenal azotemia, hypoglycemia, and normal anion gap acidosis.
The most common causes of primary adrenal insufficiency are
autoimmune destruction, infection, and metastatic disease. Other
causes include bilateral adrenal hemorrhage, particularly in the
critically ill patient with disseminated intravascular coagulation,
infiltrative disorders, and some drugs. Secondary adrenal
insufficiency due to failure of the pituitary to secrete
adrenocorticotropic hormone (ACTH) is most commonly due to
withdrawal of corticosteroids but can also result from tumors,
head trauma, cranial radiation, and infiltrative diseases. Adrenal
crisis may be confused with septic shock since the manifestations
are similar. In addition, a minor infection can lead to
decompensation. Clues to the presence of adrenal insufficiency
include the lack of predisposing conditions for infection, the lack
of an obvious source, presence of a mild infection, and poor
response to IV fluids and vasopressor agents. The treatment of
adrenal crisis includes volume resuscitation with glucose-
containing fluids and immediate administration of
corticosteroids. The most appropriate management of this
patient includes immediate administration of dexamethasone
that will not interfere with determination of plasma cortisol and
performance of a short ACTH stimulation test. Administration of
colloids or a change in vasopressor agent is unlikely to have any
significant effect in this patient. The addition of fludrocortisone is
usually not required in adrenal crisis. Once an ACTH test has been
performed, hydrocortisone, 200–300 mg/day in divided doses,
can be used, which has some mineralocorticoid activity.

RATIONALE (6) Answer: D

The low pH indicates acidemia, and the low bicarbonate level and
decreased Paco2 suggest a metabolic process. The respiratory
compensation appears appropriate: (1.5 × 7) + 8 = 18. The anion
gap is calculated as 19 mEq/L, which is increased and defines the
presence of an anion gap metabolic acidosis. This is consistent
with the patient’s worsened renal function. In the presence of an
anion gap acidosis, the delta gap should be calculated. In this
case, the difference between the measured and normal anion gap
is 7 and the bicarbonate value would be expected to decrease to
17 mEq/L (24 minus 7 mEq/L). The very low bicarbonate value
means that more bicarbonate has been lost than can be
accounted for by compensation for the increase in hydrogen ion.
Therefore, a non–anion gap metabolic acidosis is also present
that accounts for the very low bicarbonate value. The clinical
scenario suggests loss of bicarbonate through diarrhea, but this
patient was also receiving cyclosporine and may have tubular
dysfunction as well. The tachypnea in this patient is
compensation for the acidoses rather than a sign of pulmonary
disease

RATIONALE (7) Answer: B

These rhythm strips show a classic progression of severe, life-


threatening hyperkalemia that is probably due to rhabdomyolysis
from the traumatic injuries. The first sign of increased serum
potassium on the ECG is often peaked T waves. Other changes
associated with increases in serum potassium concentration
include:

 Widening of the QRS complex;

 Progressive atrioventricular conduction blocks;

 Slow idioventricular rhythm;

 An ECG tracing resembling a sine wave; and finally,

 Either ventricular fibrillation, or as in this case, asystole.

The relationship of the serum potassium concentration and ECG


changes is variable. Treatment of severe hyperkalemia with
ECG changes includes the initial administration of calcium as
calcium chloride or gluconate to antagonize the effects of the
hyperkalemia on the myocardial membranes. The effects of
calcium last about 30 minutes and allow time for other
interventions. Potassium can be redistributed from the
extracellular space to the intracellular space by administration
of insulin, along with concentrated glucose to prevent
hypoglycemia. Use of sodium bicarbonate to redistribute
potassium is less effective. Albuterol, 10–20 mg in 4 mL saline
by inhalation over 10 minutes, can also lower the serum
potassium concentration significantly. Definitive removal of
potassium from the body can be accomplished with loop
diuretics, sodium polystyrene sulfonate, or dialysis.
Epinephrine would increase the serum potassium
concentration further by redistribution to the extracellular
space.

RATIONALE (8) Answer: C

The laboratory measurements reveal acidemia with decreased


bicarbonate concentration, indicative of a metabolic acidosis.
The degree of respiratory compensation is adequate (expected
Pco2 in a simple metabolic acidosis = [1.5 × bicarbonate] + 8 ±
2), thereby excluding primary hyperventilation related to
anxiety. The anion gap is increased, and in view of the clinical
history, hyperglycemia, and ketonemia, the patient has diabetic
ketoacidosis. However, the anion gap is 17 mmol/L, and this is
5 mmol/L higher than the upper limit of normal of 12 mmol/L.
In a “pure” anion gap metabolic acidosis, the decrease in serum
bicarbonate should approximate the increase in anion gap.
Thus, the expected bicarbonate level in this patient is 19
mEq/L (normal bicarbonate level of 24 mEq/L, minus the
change in anion gap of 5 mEq/L). However, the actual
bicarbonate level of 8 mEq/L is considerably lower than
anticipated, reflective of a change in bicarbonate concentration
of 16 mmol/L. The difference between the change in
bicarbonate concentration and the change in anion gap is
known as the “delta gap” and normal values are 0 ± 6. Because
the serum bicarbonate level is much lower than expected for a
simple anion gap metabolic acidosis, the acid-base status of
this patient is best described as anion gap metabolic acidosis
(diabetic ketoacidosis) with a non–anion gap (hyperchloremic)
metabolic acidosis. From the history, diarrhea is the most likely
explanation for the hyperchloremic metabolic acidosis. The
serum bicarbonate concentration is consistent with the pH and
Paco2 (based on the Henderson-Hasselbach equation);
therefore, a laboratory error does not account for the acid-base
abnormalities.

RATIONALE (9) Answer: C

Hyponatremia with acute neurologic symptoms is considered a


medical emergency that should be treated promptly with
administration of 3% sodium chloride solution. A rapid but safe
correction must be done in order to prevent brain herniation and
osmotic demyelinization. Important steps in management are
setting the goal range for correction, calculating the rate at which
to administer the solution, monitoring changes in sodium
concentration, and establishing a diagnosis for the etiology of
hyponatremia. Current recommendations limit the increase in
sodium to approximately 8–12 mEq/L over 24 hours or 18
mEq/L in 48 hours. The appropriate rate for infusion of 3%
sodium chloride solution remains unclear as recent studies have
shown that the most commonly used formula (Adrogue-Madias)
for calculating appropriate sodium correction tends to
overestimate the correction in most cases. A retrospective study
of hyponatremia cases showed that among patients with serum
sodium levels less than 120 mEq/L, the observed rates of
correction were higher than predicted by the Adrogue-Madias
formula in almost 75% of patients. Moreover, in that study, the
lower the serum sodium concentration of the patients, the
greater was the risk of overshooting. In patients with underlying
neurologic diseases such as in this case, 2 factors that were
independently associated with reversal of impending
transtentorial herniation were an increase of sodium level of at
least 5 mEq/L within 1 hour or a serum sodium level of at least
145 mEq/L (option C). Correcting the hyponatremia over 24
hours would be appropriate for patients with neurologic
symptoms, but not for patients with underlying pathology
causing impending herniation. In this case, with CT showing
significant risk of brain herniation, the goal is to increase serum
sodium level by approximately 5 mEq/L within 1 hour (option C).
The other options do not address this immediate need and are
therefore incorrect.

RATIONALE (10) Answer: C

The finding of metabolic acidosis with increased anion gap is of


particular concern in the seriously ill patient as it can indicate
potentially fatal etiologies such as lactic acidosis from
hypoperfusion. A broad differential diagnosis must be
entertained, including lactic acidosis, ketoacidosis, uremia, and
drug intoxications (methanol, ethylene glycol, iron, etc). Recently,
a new entity has become increasingly recognized: pyroglutamic
acidosis (also called 5-oxoprolinuria). The condition occurs with
chronic depletion of glutathione stores, which results in less
inhibition of glutamate metabolism and accumulation of 5-
oxoproline. It is associated more with females, sepsis, chronic
illness, malnutrition, and chronic acetaminophen ingestion. The
prevalence of this entity is unclear. Laboratory data show an
increased anion gap acidosis with low acetaminophen level and
no other obvious cause of metabolic acidosis. The diagnosis
would be made with a urinary organic acid level, or with an
increased urinary osmolar and urinary anion gap in the right
clinical setting and without any other causes for such
abnormalities. In this patient, the abnormal transaminase levels
result from chronic acetaminophen ingestion and the high
international normalized ratio from malnutrition When
pyroglutamic acidosis is suspected, acetaminophen should be
stopped. Administration of N-acetylcysteine has been suggested
to replenish glutathione stores, but benefit is unclear Option C is
therefore correct (adverse effect of acetaminophen). The patient
does not have acute renal failure (option B is wrong) as the
increase in creatinine is less than 0.3 mg/dL above baseline and
there are no signs of sepsis (option D is wrong).

RATIONALE (11) Answer: D

The patient has tumor lysis syndrome occurring in the setting of


chemotherapy for acute lymphoblastic leukemia. It is
characterized by hyperuricemia, hyperkalemia,
hyperphosphatemia, and hypocalcemia. Risk factors include large
tumor burden, chronic kidney disease, and volume depletion.
Preventive measures are essential in decreasing the incidence of
tumor lysis syndrome and include aggressive fluid hydration and
hypouricemic agents such as allopurinol or rasburicase.
Alkalization of urine with sodium bicarbonate solution has been
advocated, but may be associated with increased calcium
phosphate deposition. There is no benefit of a sodium
bicarbonate solution over normal saline. Rasburicase is currently
approved for prevention and treatment of tumor lysis syndrome
and rapidly lowers uric acid levels by enhancing degradation to
allantoin. Allopurinol slowly lowers uric acid levels by decreasing
its formation. Neither agent would treat the hyperkalemia and
other metabolic abnormalities in this patient. In the setting of
oliguria, volume overload, and life-threatening hyperkalemia,
hemodialysis should be initiated. (option D).

RATIONALE (12) Answer: A


The finding of acute renal failure, with active urinary sediment
and dysmorphic red cells indicates glomerulonephritis. Rapid
loss of kidney function (over a few days to a few weeks) in this
setting is called rapidly progressive glomerulonephritis (RPGN)
and should be treated aggressively. Several etiologies can cause
RPGN including lupus nephritis, Goodpasture syndrome, and
vasculitis related to antineutrophil cytoplasmic antibodies
(ANCAs, which the patient has). In the setting of severe renal
failure, or severe pulmonary hemorrhage, plasmapheresis should
be initiated. Recently, a multicenter trial randomized patients
with creatinine levels greater than 5.5 mg/dL and ANCA-positive
vasculitis to pulse-dose methylprednisolone or plasmapheresis
along with oral prednisone and cyclophosphamide. The group
randomized to plasmapheresis had better survival and dialysis
independence.

RATIONALE (13) Answer: C

Early enteral nutrition in the critically ill is associated with a


decrease in infectious complications and decrease in length of
stay. Enteral nutrition should be provided to hemodynamically
stable patients and those requiring stable low doses of pressors.
Hypotensive patients (mean arterial pressure <60 mm Hg) or
those requiring escalating or high doses of vasopressors should
be stabilized before initiating enteral feeds. Hemodynamically
unstable patients are at risk for ischemia involving the intestinal
microcirculation. The presence of bowel sounds does not
correlate well with absorptive capacity and the absence of bowel
movements does not predict the ability to tolerate enteral
feeding. Several studies and meta-analyses have shown no
difference in pneumonia or mortality between gastric and
postpyloric feeding.
RATIONALE (14) Answer: B

Current guidelines recommend use of enteral immune-


modulating formulas in patients undergoing major surgery,
trauma patients, burn patients (>30% body surface area
involvement), head and neck cancer patients, and critically ill
patients on mechanical ventilation. These formulas may contain
various combinations of arginine, omega-3 fatty acids, fish oil,
glutamine, selenium, vitamin C, or vitamin E. These formulas are
associated with reduction in duration of mechanical ventilation,
infection and hospital length of stay, but not with a reduction in
mortality. The benefit is more uniform in patients undergoing
major surgery than critically ill patients on mechanical
ventilation and in surgical patients more than in medical patients.
Caution is warranted in use of immune-modulating formulas
containing arginine in patients with severe sepsis as some studies
have found a greater mortality with these formulas. Options A, C,
and D are medical patients less likely to benefit from immune-
modulating formulas compared to the surgical patient presented
in option B

RATIONALE (15) Answer: C

The approach to managing diabetic ketoacidosis (DKA) includes


fluids, insulin, and electrolyte replacement. As the first step, fluid
administration is targeted to correct volume deficits and ensure
organ perfusion. Insulin therapy is usually initiated with a bolus
of 5 to 10 units of regular insulin followed by an infusion of
regular insulin, 0.1 units/kg/h. This patient presents with severe
hypokalemia that mandates the delay of insulin administration
until the potassium concentration is greater than 3.3 mEq/L.
Administration of insulin before correction of potassium
concentration would further lower the potassium concentration
and potentially precipitate respiratory muscle weakness or
arrhythmias. Thus, options A and D are incorrect. In general,
bicarbonate therapy is not indicated in the management of DKA
and pH usually improves quickly with fluid therapy. Bicarbonate
administration has failed to show benefit in several studies, but
no studies have addressed the use of bicarbonate in patients with
DKA and pH less than 6.9. If considered for use, bicarbonate
should be administered as an infusion rather than a bolus. In this
patient, a bolus of bicarbonate would result in a shift of
potassium to the intracellular space and further lower the serum
potassium concentration. This intervention could result in the
same consequences as administering insulin with severe
hypokalemia.

RATIONALE (16) Answer: A

Thyroid storm represents the extreme manifestations of


hyperthyroidism. The distinction between severe thyrotoxicosis
and thyroid storm is often subjective, but severe clinical
manifestations should be managed similarly. This patient has
significant multiorgan signs and symptoms related to
hyperthyroidism and should be managed in an ICU. The order of
treatment in thyroid storm is very important. Propylthiouracil or
methimazole should be given as the first intervention to inhibit
new hormone production (thyroxine [T4] synthesis). Iodine in
the form of saturated solution of iodine or Lugol solution must be
administered at least 1 hour after propylthiouracil or
methimazole. If iodine is administered as the first agent, it can
cause release of preformed thyroid hormone and worsen
symptoms. Thus, option B is incorrect. Iodine administered after
propylthiouracil or methimazole inhibits release of preformed
T4. Salicylates are contraindicated in thyrotoxicosis because they
can decrease thyroid protein binding, causing an increase in free
thyroid hormone levels. Thus, option C is incorrect. Atrial
fibrillation is a frequent manifestation of hyperthyroidism but is
best managed with control of the hyperthyroidism and
administration of beta-blockers. Beta-blockers control the
peripheral effects of thyroid hormone and can be administered
intravenously. Diltiazem would not be the best agent for control
of atrial fibrillation in this clinical scenario. Hydrocortisone is
usually also administered in thyroid storm to inhibit peripheral
conversion of T4 to triiodothyronine (T3) and to treat possible
adrenal insufficiency.
PART 7: Environmental and Toxicologic Injury

Instructions: For each question, select the most correct answer.

1. A 35-year-old male window washer was moving a metal ladder


when the ladder came in contact with a 220-V electrical line. He
was holding onto the ladder with both hands at the time of
contact , was knocked clear of the ladder, and was unconscious
for approximately 1 minute. Upon awakening he was confused
and reported bilateral arm and hand pain and lower extremity
pain. He was brought to the burn center for further care. Physical
examination reveals a 5% total body surface burn to his bilateral
arms and hands, with burns to his right groin and left foot. His
pulse rate is 102, BP is 138/78 mm Hg, and RR is 24/min. His
oxygen saturation is 100% on nasal cannula at 4 L/min, and
Glasgow Coma Scale score is 15. Examination of the hands
demonstrates bilateral severe burns and contractures to the
digits and at the level of the wrist, with minimal sensation on the
digits. He has a large exit wound in the right groin area and lesser
burns on the left foot.
Figure 1 Entrance wound to right hand
Figure 2 Right groin area exit wound

Which of the following interventions in the ICU is most likely to


improve the patient’s chances for limb survival and limb salvage?

A. Administration of a calcium channel blocker to improve


perfusion into the injured area
B. Administration of sodium bicarbonate to alkalize the urine

C. Elevation of the right arm to decrease edema formation

D. Lowering of the right arm to increase arterial perfusion

2. A 2-year-old male is involved in a house fire. He is found


unconscious by fire rescue and brought to the burn center for
evaluation and treatment. On arrival, he has partial-thickness
burns to his face, upper extremities, and chest. His Glasgow Coma
Scale score is 6. Pulse rate is 132/min, BP is 112/56 mm Hg, RR is
34/min, carboxyhemoglobin is 40%, and Spo2 is 98% on a 100%
oxygen nonrebreather mask. He has soot in his nares and mouth
and his lips have a parched appearance.

The most immediate care of this patient should include:

A. High-flow oxygen therapy with immediate bronchoscopy

B. Intubation, bronchoscopy, and hyperbaric oxygen therapy

C. Immediate hyperbaric therapy, bronchoscopy, and intubation


only if the patient does not improve

D. Immediate treatment with amyl nitrate, followed by


bronchoscopy

E. Intubation followed by bronchoscopy and plasma exchange


therapy

3. Oxygen saturation by pulse oximetry (Spo2) was 98% while


the patient was receiving 100% FIO2 via face mask. The arterial
blood gas oxygen saturation (Sao2) was only 59%.
The most likely explanation for this includes:

A. Arteriovenous shunting

B. Increased tissue oxygen extraction due to low cardiac output

C. The presence of carboxyhemoglobin

D. Malfunction of the 2-channel oximeter

E. Error in Sao2 most likely representing a venous gas

4. A 48-year-old male was involved in a fire at his home when the


propane stove exploded. He sustained 70% total body surface
area burns of deep-partial to full thickness. He is admitted to the
burn center, intubated, and resuscitated. He weighs 90 kg (198
lb) and over the first 8 hours receives 12.6 L of fluid. His present
pulse rate is 145/min, BP is 90/54 mm Hg, RR is 22/min on
conventional mechanical ventilation with FIO2 of 50%, set rate of
18/min tidal, volume of 700 mL, and vpositive end-expiratory
pressure (PEEP) of 5 cm H2O. Urine output rates for the last 3
hours are 15 mL, 10 mL, and 5 mL, respectively. The ventilator is
alarming that the exhaled volumes are 450 mL and peak airway
pressures are 45 cm H2O. Physical examination is seen in the
figure.
Figure 4 70% total body surface area with multiple
escharotomies

The best next step in the management of this patient is to:

A. Increase PEEP to 15 cm H2O.

B. Convert the patient’s ventilator mode to synchronized


intermittent mandatory ventilation and decrease tidal volume.

C. Administer a 1-L bolus of lactated Ringer solution and increase


the patient’s hourly fluid rate.

D. Measure the patient’s bladder pressure.

E. Add dopamine to augment blood pressure and renal perfusion.


5. A 68-year-old man weighing 80 kg (176 lbs) was admitted to
the ICU with second-degree burns over 30% of his body from a
house fire. He was intubated in the emergency department for
evidence of inhalation injury. Currently, his HR is 120/min, BP is
108/52 mm Hg, RR is 18/min (sedated on ventilator), and
temperature is 36.4°C (97.5°F).

Which amount and type of fluid should be infused over the first 8
hours?

A. 5% dextrose lactated Ringer solution, 4,800 mL

B. Lactated Ringer solution, 4,800 mL

C. Lactated Ringer solution, 3,200 mL

D. 0.9% saline, 3,200 mL

6. Three intubated patients are admitted to the ICU following


exposure to an unknown gas in a bus station. Clinical findings
include severe hypoxemia with copious respiratory and oral
secretions. Additional findings include small pupils, bradycardia,
and diarrhea. One of the patients is able to follow commands but
appears to have significant diffuse weakness.

Which of the following would be the most beneficial initial


intervention?

A. Pyridostigmine

B. Atropine

C. Sodium nitrite and sodium thiosulfate


D. Hyperbaric oxygen

E. Pralidoxime

7. An ICU patient is suspected to have overdosed on trazodone


and fluoxetine. The patient is agitated and delirious. BP is
158/106 mm Hg, HR is 116/min, RR is 22/min, and temperature
is 38.7°C (101.6°F). She is diaphoretic with dilated pupils.
Tremors and clonus of the lower extremities are noted.

Which of the following should be administered?

A. Dantrolene

B. Lorazepam

C. Propranolol

D. Bromocriptine

E. Neuromuscular blocker

8. A man with alcohol dependence and severe alcohol-related


delirium is admitted to the ICU and a bolus of lorazepam is
initiated. Due to high requirements of lorazepam to control
agitation, the patient is intubated and a lorazepam infusion for
control of symptoms is initiated. After 24 hours, he is receiving
lorazepam, 12 mg/h. His baseline laboratory results show the
following: sodium, 132 mEq/L; chloride, 100 mEq/L; potassium,
3.4 mEq/L; bicarbonate, 22 mEq/L; blood urea nitrogen, 22
mg/dL; creatinine, 1.5 mg/dL; and glucose, 120 mg/dL. Three
days into the ICU course, the patient has a bicarbonate level of 16
mEq/L with an anion gap of 18 mEq/L, creatinine level of 1.8
mg/dL, and lactate level of 2.0 mg/dL. He has an osmolar gap of
24 mOsm/kg. He remains sedated, with HR of 90/min, BP of
104/74 mm Hg, RR of 12/min on the ventilator, and temperature
of 37.8°C (99.9° F).

Which of the following interventions is most appropriate?

A. Stop lorazepam and start midazolam infusion.

B. Stop lorazepam and start fentanyl infusion.

C. Start broad-spectrum antibiotics and continue lorazepam


infusion.

D. Initiate hemodialysis.

9. A 70-year-old woman is found unresponsive in her unheated


house in December by her neighbors. She is known to have
systolic heart failure (ejection fraction 40%–45%), chronic
kidney disease, and mild dementia. On arrival to the hospital, she
is intubated and a rectal temperature of 30°C (86°F) is measured.
HR is 44/min, BP is 88/50 mm Hg, RR is 12/min (on ventilator),
and Spo2 is 94% (FIO2 of 0.5). She remains unresponsive and
laboratory tests are pending.

Which of the following combinations of rewarming techniques is


most appropriate for this patient?

A. Forced-air rewarming blanket and warm IV fluids

B. Warm blanket, and gastric and bladder lavage with warm fluid

C. Heated, humidified oxygen and cardiopulmonary bypass

D. Forced-air rewarming blanket and hemodialysis


10. In a patient with significant heat exposure, which of the
following findings is diagnostic for heatstroke rather than heat
exhaustion?

A. Tachycardia

B. Orthostatic hypotension

C. Seizures

D. Nausea and vomiting

E. Absence of sweating

11. A 32-year-old woman ingested 50 tablets of


hydrocodone/acetaminophen, 5 mg/500 mg, approximately 7
hours before her friends brought her to the hospital. On
admission to the ICU she is somnolent with a weak gag reflex. BP
is 90/54 mm Hg, HR is 64/min, and RR is 8/min.

Which of the following is the most appropriate initial treatment


plan?

A. Administer IV naloxone, 2 mg.

B. Intubate the patient and administer IV N-acetylcysteine, 150


mg/kg over 60 min.

C. Administer IV naloxone, 2 mg, and obtain a serum


acetaminophen level.

D. Administer IV naloxone, 2 mg, and IV N-acetylcysteine, 150


mg/kg over 60 min.
12. A 62-year-old man with hypertension and hyperlipidemia has
experienced depression since the recent death of his wife. His
daughter finds him unresponsive after he does not show up for
work. On admission to the hospital, he has an HR of 40/min, BP of
86/40 mm Hg, RR of 12/min, and temperature of 36°C (96.7°F).
His home medications include sustained-release verapamil,
hydrochlorothiazide, and atorvastatin. ECG shows sinus
bradycardia with prolonged PR interval. He receives IV fluids,
atropine, calcium chloride, and glucagon with no improvement in
heart rate or blood pressure.

Which of the following should be considered for the next


intervention to improve his hemodynamic status?

A. Milrinone

B. Transcutaneous pacing

C. Insulin and glucose infusion

D. Hemodialysis

13. A 39-year-old man presents to the hospital after developing


epistaxis and difficulty swallowing. He reports that he ingested 2
boxes of rat poison the week before in a suicide attempt but
changed his mind. His physical examination is remarkable for
multiple ecchymoses on his lower extremities, mild epistaxis, and
a 2- × 3-cm hematoma in the left tonsillar area. His international
normalized ratio is 14, hemoglobin level is 11.2 g/dL, and platelet
count is normal. He is admitted to the ICU because of potential
airway compromise.
Which of the following is the best treatment option for this
patient?

A. Fresh frozen plasma and IV vitamin K

B. Fresh frozen plasma alone

C. Fresh frozen plasma and subcutaneous vitamin K

D. IV vitamin K alone

14. A 35-year-old man is admitted to the ICU with a reported


history of ethylene glycol ingestion. A friend found him comatose
after not being able to contact him for 48 hours. An empty can of
antifreeze was found in the apartment. The patient is intubated
for airway protection. BP is 110/60 mm Hg, HR is 110/min, RR is
18/min, and temperature is 36.5°C (97.6°F). Physical
examination reveals obtundation, tachycardia, and bilateral
diffuse rales. Laboratory examination shows a room air arterial
blood gas measurement with pH of 7.09, Paco2 of 30 mm Hg,
Pao2 of 65 mm Hg, anion gap of 28 mEq/L, osmolar gap of 8
mOsm/kg, creatinine level of 2.4 mg/dL, and negative serum
ethanol results. An ethylene glycol level is ordered, but results
will not be available for 72 hours.

Which of the following interventions is most likely to benefit this


patient?

A. Ethanol infusion

B. IV fomepizole alone

C. Hemodialysis alone
D. Hemodialysis and IV fomepizole

15. A 68-year-old man is admitted for repair of a traumatic femur


fracture. On postoperative day 1, he has hand tremors, mild
tachycardia, and diaphoresis. He is oriented and responds
appropriately. On questioning, he admits to drinking 2 to 3 mixed
drinks every night. While awaiting the response of the physician,
the patient suffers a tonic-clonic seizure lasting 2 minutes, which
resolves before any medications can be given. He is transferred to
the ICU. HR is 110/min, BP is 160/94 mm Hg, RR is 18/min,
temperature is 37.6°C (99.6°F), and Spo2 is 95% on oxygen, 2
L/min via nasal cannula. Which of the following is the most
appropriate intervention?

A. IV metoprolol

B. IV lorazepam

C. IV fosphenytoin

D. Observation

16. Which of the following is the recommended daily caloric


requirement for patients with major burn injury?

A. 20–25 kcal/kg

B. 25–30 kcal/kg

C. 30–40 kcal/kg

D. 40–50 kcal/kg
PART 7: Environmental and Toxicologic Injury

ANSWERS:

1–B; 2–B; 3–C; 4–D; 5–B; 6–B; 7–B; 8–A; 9–A; 10–C; 11–D; 12–C;
13–A; 14–C; 15–B; 16–C

RATIONALE (1) Answer: B

The electrical current travelled from his hands and exited from
his groin and left foot (vertical pathways). Any tissue between
these sites may have suffered injury. As a result, the extent of the
burn, 5% total body surface area (TBSA), may be misleading. The
potential for this patient to develop myoglobinemia due to
muscle necrosis is high. Fluid resuscitation should be targeted to
keep the urine output brisk in order to prevent acute renal injury.
With severe muscle necrosis or a pH lower than 7.1, sodium
bicarbonate (correct answer B) at 1–2 mEq/kg will alkalize the
urine and limit the precipitation of myoglobin in the renal
tubules. Calcium channel blockers (option A) given systemically
will cause hypotension. Elevation of the arm (option C) may
decrease perfusion, while lowering the arm (option D) will result
in increased edema formation.

RATIONALE (2) Answer: B

This patient has a significant smoke inhalation injury with


evidence of facial burns and elevated carboxyhemoglobin level
(40%). The patient’s inability to protect his airway, particularly in
the presence of facial burns, should be recognized in the primary
survey and addressed with immediate intubation. Once the
airway is secure, further workup and therapy can begin.
Inhalation injury is composed of 3 major elements. The most
common is particulate injury due to the smoke and soot that is
inhaled. This may result in reactive airway and secretion
clearance problems. Therapy is directed toward symptomatic
relief including supplemental humidified oxygen, beta nebulizers,
and time. The second component of inhalation injury is thermal
and is due to the direct effects of heat on the upper and lower
respiratory tract. This is a very serious component that may
result in respiratory epithelial sloughing, ulceration, and mucous
plugging. Severe cases may lead to tracheal stenosis due to scar
formation. Bacterial pneumonia after thermal injury is
commonplace. The upper respiratory tract is well developed to
dissipate both hot and cold air as it passes through the nasal
passages, hypopharynx, and supraglottic area. However, once the
hot air passes below the glottis, there is little to dissipate the
heat. Lower respiratory tract thermal injury including alveolar
damage is the end result. The last form of inhalation injury is due
to the toxic breakdown products of combustion. Carbon
monoxide (CO), cyanide, chlorides, ammonia, hydrogen sulfide,
and various aldehydes are the main components of these toxins.
Hemoglobin has a 200 times greater affinity for CO than for
oxygen. Treatment for CO includes high-concentration oxygen
therapy to favor off-loading of CO from the hemoglobin molecule.
Unfortunately, CO quickly dissolves into the central nervous
system and will generate the formation of reactive oxygen species
and lipid peroxidases. The long-term clinical neurologic
implications of CO poisoning include cognitive impairment,
memory loss, and neuropsychological dysfunction. Hyperbaric
therapy is aimed at lessening the possibility of central nervous
system impairment rather than normalizing serum
carboxyhemoglobin (COHb) levels. Often by the time patients
begin hyperbaric therapy, they have cleared their serum level of
COHb with simple 100% oxygen therapy. Therapy ideally is
started within the first 12 hours of injury. Amyl nitrate and
sodium thiosulfate are antidotes for cyanide poisoning. Amyl
nitrate converts hemoglobin to methemoglobin. The cyanide
binds to the methemoglobin because it contains ferric iron.
Sodium thiosulfate increases the sulfur molecule pool, favoring
the conversion of cyanide to thiocyanate, which is excreted by the
kidneys.

Bronchoscopy has a central role to play in the diagnosis and


treatment of smoke inhalation injury due to particulate and
thermal causes. In addition to the visual assessment of the airway
for a burn injury, aggressive pulmonary toilet can aid in the
directed suctioning of mucus, soot, and debris from the large and
midsized airways. The figure demonstrates the dense soot
impacted on the respiratory epithelium in a patient with
inhalation injury. It may take from 48 to 72 hours to clear all of
the particulate matter from the airways. Bronchoscopy may be
performed after the patient is intubated awaiting hyperbaric
therapy and then on a daily basis until the patient improves.

RATIONALE (3) Answer: C

The pulse oximeter oxygen saturation (Spo2) of 98% is likely to


be misleading due to the presence of carbon monoxide (CO) at
40%. Most hospital oximeters are not able to differentiate
between oxyhemoglobin, carboxyhemoglobin (COHb), and
methemoglobin. Both oxyhemoglobin and carboxyhemoglobin
absorb light at 660 nm. The pulse oximeter determines Spo2 from
the relationship of the pulsatile component of light refractance at
2 different wavelengths. It measures reduced hemoglobin and
oxyhemoglobin. If carboxyhemoglobin is present, the 2-channel
oximeter cannot distinguish the difference, and gives an artificial
reading for the Spo2. Thus the Spo2 of 98% is a measure of both
the oxyhemoglobin and carboxyhemoglobin binding. The
mathematical relationship is

Sao2 = Spo2 × (1-SaCO).


 COHb 0%–10%—Usually no symptoms

 COHb 10%–20%—Mild headache, shortness of breath

 COHb 20%–30%—Moderate headache, impaired


concentration

 COHb 30%–40%—Severe headache, cognitive difficulties

 COHb 40%–50%—Confusion, lethargy, syncope

 COHb 50%–60%—Respiratory failure, seizures

 COHb >60%—Coma, death

Administration of 100% oxygen to this patient can lessen the


half-life of carbon monoxide from 3.5 hours to 90 minutes.
Utilizing 100% FIO2 until adequate half-life degradation is
achieved should be the goal of therapy.
Figure 3 Right mainstem bronchus covered with soot from a
structure fire

RATIONALE (4) Answer: D

This patient developed an abdominal compartment syndrome


(ACS). The combination of larger fluid requirements for
resuscitation and the constellation of worsening pulmonary
mechanics, tachycardia, hypotension, and progressive oliguria
suggest this diagnosis. ACS represents an abnormal physiology
whereby increased intra-abdominal pressures impair normal
renal, cardiovascular, and pulmonary physiology. As intra-
abdominal pressure rises, pressure on the inferior vena cava
decreases right heart filling, resulting in lower blood pressure. In
a compensatory manner, heart rate increases to maintain
adequate cardiac output. Pressure on the diaphragm may result
in diaphragmatic elevation yielding lower tidal volumes and
higher airway pressures. Increased intra-abdominal pressure
also impairs the venous outflow from the kidneys, resulting in
lower renal perfusion and yielding diminished urinary output. In
patients with elevated intracranial pressure, manifestations of
high intra-abdominal pressures can be reflected in rising
intracranial pressure. The quickest method to get an accurate
measurement of intra-abdominal pressure is by measuring
bladder pressure. Placing 75 mL of fluid into the bladder,
clamping the Foley catheter at the hub, and then simply placing a
pressure transducer into the urinary catheter’s aspiration port
will give an approximation of the intra-abdominal pressure. If
surgical decompression is performed, postoperative management
will be complicated by an open abdomen. Temporary closure
with a vacuum-assisted closure (commercial or in-house
fabricated) or with a Bogota bag (sterile open IV bag sutured into
the fascia) are the standard methods to manage this problem.

RATIONALE (5) Answer: B

The most commonly used formula for the amount of fluid to be


given for resuscitation of patients with burns greater than 20% of
total body surface area is the Parkland formula. This formula uses
4 mL per percent of total body surface area involved with burns,
per kilogram of body weight. Half of this amount is administered
in the first 8 hours and the remainder is infused over the next 16
hours. Thus, in this patient the correct amount to infuse in the
first 8 hours is 4,800 mL. This calculation makes options C and D
incorrect. Lactated Ringer solution is most commonly used for
burn resuscitation, but glucose-containing fluids are avoided due
to concerns of hyperglycemia and osmotic diuresis. Thus, option
A is incorrect. It is important to remember that the formula
provides a starting point for fluid resuscitation, and modifications
of the quantity of fluid may be needed based on the patient’s
response. Although the usual endpoint of resuscitation is the goal
of urine output of 0.5 to 1.0 mL/kg/h, urine output may not
reflect adequacy of resuscitation in patients with renal
insufficiency, morbidly obese patients, and patients with very
large burns.

RATIONALE (6) Answer: B

Poisoning with nerve gases, such as sarin (O-isopropyl


methylphosphonofluoridate), result in a cholinergic syndrome
suggested by these clinical findings. A cholinergic syndrome can
also result from organophosphate or carbamate poisoning. The
toxicity is caused by inhibition of cholinesterase, which prevents
degradation of acetylcholine at the postsynaptic receptor.
Muscarinic effects include bronchorrhea, bradycardia, and a
hypersecretory syndrome (salivation, lacrimation, urination,
defecation, emesis). Nicotinic effects result in muscle
fasciculations and weakness, and central nervous system effects
lead to headache, confusion, and central respiratory depression.
Pupils are typically miotic. The primary concern in such patients
is hypoxic respiratory failure from bronchorrhea, bronchospasm,
and respiratory depression. IV atropine should be administered
in doses of 2 to 4 mg repeated every 5 minutes until
tracheobronchial secretions are controlled. The most common
cause of death in cholinergic poisonings is asphyxiation.
Continued administration of large doses of atropine may be
required as bolus or continuous infusion. Because sarin
irreversibly binds to cholinesterase, the patient is likely to
require continuous atropine infusion in an ICU setting until the
agent is completely metabolized. Atropine does not reverse
nicotinic effects and pralidoxime is used to reverse muscle
weakness by regeneration of acetylcholinesterase. It is
administered as a loading dose (1–2 g in 500 mL of normal saline
solution over 30 minutes) and then as a continuous infusion at
500 mg/h. Pyridostigmine, an anticholinesterase, may precipitate
a cholinergic crisis and is used for treatment of myasthenia
gravis. Hyperbaric oxygen may be considered for patients with
severe carbon monoxide poisoning, but patients would not
typically have evidence of a hypersecretory syndrome. Patients
with carbon monoxide poisoning will often present comatose
with an anion gap metabolic acidosis, cardiovascular instability
and increased levels of carboxyhemoglobin. Sodium nitrite and
sodium thiosulfate are used as an antidote in cyanide poisoning.
Cyanide poisoning usually presents with more life-threatening
manifestations, such as sudden cardiovascular collapse, seizures,
hypotension, arrhythmias, and severe metabolic acidosis.

RATIONALE (7) Answer: B

The patient described is suffering from selective serotonin


reuptake inhibitor syndrome. The syndrome may occur after a
single dose, high-dose overdose, or when combined with other
serotonergic agents in therapeutic dosages. Drugs associated
with serotonin syndrome include monoamine oxidase inhibitors,
other antidepressants, opiates, antiemetics, lithium, linezolid,
tramadol, drugs of abuse, and others. The pathophysiology is
probably related to excessive stimulation of serotonin receptors
in the brain. Clinical manifestations include altered mental status,
autonomic dysfunction, and neuromuscular abnormalities
ranging from tremors to myoclonus and rigidity. Although the
clinical manifestations are very similar to neuroleptic malignant
syndrome, the management differs based on the differing
pathophysiologies of the 2 syndromes. Management is largely
supportive. Intubation and mechanical ventilation may be
necessary for patients with significant altered mental status.
Benzodiazepines are the primary therapy indicated for control of
agitation. Control of autonomic instability and external cooling
for sustained hyperthermia may be needed. A neuromuscular
blocker would be contraindicated since it would preclude the
evaluation of seizure activity. Dantrolene and bromocriptine are
not recommended in this disorder. Serotonin antagonists such as
propranolol, cyproheptadine, and methysergide have been
proposed, but evidence for benefit is lacking and routine use is
not recommended

RATIONALE (8) Answer: A

This clinical scenario and the blood chemistry findings in this


patient suggest propylene glycol toxicity. Each vial of lorazepam
(2 mg/mL) contains 830 mg/mL of propylene glycol. Propylene
glycol is also a solvent in IV formulations of diazepam, etomidate,
phenytoin, nitroglycerin, esmolol, phenobarbital, and
pentobarbital. The most common manifestations of propylene
glycol accumulation are anion gap metabolic acidosis and
increased osmolar gap. Approximately 12%–45% of propylene
glycol is excreted unchanged in the urine in healthy individuals,
and the remainder is metabolized by the liver to lactate and
pyruvate. Accumulation may occur when doses exceed the upper
recommended dose of 0.1 mg/kg/h. Renal or hepatic
insufficiency may also play a role in accumulation. Serious
reported toxicities include renal dysfunction, hemolysis, cardiac
arrhythmias, seizures, and central nervous system depression or
agitation. The metabolic findings of propylene glycol toxicity may
also be confused with sepsis or severe inflammatory states.
Clinical studies suggest that the osmolar gap correlates with
propylene glycol accumulation. Although toxicity is more
common after long periods of lorazepam infusion (>3 days),
toxicity has occurred with short-term, high-dose use. The
treatment of choice is to stop the lorazepam infusion and sedate
with an agent that does not contain propylene glycol. Midazolam
(option A) does not contain propylene glycol. It would not be
appropriate to change to fentanyl alone (option B) in a patient
who requires benzodiazepines for alcohol withdrawal.
Hemodialysis would remove propylene glycol but is not required
unless severe renal dysfunction develops. Thus, option D is
incorrect. Propylene glycol is metabolized by alcohol
dehydrogenase but there are no reports of using ethanol infusion
or fomepizole for toxicity.

RATIONALE (9) Answer: A

The choice of rewarming technique for a hypothermic patient


should take into account the clinical condition of the patient
(severity of hypothermia, hemodynamic stability), presence of a
perfusing or nonperfusing rhythm, availability of resources, and
assessment of advantages and disadvantages of available
techniques. This patient has moderate hypothermia with a
perfusing rhythm. A combination of active external rewarming
and less invasive active core rewarming techniques would be a
reasonable choice for this patient. The most commonly used
active external rewarming technique is a forced-air rewarming
blanket, which has a rewarming rate of 1.5°C–2.5°C/h. Other
external rewarming techniques are warm water immersion,
heating pads, and radiant heat lamps. Active core rewarming
techniques include heated, humidified oxygen; warm IV fluids;
thoracic lavage; peritoneal lavage; gastric/bladder/ rectal lavage;
hemodialysis; cardiopulmonary bypass; and extracorporeal
membrane oxygenation. The application of a warm blanket is an
example of passive external rewarming and is usually used with
all other techniques but requires the addition of other measures
in patients with moderate to severe hypothermia. Gastric and
bladder lavage depend on surface contact area for conductive
heat transfer and have limited efficacy. Cardiopulmonary bypass
is not warranted in this patient since she has a perfusing rhythm
and blood pressure. Hemodialysis is invasive and would probably
result in further hypotension which may have adverse cardiac
effects.

RATIONALE (10) Answer: C

The presence of central nervous system (CNS) dysfunction in the


setting of significant heat exposure distinguishes heatstroke from
milder forms of heat injury such as heat exhaustion. CNS
dysfunction can include cerebellar abnormalities, seizures, and
altered level of consciousness (including coma). Although
heatstroke is usually associated with temperatures greater than
40°C (104°F), the temperature measured on arrival to a hospital
may not reflect exposure because of cooling during transport.
Tachycardia, orthostatic hypotension, and nausea and vomiting
can occur in heatstroke and heat exhaustion. Absence of sweating
is not a reliable diagnostic finding for heatstroke as patients with
exertional heat stroke may have profuse sweating

RATIONALE (11) Answer: D

This patient has overdosed on 2 potentially life-threatening


toxins: an opiate and acetaminophen. A narcotic overdose can
usually be managed by reversal of life-threatening effects
(hypoventilation, central nervous system depression) with
naloxone, a pure opioid antagonist. The goal of therapy is to
restore adequate spontaneous respirations rather than complete
arousal. Given the large quantity of hydrocodone ingested, an
initial dose of 2 mg is more appropriate than 0.4 mg of naloxone.
Doses of naloxone up to 20 mg may be required in patients who
have taken large quantities of opioids or synthetic opioids such as
hydrocodone, oxycodone, methadone, and fentanyl. Intubation is
not necessary if the patient arouses with naloxone. However,
because of the short half-life of naloxone, an infusion (onehalf to
two-thirds of the dose that reversed respiratory depression,
administered hourly and titrated as needed) may be needed until
the respiratory depressive effects of the opioid wane. The toxic
dose of acetaminophen varies by individual but any dose greater
than 140 mg/kg in an acute ingestion should be considered
potentially toxic. Although an acetaminophen level is needed to
compare with the Rumack-Matthew nomogram to determine the
need for antidote, the late presentation of this patient mandates
administering an initial dose of N-acetylcysteine while waiting for
results. N-acetylcysteine (IV or oral) is most effective at
preventing liver toxicity when administered in the first 8 hours
after ingestion. This patient requires both naloxone and
immediate administration of N-acetylcysteine, making option D
the correct choice. Options A and C are incorrect since they do not
include N-acetylcysteine administration. Option B is not correct
since intubation is not warranted as the first step.

RATIONALE (12) Answer: C

This patient’s clinical presentation is consistent with overdose of


a calcium channel blocker. Typical findings include hypotension
and bradycardia. The hypotension in calcium-channel blocker
overdose, especially verapamil, is related to negative inotropy
more than bradycardia. Atropine is frequently given in these
situations but it is rarely effective. The appropriate initial
intervention is the administration of IV calcium chloride (bolus
and infusion) but it is only effective in reversing toxic effects in
approximately 50% of patients. After failure of calcium, glucagon
is a reasonable next intervention even though data on improving
hemodynamics are better with beta-blocker overdose. Glucagon
acts as an inotropic agent by increasing intracellular cyclic
adenosine monophosphate, a mechanism similar to adrenergic
agents. After failure of these interventions, the use of regular
insulin infusion with glucose infusion to maintain euglycemia is
most appropriate. Insulin acts as an inotropic agent but the exact
mechanism is not known. Reported doses of insulin have been
variable but 0.5 to 1.0 U/kg/h have been most commonly used.
Use of insulin euglycemia has been reported with beta-blocker
toxicity, but experience is limited. Transthoracic pacing may
increase the heart rate but will not improve contractility or blood
pressure (Option B). Milrinone can be considered as an inotropic
agent in calcium channel blocker toxicity but it would most likely
worsen the hypotension (Option A). Evidence of benefit of
milrinone in this setting is limited. Calcium channel blockers are
lipophilic, highly protein bound, and primarily undergo hepatic
metabolism. Thus, hemodialysis (Option D) would be unlikely to
remove a significant amount of drug

RATIONALE (13) Answer: A

Rat poisons contain “super warfarins” that are more potent and
longer acting than warfarin. The most common agent involved in
poisonings is brodifacoum. These agents inhibit the synthesis of
vitamin K and thus the synthesis of clotting factors II, VII, IX, and
X. The risk of bleeding increases with a higher international
normalized ratio (INR). In a patient with active bleeding,
immediate factor replacement is required along with
administration of IV vitamin K. In this patient, bleeding with a
pharyngeal hematoma posed a threat to airway patency and
requires emergent treatment. Fresh frozen plasma in large doses
is required (15 mL/kg) and must be repeated every 6 to 8 hours
to maintain sufficient factor levels. Although not approved for
this use, other options for factor replacement are prothrombin
complex concentrate or activated factor VII. IV vitamin K is
indicated in this situation because it corrects the INR faster than
subcutaneous administration. Thus, Option A offers the best
interventions to quickly reverse the coagulopathy. Oral vitamin K
is indicated after stabilization for weeks to months due to the
long half-life of the “super warfarin.”

RATIONALE (14) Answer: C

Hemodialysis is most likely to benefit this patient by removing


acid metabolites of ethylene glycol. Methanol and ethylene glycol
poisonings may be characterized by the presence of an osmolar
gap and anion gap metabolic acidosis. All alcohols are osmotically
active substances, but only methanol and ethylene glycol are
metabolized to acids (see figure below). Isopropyl alcohol is
metabolized to acetone and does not result in an anion gap
metabolic acidosis. However, patients ingesting ethylene glycol
may present with a normal osmolar gap due to complete
metabolism of the alcohol to toxic metabolites (late presentation)
or errors in the performance of the serum osmolarity. Similarly,
patients ingesting ethylene glycol may present with an osmolar
gap and normal anion gap if concurrent ethanol ingestion
prevents metabolism to acid metabolites or they present very
early after ingestion. Ethanol is preferentially metabolized by
alcohol dehydrogenase. In this patient, the history is strongly
suggestive of ethylene glycol ingestion, and it is supported by the
laboratory findings of anion gap acidosis, obtundation, and renal
insufficiency. The low pH is a poor prognostic marker and
immediate hemodialysis should be initiated. The value of ethanol
infusion or fomepizole in a delayed presentation is questionable,
because ethylene glycol is likely to be completely metabolized by
alcohol dehydrogenase to toxic metabolites. However, if there is
any suspicion of the presence of ethylene glycol, oral or IV
ethanol or IV fomepizole can be initiated. IV fluids are indicated
for volume resuscitation, and mechanical ventilatory support can
be increased to offset the low pH. Delayed presentations usually
do not have urinary oxalate crystals that may be present
transiently within 4 to 6 hours after ethylene glycol ingestion.

RATIONALE (15) Answer: B

Approximately 5%–10% of patients with mild symptoms of


alcohol withdrawal develop seizures. Most seizures are brief and
self-limited but multiple seizures (2 to 6) can occur in up to 60%
of patients. Even when the seizure has terminated, IV lorazepam,
2 mg, should be administered to prevent recurrent seizures
(option B). Lorazepam would also help with preventing
progression to delirium. Fosphenytoin is not effective, which
makes option C incorrect. A beta-blocker can treat minor
withdrawal symptoms such as tachycardia and hypertension but
will not prevent seizures or prevent progression to delirium,
making option A incorrect.

RATIONALE (16) Answer: C

Burn patients with significant injuries have a hypermetabolic


state due to release of catabolic hormones such as
catecholamines, cortisol, and glucagon. Formulas for estimating
resting energy expenditure are often inaccurate in burn patients
because of confounding conditions. Indirect calorimetry can be
useful in estimating caloric needs but is not available in all
institutions. The current recommendation for daily caloric intake
is 30–40 kcal/kg/day (option C). Higher caloric intake has been
associated with decreases in infections, hospital length of stay,
and mortality. Protein should make up 20%–25% of calories with
an intake of 2–2.5 g/kg/day. Carbohydrates should make up
approximately 60% of the daily calories. Enteral nutrition is
preferred.
PART 8: Pharmacologic Issues in the ICU

Instructions: For each question, select the most correct answer.

1. A 70-year-old man has been admitted to the ICU after a


colectomy because of severe chronic obstructive pulmonary
disease and pain management. The procedure was
uncomplicated, and the patient was extubated afterward. On
examination, the patient’s RR is 16/min, and he rates his pain on
a 0–10 scale as an 8. Arterial blood gas values are pH of 7.36,
Paco2 of 45 mm Hg, Pao2 of 75 mm Hg, and bicarbonate level of
28 mEq/L on oxygen, 4 L/min by nasal cannula.

Which of the following analgesics for postoperative pain


management is most appropriate to minimize respiratory
depression?

A. Morphine

B. Meperidine

C. Butorphanol

D. Hydromorphone

E. Oxycodone

2. A 45-year-old woman with chronic renal failure is admitted to


the ICU with a hypertensive emergency. After 30 hours of
treatment with nitroprusside, the patient develops confusion and
metabolic acidosis. Administration of which of the following is
most likely to prevent/ treat these symptoms?
A. Thiosulfate

B. Cyanocobalamin

C. Pyridoxine

D. Glucagon

E. Calcium chloride

3. A 44-year-old man with HIV is admitted to the ICU with severe


community-acquired pneumonia requiring mechanical
ventilation. The patient is on an outpatient retroviral regimen
that includes a protease inhibitor; lopinavir/ritonavir; and a
nucleoside reverse-transcriptase inhibitor, zidovudine. The
patient has done very well with this regimen (viral load is
nondetectable), and his infectious disease physician recommends
that he be kept on this regimen during his ICU hospitalization.
Which of the following drugs should be avoided in this case?

A. Midazolam

B. Propofol

C. Haloperidol

D. Lorazepam

E. Morphine

4. A 47-year-old woman who is 155 cm tall (5 ft 1 in) and weighs


100 kg (222 lbs) falls down a flight of stairs. Past medical history
is positive for insulin-dependent diabetes mellitus for 20 years
and chronic alcohol dependence. Her preoperative blood urea
nitrogen level is 30 mg/dL, serum creatinine level is 3.2 mg/dL,
serum glucose level is 160 mg/dL, aspartate aminotransferase
level is 110 U/L, and international normalized ratio is 1.4.

After open reduction and internal fixation of a left femoral neck


fracture, she is admitted to the ICU because of severe alcohol
withdrawal. The following day, symptoms of withdrawal are
much decreased, but she complains of a sudden onset of
shortness of breath. A diagnosis of pulmonary embolism is made
by helical CT scan. Subcutaneous enoxaparin, 150 mg every 12
hours is begun.

Which of the following statements best characterizes her current


enoxaparin dose?

A. She is excessively anticoagulated because dosing should be


based on ideal body weight, not actual body weight.

B. She is inadequately anticoagulated because of morbid obesity


and increased volume of distribution.

C. She is excessively anticoagulated because of hepatic


dysfunction.

D. She is inadequately anticoagulated because of increased


cytochrome P-450 activity.

E. She is excessively anticoagulated because of renal dysfunction.

5. A 54-year-old man with no medical problems is admitted with


severe acute pancreatitis. He deteriorates quickly in the first 2
days of hospitalization and requires intubation. On ICU day 3, the
patient develops worsening abdominal distension, refractory
hypotension, hypoxia, and elevated peak airway pressures. His
liver function test results are normal, and his glomerular
filtration rate is estimated to be less than 30 mL/min/1.73 m2.
Bladder pressures are consistent with diagnosis of abdominal
compartment syndrome. To optimize abdominal perfusion
pressure, sedation and continuous neuromuscular blockade is
considered. Which of the following is the best choice for
neuromuscular blockade in this patient?

A. Succinylcholine

B. Vecuronium

C. Pancuronium

D. Cisatracurium

6. A 55-year-old woman presenting to the hospital with


subarachnoid hemorrhage receives management including
nimodipine, phenytoin, and famotidine. On hospital day 12, she
has a diffuse, erythematous skin rash; confusion; hypotension;
and severe distress. In the ICU, she has labored respirations at
30/min, a systolic BP palpable at 80 mm Hg, pulse rate of
150/min, and temperature of 39.5°C (103°F). Periorbital edema
is noted, with a diffuse, morbilliform, maculopapular rash
covering her entire body including the palms and soles. Which of
the following agents is associated with this disease?

A. Nimodipine

B. Phenytoin

C. Aminoglycosides

D. Famotidine
7. A 34-year-old man is admitted to the ICU after he is found
unconscious at home. He has a history of chronic back pain,
diabetes mellitus, and depression. His current medications
include the following:

Extended-release venlafaxine, 225 mg/day

Acetaminophen, 1,000 mg every 6 hours as needed for pain

Tramadol, 60 mg every 6 hours

Glipizide, 10 mg/day

Metformin, 1,000 mg every 12 hours

Blood cultures grow gram-positive cocci and chest radiography


shows bilateral lower lobe infiltrates. The patient weighs 100 kg
(220 lbs) and is 188 cm (6 feet 2 inches) tall. He has anaphylactic
allergic reaction to penicillin. His temperature is 39.8°C (104°F),
BP is 76/30 mm Hg, and HR is 120/min. His laboratory findings
include the following: blood urea nitrogen, 37 mg/dL; serum
creatinine, 2.7 mg/dL; sodium, 132 mEq/L; potassium, 3.6
mEq/L; glucose, 287 mg/dL; lactic acid, 2.8 mg/dL; and creatine
kinase, 23,000 U/L. He is diaphoretic and unresponsive.

Which of the following interventions will minimize the chances of


patient mortality?

A. Start IV daptomycin, 6 mg/kg daily.

B. Start IV linezolid, 600 mg every 12 hours.


C. Give IV vancomycin, 2,500 mg, followed by IV vancomycin,
1,500 mg daily with a goal trough of 15–20 µg/mL.

D. Start IV tigecycline, 100 mg, followed by IV tigecycline, 50 mg


every 12 hours.

8. A 72-year-old man with stage 3 chronic kidney disease had an


emergent open repair of ruptured thoracic aortic aneurysm 4
days ago. He is now oliguric with a serum creatinine level of 5.1
mg/dL. He reports incisional pain after extubation. Which of the
following agents is most appropriate for the management of his
acute pain in the ICU?

A. Morphine

B. Fentanyl

C. Methadone

D. Meperidine

9. A 37-year-old man with type 1 diabetes mellitus and a history


of kidney transplantation 7 years ago presents with delirium
caused by diabetic ketoacidosis. His calculated serum osmolarity
peaks at 304 mOsm/L, and serum glucose level reaches 528
mg/dL. Initial serum sodium level is 132 mEq/L, which
normalizes to 139 mEq/L after 18 hours. The patient’s mental
status normalizes 2 days after the administration of insulin and
hydration. Three weeks later, he notes that his right arm and leg
are “fidgety,” a symptom that rapidly increases in intensity.
Physical examination reveals right hemichorea with a ballistic
component. MRI of the brain is shown in the Figure.
Which of the following best explains the above symptoms and
radiologic findings?

A. Ischemic stroke

B. Septic emboli to the brain


C. Osmotic demyelination syndrome

D. Huntington chorea

E. Autoimmune demyelination

10. A 38-year-old, morbidly obese man is admitted to the ICU for


persistent cyanosis and reported hypoxemia after undergoing
bronchoscopy and transbronchial biopsy for suspected
sarcoidosis. He underwent the procedure with topical benzocaine
anesthetic plus midazolam and fentanyl for sedation. Shortly
afterward, he became hypoxic, with Spo2 of 85% on pulse
oximetry and cyanotic lips. His examination on arrival to the ICU
reveals equal bilateral breath sounds and bilateral rales, which
were also noted on preprocedure examination. On FIO2 of 1.0 via
nonrebreather mask, his oxygen saturations fail to
improve.Arterial blood gas studies on nonrebreather mask show
a pH of 7.30, Paco2 of 30 mm Hg, and Pao2 of 380 mm Hg.

Which of the following is the most likely etiology of the patient’s


decompensation?

A. Iatrogenic pneumothorax

B. Aspiration pneumonitis

C. Intracardiac shunt

D. Topical benzocaine

E. Hypoventilation from oversedation

11. A 39-year-old woman is brought to the emergency


department for suspected overdose after being found down at
home by her fiancé. The toxicology screen reveals tricyclic
antidepressants and benzodiazepines. Her admitting ECG is
shown in the Figure.

While in the emergency department, she has worsening mental


status and is intubated for airway protection after onset of
seizures. Shortly after intubation, she has an unstable, wide-
complex tachycardia requiring resuscitation.

Which of the following management steps is most appropriate?

A. Amiodarone for wide-complex ventricular tachycardia

B. Phenytoin for seizures

C. Flumazenil for coingestion of benzodiazepines

D. Sodium bicarbonate infusion to maintain alkalemia with pH


>7.45
12. Which of the following neuromuscular blocking agents would
be most appropriate for rapid-sequence intubation in a patient
with a contraindication to succinylcholine?

A. Pancuronium

B. Vecuronium

C. Atracurium

D. Rocuronium

13. A 25-year-old man is diagnosed with spinal osteomyelitis


after presenting to the hospital with back pain and fever. He has a
history of IV drug use and ethanol abuse.

Cultures grow methicillin-resistant Staphylococcus aureus. The


patient is discharged home after 2 weeks of vancomycin in the
hospital with linezolid, 600 mg twice daily (to complete a 6-week
course), and oxycodone, 10 mg every 6 hours as needed for pain.

Blood testing is done 4 weeks later and the patient is called by his
primary physician to be admitted as soon as possible to the
hospital. Laboratory data show the following: sodium, 132
mEq/L; potassium, 4.0 mEq/L; chloride, 108 mEq/L; bicarbonate,
7 mEq/L; blood urea nitrogen, 18 md/dL; creatinine, 0.9 mg/dL;
and glucose, 96 mg/dL. His WBC count is 8,000/µL with no left
shift, hemoglobin level is 14 g/dL, and platelet count is
98,000/µL.

An intensivist is called to evaluate the patient in the emergency


department for possible ICU admission. He is feeling fine, and
physical examination findings including vital signs are
unremarkable except for mild back tenderness on passive and
active motion.The repeat blood work is essentially unchanged,
and lactic acid level shows 6 mmol/L. Arterial blood gas results
are pH of 7.28, Paco2 of 20 mm Hg, and Pao2 of 104 mm Hg on
room air. The measured serum osmolality is 286 mOsm/kg.
Which of the following is the most likely diagnosis?

A. Early sepsis

B. Linezolid-induced lactic acidosis

C. Alcoholic ketoacidosis

D. Isopropyl alcohol intoxication

14. A 30-year-old man who was involved in a motor vehicle


collision is admitted to the ICU with deep partial- and full-
thickness burns to over 70% of his body surface area. Three days
after admission, he is in respiratory distress and requires
intubation.

Which of the following medications is contraindicated in this


patient?

A. Cisatracurium

B. Succinylcholine

C. Pancuronium

D. Vecuronium

E. Rocuronium
15. Which of the following bacteria are adequately treated with IV
colistin (colistimethate)?

A. Serratia marcescens

B. Burkholderia cepacia

C. Acinetobacter baumanii

D. Methicillin-resistant Staphylococcus aureus

E. Proteus vulgaris

16. A 63-year-old woman is admitted to the ICU for status


epilepticus. She is initially treated with lorazepam and a loading
dose of phenytoin, followed by a maintenance dose of 200 mg 3
times a day. Because of recurrent breakthrough seizures,
infusions of midazolam, 10 mg/h, and propofol, 15 mg/kg/h, are
started. In addition, she receives cefepime and linezolid for
suspected ventilator- associated pneumonia. Five days later, she
develops severe hypotension requiring norepinephrine infusion.
Her temperature is 38.6°C (101.5°F), and HR is 120/min with
intermittent bursts of supraventricular tachycardia. Laboratory
findings include the following: serum sodium, 142 mEq/L;
chloride, 100 mEq/L; bicarbonate, 12 mEq/L; potassium, 5.8
mEq/L; creatinine, 3.4 mg/dL; creatine kinase, 6,200 U/L; blood
lactate, 14 mg/dL; and hemoglobin, 8.5 g/dL. Urinalysis is
positive for hemoglobin but no red cells; chest radiograph is
unremarkable except for the presence of endotracheal and
nasogastric tubes. Arterial blood gas shows Pao2 of 90 mm Hg,
Paco2 of 26 mm Hg, and pH of 7.21.

Which of the following is the most appropriate next intervention?


A. Discontinue midazolam.

B. Discontinue propofol.

C. Discontinue linezolid.

D. Transfuse packed red blood cells to increase hemoglobin level


to 10 g/dL.

E. Start empiric antifungal coverage pending culture results.

17. Four days ago, a 59-year-old man had open repair of a


ruptured abdominal aortic aneurysm that resulted in acute
kidney injury. He is being weaned on the ventilator but is agitated
and not cooperative. His Confusion Assessment Method for the
ICU (CAM-ICU) score is positive. His current sedation is a
propofol drip, 40 µg/kg/min, and his current pain regimen is
fentanyl, 100 µg/h infusion. Dexmedetomidine is being
considered for this patient. Which of the following statements
about dexmedotomidine should guide management of this
patient?

A. The dose should be adjusted for the patient’s renal


insufficiency.

B. A loading bolus can cause transient hypotension.

C. It has been associated with osmolar gap.

D. It does not cause respiratory depression.

E. Effects are not changed by propofol or opioids.


PART 8: Pharmacologic Issues in the ICU

ANSWERS:

1–C; 2–A; 3–A; 4–E; 5–D; 6–B; 7–C; 8–B; 9–C; 10–D; 11–D; 12–D;
13–B; 14–B; 15–C; 16–B; 17–D

RATIONALE (1) Answer: C

Opioids have supraspinal and spinal modes of action. Opioid


receptors are found in the central nervous system, especially in
the midbrain and lamina I of the dorsal horn in the spinal cord, in
visceral and vascular smooth muscle, in musculoskeletal
structures, and at the terminals of peripheral sympathetic and
sensory neurons. Opioids modulate pain by 1) presynaptic opioid
receptor binding, inhibiting the release of excitatory
neurotransmitters; 2) decreased activity in interneurons; and 3)
postsynaptic hyperpolarization. Opioid receptors are classified
into groups based on their pharmacologic effects by an agonist.
The mu-receptors are subdivided into mu1 and mu2 receptors,
where the mu1 receptor is responsible for supraspinal analgesia,
and the mu2 receptor is responsible for respiratory depression,
bradycardia, and physical dependence. Stimulation of delta
receptors produces analgesia, mood alterations, and emesis.
Kappa receptors modulate spinal analgesia and have minimal
respiratory depression. Sigma receptor activation results in
excitatory symptoms, such as dysphoria, hypertonia, and
tachycardia. Morphine, meperidine, hydromorphone, and
oxycodone are nonspecific mu receptor agonists, which means all
will depress respiration to a similar extent when given as
equipotent doses. The mu opioid receptor mediates a dose-
dependent depression of ventilation, mainly via a direct action on
the medullary respiratory center. A decreased response to
hypoxic drive and a decreased response to carbon dioxide occur.
Abnormal breathing can be manifested as a shallow, rapid
breathing or slow Cheyne-Stokes respiration with normal or
large tidal volumes and intermittent apnea. These patients may
still be conscious and breathe when reminded. Butorphanol is an
opioid agonist-antagonist. Other agonist-antagonist opioids are
nalbuphine and pentazocine. The common effect seen when
giving this group of opioids is analgesia with minimal respiratory
depression. Each drug has a different effect on the other opioid
receptors. Butorphanol does not have significant mu receptor
activity, while nalbuphine and pentazocine are mu receptor
antagonists. Their analgesic activity is modulated by stimulation
of spinal kappa receptors. Use of this family of opioids on chronic
pain patients taking mu receptor agonists, such as oxycodone or
methadone, should be avoided or carefully monitored because of
the potential for causing opioid withdrawal.

RATIONALE (2) Answer: A

This patient is developing signs and symptoms consistent with


cyanide toxicity from nitroprusside. Nitroprusside has been
demonstrated to cause toxicity through the release of cyanide
and accumulation of thiocyanate. Cyanide toxicity can present
with unexplained cardiac arrest and changes in mental status,
including convulsions, encephalopathy, and coma. Metabolic
acidosis can also be present, although this may be a late event.
Risk of cyanide toxicity can be decreased by utilizing
nitroprusside at recommended doses for a short period of time. It
has also been recommended that patients receiving high doses of
nitroprusside (4–10 mg/kg/min) receive an infusion of
thiosulfate. Furthermore, hydroxocobalamin (vitamin B12a) is
safe and effective in preventing and treating cyanide toxicity
associated with use of nitroprusside. Hydroxocobalamin may be
given as a continuous infusion at a rate of 25 mg/h. It is
important to note that cyanocobalamin (vitamin B12) is not
effective as an antidote and is not capable of preventing cyanide
toxicity.

RATIONALE (3) Answer: A

With the widespread use of antiretroviral therapy in patients


with HIV, it is important for the intensivist to recognize some of
the potential drug interactions and side effects of antiretroviral
medications. Decisions regarding continuation of antiretroviral
therapy in patients admitted to the ICU should be discussed with
HIV experts. In addition, a team approach with consultation with
a critical care pharmacy specialist is important to avoid
complications related to drug interactions. Common ICU drugs
contraindicated with the use of non-nucleoside reverse-
transcriptase inhibitors include midazolam and triazolam.
Common ICU drugs, contraindicated with the use of protease
inhibitors include the following: midazolam, triazolam,
amiodarone, proton pump inhibitors (with atazanavir),
histamine2 blockers, propafenone, and quinidine.

Recognition of potential interaction between these medications


and retrovirals is important when treating patients with HIV in
the ICU. In this case, of the given list, midazolam would be
contraindicated both with the protease inhibitor and the non-
nucleoside reverse-transcriptase inhibitors. In such a patient,
alternatives to midazolam, such as lorazepam or oxazepam,
should be considered.

RATIONALE (4) Answer: E

Low-molecular-weight heparins (LMWHs) do not undergo


hepatic metabolism and primarily undergo renal elimination.
Generally, dosing of LMWH is based on actual body weight;
however, if a patient weighs more than 130 kg (286 lbs),
standard dosing may cause excessive anticoagulation. This
patient is likely to be excessively anticoagulated from LMWH
accumulation due to her renal failure, not from excessive LMWH
dose. The primary advantages of LMWH, compared with
unfractioned heparin (UFH), are better bioavailability and
consistency of action. Bioavailability is greater than 90% for
LMWHs, but only 30% for UFH. This difference is related to UFH’s
nonspecific binding to proteins and cells. This binding
contributes not only to its poor bioavailability but also affects
anticoagulant activity and decreases plasma half-life. UFH is
metabolized by a rapid, zero-order kinetic enzyme system within
the liver. The saturable hepatic kinetics results in inconsistent
elimination that also increases variability in drug effect. UFH and
its metabolites undergo first-order renal clearance kinetics. In
contrast, lessnegative-charged LMWH molecules result in
relatively little nonspecific protein binding, and dose-
independent renal clearance of LMWHs results in predictable
antithrombotic activity. Typically, anticoagulation monitoring is
not needed. Plasma half-life of LMWHs is approximately 2 to 4
times longer than that of heparin, enabling them to be
administered only once or twice daily. Subcutaneous LMWHs are
well absorbed from the subcutaneous tissue, are rapidly
distributed to most organs and tissues, and attain antithrombotic
levels within 30 minutes of administration. Subcutaneous
absorption of heparin is more variable, however, and
antithrombotic levels might not be reached for 1 to 2 hours.
Anticoagulation with LMWH can be achieved with minimal
elevation of partial thromboplastin time. When monitoring is
done, synthetic substrate-based assays for measuring thrombin
and factor Xa inhibition are used. The anti-Xa assays are more
sensitive to the effects of LMWHs and are used more commonly
than antithrombin assays. The College of American Pathologists
released a consensus report on the laboratory monitoring of
anticoagulant therapy in 1998. When monitoring LMWH, the
College of American Pathologists recommends checking anti-Xa
activity at peak effect (approximately 4 hours after
administration), because peak levels correlate with safety and
efficacy over trough levels, obtained just prior to administration
of a dose. For the acute management of venous thromboembolic
disease, the College of American Pathologists suggests a peak
target level of 0.5 to 1.1 U/mL for twice-daily recipients and 1.0
to 2.0 U/mL for once-daily patients. The College of American
Pathologists recommends monitoring for patients who have renal
insufficiency, for patients who will be receiving LMWHs for
prolonged periods (months), or for patients who are at extreme
body weights (130 kg [286 lbs]) or who are newborn.

RATIONALE (5) Answer: D

Neuromuscular blockade is available in both depolarizing and


nondepolarizing preparations. Depolarizing agents such
succinylcholine directly stimulate the acetylcholine receptors at
the postsynaptic endplate. They are not suggested for long-term
neuromuscular blockage Nondepolarizing preparations act as
competitive antagonists, binding to the postsynaptic endplate
acetylcholine receptors but not stimulating them. Several agents
have active metabolites, which can prolong neuromuscular
blockade. Both vecuronium and pancuronium are degraded by
the liver into active metabolites of 3-hydroxypancuronium and 3-
desacetylvecuronium, respectively, both of which have up to 50%
of the activity of the parent drug. These metabolites then depend
on renal excretion for clearance. So in hepatic or renal
insufficiency, the effect of the neuromuscular blockade can be
markedly prolonged. Cisatracurium is susceptible to degradation
by Hofmann elimination and ester hydrolysis, components of the
in vivo metabolic processes. Hofmann elimination is an organ-
independent chemodegradative mechanism, so there is little or
no risk from the use of cisatracurium in patients with liver or
renal disease compared to the use of other neuromuscular
blocking agents.

RATIONALE (6) ANSWER: B

Toxic epidermal necrolysis (TEN) and Stevens-Johnson syndrome


(SJS) are severe adverse cutaneous drug reactions that
predominantly involve the skin and mucous membranes. They
are characterized by mucocutaneous tenderness, hemorrhagic
erosions, erythema, and more or less severe epidermal
detachment presenting as blisters and areas of denuded skin.
These affect approximately 1 or 2 people per million annually and
are considered medical emergencies as they are potentially fatal.
The clinical diagnosis should be confirmed by a skin biopsy
showing full-thickness necrosis of the epidermis. Currently, TEN
and SJS are considered to be 2 ends of a spectrum of severe
epidermolytic adverse cutaneous drug reactions, differing only by
their extent of skin detachment.

Mycoplasma pneumoniae and herpes simplex virus infections are


well documented causes that exist alongside rare cases in which
the etiology remains unknown. Several drugs are at risk of
inducing TEN/SJS, including allopurinol,
trimethoprim/sulfamethoxazole and other sulfonamide
antibiotics, aminopenicillins, cephalosporins, quinolones,
carbamazepine, phenytoin, phenobarbital, and NSAIDs of the
oxicam type. Because of the high risk of mortality, management of
patients with SJS/TEN requires rapid diagnosis; evaluation;
identification and interruption of the culprit drug; specialized
supportive care, ideally in an ICU; and consideration of
immunomodulating agents such as high-dose IV immunoglobulin
therapy.

RATIONALE (7) Answer: C


Option A is incorrect because, while daptomycin would be an
appropriate agent for gram-positive bacteremia, it is inactivated
by lung surfactant and does not achieve therapeutic levels in the
lung. Therefore, it is not indicated for pneumonia. Daptomycin at
higher doses can cause elevation of creatine kinase
concentrations, and this patient has a significantly elevated
baseline creatine kinase level. Option B is incorrect because,
while linezolid is an appropriate agent for gram-positive
pneumonia, it is also a weak monoamine oxidase inhibitor
(MAOI). MAOIs can place the patient at risk of serotonin
syndrome when used concurrently with antidepressants. Option
C is correct. An immediate 25–30 mg/kg loading dose should be
followed by a 15–20 mg/kg dose adjusted for the patient’s renal
function. The ideal antibiotic regimen must be dosed
appropriately to treat the offending organism and reach the site
of infection with minimal drug interactions or toxicity. Increased
morbidity and mortality has been demonstrated in the literature
when the initial anti-infective is inappropriate or delayed. Option
D is incorrect because, while tigecycline is approved for
community-acquired pneumonia, its use for bacteremia is
controversial because of the risk of subtherapeutic blood levels
and treatment failure. Recent studies showing an increased risk
of all-cause mortality for high-risk infections treated with
tigecycline led the FDA to issue a warning to consider alternative
agents in high-risk infections.

RATIONALE (8) Answer: B

Option A is incorrect because, although morphine mainly


undergoes hepatic metabolism, accumulation of the metabolite
morphine-6-glucuronide may prolong the sedative effective of
morphine in renal failure. Furthermore, linear accumulation of
morphine in renal failure patients has been demonstrated.
Option B is correct. Fentanyl is metabolized by the liver to
inactive metabolites. Although an increase in sensitivity has been
ascribed to fentanyl in renal failure patients, its short half-life,
predictable kinetics, and nontoxic metabolities make it the most
appropriate choice. Option C is incorrect because, while
methadone is metabolized by the liver to inactive metabolites, the
long half-life and unpredictable kinetics of methadone make it
very undesirable for chronic pain management. These traits make
the drug more difficult to titrate for management of acute pain.
Option D is incorrect since normeperidine, the metabolite of
meperidine, can accumulate in renal failure patients, with
repeated dosing resulting in seizure, tremors, and anxiety.

RATIONALE (9) Answer: C

The osmotic demyelination syndrome refers to myelinolysis,


typically in the central pontine region, which occurs as a result of
osmotic derangements. Although the condition is typically
associated with rapid correction of hyponatremia, diabetic
ketoacidosis can also cause this syndrome as a result of osmotic
shifts generated by rapid changes in serum glucose levels. Blood
glucose should be monitored hourly in patients with diabetic
ketoacidosis and the insulin infusion should be adjusted to
ensure a fall in glucose level of 50 mg/dL/h. Controlling the rate
of glucose correction in the pediatric population is of utmost
importance because of the development of cerebral edema in this
population with overt rapid correction. The locations of these
lesions are atypical for embolic stroke or septic emboli. The
patient has a very rapid progression of symptoms and no prior
history of abnormal movements, both atypical for Huntington
disease. In the same way, the location of lesions and prior history
makes autoimmune demyelination unlikely.

RATIONALE (10) Answer: D


Local anesthetic agents such as benzocaine are the most common
drugs implicated in the etiology of acquired methemoglobinemia.
Methemoglobin is an altered state of hemoglobin in which the
ferrous (Fe2+) ions of heme are oxidized to the ferric (Fe3+)
state. The ferric hemes of methemoglobin are unable to bind
oxygen. In addition, the oxygen affinity of any remaining ferrous
hemes in the hemoglobin tetramer is increased. As a result, the
oxygen dissociation curve is “left shifted,” leading to functional
anemia and impaired oxygen delivery to the tissues. When
methemoglobin levels rise above 10%, cyanosis frequently is
present, and above this level, symptoms of severe hypoxemia
including metabolic acidosis, coma, and death can occur.
Methemoglobinemia causes disparity between Pao2 on arterial
blood gas studies and pulse oximetry readings. Oxyhemoglobin
and deoxyhemoglobin absorb light over a range of wavelengths.
Pulse oximetry uses 2 wavelengths of light; 660 nm (red region)
and 940 nm (infrared region). The ratio of the absorbance at the
two wavelengths determines the oxygen saturation. In the red
region, oxyhemoglobin absorbs less light than deoxyhemoglobin,
with the reverse in the infrared region. A ratio of absorbance
(660 nm divided by 940 nm) of 0.43 corresponds to 100%
oxygen saturation, a ratio of 1 corresponds to a saturation of
approximately 85%, and a ratio of 3.4 corresponds to 0%
saturation. When methemoglobin levels are 32%–35%, the
absorbance ratio reaches a plateau and the oximetry reading
stabilizes at 82%–85%. Clues to the diagnosis are presence of
central cyanosis with normal Pao2 on arterial blood gas analysis
and discrepancy in oxygen saturation between pulse oximetry
and arterial blood gas. Recommended treatment is IV methylene
blue, 1–2 mg/kg infused over 3–5 minutes. The presence of the
discrepancy between oxygen saturation in arterial blood gas and
pulse oximetry rules out alternate choices.
RATIONALE (11) Answer: D

Tricyclic antidepressants (TCAs) are the second most common


medication in overdose after analgesic medications and have a
high mortality rate. In overdose, the effects are most pronounced
as central nervous system (CNS) and cardiac toxicity. The
anticholinergic effects and the inhibition of the reuptake of
norepinephrine and serotonin cause CNS toxicity. CNS symptoms
include altered mental status and seizures. The anticholinergic
effects, inhibition of norepinephrine and serotonin, and
peripheral alphaadrenergenic blockade contribute to cardiac
toxicity. Cardiac toxicity can lead to sinus tachycardia with
prolonged QRS, QTc, and PR intervals, leading to ventricular
arrhythmias; cardiac toxicity is the primary cause of death. Right
bundle-branch block and hypotension can also occur. Treatment
of TCA overdose centers on supportive care. Gastric lavage and
activated charcoal can be used within 2 hours of ingestion.
Dialysis and hemoperfusion is not effective because of high lipid
solubility and protein binding. Alkalization of the urine and
serum (to a pH >7.45 and until ECG changes have resolved) is
performed with sodium bicarbonate to decrease the amount of
free drug. Lidocaine is the drug of choice for ventricular
arrhythmias, as most other drugs alter QRS, QTc, or PR intervals.
Benzodiazepines or phenobarbital are used for the treatment of
seizures, and phenytoin is only used for refractory cases.

RATIONALE (12) Answer: D

Rocuronium causes neuromuscular blockade sufficient for


intubation (80% neuromuscular blockade) in 60 to 90 seconds
and complete neuromuscular blockade usually occurs in 3
minutes. There is no histamine release associated with
rocuronium and the half-life is 20 to 35 minutes. Elimination is
both hepatic and renal with 50% uptake in the liver and
subsequent excretion in the bile. Renal elimination is less and
approximately 30% is excreted unchanged in the urine. Cirrhosis
will decrease the clearance and increase the recovery time and
half-life of rocuronium. There are reports of increased pulmonary
vascular resistance and this should be taken into consideration in
patients with pulmonary hypertension. The usual dose for
intubation is 0.6 mg/kg administered intravenously, which may
be increased to a maximum of 1.2 mg/kg if indicated. The other
agents listed require a longer time period, usually over 2 minutes,
to achieve conditions suitable for intubation. Atracurium may
cause truncal and facial flushing due to histamine release, and
transient cardiac effects including mild tachycardia and a
decrease in mean arterial pressure. Fortunately, these effects
rarely last more than 5 minutes. Vecuronium is generally well
tolerated and devoid of the adverse effects common to
atracurium. Vecuronium undergoes rapid hepatic uptake,
resulting in a shorter duration of action than pancuronium;
however, the elimination and half-life are increased in hepatic
disease. Renal disease does not interfere with elimination unless
the creatinine clearance is less than 10 mL/min. Pancuronium is
an older agent with a slower onset (3–5 minutes) and longer
duration of action (60–90 minutes). Over 80% of the dose is
excreted unchanged in the urine and renal impairment may
reduce the clearance by 35%–55 % and increase the half-life and
duration of effect.

RATIONALE (13) Answer: B

Lactic acidosis induced by linezolid is becoming increasingly


recognized. It is thought to be secondary to effects of the
antibiotic on the mithochondria and is seen more frequently in
older patients and with longer duration of treatment. This patient
has an anion gap metabolic acidosis (anion gap of 17) with
appropriate respiratory compensation. His osmolar gap is close
to normal (calculated osmolarity = [sodium × 2 + blood urea
nitrogen ÷ 2.8 + glucose ÷ 18] = 11). Our patient displays no signs
of worsening infection and has a normal white blood cell count,
so option A is wrong. Isopropyl alcohol intoxication would not
cause anion gap acidosis and would result in an osmolar gap, so
option D is wrong. The absence of nausea, vomiting, and signs of
alcohol ingestion make alcoholic ketoacidosis less likely. Thus,
option C is wrong.

RATIONALE (14) Answer: B

Neuromuscular blockers and sedatives are utilized in rapid-


sequence intubation. The use of neuromuscular blockers
improves success rates for emergency endotracheal intubation
and reduces the risk of complications. Neuromuscular blocking
agents are classified by their mechanism of action (ie,
depolarizing or nondepolarizing). Succinylcholine is a
depolarizing agent, while pancuronium, vecuronium, and
cisatracurium are nondepolarizing agents. Depolarizing agents
stimulate all cholinergic receptors by binding directly to the
postsynaptic acetylcholine receptors, causing continuous
stimulation of these receptors followed by muscular paralysis.
Nondepolarizing agents competitively inhibit the postsynaptic
acetylcholine receptors of the neuromuscular motor endplate.
Lethal hyperkalemic response to succinylcholine continues to be
reported. The depolarization of the acetylcholine receptors by
succinylcholine and its metabolites leads to potassium efflux from
the muscle, leading to hyperkalemia that can result in lethal
arrhythmias. Succinylcholine is contraindicated in patients with a
personal or family history of malignant hyperthermia and in
patients deemed to be at high risk of developing severe
hyperkalemia, such as patients with burns.

RATIONALE (15) Answer: C


Colistin is a bactericidal antibiotic that has both hydrophilic and
lipophilic moeities that bind to the lipopolysaccharides and
phospholipids on the outer cell membrane of gram-negative
bacteria, acting like a detergent. Colistin is not active against
gram-positive organisms or gram-negative cocci. Common gram-
negative bacteria that are intrinsically resistant to colistin are
Proteus, Providencia, Serratia, Neisseria, Moraxella,
Burkholderia, and some strains of Stenotrophomonas. Other
bacteria that have variable resistance against colistin include
Aeromonas, Vibrio, Prevotella, and Fusobacterium. Colistin is
used primarily for multidrug-resistant Pseudomonas, Klebsiella,
and Acinetobacter. Colistin is most commonly administered
intravenously as it is not absorbed in the stomach and is
eliminated by the kidneys. It can also be administered via
inhalation, but can cause bronchospasm in this route, so often
pretreatment with bronchodilators is performed. The most
common side effect is nephrotoxicity, which can occur in up to
40% of patients and is more likely in patients with existing
kidney dysfunction.

RATIONALE (16) Answer: B

Propofol-related infusion syndrome (PRIS) is a serious side effect


with a high mortality rate. It is characterized by dysrhythmia (eg,
bradycardia or tachycardia), heart failure, hyperkalemia, lipemia,
metabolic acidosis, and/or rhabdomyolysis or myoglobinuria
with subsequent renal failure. Risk factors include poor oxygen
delivery, sepsis, serious cerebral injury, and the administration of
high doses of propofol (usually doses >83 µg/kg/min or >5
mg/kg/h for >48 hours), but the syndrome has also been
reported following large-dose, short-term infusions during
surgical anesthesia. The onset of the syndrome is rapid, occurring
within 4 days of initiation. The mechanism of the syndrome has
yet to be determined. Initial management includes prompt
discontinuation of propofol infusion. Propofol accumulates in
tissue, so potential for prolonged and/or refractory toxicity
exists. Prolonged action has been noted following extended
midazolam infusion, possibly because of metabolite
accumulation, but there is no evidence of severe toxicity as with
propofol. Therefore, option A is incorrect. Linezolid is often
associated with serotonin syndrome in the presence of
monoamine oxidase inhibitors. It is unlikely in the current
scenario, so option C is incorrect. Liberal packed red blood cell
transfusion practice in the ICU leads to increased length of stay
and mortality. Hemoglobin levels as low as 7 g/dL should be well
tolerated, so option D is incorrect. Finally, there is no evidence of
fungal infection, so adding an antifungal agent is not indicated
and option E is also incorrect.

RATIONALE (17) Answer: D

Dexmedetomidine is a sedative, analgesic, and anxiolytic


medication that results in highly selective agonism of central
alpha2-adrenergic receptors. It has been used in the ICU for
sedation, but is also approved by the FDA for use as a procedural
sedative since it does not cause respiratory depression. Since it
has both sedative and analgesic properties, the dosage should be
decreased when started in presence of other sedatives and
opiods. Dexmedetomidine has been shown to cause hypertension
when given as a bolus because of peripheral stimulation of
alpha2-adrenergic receptors. The dose of the drug does not need
to be adjusted for renal failure since it is metabolized mostly via
glucuronidation to inactive metabolites. The diluent does not
result in an osmolal gap, as with lorazepam, regardless of the
dose administered.
PART 9: Surgical and Obstetrical Critical Care

Instructions: For each question, select the most correct answer.

1. A 54-year-old, white man weighing 84 kg (187 lbs) receives a


double lung transplant for idiopathic pulmonary hypertension
and is admitted to the ICU immediately afterward. His donor, a
47-year-old, African American woman, died of complications of
traumatic brain injury from a motor vehicle accident. She had
remained intubated for 5 days before her death and her ICU stay
had been complicated by suspected aspiration.

The transplant recipient requires increasing oxygen support as


his pulse oximetry is persistently dropping despite increasing
FIO2. He subsequently needs his positive end-expiratory
pressure (PEEP) to be increased to 10 cm H2O to keep
saturations above 90%. On day 2 after surgery, he is on a volume-
controlled mode with tidal volume set to 500 mL, RR of 18/min,
FIO2 of 1.0, and PEEP of 10 cm H2O. His morning arterial blood
gas results reveal a pH of 7.34 and Pao2 of 123 mm Hg. A portable
chest radiography confirms his auscultatory findings of bilateral
diffuse, coarse crackles, as it demonstrates bilateral 4-quadrant
airspace processes with no evidence of significant effusions.

Which of the following is the most likely explanation of his


hypoxemic respiratory failure?

A. Acute bilateral pneumonia since the donor was treated for an


aspiration process

B. Worsening heart failure considering his borderline right


ventricular dysfunction due to pretransplant pulmonary
hypertension
C. Primary graft dysfunction

D. Transfusion-related acute lung injury

2. IV morphine, 4 mg every 3 hours, fails to relieve the leg pain of


a 20-year-old man who had a gunshot wound to the leg with
femoral artery and vein injury. He had arterial repair and venous
ligation. In spite of elevation of his leg, there is swelling from the
thigh to the foot. Passive movement of his foot causes profound
calf pain. He has palpable pulses at the posterior tibial and dorsal
interosseous muscles, and his ankle brachial index (ankle blood
pressure divided by the brachial blood pressure) is normal.

Which of the following steps is most appropriate?

A. Increase the amount of pain medicine.

B. Obtain an arteriography.

C. Perform fasciotomy.

D. Place an epidural catheter for pain control.

E. Anticoagulate the patient.

3. A 25 year-old man is admitted to the ICU for observation


following a collision of his motorcycle with a tree. His Glasgow
Coma Scale score is 10. His neurologic workup is in progress.

Anticonvulsants to prevent the occurrence of early posttraumatic


seizures are contraindicated in which of the following clinical
situations?

A. Depressed skull fracture


B. Subdural hematoma

C. Subarachnoid hemorrhage

D. Penetrating brain injury

E. Cortical contusion

4. A 48-year-old man is admitted to the ICU for observation after


he falls off a porch, striking the back of his head on the hard
ground. His blood alcohol level on arrival is 0.24%. He is drowsy
but arousable and reports paresthesias in all 4 limbs. His neck is
stabilized with a cervical collar until he is more awake and
cervical spine injury can be ruled out. On physical examination,
he has 1/5 bilateral grip and wrist extension strength; 3/5 biceps,
triceps, and deltoid strength; 4/5 bilateral plantar flexion and
dorsiflexion strength; and 5/5 quadriceps, hamstring, and
iliopsoas strength.

The most likely diagnosis is:

A. Multilevel spondylosis and central cord syndrome

B. C5-C6 fracture/dislocation

C. Alcoholic polyneuropathy

D. Spinal epidural hematoma

5. A 30-year-old, female restrained driver is admitted to the ICU


following a motor vehicle collision in which she was struck
broadside by another vehicle on the driver’s side. She has a
Glasgow Coma Scale score of 9-I, opening her eyes to voice,
localizing on the right side, and having been intubated. She has a
left hemiparesis; left pneumothorax; multiple rib fractures; left
femur fracture; dislocated left shoulder; and multiple bruises and
ecchymoses over her left neck, shoulder, and trunk. She has
anisocoria with the right pupil 5 mm and reactive, and the left 3
mm and reactive with left ptosis.

Pending the results of the secondary survey, the most likely


diagnosis for the ocular findings is:

A. Traumatic right third nerve palsy

B. Carotid artery dissection in the neck

C. Brown-Sequard syndrome

D. Brachial plexus and lumbosacral plexus injuries

E. Incidental right-sided brain tumor

6. A 42-year-old woman is admitted with intracerebral and


intraventricular hemorrhage and initially treated with an
endoventricular drain. Eleven days later, she develops fever,
lethargy, and leukocytosis. In cerebrospinal fluid, glucose levels
are decreased and protein levels are increased.

The most appropriate initial antimicrobial coverage in this


patient is:

A. Daptomycin and metronidazole

B. Levofloxacin and clindamycin

C. Ertapenem and linezolid


D. Cefepime and vancomycin

E. Gentamicin and ampicillin

7. A 57-year-old man who had a double lung transplant for


emphysema 7 months ago, with cytomegalovirus (CMV)
mismatch, is transferred to the ICU from the emergency
department with a 2-day history of generalized headache. While
in the emergency department, he experienced a 4-minute
generalized seizure, which partially resolved with IV
benzodiazepines; shortly afterward, he experienced a second
seizure and was intubated for airway protection. Upon arrival to
the emergency department, systolic BP was 209 mm Hg. Capillary
blood glucose level was 154 mg/dL. The rest of his vital signs and
physical examination findings were reported as unremarkable.
Preadmission medications include prednisone, cyclosporine,
azathioprine, voriconazole, valganciclovir,
trimethoprim/sulfamethoxazole, metoprolol, furosemide, and
pravastatin. CT of the head reveals symmetrical white matter
edema in the posterior cerebral hemispheres. In addition to
appropriate blood pressure control with an IV antihypertensive
agent with close monitoring and titration, the plan of care over
the following 24 hours should include:

A. Initiation of broad-spectrum antibiotics directed toward


infectious meningitis

B. Thrombolytic therapy for suspected bilateral ischemic


posterior strokes

C. Discontinuation of cyclosporine and initiation of an alternative


calcineurin inhibitor for immunosupression
D. IV loading of antiepileptic for planned long-term therapy with
phenytoin

8. A 34-year-old woman has orthotopic liver transplant followed


by improving liver function. Four days later, she develops
abdominal pain and nausea. She is alert but tachypneic and
diaphoretic. HR is 118/min, BP is 88/51 mm Hg, and temperature
is 38.3°C (101°F). She has tenderness in the right upper quadrant
and guarding on abdominal examination. Abdominal drain
contents appear yellow-green as opposed to serosanguinous
earlier. Total bilirubin level is 4.8 mg/dL (elevated from 1.1
mg/dL 24 hours ago); alanine aminotransferase (ALT) and
aspartate aminotransferase (AST) levels are 50% higher than 24
hours ago. International normalized ratio (INR) is 2.4, up from
1.3 when measured 24 hours ago. The patient is receiving
tacrolimus and mycophenolate mofetil for immune suppression.

Which of the following is the most likely etiology of this


presentation?

A. Toxicity from immune suppressive therapy

B. Hepatic artery thrombosis

C. Primary graft dysfunction

D. Acute cellular rejection

9. A 48-year-old man is diagnosed with severe acute cellular


rejection following liver transplantation. He is treated with two 6-
day courses of high-dose corticosteroids followed by tapering
doses of corticosteroids and an increased dosage of tacrolimus.
There is little improvement in liver function or clinical status,
which is marked by jaundice, elevated bilirubin level, and
elevated international normalized ratio. A repeat biopsy shows
persistent rejection. He is started on antithymocyte globulin.
Eight hours later he has confusion, dyspnea, and wheezing. HR is
120/min, BP is 98/60 mm Hg, RR is 28/min, and temperature is
38.9°C (102°F). He appears anxious, using accessory muscles of
breathing. He is mildly confused but has no focal neurologic
deficit or nuchal rigidity. Bibasilar rales and prolonged expiration
are present on chest auscultation.

10. Which of the following causes for the patient’s acute change in
status is most likely?

A. Cytokine release and complement activation

B. Anaphylactic shock

C. Calcinuerin inhibition–mediated toxicity

D. Severe sepsis

10. Which of the following arrhythmias in the early postoperative


phase of cardiac transplantation is associated with allograft
rejection?

A. Ventricular premature contractions

B. Atrial flutter

C. Sinus tachycardia

D. New right bundle-branch block

11. A 40-year-old man is shot in the abdomen at point-blank


range. He arrives at the local trauma center in extremis with a
systolic BP of 60 mm Hg by palpation. Ten units of packed red
blood cells (PRBCs) are immediately available and transfusion is
undertaken. In the operating room, he is noted to have injuries to
his aorta, superior mesenteric artery, left kidney, and colon.

What additional blood component administration, in tandem


with surgical control, is appropriate to best achieve adequate
hemostasis?

A. Transfusion of fresh frozen plasma (FFP) based on


prothrombin time or international normalized ratio

B. Maintained 1:1:1 ratio of PRBCs to FFP to platelets throughout


resuscitation

C. Single-dose factor VIIa

D. FFP and platelets to be transfused only once surgical


hemostasis is achieved

12. A 58-year-old man had partial gastrectomy 20 days ago for a


perforated antral gastric ulcer. His postoperative course was
complicated by anastomotic leak requiring re-exploration, ileus
requiring total parenteral nutrition, and acute nonoliguric renal
failure. He is currently treated with piperacillin/tazobactam and
fluconazole but still has fever and leukocytosis. The patient’s
most recent blood culture is growing Candida species which
appear to be nonalbicans. His central venous catheter is removed.

Which of the following interventions is indicated?

A. Discontinue fluconazole and start amphotericin B.

B. Continue fluconazole.
C. Discontinue fluconazole and start voriconazole.

D. Discontinue fluconazole and start polymyxin B.

E. Discontinue fluconazole and start caspofungin.

13. A 20-year-old man is admitted to the ICU after sustaining


open comminuted pelvic fracture during a motorcycle accident.
His initial pelvic radiograph and CT with IV contrast only are
shown in the Figure.

No other thoracoabdominal injuries are noted on CT. An external


fixator is placed. His initial blood pressure upon arrival is normal
but his pulse rate remains 130/min. Over the course of several
hours, he has required several fluid boluses for hypotension,
which temporarily responds to the fluid. His hematocrit remains
at 28% despite 4 units of packed red blood cells. His chest
examination is normal to auscultation and his abdomen is slightly
distended without guarding. Which of the following is the most
appropriate next step in his management?

A. Pelvic arteriogram and embolization

B. Emergent laparotomy

C. Transesophageal echocardiography

D. Continued supportive care

14. A 19-year-old woman is admitted to the ICU with a history of


having been kicked in the upper abdomen by a horse. Her Spo2
declines from 95% to 88% on 50% mist mask. Her pulse rate is
110/min and her BP is 105/68 mm Hg. Her hematocrit is 32%.
Her radiograph is shown in the Figure. She is intbuated for
airway protection.
Which of the following is the most appropriate next step in her
care?

A. Emergent bronchoscopy and lavage

B. Emergent laparotomy

C. CT of chest, abdomen, and pelvis


D. Chest tube placement

15. An 82-year-old woman with hypertension and diabetes


mellitus is admitted to the hospital with traumatic right subdural
hematoma, rib fractures, and second and third metatarsal
fracture of her right foot. On hospital day 3, she develops
worsening oxygenation and tachycardia. Her temperature is
38.1°C (100.5°F). Her arterial blood gas results on oxygen, 3
L/min via nasal cannula are pH, 7.42; Paco2 of 38 mm Hg, and
Pao2 of 74 mm Hg, with Spo2 of 94%. Her WBC count is
13,000/µL with normal differential. Her chest radiograph is
unremarkable and her chest angiogram is shown in the Figure.
Which of the following is the most appropriate next step in her
care?

A. Begin antibiotics for aspiration pneumonitis.

B. Increase positive end-expiratory pressure and minimize excess


fluid.

C. Fully anticoagulate patient.

D. Place inferior vena cava filter.

16. A 25-year-old male is evaluated at the regional trauma center


after a high-speed car crash. He is initially hypotensive but
rapidly stabilizes after several liters of saline. CT reveals multiple
bilateral rib fractures, a right hemopneumothorax, and a grade 4
central liver laceration with moderate perihepatic fluid but no
active arterial extravasation. A right chest tube is placed and the
patient is admitted for observation. Progressive jaundice is noted
during the next several days. Three days after admission, the
patient vomits a large amount of blood with a resultant 4 g/dL
drop in his hemoglobin level.

After appropriate resuscitative measures, which of the following


is the best course of management?

A. Immediate exploratory laparotomy for control of presumed


liver bleeding

B. Exploratory thoracotomy for control of pulmonary


hemorrhage
C. Diagnostic angiography for evaluation and embolization of
hepatic arterial-biliary fistula

D. Upper GI endoscopy for diagnosis and control of peptic ulcer


bleeding

17. An 18-year-old woman presents to the emergency


department after being assaulted with a baseball bat to her head
and neck. CT reveals minimal subarachnoid hemorrhage along
with multiple complex mid-face fractures. CT angiography
reveals a focal right carotid dissection at the level of the skull
base with moderate luminal irregularity, but preserved distal
flow. She is systemically stable without evidence of neurologic
compromise.

Which of the following is the most appropriate treatment of the


carotid dissection?

A. Direct surgical repair

B. Systemic anticoagulation

C. Angiography with stenting

D. Close observation

18. A 65-year-old man with chronic obstructive pulmonary


disorder and insulin-dependent diabetes has just been extubated
on postoperative day 4, following an emergency operation for an
obstructing left-sided colon cancer. He is receiving nasal oxygen
and bronchodilator therapy. Following an episode of coughing,
his midline wound dressing is saturated with a large amount of
salmon-colored fluid.
Which of the following is the most appropriate intervention?

A. Initiation of antibiotics for a wound infection

B. Observation for any further drainage

C. CT of the abdomen

D. Returning to the operating room

E. Open skin incision at the bedside

Both questions 19 and 20 use the following information:

A 68-year-old woman is brought to the emergency department in


shock after reporting abdominal pain. A ruptured aortic
aneurysm is diagnosed, and the patient is taken for emergent
surgical repair. After a prolonged operation requiring massive
amounts of crystalloids and blood products, the aneurysm is
repaired. The patient is transferred to the ICU, intubated, and
receiving vasopressor support. On arrival, her temperature is
36.6ºC (97.8ºF), BP is 110/58 mm Hg, HR is 92/min, and RR is
20/min, with conventional mechanical ventilation 14, tidal
volume of 450 mL, positive end-expiratory pressure (PEEP) of +5
cm H2O, and FIO2 of 100%. Urine output is 75 mL/h, and the
patient continues to receive 0.9% normal saline solution at 100
mL/h. Laboratory data include the following: hemoglobin, 9.4
g/dL; normal platelet count; normal electrolytes; blood urea
nitrogen, 25 mg/dL; and creatinine, 1.6 mg/dL. Ten hours after
arrival in the ICU, the BP drops to 80/36 mm Hg, and the central
venous pressure is 18 mm Hg. Urine output has decreased to less
than 50 mL/h over the last 2 hours, despite increased IV fluids.
Peak airway pressure increases from 20 cm H2O to 56 cm H2O,
and the patient develops hypoxemia that does not respond to an
increase in FIO2 to 100% and PEEP of 12 cm H2O. Hemoglobin
obtained from an arterial blood gas measurement is 9.2 g/dL.

19. Which of the following diagnostic tests should be done first?

A. Transesophageal echocardiography

B. CT of abdomen and pelvis

C. Measurement of bladder pressure

D. Abdominal ultrasonography

E. Placement of pulmonary artery catheter

20. Following the test above, the patient continues to deteriorate.


Which of the following interventions should be done next?

A. Aggressive resuscitation with packed red blood cells, fresh


frozen plasma, and crystalloids

B. Emergent dialysis

C. Pericardiocentesis at the bedside

D. Emergent laparotomy

E. Change of ventilator to pressure-control mode

21. A 32-year-old woman, gravida 3, para 3, is admitted to the


obstetrics and gynecology department following spontaneous
vaginal delivery under epidural anesthesia. The next morning she
reports severe back pain, urinary incontinence, and lower
extremity weakness.

Which of the following immediate steps is most appropriate?

A. Radiography of the spine

B. MRI of the thoracic and lumbar spine

C. CT of the head

D. Electromyography

E. Ultrasonography of the lower extremity

22. A 93-year-old woman with a history of chronic obstructive


pulmonary disease, atrial fibrillation, and malnutrition is
admitted to the surgical ICU after a fall down a flight of stairs. Her
workup is significant for an epidural bleed, small right pulmonary
contusions, and multiple right-sided rib fractures (ribs 5–8 and
11).

Which of the following is the optimal pain management for this


patient?

A. Lidocaine patch to the affected area

B. Thoracic epidural anesthesia

C. Intercostal blocks

D. Paravertebral blocks

E. Morphine patient-controlled analgesia


23. A 35-year-old, African American woman in her third
pregnancy is admitted after spontaneous rupture of membranes
at 37 weeks’ gestation with twins. She has had no serious
concerns during this pregnancy except increasing fatigue and
pedal edema during the past few weeks. She believes her
symptoms have been primarily due to her excessive weight gain
during the pregnancy. She has an uncomplicated vaginal delivery.
Approximately 30 minutes after delivery, she reports shortness
of breath. On examination, BP is 150/80 mm Hg and RR is 35/min
with accessory muscle use. Chest examination reveals an S3
gallop, and bilateral inspiratory rales are auscultated in both lung
fields.

Which of the following is appropriate in the initial management


of this patient?

A. Diuresis with furosemide

B. Blood pressure control with labetalol

C. Inotropic support with dobutamine

D. Afterload reduction with hydralazine

24. A 32-year-old woman is brought to the hospital for evaluation


of nausea, vomiting, headache, and confusion during the past 5
days. She recently delivered a healthy baby, and her pregnancy
was characterized by hypertension and mild proteinuria. On
admission, her BP is 212/128 mm Hg, HR is 102/min, RR is 18–
22/min, temperature is 37.7°C (99.8°F), and oxygen saturation is
98% on 2 L of oxygen via nasal cannula. On neurological
examination, her Glasgow Coma Scale score is 14, and she is
drowsy but arousable, oriented to self but not to place and date,
and follows simple commands. Cranial nerve examination
findings are normal, but fundoscopy suggests papilledema. Motor
findings are normal. Reflexes are 3+ throughout, toes are
downgoing bilaterally, and no meningeal signs are noted. MRI in
the emergency department at the request of the consulting
neurologist is shown in the Figure.

T1-weighted MRI with gadolinium


Diffusion-weighted MRI

The most appropriate intervention for this patient is to start:

A. IV nicardipine to lower mean arterial pressure 15%

B. IV phenytoin

C. IV methylprednisolone

D. IV acyclovir

E. Hyperosmolar therapy with 3% saline solution


25. A 23-year-old woman is seen in the emergency department
for fatigue and mild dyspnea on exertion and 2 days of low-grade
fever. She had mild intermittent asthma during adolescence but
has not had recent exacerbations and is taking no medications.
The emergency department physician discharges her with a
diagnosis of upper respiratory tract infection. He prescribes
albuterol by metered dose inhaler as needed and recommends
follow-up with her obstetrician in a few days. Two days after
discharge, she returns with a temperature of 39°C (102.2°F),
confusion, nausea and vomiting, and dyspnea. BP is 138/92mm
Hg, HR is 112/min, RR is 25/min , and oxygen saturation is 91%.
Arterial blood gas results show pH of 7.24, Pao2 of 62 mm Hg,
Paco2 of 59 mm Hg, and oxygen saturation of 89%. Chest
radiograph reveals bilateral, scattered pulmonary infiltrates.

Which of the following interventions is most appropriate?

A. Aggressive diuresis and inotropic support

B. Broad-spectrum antibiotics

C. Noninvasive ventilation

D. Mechanical ventilation

E. Additional bronchodilator therapy


PART 9: Surgical and Obstetrical Critical Care

ANSWERS:

1–C; 2–C; 3–C; 4–A; 5–B; 6–D; 7–C; 8–B; 9–A; 10–B; 11–B; 12–B;
13–A; 14–B; 15–D; 16–C; 17–B; 18–D; 19–C; 20–D; 21–B; 22–B;
23–A; 24–A; 25–D

RATIONALE (1) Answer: C

Primary graft dysfunction is a severe form of ischemia


reperfusion injury and is the major cause of early morbidity and
mortality after lung transplantation. Recipient, donor, and
operative variables have been identified as risk factors. In this
particular patient, the donor was female, African American, older
than 45 years, and had a prolonged mechanical ventilation, all of
which are well-described donor characteristics related to
primary graft dysfunction. This patient’s Pao2 to FIO2 ratio and
radiological findings classify him as grade 3, which is associated
with the highest mortality. Treatment is mostly supportive
although large centers’ experience is being published, including
early consideration of extracorporeal membrane oxygenation.
Infectious processes from a donor that lead to hypoxemic
respiratory failure in organs that were macroscopically secretion-
free and radiologically normal prior to transplantation are
unlikely to occur within the first 5 days of transplantation.
Although heart failure and fluid mismanagement must be
considered as a possible cause of airspace processes after
transplantation, such a severe hypoxemia from heart failure
without significant pleural effusions is unlikely. There is no
mention of massive transfusion needs in this patient.

RATIONALE (2) Answer: C


This patient has compartment syndrome that occurs in the
extremity from increased pressure in a closed fascial muscle
compartment. The increased pressure causes small outflow veins
to collapse associated with increased swelling, arterial
thrombosis, and nerve and muscle death. Venous hypertension
from femoral vein ligation has caused this patient’s compartment
syndrome. Elevation of the involved extremity eliminates the
need for fasciotomy in 80% of patients but has failed in this case.
Bleeding into the compartment, crush injury, infection, edema,
snakebites, and ischemia reperfusion injury are other causes of
compartment syndrome. In the awake patient, pain is the most
frequent symptom (ie, pain out of proportion to the injury). Pain
serves as warning sign that the leg needs to be examined for
other signs of compartment syndrome. Both active and passive
muscle activity increase pain. Paresthesias (web space between
the first and second toe in lower extremity) are often rapidly
followed by paralysis. The presence or absence of the pulse does
not correlate with compartment syndrome. Compartment
pressures can be measured by manometery or commercial
devices. Compartment pressures greater than 30 mm Hg
(exceeding capillary perfusion pressure) must be addressed.
Elevation of the extremity is the first treatment, followed quickly
by fasciotomy. Early fasciotomy to decompress the compartment
will prevent neuromuscular deficits. Adequate pain control is
important but does not take the place of recognizing
compartment syndrome. The ankle-brachial index (ABI) is
obtained by measuring the blood pressure in the posterior tibal
artery and the brachial artery and is calculated using ankle blood
pressure divided by the brachial blood pressure. A normal ABI is
0.9 to 1.0. The further the ABI is from 1.0, the more arterial
disease is present. Because a pulse with normal ABI is present,
there is no indication for an arteriogram. If an arterogram is
obtained, the main arteries will be intact, and the muscle
branches will taper. Anticoagulation does not treat compartment
syndrome.

RATIONALE (3) Answer: C

Posttraumatic seizures are classified as early (within 7 days of


trauma) or late (more than 7 days after trauma). While
prevention of seizure activity is desirable, anticonvulsants are
associated with neurobehavioral and other medical side effects.
Therefore, long-term prophylaxis is not recommended. Phenytoin
has been shown to be effective for the prevention of early
posttraumatic seizures. Valproate has a similar effect but may be
associated with higher mortality rates.

RATIONALE (4) Answer: A

Central cord syndrome is characterized by weakness that is


worse distally than proximally and worse in the upper
extremities than in the lower extremities. Fracture/dislocation at
C5-C6 would more likely result in a deficit below that level, so
option B is unlikely. Alcoholic polyneuropathy is primarily
sensory, so option C is incorrect. Spinal epidural hematoma
would also likely result in a deficit below the level of
compression; thus, option D is incorrect. Chronic cervical stenosis
secondary to degenerative joint disease can result in central
spinal cord injury in the presence of an acute flexion or
hyperextension injury.

RATIONALE (5) Answer: B

The patient has significant left hemi-body trauma after being


struck on that side. External signs of trauma on the left neck
suggest potential for carotid artery injury. The abnormal pupil is
on the left , since it is associated with ptosis; therefore, right third
nerve injury (option A) is not the diagnosis. Carotid dissections
are commonly associated with Horner syndrome on the affected
side (miosis, ptosis, and anhidrosis). While Brown-Sequard
syndrome results in a hemiparesis, the side contralateral to the
motor deficit would have a sensory deficit, so it is less likely to be
associated with localization to pain on the right. Therefore,
option C is incorrect. Simultaneous brachial and lumbosacral
plexus injuries would be unusual, and while brachial plexus
injuries may be associated with a Horner syndrome, this
constellation would be very unlikely. Thus, option D is incorrect.
Right-sided brain tumor would not be expected to cause an acute
deficit in the face of trauma and would not be associated with a
left ptosis, so option E is incorrect.

RATIONALE (6) Answer: D

The patient has nosocomial meningitis related to the


endoventricular drain. The rate of infection associated with
external catheters is approximately 8%. The risk of infection is
reported to be increased with an increased duration of drainage,
but the extent of increase per unit of time is uncertain. Although
one study showed a sharp increase in the risk of infection after 5
days of external drainage, a prospective, randomized trial
showed that removing external catheters within 5 days is
unnecessary and that catheters can be left in place for longer
periods. Since infection may be acquired by the introduction of
bacteria after the insertion of a new catheter, changing
uninfected catheters might actually increase the risk of infection.
Other risk factors for infection are the routine sampling of
cerebrospinal fluid, leakage of cerebrospinal fluid at the site,
blockage of the drain, and intraventricular hemorrhage. Therapy
for patients in whom meningitis develops after neurosurgery or
for patients who are hospitalized for a prolonged period after
penetrating head trauma or basilar skull fracture should cover
staphylococci and facultative or aerobic gram-negative bacilli.
The regimen should consist of vancomycin in combination with
cefepime, ceftazidime, or meropenem; the choice of the second
agent should be based on the antimicrobial susceptibility profiles
of the local gram-negative bacilli. Meropenem is the agent of
choice if one of the carbapenems is used. Ertapenem has no
activity against Pseudomonas aeruginosa. Imipenem can lower
seizure threshold.

RATIONALE (7) Answer: C

Posterior reversible encephalopathy syndrome (PRES) is a


clinicoradiological entity known to complicate
immunosuppressive therapy, specifically related to calcineurin
inhibitors. Electrolyte disturbances and hypertension have been
implicated in facilitating the development of this neurological
complication. Recognition of this potential side effect is critical to
intensive care physicians as the therapy is often limited to
discontinuation of the offending agent and replacement with an
alternative option. Although immunosupressed, this patient had
no other clinical signs concerning for an infectious process.
Although ischemic events may present with seizures,
thrombolytic therapy wouldn’t be considered in this patient with
labile blood pressure control and no focal abnormalities after his
seizure.

RATIONALE (8) Answer: B

Early postoperative (<1 week) complications of liver


transplantation may be related to technical or anatomical causes
(vascular and hepatobiliary tree), immune suppressive therapy,
graft rejection, graft nonfunction or extrahepatic organ
dysfunction. Vascular complications include hepatic artery
thrombosis (HAT), portal vein thrombosis, and hepatic
congestion secondary to venous outflow obstruction. HAT
remains the most common vascular complication and is
manifested by elevated transaminase levels, rising bilirubin
levels, sepsis with liver abscesses, biliary anastomosis disruption,
and acute hepatic failure. Doppler ultrasonography or
arteriography may be required to diagnose HAT and differentiate
it from other less common vascular problems.

Primary bile duct leaks may present as excessive bilious drainage


from abdominal drains and disproportionate rise in bilirubin
level compared to other liver function tests. Acute cellular
rejection may occur in the first 2 weeks and can be associated
with lack of induction immunosuppressive therapy. Clinical
features range from mild nonspecific symptoms and subtle liver
function abnormalities to acute liver failure. Liver biopsy is
necessary to establish diagnosis, especially if other major
complications are ruled out. Primary graft dysfunction or
nonfunction (PGNF) presents in the immediate postoperative
period (usually up to 48 hours) with lack of improving liver
function characterized by depressed mental status, coagulopathy,
acidosis, hypoglycemia, and hypothermia, in addition to
worsening liver function test results. In this case, the stable early
postsurgical course and normal mental status argue against
PGNF, which is treated by supportive care and may require
retransplantation. Infection must be ruled out as signs of sepsis
are indistinguishable from the above mentioned complications.

RATIONALE (9) Answer: A

Acute cellular rejection in liver transplantation is treated with


high-dose corticosteroids with a good response rate. In 10% of
cases or less, rejection does not respond to steroids, necessitating
the use of agents targeting T lymphocyte function, proliferation,
or activation. These rescue medications include muromonab-CD3,
thymoglobulin (antithymocye globulin [ATG]), mycophenolate
mofetil, antiinterleukin-2 receptor antibodies (eg, basiliximab),
calcineurin inhibitors (eg, tacrolimus), and agents that inhibit the
mammalian target of rapamycin (mTOR) (eg, sirolimus). ATG is
an infusion of anti–T cell antibodies derived from horse or rabbit
sera. ATG is used in the prevention and treatment of cellular
rejection in renal and hepatic transplantation as well as for
aplastic anemia. It has been used in steroid-resistant cases of
orthotopic liver transplantation among other agents. Agents such
as muromonab and ATG can induce a syndrome of cytokine
release upon administration. Cytokine spill from T cells (tumor
necrosis factor, interleukin-2, and interferon gamma) causes the
cytokine release syndrome characterized by fever, chest pain,
gastrointestinal symptoms, wheezing, and flu-like features.
Complement activation and neutrophil sequestration can cause
acute lung injury and pulmonary edema. Hypotension may occur.
Usually seen after the first dose, the syndrome can be treated and
prevented by steroids and H1-receptor blockers. The severity of
symptoms decreases with subsequent doses. The timing of
medication use in this case does not warrant consideration of
severe sepsis in this patient. Calcinuerin inhibitors may cause
nephrotoxicity and neurotoxicity in the acute postinduction
phase. Hypertension, renal insufficiency, and seizures are notable
features.

RATIONALE (10) Answer: B

Arrhythmias are common in the early postoperative phase


following cardiac transplantation. Ventricular premature beats
are present in almost all patients early on with decreasing
incidence later in the postoperative course. Sustained ventricular
tachycardia or ventricular fibrillation can be associated with
allograft rejection.
Similarly persistent or paroxysmal atrial tachyarrhythmias (atrial
flutter/fibrillation) may be harbingers of allograft rejection in the
early postsurgical period. Atrial tachyarrhythmias may also be
associated with higher long-term risk of mortality in these
patients. In the denervated heart, loss of negative chronotropic
vagal input leads to a normal sinus resting rate greater than
80/min. The most common conduction disturbance in the
transplanted heart is a new partial or complete right bundle-
branch block seen in almost 70% of patients. Only persistent
conduction disturbances may be associated with rejection.

RATIONALE (11) Answer: B

Standard resuscitative strategies in the setting of trauma have


historically revolved around initial crystalloid resuscitation, the
institution of packed red blood cells (PRBCs) with severe
bleeding, and the correction of coagulopathy based upon empiric
clotting indices. Recent studies derived from the military
experience in Iraq and Afghanistan have demonstrated an overt
coagulation defect in severely injured patients upon presentation,
prior to the dilutional effect previously used to explain such
pathology. Given the direct interface coagulopathy has on
ongoing hemorrhage, and its contribution to the further
propagation of hypothermia and acidosis (all leading to the
“bloody vicious cycle”), the direct control of coagulopathy
beginning at the time of trauma has been studied as a means to
better ameliorate the effects of severe hemorrhagic shock. This
methodology, along with goal-directed treatment of hypothermia
and acidosis, has been termed damage control resuscitation
(DCR). Such a protocol is typically performed in tandem with
damage control laparotomy, where severe hemorrhage and
contamination is quickly controlled in the operating room, with
definitive organ or vascular repair reserved till the patient has
been systemically normalized. Although the protocol has not
been universally implemented as yet, multiple studies have
shown a marked improvement in survival with the early
transfusion of fresh frozen plasma (FFP) in an approximate 1:1
FFP to PRBC ratio (in the setting of severe hemorrhagic shock,
>10 units of PRBCs transfused). Operative blood loss has been
shown to be significantly decreased using DCR versus
conventional resuscitation schemes, with additional benefits
including less hypothermia and acidosis immediately after
surgery. Late effects including cardiac dysfunction, ventilator
dependence, acute respiratory distress syndrome, and secondary
abdominal compartment syndrome have all been tempered,
presumably because of the decreased requirement for mass
crystalloid transfusion with early hemorrhage control. Additional
work has also shown a role for the early administration of
platelets, transfused in a 1:1 or 1:2 ratio relative to PRBCs. A
number of civilian trauma centers have also demonstrated
improved outcomes implementing a DCR model, and have
developed mass transfusion protocols to immediately mobilize
the appropriate products when a severely injured patient arrives.
Factor VIIa has been extensively studied as an ancillary approach
to treating severe coagulopathy, but should not be used in
isolation without standard or DCR strategies.

RATIONALE (12) Answer: B

Although fluconazole remains one of the most commonly used


antifungal agents for the treatment of Candida infections, there is
an increasing emergence of non-albicans, fluconazole-resistant
Candida species as causes of serious fungal infections.
Candidemia is associated with mortality of 30%–40%, increased
length of hospital stay, and increased days of mechanical
ventilation. In this patient, the organism is a non-albicans
Candida, which has a high incidence of fluconazole resistance.
Another drug in the azole family is voriconazole. Cross-resistance
between fluconazole and voriconazole is clearly more
pronounced in some species of Candida than others, although all
are affected to some degree. Innate (C krusei) or emerging
(especially C glabrata and C guilliermondii) resistance to azoles
among non-albicans Candida species is an increasing problem as
non-albicans species in some series represent close to half of all
Candida infections. ARTEMIS, a global antifungal surveillance
study, found, for example, that voriconazole was active against
some fluconazole-resistant isolates of Candida (C krusei, C
inconspicua, C norvegensis, and C intermedia), but activity was
quite poor against the remaining 20 species of fluconazole-
resistant Candida (C glabrata, C tropicalis, and others). For
decades, amphotericin B deoxycholate has been the standard
therapy for invasive fungal infections. Unfortunately, it is often
poorly tolerated and associated with acute infusion-related
reactions and nephrotoxicity, which makes it less than an ideal
choice in this patient.
Polymyxin B has gram-negative, but not antifungal activity.
Echinocandins are a newer class of parenteral antifungal agents
that inhibit the synthesis of beta-(1,3)D-glucan in the fungal cell
wall. These compounds are fungicidal in vitro against C albicans
and non-albicans Candida species. No cross-resistance with
azoles has yet been reported.

RATIONALE (13) Answer: A

The patient continues to be a transient responder, suggesting that


there is ongoing blood loss. There is initial extravasation seen in
the retroperitoneum on initial CT.
Literature suggests that arterial extravasation seen on initial CT
of pelvis predicts a need for arterial embolization to achieve
hemodynamic stability. Pelvic hemorrhage appears best managed
by initial stabilization of the pelvic bones with realignment of the
fracture, followed by pelvic angiography and possible
embolization if continued hypotension occurs. Indications for
laparotomy are the same as for trauma patients without pelvic
fractures, namely signs of peritoneal irritation consistent with
intestinal perforation or ongoing intra-abdominal hemorrhage.
This patient has a relatively benign abdomen for intestinal
perforation and with recent CT, which showed only pelvic
fracture, an intra-abdominal source of bleeding is not likely.
Although sinus tachycardia is the most common dysrhymia seen
with blunt cardiac injury, in this patient the ongoing need for
blood suggests ongoing blood loss, not blunt cardiac injury as the
cause of instability.

RATIONALE (14) Answer: B

This patient’s radiograph is consistent with traumatic rupture of


the diaphragm secondary to blunt abdominal trauma. Although in
this case the findings are obvious, radiographic findings are only
diagnostic in 16%–30% of patients with diaphragmatic injury.
Radiographic findings include a very high diaphragmatic shadow,
the presence of bowel in the chest, and nasogastric tube passing
high into the left hemithorax. With blunt injury, the
diaphragmatic hernia results from a burst injury of the
diaphragm secondary to a rapid increase in abdominal pressure.
The cause may be motor vehicle crash, falls, kicks, and crush
injuries. Currently, motor vehicle crash from lateral impact
results in this injury more frequently than frontal collisions. With
blunt injuries, most diaphragmatic hernias occur on the left side
(85%), followed by the right and bilateral injuries. Stomach,
colon, omentum, spleen, and small bowel can cause herniation
into the chest. Cardiopulmonary compromise may occur from the
effect of the abdominal contents on the chest cavity. Both venous
return and vital capacity are reduced. Immediate surgical repair
is recommended. CT is the best noninvasive diagnostic modality,
but it cannot rule out subtle injury. Additional contrast studies
can be obtained to help make the diagnosis. In this case, the
diagnosis is suspected, so no further delay in surgery should
occur. Failure to recognize a traumatic diaphragmatic hernia
predisposes patients to the complication of bowel obstruction or
strangulation in the future. In the ventilated patient, a
diaphragmatic hernia may not be recognized until the patient is
extubated.
RATIONALE (15) Answer: D

This patient has bilateral pulmonary emboli. Patients with


traumatic injuries are at risk for deep venous thrombosis and
pulmonary embolus. Pulmonary embolus increases mortality of
the trauma patient nearly tenfold. When there is no
contraindication to anticoagulation, the trauma patient should be
fully anticoagulated as other patients; however, Brathwaite et al1
studied low-risk trauma patients who were treated for
pulmonary embolism with anticoagulation. There was a 36%
bleeding complication rate in this group of patients many of
which were life-threatening. At particular risk were those elderly
trauma patients who received anticoagulation. In this study the
authors also noted that there were 34 other patients who
received inferior vena cava filters with no related complications
or deaths. These authors concluded that anticoagulation for deep
venous thrombois/pulmonary embolism should be used
selectively in trauma patients and avoided in elderly patients,
who should undergo early vena cava filter placement instead.
Those trauma patients who have injuries that would be
exacerbated by bleeding (eg, incomplete spinal cord injuries,
head injuries, ocular trauma, nonoperatively managed solid
organ injuries) should be considered for mechanical interruption
of the vena cava. This patient’s traumatic subdural hematoma is a
contraindication to anticoagulation so vena cava filter should be
placed.

RATIONALE (16) Answer: C

Upper gastrointestinal bleeding after severe hepatic trauma


should be considered to be related to hemobilia (arterial-biliary
fistula) until proven otherwise. The classic triad of hemobilia
includes colicky right upper quadrant pain, gastrointestinal
hemorrhage (hematemesis or melena), and jaundice; all three
elements, however, fully present in only 22% of involved
patients. Selective celiac angiography will typically demonstrate a
segmental hepatic artery pseudoaneurysm, and establish the
appropriate access for embolization. Surgery should be reserved
for extensive lesions not amenable to angiographic ablation, and
typically requires formal hepatic resection with significant
associated morbidity. Iatrogenic trauma has surpassed other
forms of hepatic injury as the leading cause of hemobilia in recent
years. Percutaneous liver biopsy and percutaneous biliary
drainage procedures are the leading culprits, although the
associated bleeding is typically relatively minor and can be
managed conservatively. Calculous gallbladder disease rarely
presents with hemorrhagic cholecystitis, but urgent
cholecystectomy is required in such an instance. Primary hepatic
vascular malformations or neoplastic lesions have also been
reported to result in severe hemobilia. Effective biliary drainage
via endoscopic retrograde cholangiopancreatography or
percutaneous transhepatic cholangiography may be required in
certain circumstances to allow adequate decompression of clot-
obstructed ducts.

RATIONALE (17) Answer: B

Blunt cerebrovascular injury (BCVI) involving either the carotid


or vertebral artery distribution is diagnosed in about 0.1% of all
trauma patients treated in the United States. Unfortunately most
injures are diagnosed after the development of neurologic
symptoms, with significant associated long-term functional
impairment. Several models have been developed to effectively
screen those patients at greatest risk of BCVI. Typically included
in such schemes are patients with C1-3 fractures, or ligamentous
injuries at any cervical level; transverse foramen fractures
(specifically veretebral artery injuries); severe midface or
mandibular fractures; and basilar skull fractures involving the
carotid canal. Four-vessel cerebral angiography (FVCA) remains
the gold standard for diagnosis, although recent studies have
shown 16-slice CT angiography to have nearly equivalent
sensitivity and specificity. Magnetic resonance angiography and
duplex ultrasonography have been studied as alternative
screening modalities; however, published data reveal
significantly lower diagnosis rates than with FVCA or CT
angiography.

Specific treatment is predicated on the severity of injury. A


simple grading classification described by Biffl1 is shown below.
For injuries of grade II and higher, treatment with
antithrombotics should be undertaken if deemed safe in light of
the associated systemic and intracerebral bleeding risk. Systemic
anticoagulation remains the gold standard, but recent studies
suggest that antiplatelet agents (aspirin or clopidogrel) may be
equally efficacious in stroke prevention. Mandatory short-term
follow-up of BCVI 7 to 10 days after injury should be undertaken
to verify lack of progression. High-grade lesions, especially with
pseudoaneurysm formation, should be repaired by either a
surgical (if accessible) or endovascular approach; there is no
discrete data at present comparing the outcomes of specific
interventions.

Blunt Carotid and Vertebral Arterial Injury Grading Scale

I. Luminal irregularity or dissection with <25% luminal


narrowing

II. Dissection or intramural hematoma with >25% luminal


narrowing, intraluminal thrombus, or raised intimal flap

III. Pseudoaneurysm
IV. Occlusion

V. Transection with free extravasation

RATIONALE (18) Answer: D

This patient has wound dehiscence, which typically occurs on


postoperative day 4 or 5. Wound dehiscence is characterized by
profuse drainage of salmon-colored fluid that soaks the dressing
and bedding. It is a technical problem with the wound closure
technique allowing the fascia to open. It can lead to evisceration
of abdominal contents. The dressing should be removed to
examine the wound. If bowel is exposed, wet saline packs are
placed, and the patient returns to the operating room for wound
closure. There is a role for nonoperative management with moist
dressings and an abdominal binder. Risk factors for wound
dehiscence include age older than 45 years, malnutrition, morbid
obesity, cancer, uremia, diabetes, coughing, and other causes of
increased intra-abdominal pressure, infection, and hemorrhage.
Unless an intra-abdominal abscess is suspected, CT of the
abdomen is not indicated. Long-term outcome issues associated
with wound dehiscence include incisional hernias.

RATIONALE (19) Answer: C

This patient has developed hypotension, high airway pressures


with hypoxemia, and oliguria. These findings are consistent with
abdominal compartment syndrome (ACS). Commonly described
in patients after laparotomy for blunt and penetrating abdominal
trauma, ACS was initially described after repair of ruptured
abdominal aortic aneurysm. Other conditions associated with
ACS are retroperitoneal hemorrhage, burns, peritonitis, liver
transplantation, pancreatitis, and ascites. Persistent increase in
intra-abdominal pressure will produce multiple organ
dysfunction. Hypotension results from a combination of
decreased venous return and increased intrathoracic pressure
(decreases left ventricular compliance). Respiratory failure with
increased peak ventilatory pressures, hypoxemia, and eventual
hypercapnia is seen with increased intra-abdominal pressures. As
intra-abdominal pressure increases, a progression from oliguria
to anuria is commonly seen. Documentation of an increased intra-
abdominal pressure is key in establishing the correct diagnosis
and directing therapy. Currently, urinary bladder pressure
measurement is the most simple, reliable, and accepted
methodology. A Foley catheter attached to a water manometer
can be used. Intra-abdominal hypertension is seen with intra-
abdominal pressures greater than or equal to 12 mm Hg. ACS
warranting decompressive laparotomy is defined by intra-
abdominal pressure greater than or equal to 20 mm Hg in
association with new-onset single or multiple organ failure.
However, the decision to decompress the abdomen may come at
pressures less than 20 mm Hg if intra-abdominal hypertension is
seen with organ failure related to this problem. Decisions for
surgical decompression should take into account the clinical
findings and the actual pressure. A transesophageal
echocardiography would be of no value in this case, because
there is no evidence of cardiac or thoracic aorta pathology
causing clinical findings. CT or abdominal ultrasonography would
not reveal further information. A new collection of blood,
detectable by these imaging tests, is unlikely considering
hemoglobin has remained stable. Pulmonary artery catheter
pressures will not aid in making a correct diagnosis.

RATIONALE (20) Answer: D

In this patient with significant organ dysfunction from abdominal


compartment syndrome, immediate relief of increased abdominal
pressure is mandatory. Of the options provided, a bedside
laparotomy is the only treatment that will achieve this goal. The
patient is critically ill and even delaying abdominal
decompression to transport the patient to an operating room
might be deleterious. Abdominal decompression will
dramatically improve peak ventilatory pressures, hypoxemia,
hypotension, and oliguria in this patient. There is no indication of
acute hemorrhage requiring transfusion of blood products.
Although fluid resuscitation can be used to treat increased intra-
abdominal pressure in early abdominal compartment syndrome,
it will not be effective in patients with severe organ dysfunction.
Emergent dialysis is not indicated for this patient since the
oliguria is secondary to increased intra-abdominal pressure. A
pericardiocentesis is not indicated since there is no evidence of
cardiac tamponade. Finally, changing the ventilator mode to
pressure control might decrease peak airway pressure but will
not result in prolonged improvement in oxygenation.

RATIONALE (21) Answer: B

This postpartum patient has developed signs and symptoms


consistent with an epidural hematoma. Common signs and
symptoms include sharp, severe, radicular back pain followed by
neurologic symptoms of cord compression. The differential
diagnosis of this disorder includes spinal or epidural abscess,
acute herniated disk, spinal cord ischemia, spinal neoplasm,
spondylitis, lipoma, transverse myelitis, and dissecting aortic
aneurysm. MRI remains the gold standard for confirming the
diagnosis. Common risk factors for an epidural hematoma
following epidural anesthesia include age older than 50 years,
spinal stenosis, coagulopathy, platelet inhibition, arteriovenous
malformation, or multiple passes with an epidural needle.

RATIONALE (22) Answer: B


Of the five modalities listed, thoracic epidural with local
anesthetic provides superior analgesia for her rib fractures. The
choice of epidurally administered local anesthetic or local
anesthetic plus opioid should be tailored to the individual patient
and left to patient decision. Although local anesthetic opioids
provide synergistic effects and improve analgesia, the addition of
opioid could potentially lead to altered mental status in elderly
patients. Both intercostal and paravertebral blocks would require
repeat placement for pain control greater than 48 hours.

RATIONALE (23) Answer: A

Peripartum cardiomyopathy is a dilated cardiomyopathy that


develops in the last gestational month of pregnancy or in the first
5 months after delivery. It occurs more frequently in older
women, obese women, and multiparous women with twin
pregnancies. The etiology of peripartum cardiomyopathy is
unknown, although many hypotheses have been suggested, such
as viral myocarditis, immune-mediated injury, selenium
deficiency, and the hemodynamic stress of pregnancy.1 The
presentation of peripartum cardiomyopathy is similar to other
dilated cardiomyopathies and most patients present in New York
Heart Association class III or IV functional status. Patients who
present early after delivery often have dramatic symptoms and
signs of congestive heart failure. The diagnosis is usually more
difficult during the late stages of pregnancy because of overlap
with symptoms of pregnancy. The treatment of peripartum
cardiomyopathy is similar to the treatment of acute and chronic
heart failure due to other causes of left ventricular systolic
dysfunction. Patients who are congested but have adequate
perfusion require initial treatment with IV diuretics alone or in
combination with vasodilators such as nitroglycerin,
nitroprusside, or nesiritide.2 Patients with diminished perfusion
require augmentation of their cardiac output with inotropic
drugs such as IV dobutamine or milrinone. Patients should then
be started on an angiotensin-converting enzyme inhibitor if the
diagnosis is made postpartum. More than half of these patients
normalize their left ventricular ejection fraction during the first 6
months of presentation. Indicators of poor prognosis in
peripartum cardiomyopathy include a decreased left ventricular
ejection fraction 6 months postpartum, larger left ventricular and
diastolic dimension, clinical presentation more than 2 weeks
postpartum, age older than 30 years, African American descent,
and multiparity.3

RATIONALE (24) Answer: A

Posterior reversible encephalopathy syndrome (PRES), or


reversible posterior leukoencephalopathy syndrome, is a well-
established neurological disorder with characteristic MRI
findings. It is associated with a multitude of diverse clinical
entities, including preeclampsia and eclampsia, acute
glomerulonephritis, systemic lupus erythematosus, thrombotic
thrombocytopenic purpura, hemolytic-uremic syndrome, and
drug toxicity from various immunosuppressive and
chemotherapeutic agents. Clinically the syndrome is
characterized by headache, confusion, seizures, and vision loss,
usually in the setting of these clinical entities. Even though PRES
is considered reversible, prompt recognition and treatment is
necessary to prevent cerebral ischemia and bad outcome.
Treatment includes removing precipitating factors and
controlling blood pressure. Because of possible loss of
autoregulation, particularly in posterior circulation, lowering of
mean arterial pressure should be done with extreme caution in
order to avoid cerebral ischemia.

RATIONALE (25) Answer: D


Respiratory failure in pregnancy is responsible for significant
morbidity and mortality of both the mother and the fetus.
Respiratory failure can result from a number of pregnancy-
related complications, such as pneumonia, preeclampsia,
pulmonary embolism, peripartum cardiomyopathy, asthma,
aspiration, and cardiogenic and noncardiogenic pulmonary
edema. Maternal physiologic adaptations to pregnancy reduce
tolerance to all types of respiratory failure. Increased oxygen
consumption and decreased functional residual capacity place
both the mother and the fetus at risk from hypoventilation and
apnea. The onset of pneumonia is not gestational-age dependent.
Pregnancy increases pneumonia complications such as the need
for mechanical ventilation and intubation, bacteremias, and
empyemas. Fetal complications include higher rates of preterm
labor, lower average birth weights, and higher neonatal mortality
rates. The most common bacterial agents causing pneumonia in
pregnancy include Streptococcus pneumoniae and Haemophilus
influenzae.1 Viral pneumonias such as varicella and influenza are
less common. Risk factors for developing pneumonias during
pregnancy include preexisting diseases such as HIV infection,
asthma, cystic fibrosis, anemia, cocaine and alcohol abuse, and
use of tocolytic agents and corticosteroids. Guidelines developed
by the British Thoracic Society for assessing the course of the
pneumonia, the need for hospitalization, and mortality prediction
consider the presence of 2 of the following 4 criteria to indicate
severe illness. These include RR greater than 30/min, diastolic BP
less than 60 mm Hg, blood urea nitrogen level greater than 19.1
mg/dL, and confusion.2 The combination of a macrolide and a
beta-lactam antibiotic has proven effective for treatment of
bacterial pneumonias in pregnancy and has a favorable fetal
safety profile. Peripartum cardiomyopathy most often presents
abruptly after delivery of the fetus with the development of
pulmonary edema and evidence of acute left heart failure that is
treated with diuresis and inotropic support once the diagnosis is
confirmed by echocardiography. Asthmatic symptoms may
worsen, improve, or remain unchanged during pregnancy but
would be unlikely to occur de novo in late pregnancy. Caution
should be exercised using noninvasive ventilation in pregnancy
because of the increased risk of aspiration, especially in advanced
pregnancy, and in particular if the patient has a recent history of
emesis.3
PART 10: Administrative and Ethical Issues in the Critically Ill

Instructions: For each question, select the most correct answer.

1. Which of the following components most helpful in framing


quality improvement programs in the ICU?

A. Structure, process, and outcome

B. Problem, question, and hypothesis

C. Vision, objective, and mission

D. Exposure, confounder, and disease

E. Problem, analysis, and plan

2. A 55-year-old man is admitted to the ICU after sustaining a


closed head traumatic injury. He has a depressed level of
consciousness, with stable vital signs. He opens his eyes to
speech, localizes to pain, and moans but does not speak. There
are no lateralizing signs.

What is his Glasgow Coma Scale score?

A. 15

B. 13

C. 10

D. 8

E. 6
3. A 56-year-old man is admitted to the hospital after a motor
vehicle collision. His Glasgow Coma Scale score after
resuscitation is 4 and he has flexor posturing bilaterally. His
pupils are asymmetric and the right is dilated to 5 mm. His
temperature is 36°C (96.7°F), BP is 90/50 mm Hg, HR is 110/min,
RR is 18/min, and oxygen saturation is 98% (1.0). CT of the head
shows bilateral frontal contusions and subarachnoid hemorrhage
with a large right subdural hematoma. The cervical spine was
immobilized in the usual fashion. Focused assessment with
sonography for trauma shows a small collection at the right
retrocolic recess. The neurosurgery team has evaluated the
patient and is planning to take him to the operating room.

Which of the following prediction scores will best evaluate the


patient’s operative risk?

A. American Society of Anesthesiologists Physical Status


Classification System

B. World Federation of Neurological Surgeons scale

C. Injury Severity Score (ISS)

D. Sequential Organ Failure Assessment score (SOFA)

E. Multiple Organ Dysfunction Score (MODS)

4. Which of the following processes is best supported by ICU


scoring systems?

A. Adjustments of medication dosing and delivery

B. Support for decision analysis and resource allocation


C. Improving testing accuracy by improving pre-test probabilities

D. Increasing diagnostic power

5. ICU scoring systems help with estimation and prediction of


mortality based on statistical models and their ability to
discriminate between groups. The performance of these models
and the ability to discriminate between survivors and
nonsurvivors is best evaluated by which of the following
characteristics of the model?

A. Odds ratio and 95% confidence interval

B. Standardized mortality rate

C. Frequency estimate of the disease

D. Alpha of the test statistic

E. Receiver operating characteristic

6. A 32-year-old man with acute myeloid leukemia is in the ICU


after being admitted to the ICU for severe sepsis. The resident
discusses with the patient the need for inserting a central line for
fluid and medication administration. The patient is thankful for
the resident’s explanation of the risk-to-benefit ratio and
alternatives.

The patient agrees that he is critically ill but refuses the


intervention, arguing that there is no need for it and that he can
get two peripheral IV lines for the same purpose. The resident
documents his discussion and proceeds to insert 2 peripheral IV
lines. Which of the following ethical principles supports the
resident’s response?

A. Autonomy

B. Independence

C. Utilitarianism

D. Beneficence

E. Virtue

7. An 85-year-old woman with Alzheimer dementia is admitted to


the ICU for severe community-acquired pneumonia and acute
hypoxemic respiratory failure. The patient has no advance
directive or additional family members, and the team realizes
that she has been living with a son-in-law who did not know her
well. In the ICU she continues to deteriorate, developing acute
respiratory distress syndrome and multiple organ dysfunction
syndrome requiring further renal replacement therapy. Given the
physiologic deterioration refractory to aggressive medical
management, the team approaches the son-in-law and asks if he
would like to continue with advanced life support in the setting of
what the treating physicians consider futile care. He says that,
though he never talked about end-of-life issues with his mother-
in-law, he thought that she was the type of person that “would
not have liked to be kept alive artificially.”

Which of the following approaches should the team take at this


point?

A. Continue with current therapy as there is no advance directive


and the patient’s family may file legal suit.
B. Get a court order to assign a representative to the patient as
the son-in-law is unaware of the patient’s wishes.

C. Change goals of care from full medical management to “comfort


care” and issue a do-not-resuscitate order after documenting
discussion on the chart.

D. Request an ethics consultation from the hospital.

E. Perform a tracheostomy, percutaneous endoscopic


gastrostomy, and send the patient to a nursing home.

8. Which of the following principles was established by the


Declaration of Helsinki?

A. The right to adequate medical services and health care systems

B. Ethical principles for medical research involving human


subjects

C. The right to refuse to eat and not be fed artificially against


one’s will

D. The need for the development of guidelines for the ethical


practice of medicine

E. The right to die with dignity

9. In the case of Karen Quinlan (In re Quinlan, 70 NJ 10 [1976]),


the Supreme Court of New Jersey established that:

A. The principle of clear and convincing evidence may be used to


withdraw life support
B. The physician has a duty to warn against potential harm

C. The principle of beneficence supercedes the patient’s wishes

D. Patients have a right to privacy and that therapeutic decisions


can be based on a substituted judgment standard

E. Physicians cannot withdraw life support unless supported by a


court order

10. An ICU team was interested in knowing about patient


satisfaction after the institution of a rapid response team (RRT).
Every time the RRT was activated, the investigator collected
information from patients and/or family members using a sample
questionnaire. The investigators asked on a scale of 1–6 if the
patient or family members were satisfied with the service during
urgent situations and dichotomized this outcome into 1–3
(satisfied) and 4–6 (not satisfied). They further divided the
cohort into patients younger and older than 50 and compared the
satisfaction results. After 6 months, the investigators collected
information from 632 subjects and reported that 78% of them
were satisfied with the service. Among those older than 50, the
RRT had a significant impact on patient/family satisfaction (odds
ratio, 0.8; 95% confidence interval, 0.7–0.9).
This example illustrates an observational study with which of the
following designs?

A. Prospective cohort

B. Retrospective cohort

C. Case-control
D. Case-cohort

E. Cross-sectional

11. During his 2-year ICU training, a fellow collected information


on all the procedures performed and associated complications.
Specifically, this fellow inserted 45 central lines and 5 pulmonary
artery catheters (PACs) during the first year, and 40 central lines
and 10 PACs during the second year. Of these insertions, 1 was
associated with a pneumothorax and 4 with line-related sepsis.
Among the 4 with line-related sepsis, the fellow recorded that 2
patients had candidemia.

Which of the following best characterizes the incidence rate per


year of complications for this fellow at the end of the fellowship
and the prevalence of candidemia within his patient group?

A. 1% per year and 5%

B. 5% per year and 1%

C. 2% per year and 2.5%

D. 2.5% per year and 2%

E. Can’t be calculated

12. An experienced investigator is developing a new urine test for


diagnosing seizures in critically ill patients. The innovative test
detects specific neuronal proteins in the urine that are spilled
into the circulation in an epileptic convulsion. In the presence of
urine of a patient with a seizure, the test turns green; it remains
unchanged if there has been no seizure. This could be a
revolutionary test as physicians will not need to rely solely on
clinical examination and electroencephalogram, and it could be
done at the bedside. The following Figure describes the results
obtained with the new test.

What is the specificity of this test?

A. 425/1,550

B. 1,575/3,250

C. 1,575/2,000

D. 425/2,000

E. 1,575/3,250

13. A 45-year-old woman with a history of factor V Leiden is


admitted to the hospital for evaluation of “pleuritic” chest pain, a
swollen leg, and shortness of breath. On admission, BP was
120/85 mm Hg, HR was 122/min, RR was 24/min, and oxygen
saturation was 90% on oxygen, 2 L via nasal cannula. Arterial
blood gas results showed pH of 7.47, Pao2 of 30 mm Hg, Paco2 of
60 mm Hg, and 90% oxygen saturation. Chest radiograph
revealed normal lung fields and cardiomediastinal figure.
Which of the following tests would be most appropriate to
establish a diagnosis of pulmonary embolism in this patient?

A. Test A

B. Test B

C. Test C

D. Test D

14. A critical care fellow in the ICU was interested in whether the
implementation of a daily checklist was associated with reduced
ICU length of stay (LOS). To test his hypothesis. he measured the
LOS in days in 2 different groups of critical care patients. Group 1
had the checklist implemented during working rounds and Group
2 had no checklist implemented for working rounds. Patients
admitted to the ICU between 7 am and 7 pm were in Group 1, and
those admitted between 7 pm and 7 am were in Group 2. After 6
months, he collected information on 258 patients. The results
showed that the ICU LOS of Group 1 was 4.5 ± 2.1 days and in
Group 2 was 5.1 ± 1.8.

Which of the following test statistics would this fellow use to


determine a statistical difference between the mean LOSs of the 2
groups?
A. Chi-square test

B. Student t test for mean

C. Mann-Whitney test

D. Linear regression

E. Logistic regression

15. Which of the following will best determine the probability


that a test will reject the null hypothesis when this hypothesis is
false?

A. Alpha

B. Beta

C. 1 – beta

D. 1 – alpha
PART 10: Administrative and Ethical Issues in the Critically Ill

ANSWERS:

1–A; 2–D; 3–A; 4–B; 5–E; 6–A; 7–C; 8–B; 9–D; 10–E; 11–D; 12–C;
13–D; 14–B; 15–C

RATIONALE (1) Answer: A

Classic quality-of-care components have been described by the


Donabedian model. Structure represents the first component of
the quality-of-care model and can be defined as the way we
organize care. Process generally refers to what we do, or fail to
do, for patients and their families. Outcomes represent the third
component of the quality-of-care model and refer to the results
we achieve. Critical care clinicians interested in quality
improvement should understand the structure-process-outcome
model and select aspects they are both interested in and able to
improve.

RATIONALE (2) Answer: D

The Glasgow Coma Scale (GCS) score was developed to provide a


consistent description of patients with head injury, as well as to
provide a measure of severity. Patients are scored on eye
opening, best motor response, and verbal response. Points are
given for each area and the sum of the points provides the score.
The best possible score is 15 and the worst score is 3. The
severity of head injury can be characterized as mild if it scores
14–15 on the GCS, moderate if the score is 9–13, and severe if the
score is 3-8. Repeated examination and GCS scoring in response
to treatment provides information for prognosis, morbidity, and
mortality. A GCS score of 8 or less describes a coma; at that point,
early tracheotomy is considered. The GCS is not a substitute for a
complete neurologic examination with cranial and peripheral
nerve evaluation for isolated or lateralizing signs. By adding the
scores highlighted on the scale below, it is clear that the patient
described in this question has a score of 10, which indicates a
moderate head injury.

RATIONALE (3) Answer: A

The American Society of Anesthesiologists (ASA) physical status


classification system is used to predict perioperative mortality
based on functional status. In 1963, the ASA adopted the 5-
category physical status classification system; a sixth category
was later added. The categories are:
1. A normal healthy patient
2. A patient with mild systemic disease
3. A patient with severe systemic disease
4. A patient with severe systemic disease that is a constant threat
to life
5. A moribund patient who is not expected to survive without the
operation
6. A patient declared brain dead whose organs are being removed
for donor purposes
The most commonly used organ dysfunction scoring systems are
the Multiple Organ Dysfunction Score (MODS), the Sequential
Organ Failure Assessment (SOFA), and the Logistic Organ
Dysfunction (LOD) score, but these scores are not used to predict
perioperative mortality. Thus, options D and E are incorrect. The
clinical grading system proposed by the World Federation of
Neurological Surgeons (option B) is intended to be a simple,
reliable, and clinically valid way to grade a patient with
subarachnoid hemorrhage (not traumatic brain injury) and does
not offer any operative risk prediction utility. The Injury Severity
Score (ISS) is an anatomical scoring system that provides an
overall score for patients with multiple injuries. Each injury is
assigned an abbreviated injury scale (AIS) score and is allocated
to one of 6 body regions (head, face, chest, abdomen, extremities
[including pelvis], external). Only the highest AIS score in each
body region is used. The three most severely injured body
regions have their score squared and added together to produce
the ISS score. The ISS offers no operative risk prediction utility, so
option C is incorrect.

RATIONALE (4) Answer: B


Adult ICU prognostic models have been used for predicting
patient outcome for 3 decades. Scoring systems are also used to
provide information about disease severity, aid in clinical
management by supporting decisions for resource allocation,
interpret data by adjusting for case-mixture and disease severity,
and may be employed for surveys or comparative audits. ICU
scoring systems do not provide information to perform
adjustments in medication delivery or increase diagnostic power
by helping with testing accuracy.

RATIONALE (5) Answer: E

The relationships between the predictor and outcome variables


of the development model need independent validation. A
mortality prognostic model must differentiate between survivors
and nonsurvivors and be well calibrated and reliable. It also has
to be periodically updated to reflect the change in medical
practice and case mix over time. The performance of the ICU
prognostic models is assessed for discrimination by the area
under the receiver operating characteristic (ROC) curve, and for
calibration by the Hosmer-Lemeshow statistic for calibration. An
ROC curve is a graphical plot of the sensitivity or true-positive
rate, versus false-positive rate (1-specificity), for a binary
classifier system as its discrimination threshold is varied. The
area under the ROC curve is the measure of how well a model
differentiates between groups (areas under the ROC curve of 1,
0.90–0.99, 0.80–0.89, 0.70–0.79, 0.60–0.69, and 0.60 are
considered to be perfect, excellent, very good, good, moderate,
and poor, respectively). Calibration refers to the correlation
between the predicted and actual outcome for the entire range of
risk. The calibration is considered good if the Hosmer-Lemeshow
statistic P value is less than .05

RATIONALE (6) Answer: A


Autonomy refers to the principle of self-rule, self-determination,
or self-governance. It supports the notion that rational
individuals with decisional capacity (or competency in legal
terms) are uniquely qualified to decide what is best for
themselves and should be allowed to do whatever they want. This
holds true even if doing so involves considerable risk or would be
deemed foolish by others, providing their decision does not
infringe on the autonomy of another. In the US, the principle of
autonomy is strongly guarded by the Constitution. The universal
principles of autonomy, beneficence, and justice were established
by the Belmont Report in 1976. The principle of beneficence
(option D) is inherent to the role of physicians and determines
the duty to prevent evil or harm by promoting good and welfare
for others. The principle of justice demands that one act to
promote the greatest benefit to the greatest number of
individuals while inflicting the least amount of harm. The
principle of nonmaleficence was later introduced by Beauchamp
and Childress in their landmark textbook on bioethics.2 This
principle establishes the duty to refrain from inflicting harm on
others and is sometimes defined by the maxim “primum non
nocere” (first do no harm).

RATIONALE (7) Answer: C

The team should switch the goals of care from full medical
management to “comfort care” and issue a do-not-resuscitate
order after documenting discussion on the chart based on the
best-interest standard. In the case of In re Conroy, (486 A.2d
1209 NJ 1985), Claire Conroy, an elderly mentally and physically
incapacitated woman residing in a nursing home had a guardian
(nephew) who did not know her explicit wishes but felt she
would not have liked to have a feeding tube. In this case, the New
Jersey Supreme Court permitted the use of the best-interest
standard, which is applicable in those cases where the burden of
a therapy outweighs the benefits and the pain of interventions
which would make them inhumane. The standard is based on the
principle of nonmaleficence, or primum non nocere (first do no
harm).

RATIONALE (8) Answer: B

The Declaration of Helsinki was adopted during the 18th World


Medical Association General Assembly in June 1964. It is a
statement of ethical principles for medical research involving
human subjects, including research on identifiable human
material and data. It follows the principles of the Nuremberg
Code developed after the Nuremberg Trials. Different countries
and political systems consider the right to adequate medical
services and health care systems as a fundamental human right;
however, in some countries, even developed ones, health care
coverage and access to health care systems is not possible for all
individuals. Thus, option A is incorrect. The Declaration of Tokyo
established “torture to be contrary to the laws of humanity” and
that it was unethical to artificially feed a prisoner who refuses to
eat (option C). The right to die with dignity (option E) is derived
from the ethical principle of autonomy and is endorsed by many
medical and ethical societies (In re Quinlan, 70 NJ 10 [1976]).

RATIONALE (9) Answer: D

Karen Quinlan was a 21-year-old woman in a persistent


vegetative state (PVS) from severe anoxic brain injury secondary
to drug intoxication during a party. She was kept alive and on a
ventilator for several months. Because of her lack of
improvement, her parents requested that the hospital
discontinue medical care and allow her to die from natural
causes, but the hospital refused. After much rhetoric and a legal
battle, the New Jersey Supreme Court determined that a guardian
ad litem (appointed by court order) was not necessary to
represent Karen independently in this particular case and
allowed her father to make all decisions on her behalf. The basis
of the court decision is rooted in an individual’s legal right to
privacy and the notion that her father could make the assertion
based on the family’s best judgment (substituted judgment
standard). The decision included legal immunity for the
physicians and the suggestion to involve ethics committees in
such cases. This particular case sparked the development of
ethics committees, advance directives, and nursing homes for
palliative care. After weaning from the ventilator in 1976, Karen
continued to breathe on her own until she died in 1985.
The principle of clear and convincing evidence (option A) is a
legal principle used in the US legal system (2 others being the
standards of reasonable doubt and preponderance of the
evidence). This principle can be used by physicians in certain
states (Missouri, New York, Florida, among others) to withdraw
life support or any other intervention when there is clear and
convincing evidence of a patient’s previous statements and in the
absence of a declaration such as a living will, advance directive, or
durable power of attorney. The decision is based on Cruzan v.
Director, Missouri Department of Health (497 US 261[1990]), in
which the court endorsed the right of a competent person to
refuse medical therapy even if the refusal results in the patient’s
death; the decision is also based on the liberty interest set forth
in the Fourth Amendment. The case of Terry Schiavo (Schindler v.
Schiavo, 866 So2d 140 [Fla Dist Ct App 2004]) was ruled
following the same principle and endorsing the historic Cruzan
decision. In Tarasoff v. Regents of the University of California
(551 P2d 334 [1976]), the court recognized the duty of a
physician to warn another person of a dangerous patient’s intent
on doing harm to that person even if this meant violating the
patient-physician confidentiality agreement. The principle of
beneficence is inherent to the role of physicians and determines
the duty to prevent evil or harm by promoting good and welfare
for others. One’s perceptions and intentions even if in the best
interest should not supercede the wishes of another competent
human being. Thus, option C is incorrect.

RATIONALE (10) Answer: E

A cross-sectional study is a type of observational study and


involves the collection of data at a defined point in time. It is often
used to test the prevalence of acute or chronic conditions or to
answer questions about the causes of disease or the results of an
intervention. Cross-sectional studies are also called prevalence
studies. There are 2 primary types of nonexperimental or
observational studies in epidemiology. The cohort study or
incidence study, is a direct analogue of the experiment, where
different exposure groups are compared but the investigator does
not assign the exposure. In this sense, the cohort study is the gold
standard of observational studies. When the outcome has not
occurred and the investigator follows it up into the future, it is
called a prospective cohort. When the outcome has occurred and
the investigator goes back in time to identify the exposure, it is
called a retrospective cohort. The second type of
nonexperimental, observational study is the case-control study. It
employs an extra step of sampling according to the outcome of
individuals in a population where the outcome is known (cases).
The cases are compared to individuals without the outcome
(controls), who are sampled independent of the exposure. A case-
cohort study is a variant of the cohort study where individuals
are randomly sampled from the cohort at baseline, creating a
subcohort, rather than when cases occur (incidence density
sampling).

RATIONALE (11) Answer: D


In epidemiology, the incidence is a measure of the risk of
developing a new condition within a specified period of time. The
incidence proportion (cumulative incidence) is the number of
new cases within a specified time period divided by the size of
the population at risk (ie, for complications [5/100] × 100 = 5%).
The incidence rate (incidence density or person-time incidence
rate), on the other hand, is expressed as the number of new cases
per population in a given time period (ie, 5/100 ÷ 2 years = 2.5%
per year). Similarly, the prevalence of a disease is defined as the
total number of cases of the disease in the population at a given
time and is used as an estimate of how common a condition is
within a population (ie, for candidemia, [2/100] × 100% = 2%).

RATIONALE (12) Answer: C

The false-positive rate (FP) is the proportion of absent events


that yield a positive test outcome or the conditional probability of
a positive test result given an absent disease or condition (ie,
425/2,000 = 0.2125). The specificity of a test is the proportion of
negatives which are correctly identified by a test or the
conditional probability of a negative result given an absent
disease or condition (ie, 1,575/2,000 = 0.7875) and is equal to 1 –
FP (True negatives ÷ [True negatives + FP]). In hypothesis
testing, the FP equals the significance level, which is expressed as
alpha. Hence, increasing the specificity of a test will decrease the
probability of FP or type I error (the probability of rejecting a null
hypothesis when it is true). Similarly, the false-negative rate (FN)
is the proportion of positive or present events that yield a
negative test outcome or the conditional probability of a negative
test result given a present or positive disease or condition (ie,
125/1,250 = 0.1 ). The sensitivity of a test is the proportion of
positives which are correctly identified by a test or the
conditional probability of a positive result given a present or
positive disease or condition (ie, 1,125/1,250 = 0.9) and is equal
to 1 – FN (True positive ÷ [True positive + FN]). In hypothesis
testing, the FN equals the beta error. Hence, increasing the
sensitivity of a test will decrease the probability of FN or type II
error (the probability of failing to reject a null hypothesis when it
is false).

RATIONALE (13) Answer: D

The positive predictive value (PPV) of a test is the proportion of


subjects with a positive result that are correctly identified as
positive for a diseaseor condition, or the conditional probability
of the presence of a disease or condition given a positive result.
The PPV is also known as the precision of the test, and its value
depends on the prevalence of the outcome of interest, also known
as the pretest probability of the disease. Sensitivity and
specificity are said to be unchangeable properties of a test
because they do not vary with pretest probabilities (or
prevalence) of the disease. It is much more useful to know the
probability of disease in a patient who has a positive test result
(PPV) and/or the probability of nondisease in a patient with a
negative test result (NPV). In other words, as the pretest
probability of disease falls, the predictive value of a positive
result also falls, and the predictive value of a negative result rises.
As the pretest probability of disease increases, a positive test
result is more informative than a negative result.

RATIONALE (14) Answer: B

Hypothesis testing is a method of making decisions from data


collected from controlled experiments or observational studies.
The first step in hypothesis testing is to determine a null
hypothesis (H0) and the alternate hypothesis (HA). Next, the
investigator needs to pick a test statistic and a significance level
at which rejection of the null hypothesis would result not by
chance alone. The significance level is usually referred as alpha or
P-value. The test statistic is determined based on the number of
subjects, variance, and distribution of the data. The t test follows
a Student t distribution, and is commonly applied when the test
statistic would follow a normal distribution. Test statistics that
follow normal distributions are considered as parametric tests.
Several tests can be employed to determine normality of the
sample, but usually the t test is robust for samples of more than
30 subjects or observations. When the data is not known to be in
a normal distribution, then nonparametric tests should be
applied to avoid the possibility of type I error (rejecting a null
hypothesis that is true). The Mann-Whitney or Wilcoxon rank
sum test is a type of nonparametric test statistic employed for
assessing whether 2 independent samples of observations have
equally large values. The chi-square test statistic, in which the
sampling distribution of the test statistic follows a chi-square
distribution, could be used to determine statistical differences
among proportions and not means or medians. Finally, linear and
logistic regression are statistical methods employed in modeling
the relationship between a scalar variable Y and one or more
variables denoted as X or covariates, and are in the domain of
multivariate analysis.

RATIONALE (15) Answer: C

In hypothesis testing, power is the probability that a test will


reject the null hypothesis when it is false or the probability of a
test’s not to making a type II error. Type II error, or beta, refers to
the probability of failing to reject a null hypothesis when it is
false (false-negative rate). Therefore, power equals 1 – beta, or
the sensitivity of the test. Hence, increasing the sensitivity of a
test will decrease the probability of false negatives or type II
error.

Das könnte Ihnen auch gefallen